「Lancet」の記事一覧

重症急性胆石性膵炎が疑われる症例に用いる緊急内視鏡的逆行性膵胆管造影による括約筋切開と保存的治療の比較(APEC試験) 多施設共同無作為化比較試験
重症急性胆石性膵炎が疑われる症例に用いる緊急内視鏡的逆行性膵胆管造影による括約筋切開と保存的治療の比較(APEC試験) 多施設共同無作為化比較試験
Urgent endoscopic retrograde cholangiopancreatography with sphincterotomy versus conservative treatment in predicted severe acute gallstone pancreatitis (APEC): a multicentre randomised controlled trial Lancet. 2020 Jul 18;396(10245):167-176. 原文をBibgraph(ビブグラフ)で読む 上記論文の日本語要約 【背景】緊急内視鏡的逆行性膵胆管造影(ERCP)による括約筋切開術によって胆管炎の合併がない胆石性膵炎の転帰が改善するかはいまだに明らかになっていない。著者らは、無作為化試験を実施し、重症急性胆石性膵炎患者に用いる緊急ERCPによる括約筋切開と保存的治療を比較した。 【方法】オランダの病院26施設で実施した多施設共同並行群間評価者盲検無作為化比較優越性試験では、胆管炎の合併がなく重症胆石性膵炎(APACHE IIスコア8点以上、Imrieスコア3点以上、CRP 150mg/L超)の疑いがある患者を適格とした。Webベースの無作為化モジュールを用いて無作為に選択したブロックサイズで被験者を緊急ERCPによる括約筋切開(受診後24時間以内)と保存的治療に(1対1の割合で)無作為に割り付けた。主要評価項目は、無作為化後6カ月以内の死亡または主要合併症(新たに発症した臓器不全の持続、胆管炎、菌血症、肺炎、膵臓壊死または膵機能不全)の複合としたintention to treatで解析した。この試験は、ISRCTNレジストリにISRCTN97372133番で登録されている。 【結果】2013年2月28日から2017年3月1日までの間に、232例を緊急ERCPによる括約筋切開(118例)と保存的治療(114例)に無作為に割り付けた。入院時、緊急ERCP群の1例に胆管炎、保存的治療群の1例に慢性膵炎があったため、ともに最終解析から除外した。主要評価項目は、緊急ERCP群117例中45例(38%)、保存的治療群113例中50例(44%)に発生した(リスク比0.87、95%CI 0.64-1.18、P=0.37)。胆管炎の発症[緊急ERCP群117例中2例(2%)、保存的治療群113例中11例(10%)、RR 0.18、95%CI 0.04-0.78、P=0.010]を除き、主要評価項目を構成する個々の要素に重要な差はなかった。緊急ERCP群118例中87例(74%)、保存的治療群114例中91例(80%)に有害事象が報告された。 【解釈】石性膵炎の疑いがあるが胆管炎の合併がない患者で、緊急ERCPによる括約筋切開によって主要合併症と死亡の複合とした評価項目は減少しなかった。この結果から、重症急性胆石性膵炎が疑われる患者には保存的治療を実施し、胆管炎の合併または胆汁うっ滞が示唆される場合のみ緊急ERCPを施行することが推奨される。 第一人者の医師による解説 胆管炎合併の有無の判断に迷う患者も多く 患者に応じた方針決定が必要 三箇 克幸 横浜市立大学医学部消化器内科学/前田 愼 横浜市立大学医学部消化器内科学主任教授 MMJ. February 2021;17(1):23 胆石性膵炎は急性膵炎の最も一般的な成因であり、胆管炎、臓器不全、およびその他の生命を脅かす合併症を発症することがあるため、治療戦略は重要である。ガイドラインによると、胆管炎を伴う胆石性膵炎では、緊急内視鏡的逆行性胆管膵管造影(ERCP)が推奨されており、胆汁うっ滞を合併する胆石性膵炎でも緊急 ERCPが有用である可能性が示唆されている(1)。一方で、胆管炎や顕著な胆汁うっ滞を合併しない胆石性膵炎の症例に対する、緊急ERCPは保存的治療よりも有用か否かは証明されていないのが現状である。 本論文は、胆管炎を合併しない重篤な経過が予測される胆石性膵炎に対して、乳頭括約筋切開術を伴う緊急 ERCPが保存的治療に比べ有用か否かを評価することを目的に、オランダの26施設で施行された無作為化対照試験(APEC試験)の報告である。APEC試験では胆管炎を合併しない重篤な経過が予測される胆石性膵炎患者を、診断後24時間以内および発症後72時間以内に緊急 ERCPを施行した群(117人)と保存的治療群(113人)に無作為化し評価した。主要評価項目は観察期間の6カ月間における死亡と合併症の複合エンドポイントであった。合併症は、持続する臓器不全、胆管炎、膵実質壊死、菌血症、肺炎、膵内分泌または外分泌機能不全と定義された。 その結果、主要評価項目である死亡・合併症の発生率は、緊急 ERCP群では38%(45/117)、保存的治療群では44%(50/113)であり、統計学的有意差は示されなかった。合併症の多くは両群間で発生率に統計学的有意差は認めなかったが、胆管炎の発生率のみ、保存的治療群の10%(11/113)と比較し、緊急 ERCP群で2%(2/117)と有意に低下した。 今回の研究結果から、重篤な経過が予測される急性胆石性膵炎において、胆管炎を合併する患者においてのみ、緊急ERCPを施行し、それ以外は保存的治療が推奨される。 本研究の限界は、胆石性膵炎自体にも発熱を伴うことがあるため、胆管炎を合併しているか否かの診断が困難な患者も多く存在する点である。また総胆管結石の正診率は超音波内視鏡検査が、生化学検査や放射線検査よりも高いとされている(2)。しかし、超音波内視鏡検査の汎用性の問題から実臨床では生化学検査や放射線検査を用いて評価することが多く、本研究でも同様である。これらの影響に関して、本研究では明らかでないため、今後のさらなる検討が必要である。 1.Tenner S, et al. Am J Gastroenterol.2013 Sep;108(9):1400-1415. 2.Giljaca V,et al.Cochrane Database Syst Rev.2015 Feb 26;2015(2):CD011549.
物理的距離の保持、マスクおよび保護めがね着用によるヒトからヒトへのSARS-CoV-2感染およびCOVID-19の予防 系統的レビューおよびメタ解析
物理的距離の保持、マスクおよび保護めがね着用によるヒトからヒトへのSARS-CoV-2感染およびCOVID-19の予防 系統的レビューおよびメタ解析
Physical distancing, face masks, and eye protection to prevent person-to-person transmission of SARS-CoV-2 and COVID-19: a systematic review and meta-analysis Lancet. 2020 Jun 27;395(10242):1973-1987. doi: 10.1016/S0140-6736(20)31142-9. Epub 2020 Jun 1. 原文をBibgraph(ビブグラフ)で読む 上記論文の日本語要約 【背景】重症急性呼吸器症候群コロナウイルス2(SARS-CoV-2)はCOVID-19の原因であり、密接な接触を通してヒトとヒトとの間で広がる。著者らは、医療現場および非医療現場(地域など)での物理的距離の保持、マスク着用および眼の保護がウイルス伝播にもたらす効果を調べることを目的とした。 【方法】ヒトからヒトへの感染を避けるための最適な距離を調べ、マスクおよび保護めがね着用によるウイルス伝播予防効果を評価するため、系統的レビューおよびメタ解析を実施した。WHOおよびCOVID-19関連の標準的データベース21件から、SARS-CoV-2、重症急性呼吸器症候群を引き起こすベータコロナウイルス、中東呼吸器症候群のデータを取得した。データベース開始から2020年5月3日までのデータを検索し、言語は問わないが、比較試験や、妥当性、実行可能性、資源の利用および公平性に関する因子を特定した。記録をふるいにかけデータを抽出し、二重にバイアスリスクを評価した。頻度論的統計、ベイズ・メタ解析、ランダム効果メタ回帰解析を実施した。Cochrane法とGRADEアプローチに従って、根拠の確実性を評価した。この試験は、PROSPEROにCRD42020177047番で登録されている。 【結果】16カ国6大陸の観察的研究172件を特定した。無作為化試験はなく、44件が医療現場および非医療現場で実施した比較試験(対象計2万5697例)だった。1m未満の物理的距離と比べると、1m以上の距離でウイルスの伝播率が低く(1万736例、統合調整オッズ比[aOR]0.18、95%CI 0.09-0.38、リスク差[RD]-10.2%、95%CI -11.5--7.5、中等度の確実性)、距離が長くなるほど保護効果が高くなった(1m当たりの相対リスク[RR]の変化2.02、交互作用のP=0.041、中等度の確実性)。マスクの着用で感染リスクが大幅に低下し(2697例、aOR 0.15、95%CI 0·07-0.34、RD -14.3%、-15.9--10.7、低度の確実性)、使い捨てサージカルマスクや同等のマスク(再利用可能な12-16層の綿マスクなど)と比べるとN95や同等のマスクの方が関連が強かった(交互作用のP=0.090、事後確率95%未満、低度の確実性)。このほか、保護めがねでも感染率が低下した(3713例、aOR 0.22、95%CI 0.12-0.39、RD -10.6%、95%CI -12.5--7.7、低度の確実性)。未調整の試験、下位集団解析や感度解析からほぼ同じ結果が得られた。 【解釈】この系統的レビューおよびメタ解析の結果は、1m以上の物理的距離を支持し、政策決定者にモデルおよび接触者追跡の定量的推定を提供するものである。この結果および関連因子から、公共の場および医療現場でのマスク、医療用レスピレーターおよび保護めがねの最適な着用を広めるべきである。この介入の根拠を示す頑強な無作為化試験が必要であるが、現在の入手可能な科学的根拠の系統的評価は中間的なガイダンスになると思われる。 第一人者の医師による解説 44試験のメタ解析の結果 有用なエビデンス さらに今後の研究の蓄積が必要 荒岡 秀樹 虎の門病院臨床感染症科部長 MMJ. February 2021;17(1):14 新型コロナウイルス(SARS-CoV-2)の主な感染様式は飛沫感染と接触感染であり、特殊な状況下(例、換気の悪い空間)では一部空気感染を示唆する報告もある。なかでも飛沫感染をどのように防止するかが感染予防の最優先事項と考えられている。本論文は、コロナウイルス(SARS-CoV-2、SARS、MERS)における感染予防法(身体的距離[physical distancing]、マスク、眼の保護)の効果を調べた44試験(患者25,697人)のメタ解析の報告である。 解析の結果、1m以上の身体的距離は、1m未満の身体的距離と比較して、ウイルス伝播を低下させることが証明された(補正オッズ比[aOR], 0.18;95%信頼区間[CI], 0.09~0.38;中程度の確実性)。この予防効果は距離が離れるほど強くなることも明らかとなった(中程度の確実性)。また、マスクの着用は感染のリスクを大幅に低下させた(aOR, 0.15;95% CI, 0.07~0.34;低い確実性)。マスクの感染リスク低下効果は市中よりも医療現場でより大きいことが示唆された。N95マスクはサージカルマスクと比較して予防効果が大きいことが示された。さらに、眼の保護も感染の減少と関連していた(aOR, 0.22;95% CI, 0.12~0.39;低い確実性)。 本研究は世界保健機関(WHO)の支援を受けて実施され、完全な系統的レビューの方法を遵守していることが強みである。また、GRADEアプローチに従って、証拠の確実性を評価している。2020年5月3日までの関連データを検索し、新型コロナウイルスのパンデミックが他の世界的な地域に広がる前に中国からのデータを含めて評価している。一方、本研究の限界として、対象の研究はいずれもランダム化されていないこと、多くの研究で身体的距離に関する正確な情報が提供されておらず推測が含まれること、ほとんどの研究がSARSとMERSについて報告したものであること、などが挙げられる。 今回のメタ解析の結果は、SARS-CoV-2に対する感染予防のランダム化試験がない状況下で、非常に有益な情報を私たちに与えるものである。1m以上の身体的距離が重要で、2m以上の距離はより有用な可能性がある。さらに、医療現場および市中でのマスク着用、眼の保護の重要性を明らかにした。本論文は、2020年5月3日までのエビデンスをまとめ、大変有用なものであるが、これ以降のSARS-CoV-2に特化した感染予防のエビデンスにも注目していきたい。
サイトメガロウイルス感染妊婦の胎児への垂直感染予防に用いるバラシクロビル 無作為化二重盲検プラセボ対照試験
サイトメガロウイルス感染妊婦の胎児への垂直感染予防に用いるバラシクロビル 無作為化二重盲検プラセボ対照試験
Valaciclovir to prevent vertical transmission of cytomegalovirus after maternal primary infection during pregnancy: a randomised, double-blind, placebo-controlled trial Lancet. 2020 Sep 12;396(10253):779-785. doi: 10.1016/S0140-6736(20)31868-7. 原文をBibgraph(ビブグラフ)で読む 上記論文の日本語要約 【背景】サイトメガロウイルスはよく見られる先天性感染であり、妊娠早期の初回感染の後の死亡率が高い。胎児への垂直感染を予防するのに有効な手段はない。著者らは、妊娠早期にサイトメガロウイルスに感染した妊婦から胎児への垂直感染がバラシクロビルで予防できるかを調査することを試みた。 【方法】この前向き二重盲検プラセボ対照無作為化試験は、Infectious Feto-Maternal Clinic of Rabin Medical Center(イスラエル・ペタフ ティクヴァ)で実施された。妊娠初期または第1三半期中のサイトメガロウイルス初回感染が血清学的検査で確定した18歳以上の妊婦を経口バラシクロビル(1日8g、1日2回)またはプラセボに無作為に割り付け、登録時から21週または22週時に羊水穿刺検査実施まで投与した。妊娠初期と第1三半期中に感染した被験者を別々に無作為化し、4つのブロックで無作為化した。試験期間中、被験者と研究者に治療の割り付けを伏せた。主要評価項目は、サイトメガロウイルスの垂直感染率とした。per-protocol原理にしたがって統計学的解析を実施した。この試験は、ClinicalTrials.govにNCT02351102番で登録されている。 【結果】2015年11月15日から2018年10月8日にかけて100例を無作為化し、バラシクロビルまたはプラセボを投与した。10例を除外し(各群5例ずつ)、最終解析にはバラシクロビル群45例(いずれも単胎妊娠)、プラセボ群45例(単胎妊娠43例、2例が双胎妊娠)を対象とした。バラシクロビル群は第1三半期中および妊娠初期の感染例を含み、羊水穿刺45件中5件(11%)がサイトメガロウイルス陽性、プラセボ群では羊水穿刺47件中14件(30%)が陽性であった(P=0.027、垂直感染のオッズ比0.29、95%CI 0.09-0.90)。第1三半期中のサイトメガロウイルス初回感染例で、バラシクロビル群(羊水穿刺19件中2件[11%])の方がプラセボ群(羊水穿刺23件中11件[48%])よりも羊水穿刺でのサイトメガロウイルス陽性が有意に少なかった(P=0.020)。臨床的に有意な有害事象は報告されなかった。 【解釈】バラシクロビルは、妊娠早期にサイトメガロウイルスに感染した妊婦のから胎児への垂直感染率を抑制するのに有効である。初回感染妊婦の早期治療によって、妊娠喪失や先天性サイトメガロウイルス感染乳児の出産を防ぐことができると思われる。 第一人者の医師による解説 妊娠初期のCMVスクリーニング検査と バラシクロビル投与の広まりに期待 田中 守 慶應義塾大学医学部産婦人科教授 MMJ. February 2021;17(1):28 先天性サイトメガロウイルス感染症は、世界で最も多い母児感染の原因の1つであり、器官形成期である妊娠初期に感染すると児に重篤な神経学的障害を残す疾患である。有効なワクチンもないため、初回感染妊婦のスクリーニング法と治療法の確立が求められてきた(1)。出生後の先天性サイトメガロウイルス感染症に使用されているアシクロビルやバラシクロビルは、米国食品医薬品局(FDA)分類でカテゴリーBに分類され、妊娠中の安全性が認められている。また、母体へ投与されたバラシクロビルは、胎盤を通過し、母体、胎児、羊水中に十分な治療濃度が確保されると報告されている(2)。 本論文は、妊娠初期に母体サイトメガロウイルス初回感染が確認された妊婦に対してバラシクロビルの有効性と安全性を検討した無作為化二重盲検プラセボ対照試験のイスラエルからの報告である。100人のサイトメガロウイルス初回感染妊婦に対して、1日8gのバラシクロビル群とプラセボ群の2群に分け、治療効果を判定する羊水 PCR検査まで最低6週間の薬剤服用を行った。最終的に45人のバラシクロビル群と45人のプラセボ群の結果が解析された。妊娠中期に実施された胎児感染を調べるための羊水 PCR検査において、バラシクロビル群では45検査中5例(11%)がサイトメガロウイルス陽性となり、プラセボ群の44検査中14例(30%)に比較して有意に低値となった。特に妊娠初期に判明した母体サイトメガロウイルス初回感染例に対しては、顕著な胎児垂直感染予防効果が認められた(羊水PCR検査陽性率:バラシクロビル群11%、対するプラセボ群48%)。一方、バラシクロビル群に明らかな有害事象は認められなかった。 日本では、イスラエルほど妊娠初期のサイトメガロウイルス初回感染スクリーニング検査が普及していない現状であり、いまだ多くの先天性サイトメガロウイルス感染症の子どもが生まれてきている。妊娠初期にスクリーニング検査を行い、陽性者にはバラシクロビルを投与することで垂直感染が予防できる可能性が示されたことは意義深い。今後、日本での妊婦に対するサイトメガロウイルス感染症スクリーニング検査とバラシクロビル投与による垂直感染予防策が広まっていくことが望まれる。 1. Tanimura K, et al. J Obstet Gynaecol Res. 2019 45(3):514-521. 2. Jacquemard, F, et al. BJOG 2007 114(9): 1113-1121.
根治的前立腺全摘除術後の放射線治療のタイミング(RADICALS-RT) 第III相無作為化比較試験
根治的前立腺全摘除術後の放射線治療のタイミング(RADICALS-RT) 第III相無作為化比較試験
Timing of radiotherapy after radical prostatectomy (RADICALS-RT): a randomised, controlled phase 3 trial Lancet. 2020 Oct 31;396(10260):1413-1421. doi: 10.1016/S0140-6736(20)31553-1. Epub 2020 Sep 28. 原文をBibgraph(ビブグラフ)で読む 上記論文の日本語要約 【背景】前立腺がんの根治的前立腺全摘除術後の放射線治療の最適なタイミングは明らかになっていない。著者らは、前立腺特異的抗原(PSA)生化学的再発時の救済放射線療法と併用する経過観察と比較した補助放射線療法の有効性と安全性を比較すること。 【方法】根治的前立腺全摘除術後に生化学的進行が見られる1項目以上の危険因子(病理学的T分類3または4、グリーソンスコア7-10点、断端陽性、術前PSAが10ng/mL以上のいずれか)がある患者を組み入れた無作為化比較試験を実施した(RADICALS-RT試験)。試験は、試験実施の認可を受けたカナダ、デンマーク、アイルランドおよび英国の施設で実施した。患者を補助放射線療法とPSAで判定した再発(PSA 0.1ng/mL以上または連続3回以上で上昇)に応じて救済放射線療法を用いる経過観察に1対1の割合で無作為に割り付けた。盲検化は実効不可能と判断した。グリーソンスコア、切除断端、予定していた放射線スケジュール(52.5Gy/20分割または66Gy/33分割)および施設を層別化因子とした。主要評価項目は無遠隔転移生存期間に規定し、救済放射線療法(対照)による90%の改善から補助放射線療法による10年時の95%の改善を検出するデザイン(検出力80%)とした。生化学的無増悪生存期間、プロトコールにないホルモン療法非実施期間および患者方向転帰を報告する。標準的な生存解析法を用いた。ハザード比(HR)1未満を補助放射線療法良好とした。この試験は、ClinicalTrials.govにNCT00541047で登録されている。 【結果】2007年11月22日から2016年12月30日の間に、1396例を無作為化し、699例(50%)を救済放射線療法群、697例(50%)を補助放射線療法群に割り付けた。割り付け群は年齢中央値65歳(IQR 60-68)で釣り合いがとれていた。追跡期間中央値4.9年(IQR 3.0-6.1)であった。補助放射線療法群に割り付けた697例中649例(93%)が6カ月以内、救済放射線療法群に割り付けた699例中228例(33%)が8カ月以内に放射線療法を実施したことを報告した。イベント169件で、5年生化学的無増悪生存率が補助放射線療法群で85%、救済放射線療法群で88%であった(HR 1.10、95%CI 0.81-1.49、P=0.56)。5年時のプロトコールにないホルモン療法非実施期間が補助放射線療法群で93%、救済放射線療法群で92%であった(HR 0.88、95%CI 0.58-1.33、P=0.53)。1年時の自己報告の尿失禁は補助放射線療法群の方が不良であった(平均スコア4.8 vs. 4.0、P=0.0023)。が補助放射線療法群の6%、救済放射線療法群の4%に2年以内にグレード3-4の尿道狭窄が報告された(P=0.020)。 【解釈】この初期結果は、根治的前立腺全摘除術後の補助放射線療法のルーチンの実施を支持するものではない。補助放射線療法によって泌尿器合併症リスクが上昇する。PSA生化学的再発時に救済放射線療法を実施する経過観察を根治的前立腺全摘除術後の現行の標準治療とすべきである。 第一人者の医師による解説 適切な救済放射線治療により 補助放射線治療とPSA制御に差はない 伊丹 純 元国立がん研究センター中央病院放射線治療科科長 MMJ. April 2021;17(2):54 前立腺全摘術は前立腺がんに対する根治療法の1つであるが、高リスク患者では半分程度に前立腺特異抗原(PSA)再発が見られる。切除断端陽性、前立腺被膜外浸潤陽性、精嚢浸潤陽性、Gleason score8以上などの再発高リスク患者には、手術に引き続き補助放射線治療が実施されることがある。それに対して、術後は経過観察とし、PSA再発をきたした場合にのみ救済放射線治療を実施する方が、放射線治療の対象を限定することができ、長期成績は補助放射線治療と変わらないとするものもある。 今回報告されたRADICALS-RT試験は術後の補助放射線治療群と経過観察群を比較した無作為化第3相試験であり、対象は再発危険因子としてpT3/pT4、Gleason score 7~10、断端陽性、治療前PSA 10ng/mL以上のいずれか1個以上を持つ前立腺全摘術の前立腺がん患者で、通常の術後照射の対象より再発リスクの低い患者も含まれる。無作為割り付け後、補助放射線治療群は2カ月以内に前立腺床に対する放射線治療を開始し、経過観察群はPSAが2回続けて0.1ng/mL以上に上昇した場合、2カ月以内に救済放射線治療を開始した。救済放射線治療はPSA 0.2ng/mL以下でより有効であることが示されており当試験の重要なポイントである。補助放射線治療、救済放射線治療ともに前立腺床±骨盤リンパ節に66Gy/33分割、または52.5Gy/20分割(約62Gy/31分割相当)の照射が実施された。2007年11月~16年12月に英連邦諸国およびデンマークから1,396人が登録され、追跡期間中央値は4.9年。無作為割り付け後5年で経過観察群のうち32%の患者で救済放射線治療が開始されていた。5年PSAの無増悪生存率は 補助放射線治療群で85%、経過観察群88%で有意差はなかった。しかし、泌尿器症状、消化器症状などは2年以内の早期およびそれ以降の晩期ともに経過観察群で有意に少なかった。 今回の試験と同時期にLancet Oncologyに同様な2件の第3相試験(1),(2)が報告され、それらを併せた3試験のメタアナリシス(3)も発表された。いずれの報告でもPSA値が0.2ng/mL程度の段階で救済療法が実施されれば経過観察群はPSA無増悪生存率で補助放射線治療群と差はないという結果であった。術後照射を必要とする高リスク群も抽出できなかった。これら3件の第3相試験とそのメタアナリシスを踏まえると、前立腺全摘術後の補助放射線治療はルーティンで実施されるべきではなく、救済療法はPSAが0.2ng/mL程度の段階で早期に開始すべきである。また、救済放射線治療の際にはホルモン療法の同時併用も考慮されるべきである。 1. Kneebone A, et al. Lancet Oncol. 2020;21(10):1331-1340. 2. Sargos P, et al. Lancet Oncol. 2020;21(10):1341-1352. 3. Vale CL, et al. Lancet. 2020;396(10260):1422-1431.
原発性自然気胸の外来治療:非盲検無作為化対照試験
原発性自然気胸の外来治療:非盲検無作為化対照試験
Ambulatory management of primary spontaneous pneumothorax: an open-label, randomised controlled trial Lancet. 2020 Jul 4;396(10243):39-49. doi: 10.1016/S0140-6736(20)31043-6. 原文をBibgraph(ビブグラフ)で読む 上記論文の日本語要約【背景】原発性自然気胸は健康な若年患者にも起こる。最適な管理法は定義されておらず、入院期間が長引くことが多い。外来治療の有効性に関するデータは少ない。著者らは、外来治療による入院期間および安全性を標準治療と比較することを目的とした。【方法】この非盲検無作為化試験では、英国の病院24施設で3年間にわたり、症候性の原発性自然気胸成人患者(16~55歳)を登録した。患者を外来用デバイスと標準ガイドラインに従った管理(吸引および標準的な胸腔チューブ挿入、またはそのいずれか)に無作為化により(1対1の割合で)割り付けた。主要評価項目は、無作為化後最長30日間の再入院を含めた入院期間とした。主解析はデータが入手できた患者を対象とし、安全性解析は割り付けた全患者を対象とした。この試験はInternational Standard Randomised Clinical Trialsの番号ISRCTN79151659で前向きに登録されている。【結果】2015年7月から2019年5月までの間にスクリーニングした776例のうち236例(30%)を外来治療(117例)、標準治療(119例)に無作為化により割り付けた。30日時点で、入院期間中央値は、外来治療を受けデータが入手できた114例(0日[IQR 0~3])の方が標準治療を受けデータを入手できた113例(4日[IQR 0~8])よりも短かった(P<0.0001;差の中央値2日[95%CI 1~3])。236例中110例(47%)に有害事象が発現し、内訳は外来治療群117例中64例(55%)、標準治療群119例中46例(39%)であった。外来治療を受けた患者に重篤な有害事象が計14件発現し、このうち8件(54%)が気胸の増大、無症候性の肺浮腫およびデバイスの不具合、漏出、ずれなどの介入によるものであった。【解釈】原発性自然気胸の外来治療によって最初の30日間の再入院を含め入院期間が有意に減少したが、有害事象が増えた。このデータからは、原発性自然気胸は、介入が必要な患者に外来用デバイスを用いることにより、外来での治療が可能であることを示唆している。 第一人者の医師による解説 患者自身の自己管理のため 合併症や事故のリスクが大きいことに留意 栗原 正利 公益財団法人日産厚生会玉川病院 気胸研究センター 気胸研究センター長 MMJ. December 2021;17(6):175 原発性自然気胸患者の外来通院ドレナージ治療は次第に普及しつつあるがその評価はまだ明確でない。本論文は、原発性自然気胸に対する外来通院治療の安全性と入院期間短縮効果を入院標準治療と比較した、英国の多施設共同ランダム化非盲検対照試験(RandomisedAmbulatoryManagementofPrimaryPneumothorax;RAMPP)の報告である。入院標準治療は英国胸部疾患学会(BTS)ガイドラインに準じた初期治療を意味し、胸部X線写真において肺門のレベルで胸壁から肺の輪郭までの距離が2cm以上の患者および/または呼吸器症状のある患者に対して、1回吸引治療または胸腔ドレナージ治療を行うものである。それに対して外来通院治療では携帯型ドレナージキットとしてハイムリッヒバルブとボトルがついたRocketPleuralVent(RocketMedical社、英国)が用いられた。RAMPP試験には英国の病院24施設が参加し、携帯型ドレナージキットによる外来通院治療群(117人)と入院標準治療群(119人)の間で有用性が比較検討された。主要評価項目として30日までの入院期間、副次評価項目として追加治療の必要性、有害事象、痛み・息切れの評価、再発率、欠勤期日などが検討された。その結果、外来通院治療群では、入院標準治療群に比べ、入院期間は有意に短縮し、再発率(7日目:7%対19%)は低く、手術紹介の頻度は同程度であった(28%対22%)。著者らは、ドレーンにまつわる有害事象はやや認められるものの、携帯型の胸腔ドレナージキットによる外来通院治療は推奨できると結論づけている。多施設共同ランダム化試験で、外来通院ドレナージ治療の有用性を科学的に検討した試みは評価したい。一方で、外来通院ドレナージ治療は、患者自身の自己管理になる。したがって合併症や事故のリスクは入院治療の場合よりはるかに大きい。ドレナージ装置取り扱いの十分な説明と問題が生じた時には24時間病院に連絡できる体制を取らなければならない。外来通院治療はリスクを伴うことを肝に銘じておくことが重要である。今回の外来通院治療は英国BTSのガイドラインに基づいて行われているが、米国には米国胸部疾患学会(ACCP)によるガイドライン(1)があり、日本には独自のガイドラインがある。各国の医療制度の違いにより診断および治療方針が異なるので、今後は各国で共有できる情報の整理を行っていく方向性が重要と考えられる。外来通院ドレナージ治療は、その携帯装置が日本でいち早く開発され世界に先駆けて導入されている(2)。日本において携帯型ドレナージキット治療が一番進んでいるのが実情である。 1. Baumann MH, et al. Chest. 2001;119(2):590-602. 2. 江花弘基 , et al. 日救急医会誌(JJAAM). 2011; 22: 803-809.
1型糖尿病成人患者の持続血糖モニタリングに用いるリアルタイムスキャンと間歇スキャンの比較(ALERTT1):6カ月間の前向き多施設共同無作為化対照試験
1型糖尿病成人患者の持続血糖モニタリングに用いるリアルタイムスキャンと間歇スキャンの比較(ALERTT1):6カ月間の前向き多施設共同無作為化対照試験
Comparing real-time and intermittently scanned continuous glucose monitoring in adults with type 1 diabetes (ALERTT1): a 6-month, prospective, multicentre, randomised controlled trial Lancet. 2021 Jun 12;397(10291):2275-2283. doi: 10.1016/S0140-6736(21)00789-3. Epub 2021 Jun 2. 原文をBibgraph(ビブグラフ)で読む 上記論文の日本語要約【背景】1型糖尿病患者は、間歇スキャン持続血糖モニタリング(isCGM)またはリアルタイム持続血糖モニタリング(rtCGM)により血糖値を持続的に測定することができる。しかし、isCGMからアラート機能付きrtCGMに変更することによる便益があるかどうかは不明である。そこで今回、1型糖尿病成人患者を対象に、rtCGMとisCGMを比較する試験を実施した(ALERTT1)。【方法】ベルギー国内6施設で、前向き2群並行群間多施設無作為化比較試験を実施した。これまでisCGMを使用していた1型糖尿病成人患者をrtCGM(介入)とisCGM(対照)に1対1の割合で無作為化により割り付けた。中央機関で無作為化を実施し、試験施設、年齢、性別、糖化ヘモグロビン(HbA1c)、time in range(3.9~10.0mmol/L)、インスリン投与方法、低血糖の認識に基づく最小化法を用いた。参加者、治験責任医師、試験チームに治療の割り付けを伏せなかった。主要評価項目は、6カ月後のtime in rangeの平均群間差とし、intention-to-treatで評価した。本試験は、ClinicalTrials.govにNCT0377260で登録されている。【結果】2019年1月29日から7月30日までの間に参加者269例を登録し、そのうち254例を無作為化によりrtCGM群(127例)およびisCGM群(127例)に割り付けた。それぞれ124例および122例が試験を完了した。6カ月後、rtCGMの方がisCGMよりもtime in rangeが高かった(59.6% vs. 51.9%;平均差6.85ポイント[95%CI 4.36~9.34];P<0.0001)。(rtCGMの方が)6カ月後のHbA1c(7.1% vs. 7.4%;P<0.0001)、3.0mmol/L未満の時間の割合(0.47% vs. 0.84%;P=0.0070)およびHypoglycaemia Fear Survey(低血糖恐怖感調査:HFS)IIの心配下位尺度スコア(15.4 vs. 18.0;P=0.0071)が低かった。rtCGM群の方が重度の低血糖を来した参加者が少なかった(3例 vs. 13例;P=0.0082)。isCGMの方が皮膚反応が多く、rtCGM使用者の方がセンサー挿入後の出血が多かった。【解釈】任意に抽出した1型糖尿病成人患者で、isCGMからrtCGMへの切り替えにより、治療開始6カ月後のtime in rangeが有意に改善した。1型糖尿病患者の健康と生活の質を改善するため、isCGMに代わってrtCGMを検討すべきであることが示唆される。 第一人者の医師による解説 目標のTIRは患者ごとに異なり いずれのCGMが適切かは主治医の判断による 浦上 達彦 日本大学小児科学系小児科学分野診療教授 MMJ. December 2021;17(6):177 1型糖尿病の患者は、間歇スキャン持続的血糖モニタリング(isCGM)あるいはリアルタイムCGM(rtCGM)を用いることにより持続的に皮下のグルコース値をモニターすることが可能になったが、isCGMからアラート機能を有するrtCGMに切り替えた際の有用性については明らかでない。本論文は、成人1型糖尿病患者を対象にisCGMとrtCGMを比較する目的でベルギーの6病院で実施された、前向き、二重盲検、並行群間比較、多施設共同無作為化対照試験(ALERTT1)の報告である。以前にisCGMを使用していた成人1型糖尿病患者254人を、rtCGM(介入)群127人とisCGM(対照)群127人に無作為に1:1の比で割り付け、そのうちそれぞれ124人と122人が試験を終了した。試験開始後6カ月の時点で、主要評価項目であるtimeinrange(TIR;グルコース値が70~180mg/dLの範囲内にある時間の割合)はisCGM群に比べrtCGM群の方が有意に高く(51.9%対59.6%;平均差6.85%;95%信頼区間[CI],4.36?9.34;P<0.0001)、HbA1c値もisCGM群に比べrtCGM群の方が有意に低かった(7.4%対7.1%;P本試験では、成人1型糖尿病患者において、血糖コントロールの改善と低血糖頻度の低下に関して、isCGMに対するrtCGMの優越性が示された。rtCGMでは連続したグルコーストレンドが表示されることにより、高血糖時の補正インスリンの投与あるいは低血糖時の補食摂取などが的確に行えることと、特に無自覚性低血糖や重症低血糖の既往のある患者に対する低血糖アラート機能が低血糖、重症低血糖の発生を抑制するのに有効であることが、今回の結果につながったものと思われる。一方、TIRは推奨される70%以上の達成を単に目指すのではなく、低血糖の発生を最小限に抑え、患者の生活の質(QOL)が改善することを最大の目標としており(1)、目標とすべきTIRは患者ごとに個別化されるべきである(2),(3)。したがって、厳格な血糖目標を達成しやすいrtCGMに無理やり切り替えるのではなく、患者の要求度と治療・管理の内容によって、操作が簡便で利便性に優れるisCGMを使用するのか、低血糖の減少と血糖コントロールの改善に優れるrtCGMを使用するかを患者ごとに主治医が判断する必要がある。 1. Battelino T, et al. Diabetes Care. 2019;42(8):1593-1603.2. Urakami T, et al. Endocr J. 2020;67(10):1055-1062.3. Urakami T, et al. Horm Res Paediatr. 2020;93(4):251-257
検診規模拡大前後にみられる2型糖尿病の心血管リスク予測:導出および検証研究
検診規模拡大前後にみられる2型糖尿病の心血管リスク予測:導出および検証研究
Cardiovascular risk prediction in type 2 diabetes before and after widespread screening: a derivation and validation study Lancet. 2021 Jun 12;397(10291):2264-2274. doi: 10.1016/S0140-6736(21)00572-9. Epub 2021 Jun 2. 原文をBibgraph(ビブグラフ)で読む 上記論文の日本語要約【背景】近年まで、世界のほとんどの糖尿病患者は症状が発現してから診断を受け、心血管リスクが高い。これは、ほとんどの患者が心血管予防薬を処方されていることを意味する。しかしニュージーランドでは、世界初の国家プログラムにより、2016年までに、対象となる成人の90%が糖尿病検診を受け(2012年の50%から上昇)、糖尿病を発症して間もない無症候性の患者が多く発見されている。著者らは、検診が広く実施される前に導出した心血管リスク予測式が現在検診で発見される患者のリスクをいくぶん過剰に見積もっているという仮説を立てた。【方法】検診規模拡大前後の期間を含む2004年10月27日から2016年12月30日までを検討した40万人規模のPREDICTプライマリケアコホート研究から、2型糖尿病があり、既知の心血管疾患、心不全および重大な腎機能障害がない30~74歳のニュージーランド人を特定した。糖尿病関連および腎機能の指標など事前に定めた18の予測因子とともにCox回帰モデルを用いて心血管疾患の5年リスクを推定する性別の式を導出した。検診が広く実施される前の2000~2006年に募集したNew Zealand Diabetes Cohort Study(NZDCS)で導出した同等の式と性能を比較した。【結果】4万6,652例をPREDICT-1 Diabetesサブコホートとし、このうち4,114例に追跡期間中(中央値5.2年、IQR 3.3~7.4)初発の心血管事象が発現した。ベースラインで1万4,829例(31.8%)が経口血糖降下薬およびインスリンを使用していなかった。新計算式で推定した心血管5年リスクの中央値は、女性が4.0%(IQR 2.3~6.8)、男性が7.1%(4.5~11.2)であった。古いNZDCSの式では、女性で3倍(中央値14.2%[9.7~20.0])、男性で2倍(中央値17.1%[4.5~20.0])、心血管リスクの中央値が過剰に推定された。PREDICT-1 Diabetesの式は、モデルおよび判別性能の指標もNZDCSの式よりも有意に優れていた(例、女性:R2=32%[95%CI 29~34]、HarrellのC=0.73[0.72~0.74]、RoystonのD=1.410[1.330~1.490]vs. R2=24%[21~26]、C=0.69[0.67~0.70]、D=1.147[1.107~1.187])。【解釈】国際的な治療ガイドラインではいまだに、ほとんどの糖尿病患者の血管リスクが高いと考えている。しかし、今回、ニュージーランドでは近年の糖尿病検診規模拡大によって糖尿病患者の心血管リスクが根本的に変化していることが示された。患者の多くは腎機能が正常であり、血糖降下薬を投与されておらず、心血管リスクが低い。肥満の増加、検診に用いる検査の簡易化および心血管事象を予防する新世代血糖降下薬の導入によって糖尿病検診の増加が必然となるに伴い、この結果の国際的な意味が明らかになる。現代の糖尿病集団を対象に、糖尿病関連および腎機能の予測因子を多数用いて導出した心血管リスク予測式により、不均一性を増しつつある集団から低リスク患者と高リスク患者を判別し、しかるべき非薬物療法管理や費用効果を踏まえた高価な新薬の対象者に関する情報を提供することが求められている。 第一人者の医師による解説 日本でもリスク変化の可能性 他国・以前のリスク予測式の適用には留意が必要 杉山 雄大 国立国際医療研究センター研究所糖尿病情報センター・医療政策研究室長 MMJ. December 2021;17(6):178 従来の糖尿病患者を対象とした心血管リスクスコアは、有症状で心血管リスクがすでに高い段階で糖尿病が発見される社会における糖尿病患者から導出されてきた。一方で、健診機会が広がり無症状でも糖尿病の診断がされるようになると、全体として糖尿病患者における心血管リスクが下がり、従来のリスクスコアではリスクの過大評価となっている可能性がある。本研究では、政策により糖尿病健診受診割合が急拡大(2012年50%→16年90%)したニュージーランドにおいて、PREDICT研究の部分コホートにおける新規心血管疾患の発症率を算出、新たな予測式を導出(derivation)・検証(validation)し、さらに2002~06年のコホートデータ(NewZealandDiabetesCohortStudy;NZDCS)から導出された2型糖尿病患者に対するリスク予測式をPREDICT研究のコホートを用いて検証することにより、上記の仮説を検討した。PREDICT研究では、ニュージーランドのプライマリケア医が心血管リスク評価を行うための標準化されたソフトウエア(PREDICT意思決定支援ソフトウエア)に情報を入れた際に患者がコホートに登録される。本研究は、PREDICT研究のコホートのうち、2004?16年に登録された心血管疾患既往のない46,652人の2型糖尿病患者を対象とした。結果として、244,840人・年において4,114件の心血管疾患が発症した(中央値5.2年;4分位範囲3.3~7.4年)。PREDICT研究、NZDCSそれぞれのリスク予測式から計算された心血管疾患の予測5年累積発症率と実際の発症率を10分位の較正プロットで比較すると、PREDICT研究からのリスク予測は観測値とほぼ一致するのに対し、NZDCSの予測式はリスクを大幅に過大評価する結果となった(女性:約3倍、男性:約2倍)。本研究は、糖尿病に対する健診が発達していない社会・時代から導出されたリスクスコア(例:フラミンガムリスクスコア)を健診の発達した社会・時代に適用すると、糖尿病患者に対するリスクを過大評価してしまうことを示唆している。また、投薬などの推奨(例:40歳以上の糖尿病患者全体に対するスタチン推奨)もリスク評価に基づくものであり、リスクの異質性を認識し、正確なリスク評価に基づいて費用対効果の高い診療をする方が望ましい、と著者らは論じている。日本でも特定健診の導入などにより未治療糖尿病患者の割合は低下しており(1)、糖尿病患者のリスクに変化が生じている可能性がある。他国・以前のリスク予測式を適用する際には、リスク予測式導出時との違い・変化に留意する必要がある。 1. 厚生労働省 . 平成 28 年国民健康・栄養調査結果の概要 . https://www.mhlw.go.jp/file/04-Houdouhappyou-10904750-KenkoukyokuGantaisakukenkouzoushinka/kekkagaiyou_7.pdf(2021 年 10 月 18 日アクセス可能)
ループス腎炎に用いるvoclosporinとプラセボの有効性および安全性の比較(AURORA 1) 多施設共同二重盲検無作為化プラセボ対照第III相試験
ループス腎炎に用いるvoclosporinとプラセボの有効性および安全性の比較(AURORA 1) 多施設共同二重盲検無作為化プラセボ対照第III相試験
Efficacy and safety of voclosporin versus placebo for lupus nephritis (AURORA 1): a double-blind, randomised, multicentre, placebo-controlled, phase 3 trial Lancet. 2021 May 29;397(10289):2070-2080. doi: 10.1016/S0140-6736(21)00578-X. Epub 2021 May 7. 原文をBibgraph(ビブグラフ)で読む 上記論文の日本語要約【背景】ループス腎炎成人患者の治療薬として承認された新たなカルシニューリン阻害薬voclosporinによって、第II相試験でループス腎炎患者の腎奏効が改善した。この試験は、ループス腎炎の治療に用いるvoclosporinの有効性と安全性を評価することを目的とした。【方法】この多施設共同、二重盲検、無作為化第III相試験は、27カ国の142施設で実施された。米国リウマチ学会の基準に基づきループス腎炎を呈する全身性エリテマトーデスと診断され、2年以内の腎生検でクラスIII、IVまたはV(単独またはクラスIII、IVとの併存)の患者を適格とした。自動ウェブ応答システムを用いて、ミコフェノール酸モフェチル(MMF、1gを1日2回)と急速に減量する低用量経口ステロイドによる基礎治療を実施した上で、患者を経口voclosporinとプラセボに(1対1の割合で)無作為化により割り付けた。主要評価項目は、52週時の腎の完全寛解とし、主要評価項目評価直前の尿蛋白/クレアチニン比0.5mg/mg未満、腎機能安定(eGFR値60mL/min/1.73m^2または治療前からの低下度20%以下と定義)、レスキュー薬非投与および44~52週に3日以上連続または7日間以上にわたるprednisone 10mg/日相当量未満の投与の複合と定義した。このほか、安全性を評価した。intention-to-treatで有効性解析、無作為化し試験治療を1回以上実施した患者で安全性解析を実施した。試験は、ClinicalTrials.govにNCT03021499で登録されている。【結果】2017年4月13日から2019年10月10日の間に、179例をvoclosporin群、178例をプラセボ群に割り付けた。主要評価項目に定めた腎の完全寛解は、voclosporin群の患者の方がプラセボ群の患者よりも多く達成した(179例中73例[41%]vs. 178例中40例[23%];オッズ比2.65;95%CI 1.64~4.27;P<0.0001)。有害事象は両群が拮抗していた。voclosporin群178例中37例(21%)とプラセボ群178例中38例(21%)に重篤な有害事象が発現した。最も多く見られた感染症などの重篤な有害事象は肺炎であり、voclosporin群の7例(4%)とプラセボ群の8例(4%)に発現した。試験期間中または試験追跡期間中に計6例が死亡した(voclosporin群1例[1%未満]とプラセボ群5例[3%])。死亡に至る事象に、試験担当医師が試験治療に関連があると考えたものはなかった。【解釈】MMF+低用量ステロイドとvoclosporinの併用は、MMF+低用量ステロイドのみよりも、臨床的にも統計的にも腎の完全寛解率が良好であり、安全性のデータも同等であった。この結果は、活動性ループス腎炎治療の重要な成果である。 第一人者の医師による解説 標準薬のMMFにボクロスポリン併用で寛解率改善 国内での保険収載を期待 廣村 桂樹 群馬大学大学院医学系研究科内科学講座腎臓・リウマチ内科学分野教授 MMJ. December 2021;17(6):180 活動性ループス腎炎の治療では、まず寛解導入療法を実施して腎炎を鎮静化し、その後維持療法により長期間の寛解維持を目指す。寛解導入療法は、中等量~大量のグルココルチコイドと免疫抑制薬の投与が基本である。2009年に報告されたALMS試験では、ミコフェノール酸モフェチル(MMF)とシクロホスファミド静注療法(IVCY)が比較され、両者がほぼ同等の寛解率を示し(1)、その結果をもとにMMFまたはIVCYが寛解導入における免疫抑制薬の標準薬となった。その後、より高い有効性を求め、標準薬に生物学的製剤などを併用する臨床試験がいろいろ試みられたが、ほとんどの試験が失敗に終わっている。そうした中、2015年に、MMFとカルシニューリン阻害薬であるタクロリムスの併用により、IVCYに比べ、寛解率が有意に高まることが中国の多施設共同試験で示され注目を集めた(2)。本論文で報告されたAURORA1試験は、シクロスポリン誘導体で新規カルシニューリン阻害薬のボクロスポリン(VCS)とMMFの併用療法の効果を検討した、日本も含めた国際的な第3相臨床試験である。ISN/RPS2003年分類III、IV、V型のループス腎炎患者357人を対象に、MMF(1回1g、1日2回)をベース薬としてVCS(1回23.7mg、1日2回)投与群とプラセボ投与群の2群に無作為に割り付け(1;1)、検討がなされた。なお本試験ではグルココルチコイド投与量がかなり少ないことが特徴である。治療開始時にメチルプレドニゾロン(0.5g/日、2日間)を投与後、プレドニゾン25mg/日(体重45kg以上の場合)から開始して漸減し、16週目以降は2.5mg/日に減量するプロトコールであり、通常投与量の半分以下となる。主要評価項目である52週後の完全腎奏効(早朝尿での尿蛋白/尿Cr比0.5mg/mgCr以下、eGFR60mL/分/1.73m2以上またはベースラインからのeGFR低下20%以下など)が得られた患者は、VCS群41%に対してプラセボ群23%であり、VCS群が有意に優れていた(オッズ比,2.65;95%信頼区間,1.64~4.27;P<00001)。一方、有害事象は両群で差がみられず、感染症関連の重篤な有害事象として肺炎が最も多くみられたが、VCS群、プラセボ群ともに4%であった。本試験の結果を受けて、2021年1月に米食品医薬品局(FDA)はループス腎炎の治療薬としてボクロスポリンを承認し、米国では販売されている。日本の大塚製薬は日本と欧州でのVCSの独占販売権を取得しており、今後国内での保険収載が期待される。 1. Appel GB, et al. J Am Soc Nephrol. 2009;20(5):1103-1112. 2. Liu Z, et al. Ann Intern Med. 2015;162(1):18-26.
糖尿病の有無を問わない成人の代謝減量手術と長期生存の関連 計174,772例を検討したマッチドコホート研究および前向き比較対照研究の1段階法メタ解析
糖尿病の有無を問わない成人の代謝減量手術と長期生存の関連 計174,772例を検討したマッチドコホート研究および前向き比較対照研究の1段階法メタ解析
Association of metabolic-bariatric surgery with long-term survival in adults with and without diabetes: a one-stage meta-analysis of matched cohort and prospective controlled studies with 174 772 participants Lancet. 2021 May 15;397(10287):1830-1841. doi: 10.1016/S0140-6736(21)00591-2. Epub 2021 May 6. 原文をBibgraph(ビブグラフ)で読む 上記論文の日本語要約【背景】代謝減量手術によって体重が大幅に減少し、肥満関連のリスクや合併症の寛解や改善につながる。しかし、政策の指針や患者カウンセリングに利用するために、術前の糖尿病併存状態で層別化した長期死亡率や平均余命にもたらす効果の推定を強固なものにする必要がある。著者らは、重度肥満患者で代謝減量手術と標準治療の長期生存転帰を比較した。【方法】前向き比較対照研究および高品質のマッチドコホート研究から再構成した患者個別の生存データを用いて、事前に定めた1段階法によるメタ解析を実施した。PubMed、ScopusおよびMEDLINE(Ovid経由)で、2021年2月3日までに非外科的肥満管理と代謝減量手術後の全死因死亡を比較した無作為化試験、前向き比較対照研究およびマッチドコホート研究を検索した。総説とともに、組み入れた研究の参考文献一覧から灰色文献も検索した。共有異質性(ランダム効果など)および層別Coxモデルを用いて、試験レベルでの参加者のクラスタリングを考慮に入れた上で、代謝減量手術を施行した肥満成人の全死因死亡率をマッチさせた標準治療を実施した対照と比較した。このほか、治療必要数を算出し、Gompertz比例ハザードモデルを用いて平均余命を推定した。試験のプロトコールは、PROSPEROにCRD42020218472番で予め登録されている。【結果】特定した論文1,470編のうち、マッチドコホート研究16編および前向き比較対照試験1編を解析対象とした。1,200万人の間に7,712例が死亡した。全174,772例では、代謝減量手術で死亡のハザード率が49.2%(95%CI 46.3~51.9、P<0.0001)低下し、平均余命中央値が通常治療より6.1年(95%CI 5.2~6.9)長かった。部分集団解析で、代謝減量手術を施行した患者は、ベースラインで糖尿病があった患者(ハザード比0.409、95%CI 0.370~0.453、p<0.0001)、なかった患者(0.704、0.588~0.843、P<0.0001)ともに全死因死亡率が低かったが、治療効果は糖尿病があった患者の方が高かった(部分集団間のI^2 95.7%、P<0.0001)。手術群の方が非手術群よりも平均余命の中央値が9.3年(95%CI 7.1~11.8)長く、糖尿病がなかった患者の平均獲得余命は5.1年(2.0~9.3)であった。10年間で死亡1例を予防するための必要治療数は、糖尿病がある患者が8.4(95%CI 7.8~9.1)、糖尿病がない患者が29.8(21.2~56.8)であった。胃バイパス、バンディングおよび袖状胃切除の間に治療効果の差が見られなかった(I^2 3.4%、P=0.36)。このメタ解析の結果や他のデータを利用することにより、今回の解析で統合した世界の代謝減量手術適応患者間で手術率が1.0%上昇するごとに、糖尿病がある患者とない患者でそれぞれで5,100万人年と6,600万人年の平均余命が獲得できることが推定された。【解釈】代謝減量手術によって、通常の肥満管理よりも肥満成人患者の全死因死亡率および平均余命が大きく改善する。糖尿病がない患者よりも糖尿病を併存する患者の方が生存に関する便益が顕著に見られる。 第一人者の医師による解説 国内でも3学会合同のコンセンサスステートメント発刊 本手術の普及を期待 岡住 慎一 東邦大学医療センター佐倉病院外科教授 MMJ. December 2021;17(6):176 1950年代に欧米で減量を目的として開始された高度肥満症に対する外科治療は、減量効果とともに糖尿病をはじめとする肥満関連合併症の改善も得られることが判明し、metabolicsurgery(代謝改善手術)として新たに位置づけられ、現在、全世界で年間約80万人に行われるほどに拡大している(1)。高度肥満症では、関連疾患による高死亡率対策が課題である。本論文は、外科治療における生存期間延長効果について計174,772人が参加した17試験でメタ解析し、さらに2型糖尿病合併の有無において検討した大規模研究である。結果は、内科治療に比べ外科治療による生存期間延長効果は6.1年(95%信頼区間[CI],5.2~6.9)であり、致死危険率(hazardrateofdeath)で49%(95%CI,46.3~51.9;P<0.0001)の低下が得られていた。治療後20年、30年累積死亡率は、内科治療群でそれぞれ20.0、46.0%であったのに対し、外科治療群ではそれぞれ8.8、29.5%であった。さらに、2型糖尿病合併の有無別にみると、外科治療群では両者ともに全死亡率が低下し、特に、2型糖尿病合併例における効果は顕著であった(P<0.0001)。2型糖尿病の合併例における治療後20年累積死亡率は、内科治療群の35.2%に対し、外科治療群では21.1%、一方、非合併例ではそれぞれ19.3、11.9%であり、内科治療に対する外科治療の生存期間延長効果は糖尿病合併例で9.3年(95%CI,7.1~11.8)、非合併例で5.1年(2.0~9.3)であった。これらの生存期間延長効果は、肥満外科の主要な術式(胃バイパス、袖状胃切除、胃バンディング)すべてにおいて示されたとしている。Metabolicsurgeryの効果の周知により、2016年米国糖尿病学会(ADA)のガイドラインでは高度肥満症(体格指数[BMI]35以上、アジア系:BMI32.5以上)を伴う制御困難な2型糖尿病に対して外科治療が「推奨」され、世界45学会(日本糖尿病学会を含む)が承認した(2)。2021年7月には、日本肥満症治療学会、日本肥満学会、日本糖尿病学会の3学会合同により、「日本人の肥満2型糖尿病に対する減量・代謝改善手術に関するコンセンサスステートメント」が発刊された(3)。日本においても、今後さらに本手術の普及が進むことが期待されている。 1. Fifth IFSO Global Registry Report 2019(IFSO & Dendrite Clinical Systems) 2. 糖尿病診療ガイドライン2019(日本糖尿病学会) 3. 日本人の肥満2型糖尿病に対する減量・代謝改善手術に関するコンセンサスステートメント(日本肥満症治療学会/日本肥満学会/日本糖尿病学会,2021)
さまざまな血圧値の患者に用いる心血管疾患の1次予防および2次予防を目的とした薬剤による降圧治療 個別患者データのメタ解析
さまざまな血圧値の患者に用いる心血管疾患の1次予防および2次予防を目的とした薬剤による降圧治療 個別患者データのメタ解析
Pharmacological blood pressure lowering for primary and secondary prevention of cardiovascular disease across different levels of blood pressure: an individual participant-level data meta-analysis Lancet. 2021 May 1;397(10285):1625-1636. doi: 10.1016/S0140-6736(21)00590-0. 原文をBibgraph(ビブグラフ)で読む 上記論文の日本語要約【背景】心血管疾患の併存を問わない正常血圧または正常高値血圧の患者に用いる薬剤による血圧降下作用は明らかになっていない。著者らは、治療前の収縮期血圧値別に、降圧治療が主要心血管事象リスクにもたらす作用を明らかにすべく、無作為化試験の個別患者データを解析した。【方法】薬剤による降圧治療をプラセボまたは他のクラスの降圧薬と比較した無作為化試験または治療の強度別に比較した無作為化試験計48件(各群の追跡期間1,000人年以上)の個別患者データのメタ解析を実施した。心不全患者、急性心筋梗塞などの急性期疾患の短期治療を対象とした試験を除外した。Blood Pressure Lowering Treatment Trialists' Collaboration(英オックスフォード大学)から1972年から2013年までに発表された51試験のデータを取得した。データを統合し、心血管疾患併存(無作為化割り付け前の脳卒中、心筋梗塞、虚血性心疾患などの報告)の有無、収縮期血圧全体および7段階の収縮期血圧値分類(120 mmHg未満から170mmHg以上まで)別に降圧治療効果を層別化した。主要評価項目は、主要心血管事象(脳卒中、心筋梗塞または虚血性心疾患、致命的または入院を要する心不全の複合と定義)とし、intention-to-treatで解析した。【結果】この解析では、48試験の参加者計344,716例のデータを対象とした。無作為化前の平均収縮期血圧および拡張期血圧は、心血管疾患既往歴がある参加者(157,728例)が146/84 mm Hg、心血管疾患既往例がない参加者(186,988例)が157/89 mmHgであった。試験前の参加者の血圧に大きなばらつきを認め、心血管疾患既往歴がある参加者31,239例(19.8%)および心血管疾患既往例がない参加者14,928例(8.0%)の収縮期血圧が130mmHg未満であった。降圧治療の相対的効果は、収縮期血圧低下の程度と比例していた。中央値で4.15年の追跡後(Q1~Q3 2.97~4.96)、42,324例(12.3%)に主要心血管事象が発生した。試験前に心血管疾患既往歴がなかった参加者の1000人年当たりの主要心血管事象発症率は、比較対照群が31.9(95%CI 31.3~32.5)、介入群が25.9(25.4~26.4)であった。試験前に心血管疾患既往歴があった参加者の発症率は、比較対照群39.7(95%CI 39.0~40.5)および介入群36.0(95%CI 35.3~36.7)であった。収縮期血圧5mmHgの低下による主要心血管事象のハザード比(HR)は、心血管疾患既往がない参加者が0.91(95%CI 0.89~0.94)、心血管疾患既往がある参加者が0.89(0.86~0.92)であった。層別解析で、試験前の心血管疾患既往歴の有無や収縮期血圧分類別による主要心血管事象に対する治療効果の異質性について、信頼性の高い科学的根拠はなかった。【解釈】この無作為化試験の大規模解析では、収縮期血圧5mmHg低下により、心血管疾患既往歴の有無とは関係なく、正常血圧や正常高値血圧でさえ主要心血管事象リスクが約10%低下した。この結果は、現在治療の対象外となる血圧値でも、薬剤による一定の血圧降下が心血管疾患の1次予防および2次予防に等しく有効であることを示唆している。降圧治療の適応について患者と話し合う医師は、血圧を下げることよりも心血管リスク低下の重要性を重視すべきである。 第一人者の医師による解説 降圧療法は心血管病が存在し 血圧が低くても有用なことを確認 平田 恭信 東京逓信病院名誉院長 MMJ. December 2021;17(6):170 世界的に高血圧者の数はこの30年間で倍増しており、降圧療法の重要性が高まっている。高血圧の治療法については降圧薬の使用方法などにまだ改善の余地があるが、その恩恵については議論の余地は少ない。しかし未解決の重要な問題も残っており、そのうち(1)すでに心血管病を有する高血圧者と有さない高血圧者の間で降圧療法の効果は異なるのか否か(2)その効果は投与前の血圧値によって差があるのか、特に正常~正常高値の血圧レベルでどうか──という2つの疑問を本研究では解明しようとしている。それに答えるには相当数の対象者が必要である。というのは血圧レベルが正常に近いほど、降圧治療によるリスク低減効果は小さくなることが知られているからである。臨床上のエビデンスレベルとしては関連研究のメタアナリシスが最上位に置かれているのは周知のことであるが、メタアナリシスにも弱点はあり、どの論文を解析対象とするかの選択バイアスがありうること、対象者数が他より圧倒的に多い論文が含まれると結論がそれに引っ張られてしまうことである。その点、著者であるBloodPressureLoweringTrialists’Collaborationグループはあらかじめ質の保証された降圧療法に関する臨床研究を結果の出る前から組み入れることを表明しておき、それを徐々に積み上げてきた。このことによって少なくとも論文の選択バイアスは避けられる。さらに研究対象者の個々のデータ(individualparticipant-leveldata)も解析可能なシステムを構築した。これまでも同グループにより降圧療法による合併症の抑制効果は到達した血圧値に依存し、降圧薬の種類によらないことが示されてきた。本研究でも約34万人の解析により、収縮期血圧が5mmHg低下すると心血管合併症(イベント)の発生リスクが約10%低下し、この効果は投与前に心血管病のある場合(2次予防)、未罹患の場合(1次予防)のいずれでも同様に認められた。また心血管合併症の発症リスクも脳卒中で13%、心不全で13%ならびに虚血性心疾患で8%抑制された。さらにこの効果は投与前の血圧値を7段階に分けて解析しても各レベル間の差は明らかでなく、収縮期血圧が120mmHg未満や120~129mmHgであっても認められた。このことはいわゆる治療効果のJカーブ現象は一般的には心配はないことを示している。降圧療法の目的は心血管合併症の予防にあることより、心血管病が存在して、血圧が低めであっても治療が有用なことが確認された。
再発または難治性多発性骨髄腫に用いるカルフィルゾミブ+デキサメタゾンと比較したカルフィルゾミブ+デキサメタゾン+ダラツムマブ(CANDOR) 第III相多施設共同非盲検無作為化試験の結果
再発または難治性多発性骨髄腫に用いるカルフィルゾミブ+デキサメタゾンと比較したカルフィルゾミブ+デキサメタゾン+ダラツムマブ(CANDOR) 第III相多施設共同非盲検無作為化試験の結果
Carfilzomib, dexamethasone, and daratumumab versus carfilzomib and dexamethasone for patients with relapsed or refractory multiple myeloma (CANDOR): results from a randomised, multicentre, open-label, phase 3 study Lancet. 2020 Jul 18;396(10245):186-197. doi: 10.1016/S0140-6736(20)30734-0. 原文をBibgraph(ビブグラフ)で読む 上記論文の日本語要約 【背景】レナリドミドおよびボルテゾミブを用いた1次治療によって、再発または難治性多発性骨髄腫の新たな治療の必要性が高まっている。カルフィルゾミブとダラツムマブの併用は、第I相試験で、再発または難治性多発性骨髄腫での高い有効性が示されている。この試験では、再発または難治性多発性骨髄腫に用いるカルフィルゾミブ、デキサメタゾンおよびダラツムマブの併用をカルフィルゾミブとデキサメタゾンの併用と比較した。 【方法】この第III相多施設共同非盲検無作為化試験では、北米、欧州、オーストラリアおよびアジアの102施設から組み入れた再発または難治性多発性骨髄腫患者466例をカルフィルゾミブ+デキサメタゾン+ダラツムマブ(KdD)とカルフィルゾミブ+デキサメタゾン(Kd)に2対1の割合で無作為に割り付けた。全例にカルフィルゾミブ56mg/m2(第1サイクルの第1、2日は20mg/m2)を週2回投与した。ダラツムマブは、第1サイクルの第1、2日に8mg/kg、残りの第1サイクルと第2サイクルでは週1回16mg/kg、第3-6サイクルでは2週に1回16mg/kg、その後は4週に1回16mg/kgを投与した。デキサメタゾン40mgを週1回(第2週以降、75歳以上の患者に20mg)投与した。主要評価項目は、intention-to-treatで評価した無増悪生存期間とした。安全性解析集団で有害事象を評価した。この試験(NCT03158688)は、ClinicalTrials.govに登録されており、現在進行中であるが、登録は終了している。 【結果】2017年6月13日から2018年6月25日にかけて、適格性を評価した569例のうち466例を組み入れた。追跡期間中央値約17カ月後、KdD群は無増悪生存期間未達であったのに対し、Kd群は15.8カ月であった(ハザード比0.63、95%CI 0.46-0.85、P=0.0027)KdD群の治療期間中央値はKd群よりも長かった(70.1週 vs 40.3週)。KdD群の253例(82%)、Kd群の113例(74%)にグレード3以上の有害事象が報告された。治療中止に至った有害事象の発生頻度は、両群同等であった(KdD群69例[22%]、Kd群38例[25%])。 【解釈】KdD療法によって、Kd療法と比べて再発または難治性多発性骨髄腫患者の無増悪生存期間が長くなり、リスク便益のデータも良好であった。 第一人者の医師による解説 他の3剤併用療法との比較は未実施 併用療法の選択や順番は臨床現場での判断 中澤 英之 信州大学医学部血液内科講師 MMJ. February 2021;17(1):24 多発性骨髄腫(MM)の治療は、20年ほど前まで選択肢が限られていたが、その後、プロテアソーム阻害薬(PI)のボルテゾミブ、免疫調整薬(IMiDs)のレナリドミドが登場し、さらにこの10年間に7種類の新規薬剤が利用可能になった。現在、新規薬剤を含めた複数の選択肢の中から、患者ごとに治療を選ぶことが、臨床医の課題となっている。一方、MM患者の治療開始からの平均余命は約6年とされ、特に再発・難治性 MMの治療にはまだ改善の余地がある。 本論文は、再発・難治性 MMに対する新たな併用療法として、PIのカルフィルゾミブ(K)、抗CD38抗体薬のダラツムマブ(D)、デキサメタゾン(d)の3剤併用療法(KdD療法)の有用性を評価したランダム化国際共同非盲検第 III相 CANDOR試験の報告である。この試験結果に基づき、日本では2020年11月からKdD療法が保険診療で実施可能となった。本試験には北米、欧州、豪州、アジアの102施設から、前治療レジメン数1~3の再発・難治性MM患者が登録され、KdD群とKd(対照)群に割り付けられた。その結果、主要評価項目の無増悪生存期間(PFS)は追跡期間中央値17カ月時点において、KdD群は未到達、Kd群は15.8カ月(ハザード比[HR], 0.63)で、KdD療法の優位性が示された。頻度の高い有害事象(AE)として血小板減少、貧血、消化管症状、高血圧、感染症、疲労感などが認められ、ほかに注目すべきAEとして、末梢神経障害、注射反応、心不全、急性腎不全、虚血性心疾患などが報告された。有害事象による中止率はKdD群22%、Kd群25%で、Kの中止要因は心不全、Dの中止要因は肺炎が最多であった。 本試験の追跡調査結果が2020年秋の米国血液学会(ASH)で報告された。観察期間およそ28カ月時点の解析(1)によると、PFS中央値はKdD群28.6カ月、Kd群15.2カ月であり、KdD群の優位性はその後も維持されていることが明らかになった(HR, 0.59;95% CI, 0.45~0.78)。サブグループ解析では、細胞遺伝学的に高リスク患者、前治療歴2レジメン以上の患者、レナリドミド不応性患者でもKdD療法の優位性が示された。新たなAEの報告はなかった。 CANDOR試験によって、KdD療法は再発・難治性MMに対して有効で認容性の良好な治療選択肢であることが明らかになった。しかし、他の3剤併用療法との直接比較は現時点では実現していない。どの併用療法を、どのような順番で行うかは、まだ臨床現場での判断に任されている。MMの治療選択肢が劇的に増えたこの10年間は、解決すべき多くの課題を再認識した10年間であったとも言えるだろう。 1. Meletios A Dimopoulos, et al. Blood 2020; 136 (Supplement 1): 26-27.
SARS-CoV-2に対するChAdOx1 nCoV-19ワクチンの安全性および免疫原性 第I/II相単盲検無作為化比較試験の仮報告
SARS-CoV-2に対するChAdOx1 nCoV-19ワクチンの安全性および免疫原性 第I/II相単盲検無作為化比較試験の仮報告
Safety and immunogenicity of the ChAdOx1 nCoV-19 vaccine against SARS-CoV-2: a preliminary report of a phase 1/2, single-blind, randomised controlled trial Lancet. 2020 Aug 15;396(10249):467-478. doi: 10.1016/S0140-6736(20)31604-4. Epub 2020 Jul 20. 原文をBibgraph(ビブグラフ)で読む 上記論文の日本語要約 【背景】SARSコロナウイルス2(SARS-CoV-2)の世界的流行は、予防接種によって縮小できると思われる。著者らは、SARS-CoV-2のスパイク蛋白を発現するウイルスベクターコロナウイルスワクチンの安全性、反応性および免疫原性を評価した。 【方法】英国5施設で、SARS-CoV-2のスパイク蛋白を発現するチンパンジーアデノウイルスをベクターに用いたワクチン(ChAdOx1 nCoV-19)を対照の髄膜炎菌結合型ワクチン(MenACWY)と比較する第I/II相単盲検無作為化比較試験を実施した。検査によるSARS-CoV-2感染確定歴やCOVID-19様症状がない18~55歳の健康成人をChAdOx1 nCoV-19群とMenACWY群に(1対1の割合で)無作為に割り付け、いずれも5×1010ウイルス粒子を筋肉内に単回投与した。5施設中2施設でプロトコールを修正し、接種前にパラセタモルを予防投与してもよいこととした。10例を非無作為化非盲検のChAdOx1 nCoV-19プライムブースト群に割り付け、2回の接種日程を設け、初回投与28日後に追加投与した。SARS-CoV-2スパイク蛋白三量体に対する標準総IgG ELISA、多重免疫アッセイ、3通りの生SARS-CoV-2中和アッセイ(50%プラーク減少中和アッセイ[PRNT50]、マイクロ中和アッセイ[MNA50、MNA80、MNA90]およびMarburg VN)を用いて、偽ウイルス中和アッセイを用いて、試験開始時および追加接種時の液性応答を評価した。体外インターフェロンγenzyme-linked immunospot(ELISPOT)アッセイを用いて、細胞性応答を評価した。主要評価項目は、ウイルス学的に確定した症候性COVID-19で測定した有効性および重度有害事象の発生率で測定した安全性とした。患者を割り付けたグループごとに解析した。ワクチン投与後28日間にわたって安全性を評価した。ここに、安全性、反応性および細胞性および液性免疫反応に関する仮の結果を報告する。この試験は進行中であり、ISRCTN(15281137)、およびClinicalTrials.gov(NCT04324606)に登録されている。 【結果】2020年4月23日から同年5月21日にかけて1077例を登録し、ChAdOx1 nCoV-19(543例)とMenACWY(534例)に割り付、そのうち10例を非無作為化ChAdOx1 nCoV-19プライムブースト群に組み入れた。ChAdOx1 nCoV-19群では局所および全身反応が対照群より多く、熱っぽさ、寒気、筋肉痛、頭痛や倦怠感など多くの症状がパラセタモルの予防投与によって改善した(いずれもP<0.05)。ChAdOx1 nCoV-19による重度の有害事象はなかった。ChAdOx1 nCoV-19群では、スパイク特異的T細胞応答が14日目にピークに達した(末梢血単核球100万個当たりのスポット形成細胞数中央値856個、IQR 493~1802、43例)。28日時までに高スパイクIgG反応が上昇し(中央値157 ELISA単位(EU)、96~317、127例)、2回目の投与後にさらに上昇した(639EU、360~792、10例)。単回投与後、MNA80で測定した35例中32例(91%)、PRNT50で測定した35例(100%)でSARS-CoV-2中和抗体反応が検出された。ブースター投与後、全例に中和活性が認められた(42日時にMNA80で測定した9例全例、56日時にMarburg VNで測定した10例全例)。中和抗体反応にELISAで測定した抗体値と強い挿管が認められた(Marburg VNによるR2=0.67、P<0.001)。 【解釈】ChAdOx1 nCoV-19は許容できる安全性を示し、同種ブースター投与によって抗体反応が増強された。この結果を液性および細胞性免疫反応の誘導を併せて考えると、このワクチン候補を進行中の第III総試験で大規模な範囲で評価する妥当性を支持するものである。 第一人者の医師による解説 第3相試験でも70%の感染防御能確認 過去の研究の積み重ねの成果 中山 哲夫 北里大学大村智記念研究所特任教授 MMJ. February 2021;17(1):10 2019年12月に中国・武漢市で発生した重症肺炎の原因がコロナウイルスと判明し、SARSCoV-2と命名された。そのスパイク蛋白を主要な感染防御抗原としてワクチン開発が進んでいる。本論文はアストラゼネカ社とオックスフォード大学の共同開発によるチンパンジーアデノウイルス(ChAd)をベクターとしてSARS-CoV-2のスパイク抗原を発現する組換えワクチン(ChAdOx1nCoV-19)を健常成人543人に接種し、対照として4価髄膜炎ワクチンを534人に接種した第1/2相試験の免疫原性と安全性に関する報告である。 SARS-CoV-2スパイク蛋白に対するIgG抗体は接種28日後までに全例陽転化し、SARS-CoV-2中和抗体は80%抑制法で1回接種後に35人中32人(91%)が陽転化し、2回接種後に全例陽転化した。細胞性免疫能(ELISPOT)は1回接種14日後には43例全例に検出された。副反応としては487人中に接種部位の疼痛が67%(328人)、圧痛が83%(403人)、全身反応として倦怠感や頭痛がそれぞれ70%(340人)と68%(331人)に認められたが軽度で第3相試験に進むこととなった。 本論文以降、4月から英国、ブラジル、南アフリカで実施された4件の第III相試験の中間統合解析による有効性、免疫原性、安全性の結果が報告された(1)。感染者は標準量ワクチン2回接種群では27/4,440(0.6%)、対照(髄膜炎菌ワクチンまたは生食)群では71/4,455(1.6%)で有効率は62.1%(95% CI, 41.0~75.7)であった。低用量で1回接種し、その28日後に標準量を接種した群では3/1,367(0.2%)、対照群では30/1,374(2.2%)でワクチン有効率は90.0%(95% CI, 67.4?97.0)、全体のワクチン有効率は70.4%(54.8~80.6)であった(1)。また、英国で実施された70歳以上の高齢者も含めた同ワクチンの第2/3相試験では、18~55歳群と比較し、70歳以上では局所反応、全身反応ともに発現率が低いことが報告されている(2)。中和抗体は2回接種2週後に高値を示した。細胞性免疫能も検出され、英国では昨年12月8日から予防接種が始まった。 COVID-19発生後1年でワクチンができたように報道されているが、オックスフォード大学グループは種痘ワクチンをベクターとしたHIVワクチンの開発からヒトアデノウイルスをベクターとするシステムの構築へと進み、そして既存抗体陽性例で免疫能が低下する問題を解決すべく今回のChAdベクターを開発した。インフルエンザ、エボラ、MERS、SARS、Zikaウイルスなどに対するワクチンを研究開発し、一部は臨床試験まで実施した。こうした研究の積み重ねが今の成果につながっていることを忘れてはいけない(3)。 1. Voysey M, et al. Lancet. 2020:S0140-6736(20)32661-1. 2. Ramasamy MN, et al. Lancet. 2021;396(10267):1979-1993. 3. Gilbert SC, et al. Vaccine. 2017;35(35 Pt A):4461-4464.
高リスクまたは超高リスクの重症大動脈弁狭窄症に用いる自己拡張型intra-annular留置大動脈弁と市販の経カテーテル心臓弁の比較 無作為化非劣性試験
高リスクまたは超高リスクの重症大動脈弁狭窄症に用いる自己拡張型intra-annular留置大動脈弁と市販の経カテーテル心臓弁の比較 無作為化非劣性試験
Self-expanding intra-annular versus commercially available transcatheter heart valves in high and extreme risk patients with severe aortic stenosis (PORTICO IDE): a randomised, controlled, non-inferiority trial Lancet. 2020 Sep 5;396(10252):669-683. doi: 10.1016/S0140-6736(20)31358-1. Epub 2020 Jun 25. 原文をBibgraph(ビブグラフ)で読む 上記論文の日本語要約 【背景】自己拡張型intra-annular留置経カテーテル大動脈弁置換システムPortico(Abbott Structural Heart社、米ミネソタ州セントポール)と市販大動脈弁のデザインの違いによる性能を比較する無作為化試験データが必要とされる。 【方法】この多施設共同前向き非劣性無作為化試験(Portico Re-sheathable Transcatheter Aortic Valve System US Investigational Device Exemption:PORTICO IDE)では、経カテーテル大動脈弁置換の経験が豊富な米国およびオーストラリアの医療機関52施設から、高リスクまたは超高リスクの重症症候性大動脈弁狭窄症患者を登録した。NYHA心機能分類II以上で、重症大動脈弁狭窄症の既往がない21歳以上の患者を適格とした。実施施設、手術のリスク、カテーテル挿入部位で層別化し、置換ブロック法を用いて、適格患者を第1世代Portico弁およびその送達システムと市販の弁システム(intra-annularバルーン拡張型弁Edwards-SAPIEN、SAPIEN XT、SAPIEN 3 valve[いずれも米Edwards LifeSciences社]、supra-annular自己拡張型弁CoreValve、Evolut-R、Evolut-PRO[いずれも米Medtronic社]のいずれか)に(1対1の割合で)無作為に割り付けた。施設の職員、植込み担当医師および被験者には治療の割り付けが分かるようにした。重要な検査データおよび臨床的イベントの評価者には治療の割り付けを伏せた。主要安全性評価項目は、30日時の全死因死亡、障害が残る脳卒中、輸血を要する生命を脅かす出血、透析を要する急性腎障害、重大な血管合併症の複合とした。主要評価項目は、1年時の全死因死亡または障害が残る脳卒中とした。処置後最長2年間、臨床転帰および弁の機能を評価した。Intention-to-treatで主要解析を実施し、カプラン・マイヤー法でイベント発生率を推算した。非劣性のマージンは、主要安全性評価項目で8.5%、有効性評価項目で8.0%に設定した。この試験はClinicalTrials.govにNCT02000115番で登録されており、現在も進行中である。 【結果】資金提供者による11カ月間の組み入れ停止期間があったため、2014年5月30日から2015年9月12日までの間および2015年8月21日から2017年10月10日までの間に、1034例を登録し、適格基準を満たした750例をPortico弁群(381例)と市販の弁群(369例)に割り付けた。平均年齢83(SD 7)歳、395例(52.7%)が女性であった。30日時の主要安全性評価項目をみると、Portico弁群のイベント発生率が市販の弁群よりも高かった(52例[13.8%] vs 35例[9.6%]、絶対差4.2、95%CI -0.4~8.8、信頼区間の上限[UCB]8.1%、非劣性のP=0.034、優越性のP=0.071)。1年時の主要有効性評価項目発生率は、両群同等であった(Portico弁群55例[14.8%] vs 市販の弁群48例[13.4%]、差1.5%、95%CI -3.6~6.5、UCB 5.7%、非劣性のP=0.0058、優越性のP=0.50)。2年時の死亡率(80例[22.3%]vs 70例[20.2%]、P=0.40)や障害の残る脳卒中発生率(10例[3.1%]vs 16例[5.0%]P=0.23)は両群同等であった。 【解釈】Portice弁の2年時の死亡率や障害の残る脳卒中発生率は市販されている弁とほぼ同じであったが、30日時の死亡など主要複合安全性評価項目の発生率が高かった。第1世代Portico弁およびその送達システムに市販されている他の弁を上回る優越性は認められなかった。 第一人者の医師による解説 医療機器の成績は機器性能と使用する医療者の技術に依存 learning curveの考慮が必要 戸田 宏一 大阪大学大学院医学系研究科心臓血管外科准教授(病院教授) MMJ. February 2021;17(1):21 高齢化に伴い高齢者の大動脈弁狭窄症に関する関心が高まっている。開心術を要さない経カテーテル大動脈弁置換術(TAVI)は日本では2009年に導入され(1)、現在190以上の施設で年間8,000件以上実施されている。TAVIの成績は手術死亡率2%以下と優れているが、克服する問題もあり新しいデバイスに対する期待は高い。 本論文は、手術リスクの高い(STS-PROMスコア8%以上)重症大動脈弁狭窄症患者を対象に、弁植え込み部位での位置調整を容易にしたTAVI弁Portico(Abbott社)を市販のTAVI弁(SAPIENシリーズ[Edwards社]、CoreValveシリーズ[Medtronic社])と比較した多施設共同無作為化対照非劣性試験の報告である。その結果、安全性の主要評価項目である30日以内の全死亡・重症脳梗塞・輸血を要する出血・透析を要する急性腎障害・血管合併症の複合イベントの発生率は、Portico弁群の方が市販弁群よりも高かったが、非劣性の基準を満たした。30日以内全死亡率は、intention-to-treat解析ではPortico弁群の方が市販弁群よりも高いものの有意差はなかったが、per protocol解析では有意に高かった(4.3% 対 1.2%;P=0.01)。一方、有効性の主要評価項目である術後1年の全死亡・重症脳梗塞に関しては非劣性が示された。副次評価項目である術後1年の中等度以上の大動脈弁逆流はPortico弁群で有意に高く(7.8% 対 1.5%;P=0.0005)、術後30日以内のペースメーカー埋め込み率もPortico弁群で有意に高かった(27.7% 対 11.6%;P<0.001)。そのほか、術後2年までの生活の質(QOL)や心不全症状の改善について有意な群間差はなく、術後1年の6分間歩行距離でも差を認めなかった。 今回の試験においてPortico弁の優位性は示されなかったが、著者らは、①各施設のPortico弁使用数は5例と少なく経験が不十分であった可能性②研究期間の前半に比べ、後半の手術症例では、Portico弁群の術後死亡率、主要安全評価項目の改善がみられたと述べている。さらに、術後1年の中等度以上の大動脈弁逆流に関しても経験豊富な施設の多施設研究では改善がみられることから、医療技術のlearning curveの重要性を指摘している。一方、次世代Portico-FlexNav TAVI弁は今回の第1世代Portico弁よりも安全性の向上がみられ、次世代Portico弁の臨床試験が進められていると報告している。これら医療機器を用いた治療は医薬品と異なり、その成績は機器の性能と使用する医療者の技術に依存するため、機器の進歩を技術の進歩が追いかけ続けるmoving targetの状態となり、その評価を複雑にしていると思われる。 1. Maeda K, et al. Circ J. 2013;77(2):359-362.
左室駆出率が低下した心不全に用いるSGLT2阻害薬 EMPEROR-Reduced試験とDAPA-HF試験のメタ解析
左室駆出率が低下した心不全に用いるSGLT2阻害薬 EMPEROR-Reduced試験とDAPA-HF試験のメタ解析
SGLT2 inhibitors in patients with heart failure with reduced ejection fraction: a meta-analysis of the EMPEROR-Reduced and DAPA-HF trials Lancet. 2020 Sep 19;396(10254):819-829. doi: 10.1016/S0140-6736(20)31824-9. Epub 2020 Aug 30. 原文をBibgraph(ビブグラフ)で読む 上記論文の日本語要約 【背景】DAPA-HF試験(ダパグリフロジンを評価)とEMPEROR-Reduced試験(エンパグリフロジンを評価)から、糖尿病の有無を問わない左室駆出率が低下した心不全(HFrEF)患者ナトリウム・グルコース共輸送体-2(SGLT2)阻害によって心血管死や心不全による入院の複合リスクが低下したことが示された。しかし、いずれの試験も、心血管死または全死因死亡にもたらす効果の評価および臨床的に重要な下位集団に見られた効果の特徴を明らかにするのに十分な検出力はなかった。そこで著者らは、DAPA-HF試験のデータおよびEMPEROR-Reduced試験の患者データを用いて、両試験で無作為化したHFrEF患者全例および関連下位集団を対象に、SGLT2阻害が非致命的心不全イベントおよび腎転帰にもたらす効果を推定することを目的とした。 【方法】SGLT2阻害薬が糖尿病の有無を問わないHFrEF患者の心血管転帰にもたらす効果を検討した大規模試験2件、DAPA-HF試験(ダパグリフロジンを評価)とEMPEROR-Reduced試験(エンパグリフロジンを評価)のメタ解析を実施した。主要評価項目は、全死因死亡までの時間とした。さらに、事前に規定した下位集団を対象に、心血管死または心不全による入院の複合リスクにもたらす効果も検討した。この下位集団は、2型糖尿病の状態、年齢、性別、アンジオテンシン受容体・ネプリライシン阻害薬(ARNI)治療、NYHA心機能分類、人種、心不全による入院歴、推算糸球体濾過量(eGFR)、BMIおよび地域(事後)を基にしている。イベント発生までの時間にCox比例ハザードモデル、治療の相互作用にコクランのQ検定を用いてハザード比(HR)を求めた。イベント再発の解析は、Lin-Wei-Yang-Yingモデルで得た率比を基にした。 【結果】両試験の被験者計8474例でみた推定治療効果は、全死因死亡率低下率13%(統合HR 0.87、95%CI 0.77~0.98、P=0.018)、心血管死亡率低下14%(0.86、0.76~0.98、P=0.027)であった。SGLT2阻害によって、心血管死亡または心不全による初回入院の複合リスクの相対的低下率は26%(0.74、0.68~0.82、P<0.0001)、心不全による再入院または心血管死の複合減少率は25%であった(0.75、0.68~0.84、P<0.0001)。このほか、腎転帰の複合リスクも低下した(0.62、0.43~0.90、P=0.013)。試験間の効果量の異質性を検討した検定はいずれも有意ではなかった。統合した治療効果からは、年齢、性別、糖尿病の有無、ARNIを用いた治療および試験開始時のeGFRで分類した下位集団で、一貫して便益が見られたが、NYHA心機能分類および人種で分類した下位集団では、集団による治療の交互作用が認められた。 【解釈】エンパグリフロジンとダパグリフロジンが心不全による入院にもたらす効果は、両試験を通して一貫して見られ、両薬剤によってHFrEF患者の腎転帰が改善し、全死因死亡および心血管死を抑制することが示唆された。 第一人者の医師による解説 SGLT2阻害薬は糖尿病薬に収まらず 心不全治療薬としてエビデンスの構築へ 深谷 英平 北里大学医学部循環器内科学講師 MMJ. February 2021;17(1):19 元来、糖尿病薬として登場したSGLT2阻害薬であったが、近年報告された大規模臨床試験において心不全による入院、心血管イベント、腎機能悪化の抑制を認めたことから、多面的効果が期待される薬剤へと変わってきた。さらに糖尿病の有無にかかわらず、左室駆出率の低下した心不全(HFrEF)患者を対象に同様の効果が期待できるかを検証した、エンパグリフロジンを用いたEMPERORReduced試験(1)とダパグリフロジンを用いたDAPA-HF試験(2)が相次いで発表された。両試験ともに、糖尿病の有無にかかわらずSGLT2阻害薬群において心不全入院の有意な減少を認めた一方で、患者数の限界から、心血管死や全死亡における差を見出すには至らなかった。そこで本論文では両試験を統合解析し、HFrEF患者8,474人を対象にSGLT2阻害薬の有効性を再検討した。 その結果、SGLT2阻害薬群ではプラセボ群に比べ全死亡リスクは13%低下、心血管死リスクは14%低下した。心血管死と初回心不全入院の複合リスクは26%低下、心不全再入院と心血管死の複合リスクも25%低下した。腎関連の複合エンドポイント(eGFRの低下、透析への移行、腎移植、腎不全死)についても38%のリスクの低下が認められた。サブグループ解析でも、糖尿病の有無、性別、年齢(65歳超対以下)、心不全入院の既往、腎機能低下の有無、肥満の有無(BMI 30kg/m2以上対未満)にかかわらずSGLT2阻害薬の一貫した有効性が実証された。一方、55歳未満の比較的若い年齢層、心不全機能分類 NYHA III-IVの重症例では、SGLT2阻害薬の有効性についてプラセボよりも優位な傾向はみられるが有意差は認めなかった。異なる人種間でも一貫した有効性が示され、アジア人や黒人では白人に比べSGLT2阻害薬のイベント抑制効果がさらに強い可能性も示された。ダパグリフロジンとエンパグリフロジンの間で有効性の差は認められず、SGLT2阻害薬のクラスエフェクトの可能性が示唆された。 今回のメタ解析により、HFrEF患者に対するSGLT2阻害薬のイベント抑制効果、予後改善効果が明らかとなった。日本では2020年11月にダパグリフロジンは慢性心不全への適応追加が承認され、エンパグリフロジンも適応追加が申請された。SGLT2阻害薬は単なる糖尿病の薬に収まらず、イベント抑制効果が実証された心不全治療薬の1つになった。今後も心不全治療薬としてさらなるエビデンスが構築されていくことが予想される。 1. Packer M, et al. N Engl J Med. 2020;383(15):1413-1424. 2. McMurray JJV, et al. N Engl J Med. 2019;381(21):1995-2008.
若年者・高齢者を対象としたプライムブーストレジメンで投与するChAdOx1 nCoV-19ワクチンの安全性および免疫原性(COV002):第II/III相単盲検無作為化対照試験
若年者・高齢者を対象としたプライムブーストレジメンで投与するChAdOx1 nCoV-19ワクチンの安全性および免疫原性(COV002):第II/III相単盲検無作為化対照試験
Safety and immunogenicity of ChAdOx1 nCoV-19 vaccine administered in a prime-boost regimen in young and old adults (COV002): a single-blind, randomised, controlled, phase 2/3 trial Lancet. 2021 Dec 19;396(10267):1979-1993. doi: 10.1016/S0140-6736(20)32466-1. Epub 2020 Nov 19. 原文をBibgraph(ビブグラフ)で読む 上記論文の日本語要約 【背景】高齢者(70歳以上)がCOVID-19を発症すると重症化リスクや死亡リスクが高く、有効なワクチンを開発すれば、優先的接種の対象となる。ワクチンの免疫原性は、免疫老化の結果として高齢者で悪化することが多い。若年成人を対象とした新たなチンパンジーアデノウイルスベクター型ワクチンChAdOx1 nCoV-19(AZD1222)の免疫原性は既に報告している。今回は、対象者を70歳以上の高齢成人にも広げて、このワクチンの安全性と免疫原性を報告する。 【方法】この第II/III相単盲検無作為化対照試験(COV002)の第II相段階の報告では、英国の臨床研究施設2施設で、18歳以上の健康な成人を18~55歳、56~69歳、70歳以上の下位集団に分けて登録した。重度または治療不応の併存疾患やフレイルスコア高値(65歳以上の場合)がない被験者を適格とした。まず、被験者を低用量コホートに組み入れ、ブロック無作為化法を用いて、年齢、用量群および施設で層別化して、各年齢群内でChAdOx1 nCoV-19筋肉内投与(ウイルス粒子2.2×10^10個)と対照ワクチンMenACWYに割り付けることとし、18~55歳群はChAdOx1 nCoV-19の2回投与とMenACWYの2回投与に1対1の割合、56~69歳群はChAdOx1 nCoV-19単回投与、MenACWY単回投与、ChAdOx1 nCoV-19の2回投与、MenACWYの2回投与に3対1対3対1の割合、70歳以上群はChAdOx1 nCoV-19単回投与、MenACWY単回投与、ChAdOx1 nCoV-19の2回投与、MenACWYの2回投与に5対1対5対1の割合で無作為に割り付けた。初回投与と2回目投与の間隔は、28日間空けることとした。その後、被験者を標準投与コホート(ChAdOx1 nCoV-19ウイルス粒子3.5-6.5×10^10個)に組み入れ、18~55歳群をChAdOx1 nCoV-19の2回投与とMenACWYの2回投与に5対1の割合で割り付けたほかは、同じ無作為化手順を採用した。被験者と治験責任医師にワクチンの割付を伏せ、ワクチンを投与するスタッフには割り付けを伏せずにおいた。この報告の目的は、55歳以上の成人を対象とした単回投与および2回投与スケジュールの安全性、液性免疫および細胞性免疫原性を評価することである。施設内標準化ELISA、複合免疫アッセイおよび生重症急性呼吸器症候群コロナウイルス2(SARS-CoV-2)マイクロ中和アッセイ(MNA80)を用いて、ベースラインおよび各ワクチン接種から追加免疫投与1年後までの液性反応を評価した。ex-vivoのIFN-γ酵素結合免疫スポットアッセイを用いて、細胞応答を評価した。試験の複合主要転帰は、ウイルス学的に判定した症候性COVID-19症例数で測定した有効性および重篤な有害事象の発現で測定した安全性とした。ワクチンを投与した被験者を割り付けた投与群別に解析した。ここに、安全性、反応原性、細胞性および液性免疫反応に関する初期結果を報告する。本試験は現在進行中であり、ClinicalTrials.gov(NCT04400838)、ISRCTN(15281137)に登録されている。 【結果】2020年5月30日から8月8日の間に被験者560例を組み入れ、18~55歳群160例をChAdOx1 nCoV-19群100例、MenACWY群60例に、56~69歳群160例をそれぞれ120例、40例に、70歳以上群240例をそれぞれ200例、40例に割り付けた。7例が割り付けた2回投与レジメンの追加免疫投与を受けず、1例が誤ったワクチン接種を受け、3例に検体の表示に誤りがあったため、免疫原性の解析から除外した。解析対象とした552例のうち280例(50%)が女性であった。ChAdOx1 nCoV-19群の方が対照ワクチン群よりも局所反応と全身反応が多く、性質は既報とほぼ同じ(注射部位疼痛、発熱感、筋肉痛、頭痛)であったが、高齢者(56歳以上)では若年者よりも少なかった。ChAdOx1 nCoV-19の標準的な2回投与を受けた被験者では、初回免疫投与後、18~55歳群49例中43例(88%)、56~69歳群30例中22例(73%)、70歳以上群49例中30例(61%)が局所反応、それぞれ42例(86%)、23例(77%)、32例(65%)が全身反応を報告した。2020年10月26日現在、試験期間中に重篤な有害事象が13件発生したが、試験ワクチンとの関連があるものはないと考えられた。ワクチン2回投与群では、年齢層別コホート3群の追加免疫投与28日後の抗スパイクSARS-CoV-2 IgG反応中央値がほぼ同じであった(標準用量群:18~55歳20,713任意単位[AU]/mL[IQR:13,898~33,550]、39例;56~69歳16,170AU/mL[10,233~40,353]、26例;70歳以上17,561AU/mL[9,705~37,796]、47例;P=0.68)。全年齢層群の追加免疫投与後の中和抗体価がほぼ同じであった(標準投与群の第42日のMNA80中央値:18~55歳193[IQR 113~238]、39例;56~69歳144[119~347]、20例;70歳以上161[73~323]、47例;P=0.40)。追加免疫投与後14日目までに209例中208例(99%以上)に中和抗体反応が認められた。ChAdOx1 nCoV-19標準単回投与後14日目にT細胞反応がピークとなった(末梢血単核球100万個当たりのスポット形成細胞[SFC]中央値は、18~55歳1187個[IQR 841~2428]、24例;56~69歳797個[383~1817]、29例;70歳以上977個[458~1914]、48例)。 【解釈】ChAdOx1 nCoV-19の忍容性は、高齢者の方が若年成人よりも高いと考えられ、全年齢層群の追加免疫投与後の免疫原性がほぼ同じであった。全年齢層と併存疾患がある者を対象にこのワクチンの有効性をさらに詳細に評価することが必要である。 第一人者の医師による解説 日本での本ワクチンの承認や接種の検討において 非常に重要なデータ 齋藤 昭彦 新潟大学大学院医歯学総合研究科小児科学分野教授 MMJ. April 2021;17(2):41 アデノウイルスベクターワクチン(ChAdOx1nCoV-19)の安全性と免疫原性については、第1・2相試験ですでに報告されたが(1)、今回報告された第2・3相単盲検ランダム化対照試験では、対象年齢を広げ、70歳以上の高齢者を含めた集団を対象に同様の検討を行った。試験参加者は計560例で、英国の2つの臨床研究施設で行われた。対象は、18~55歳、56~69歳、70歳以上の3群に分けられ、年齢が高くなるほどSARS-CoV-2ワクチンへの割り付け比が高くなるように設定された(18~55歳では62.5%、56~69歳では75.0%、70歳以上では83.3%)。重症またはコントロールできていない基礎疾患を有する人は除外された。参加者はSARS-CoV-2ワクチン群(2.2X1010ウイルス粒子/ワクチン)もしくは対照ワクチン(髄膜炎菌ワクチン)を接種された後、液性・細胞性免疫の評価、有効性、そして安全性が評価された。 その結果、局所と全身の反応の頻度は、SARSCoV-2ワクチン群の方が対照ワクチン群に比べ高かったが、それぞれの副反応の頻度は過去の報告と同様で、56歳以上の成人ではそれより若い人に比べ低かった。SARS-CoV-2ワクチン2回接種者における初回接種後の局所反応の頻度は、18~55歳で88%、56~69歳で73%、70歳以上で61%、全身反応はそれぞれ86%、77%、65%であった。ワクチンと関係のある重篤な副反応はみられなかった。SARS-CoV-2ワクチン2回接種者において、追加接種後28日目の抗スパイク蛋白IgG値の中央値、中和抗体価に関して年齢群間に差は認めなかった。追加接種後14日目までに99%超の接種者に中和抗体の反応を認めた。T細胞の反応は、初回接種後14日目にピークとなった。 ChAdOx1 nCoV-19は、アストラゼネカ社がこれまで研究開発してきたアデノウイルスベクターを用いたワクチンの技術(2)を応用した製品である。新興コロナウイルスであるSARS、MARSに対するワクチンの研究を行ってきた成果が活かされている。今回の結果は、若年成人だけでなく、高齢者でもより安全に接種でき、追加接種後の免疫原性は全年齢において同様であった。今回のSARSCoV-2感染は、年齢が高くなると重症化リスクが高まるため(3)、特に高齢者におけるワクチンの安全性と免疫原性のデータは非常に重要である。このワクチンは、今後、日本に輸入され、また、国内のワクチン会社でも技術移転により製造され、それらが国内で接種される予定である(執筆時点)。このデータは、今後の日本における本ワクチンの承認、接種を検討する上で、非常に重要なものとなるであろう。 1. Folegatti PM, et al. Lancet. 2020;396(10249):467-478. 2. Gilbert SC, et al. Vaccine. 2017;35(35 Pt A):4461-4464. 3. O'Driscoll M, et al. Nature. 2021;590(7844):140-145.
COVID-19に用いる体外式模型人工肺支援 体外生命維持機構レジストリの国際コホート研究
COVID-19に用いる体外式模型人工肺支援 体外生命維持機構レジストリの国際コホート研究
Extracorporeal membrane oxygenation support in COVID-19: an international cohort study of the Extracorporeal Life Support Organization registry Lancet. 2020 Oct 10;396(10257):1071-1078. doi: 10.1016/S0140-6736(20)32008-0. Epub 2020 Sep 25. 原文をBibgraph(ビブグラフ)で読む 上記論文の日本語要約 【背景】COVID-19に伴う急性低酸素血症性呼吸不全に対して、主要医療機関の多くが体外式模型人工肺(ECMO)による支援を推奨している。しかし、COVID-19に対するECMO使用の初期報告から、死亡率が非常に高いことが明らかになり、これまでにCOVID-19に用いるECMOを検討する大規模な国際的コホート研究は報告されていない。 【方法】2020年1月16日から5月1日の間に36カ国213施設でECMOによる支援を開始したCOVID-19確定患者(16歳以上)の疫学、入院経過および転帰を明らかにするため、体外生命維持機構(ELSO)レジストリのデータを用いた。主要転帰は、ECMO開始90日時に生存時間解析で評価した院内死亡とした。多変量Coxモデルを用いて、患者および病院因子に院内死亡率と関連があるかを検討した。 【結果】ECMO支援を用いたCOVID-19患者1035例のデータをこの研究の対象とした。このうち、67例(6%)が入院中、311例(30%)が自宅に退院または急性期リハビリ施設に転院、101例(10%)は長期急性期医療施設に転院または転院先不明、176例(17%)が他院に転院、380例(37%)が死亡した。ECMO開始90日後の院内死亡の推定累積発生率は37.4%(95%CI 34.4~40.4)であった。最終記録が死亡または退院だった患者の死亡率が39%(968例中380例)であった。循環補助を目的としたECMOの使用には、院内死亡率上昇と独立の関連があった(ハザード比1.89、95%CI 1.20~2.97)。静脈-静脈方式ECMO支援を受け、急性呼吸窮迫症候群の特徴が見られたCOVID-19患者の下位集団では、ECMO開始90日後の院内死亡の累積発生率は38.0%(95%CI 34.6~41.5)であった。 【解釈】ECMOを実施したCOVID-19患者で、ECMO開始90日後の推定死亡率および最終記録が死亡または退院だった患者の死亡率はいずれも40%未満であった。世界213施設から得られた今回のデータは、COVID-19に用いるECMOの死亡率の一般化可能な推定値を提示するものである。 第一人者の医師による解説 非 COVID-19関連 ARDSのECMO導入後死亡率と同程度 肺保護人工呼吸器療法との比較にはRCTが必要 佐藤 ルブナ(フェロー)/大曲 貴夫(センター長) 国立国際医療研究センター病院国際感染症センター MMJ. April 2021;17(2):44 新型コロナウイルス感染症(COVID-19)の世界的流行以前には、急性呼吸促迫症候群(ARDS)患者に対する体外式膜型人工肺(ECMO)使用は肺保護を目指した従来の人工呼吸器療法に比べ死亡率を低下させることが報告されている(1)。しかし、ECMOを導入したCOVID-19患者を対象とした初めての報告では、ECMO導入群の死亡率が90%を超える結果であった(2)。この報告はARDSの定義を満たしてECMOを導入されたCOVID-19患者の死亡率を検討したプール解析であったが、病態が解明されていなかった流行初期の中国の報告のみが解析の対象となった背景があり、より大規模な国際コホート研究が望まれていた。 本論文はExtracorporeal Life Support Organization(ELSO)レジストリをもとに、ECMO導入COVID-19患者の予後を明らかにすることを主な目的としたコホート研究である。対象は2020年1~5月に36カ国のECMOセンター213施設でECMOを導入された16歳以上のCOVID-19患者1,035人で、研究期間終了(解析)時点において生存退院、死亡退院、ECMO導入後90日間の追跡終了のいずれかに該当した患者は968人(94%)であった。主要評価項目はECMO導入後90日間の院内死亡率(生存時間解析)であった。 対象患者の年齢中央値は49歳、男性が74%を占めていた。70%の患者が肥満、糖尿病、喘息、呼吸器疾患、腎不全、心疾患、免疫不全などの基礎疾患を有していた。ARDSのベルリン定義を満たしたのは79%であった。患者全体の90日院内死亡率は37.4%(95% CI, 34.4~40.4)であり、ARDSのベルリン定義を満たし、静脈脱血-静脈送血(V-V)ECMOを導入された群では38.0%(34.6~41.5)であった。これはCOVID-19以外の病態を誘因としたARDSに対してECMOを導入された患者における死亡率の既報(3)と同程度であった。また、ECMO使用中の出血性合併症として、6%の患者が脳出血を発症したが、これも非COVID-19患者の既報(1)と同程度であった。 本論文は重症呼吸不全を呈したCOVID-19患者に対してECMOの導入とECMOセンターでの管理を検討するよう勧めている世界保健機関(WHO)のガイドラインを支持する結果を示した。本論文の対象期間はステロイドや抗ウイルス薬に関する知見が集積する以前であったため、ステロイドは41%、レムデシビルは8%の使用にとどまり、現在の標準的治療が反映されていない点もある。標準化治療を行った上でのECMO導入率や死亡率の解明、肺保護戦略を徹底した人工呼吸器管理とECMOの有効性の比較にはさらなる研究が必要である。 1. Munshi L, et al. Lancet Respir Med. 2019;7(2):163-172. 2. Henry BM, et al. J Crit Care. 2020;58:27-28. 3. Combes A, et al. N Engl J Med. 2018;378(21):1965-1975.
2017年から2100年までの195の国と地域の出生率、死亡率および人口の推移 国際疾病負荷研究の予測研究
2017年から2100年までの195の国と地域の出生率、死亡率および人口の推移 国際疾病負荷研究の予測研究
Fertility, mortality, migration, and population scenarios for 195 countries and territories from 2017 to 2100: a forecasting analysis for the Global Burden of Disease Study Lancet. 2020 Oct 17;396(10258):1285-1306. doi: 10.1016/S0140-6736(20)30677-2. Epub 2020 Jul 14. 原文をBibgraph(ビブグラフ)で読む 上記論文の日本語要約 【背景】年齢構造の変化、資源および医療のニーズ、環境および経済的展望を予測し計画を立てるために、将来の人口推移の可能性を理解することが重要である。将来の出生率パターンは、将来の人口規模を予測するために重要なデータであるが、不確実性が大きく、推定や予測の方法が異なるため、世界の人口予測に重要な差が生じる可能性がある。人口規模および年齢構造が変化することによって、多くの国に経済的、社会的および地政学的に大きな影響を及ぼすと思われる。この研究では、死亡率、出生率、移住および人口を予測する新たな方法を開発した。このほか、将来の人口統計学的変化の経済的および地政学的影響の可能性を評価した。 【方法】出生率、移住率および死亡率を表す参照および代替シナリオで将来の人口をモデル化した。50歳時のコホート合計特殊出生率(CCF50)を求めるため、統計モデルを開発した。コホート合計特殊出生率は、合計特殊出生率(TFR)の期間指標よりも長期的にみてはるかに安定している。学歴および避妊手段使用を時系列ランダムウォークの関数に用いて、CCF50をモデル化した。CCF50および共変量を関数として、2100年までの年齢別死亡率をモデル化した。潜在的な死亡率、危険因子、自己回帰和分移動平均(ARIMA)モデルを用いて、2100年までの年齢別死亡率をモデル化した。社会人口統計学的特性指数、人口増加率および戦争と自然災害による死亡を関数として、ARIMAモデルを用いて純移動をモデル化した。モデルの枠組みは、学歴と避妊手段使用の変化の速度を基にした参照シナリオと代替シナリオを開発するために用いた。参照シナリオでの各国および地域の国内総生産のサイズを推定した。予測した不確定区間(UI)で過去のデータ、モデル推定および予測したデータ分布から伝播した不確かさを組み込んだ。 【結果】参照シナリオでは、2100年の世界のTFRは1.66(95%UI 1.33~2.08)と予測された。参照シナリオでは、世界の人口は2064年の97億3000万人(88億4000万-109億)をピークに減少に転じ、2100年に87億9000万人(68億3000万~118億)になると予測された。2100年の上位5カ国の参照予測は、インド(10.9億人[7億2000万~17億1000万])、ナイジェリア(7億9100万人[5億9400万~10億5600万])、中国(7億3200万[4億5600万~14億9900万])、米国(3億3600万[2億4800万~4億560-万])およびパキスタン(2億4800万[1億5100万~4億2700万])であった。このほか、世界の多くの地域で年齢構造が変化することが示唆され、2100年には65歳以上が23億7000万人(19億1000万~28億7000万)、20歳未満が17億人(11億1000万~28億1000万)になることが予測された。2050年までに151カ国で、2100年までに183カ国でTFRが人口置換水準(TFR 2.1未満)を下回ることが予想された。参照シナリオでは、日本やタイ、スペインなどの23カ国で、2017-2100年の間に人口が50%以上減少し、中国の人口が48.0%(-6.1~68.4)低下する見通しが立った。参照シナリオでは、2035年までに中国が最大の経済国になり、2098年に米国が再び最大の経済国になることが予想された。代替シナリオから、教育と避妊手段へのアクセス改善の持続可能な開発目標(SDG)が達成されると、2100年の世界人口が62億9000万人(48億2000万~87億3000万)になり、この推進因子の変化率の99%パーセンタイル値を推測すると、人口が68億8000万人(52億7000万~95億1000万)になると予測された。 【解釈】この結果から、女性の学歴と避妊手段へのアクセス改善の傾向が続くと、出生率低下が加速し、人口増加が鈍化する。中国やインドなどの多くの国でTFRが人口置換水準を下回り続けると、経済的、社会的、環境的および地政学的な影響があるであろう。女性の性と生殖に関する健康を維持し増進すると同時に、低出生率の持続に対応する政策が今後重要になってくると思われる。 第一人者の医師による解説 増える非労働力人口比率 社会保障制度などの財政的持続がますます深刻 野村 周平 慶應義塾大学医学部医療政策管理学・特任准教授 MMJ. April 2021;17(2):60 1950年代以降、世界人口の予測は国連経済社会局人口部(UNPD)などによって行われている。UNPDの最新統計では2100年の世界人口は108.8億人と予測されている。本論文は世界の疾病負荷研究(Global Burden of Disease;GBD)プロジェクトの研究成果からの1編であり、GBD2017の枠組みに基づき(1)、UNPDなどの方法論に改善を加え、世界195の国・地域における2018年から2100年までの人口を予測したものである。 GBD人口予測モデルは大きく死亡率、移民率、出生率の3要素からなり、それぞれも別個の予測モデルで推定されている。死亡率は危険因子の保有率や社会人口指数(SDI:収入レベル、教育レベル、出生率の混合指標)の関数として、移民率はSDIや紛争・自然災害による死亡数、出生率と死亡率の差の関数としてモデル化されている。出生率のモデル化が特筆すべき点であり、従来人口予測で多く使われる合計特殊出生率(TFR)ではなく、「特定の集団における女性が50歳を迎えたときに出産した子供の数の平均」と定義される生涯出生率(CCF50)が、今回の人口予測モデルで使われている。CCF50は女性が出産可能年齢の終わりまでの実際の出産数を表すという点で、女性教育の進展に伴う妊娠年齢上昇の影響をTFRよりも受けづらく、推定がより安定する(注 TFRはある年における教育水準の異なる世代別の出生率の合計)。CCF50は女性の教育レベルと避妊へのアクセスの関数としてモデル化され、年齢別出生率およびTFRもCCF50の関数として推定された。 本研究では、世界人口は2064年にピーク(約97億人)を迎えた後、2100年には約88億人にまで減少すると推定された。2050年までに195カ国中151カ国で、2100年までに183カ国でTFRが2.1*を下回るとしている(*人口が減少し始めるとされる閾値)。2017年時に約1億2800万人であった日本の人口は、2100年までに5300万人以下に減少すると予測された。日本、タイ、スペインなど23カ国では、人口が半減すると予測されている。 人口減少は二酸化炭素の排出量減や、地球の食糧システムへの負荷が減るメリットだけではなく、経済成長とも密接に関係する。本研究で非労働力人口の労働力人口(20~64歳と定義)に対する比率は、2017年の0.80から世界全体で2100年には1.16に達すると予測された。国民健康保険や社会保障制度の財政的持続の課題がますます深刻になる。女性のリプロダクティブ・ヘルスを維持・向上させつつ、低出生率の持続に適応するための政策オプションが今後重要であると本稿は締め括られている。 1. GBD 2017 Population and Fertility Collaborators. Lancet.2018;392(10159):1995-2051.(MMJ 2019年12月号)
発症時刻不明の脳梗塞に対して高度画像診断を基に実施するアルテプラーゼ静注 個別被験者データの系統的レビューとメタ解析
発症時刻不明の脳梗塞に対して高度画像診断を基に実施するアルテプラーゼ静注 個別被験者データの系統的レビューとメタ解析
Intravenous alteplase for stroke with unknown time of onset guided by advanced imaging: systematic review and meta-analysis of individual patient data Lancet. 2020 Nov 14;396(10262):1574-1584. doi: 10.1016/S0140-6736(20)32163-2. Epub 2020 Nov 8. 原文をBibgraph(ビブグラフ)で読む 上記論文の日本語要約 【背景】発症時刻不明の脳梗塞は、血栓溶解療法の対象から除外されている。今回、このような患者で、画像バイオマーカーから救済可能な組織が特定できた場合に用いるアルテプラーゼ静注が安全かつ有効であるかを明らかにすることを目的とした。 【方法】2020年9月21日以前に公表された試験の個別被験者データの系統的レビューとメタ解析を実施した。灌流・拡散MRI、灌流CTまたはDWI-FLAIR1 MRIでミスマッチ所見が認められた発症時刻不明の成人脳梗塞患者を対象に、アルテプラーゼ静注を標準治療またはプラセボと比較した無作為化試験を適格とした。主要評価項目は、90日時の機能的転帰良好(修正ランキン尺度[mRS]0~1点、後遺症がなしを示す)とし、調整した無条件混合効果ロジスティック回帰モデルを用いて治療効果を推定した。90日時のmRS改善と患者個別の転帰(mRS 0~2点)を副次評価項目とした。安全性評価項目を死亡、重度の後遺症または死亡(mRS 4~6点)、症候性頭蓋内出血とした。本試験は、PROSPEROに登録されている(CRD42020166903)。 【結果】特定した抄録249報のうち、WAKE-UP、EXTEND、THAWS、ECASS-4の4試験が適格基準を満たした。4試験から843例分の個別被験者データが得られ、そのうち429例(51%)がアルテプラーゼ、414例(49%)がプラセボまたは標準治療に割り付けられていた。アルテプラーゼ群420例中199例(47%)、対照群409例中160例(39%)の転帰が良好であり(調整後オッズ比[OR]1.49[95%CI 1.10~2.03]、P=0.011)、4試験の異質性は低かった(I^2=27%)。アルテプラーゼに機能的転帰の有意な改善(調整後共通OR 1.38[95%CI 1.05~1.80]、P=0.019)および患者個別転帰の高オッズ(調整後OR 1.50[同1.06~2.12]、P=0.022)との関連が認められた。アルテプラーゼ群では90例(21%)に重度の後遺症または死亡(mRS 4~6点)が発生したのに対して、対照群では102例(25%)であった(調整後OR 0.76[同0.52~1.11]、P=0.15)。アルテプラーゼ群の27例(6%)、対照群の14例(3%)が死亡した(調整後OR 2.06[同1.03~4.09]、P=0.040)。症候性頭蓋内出血発生率は、アルテプラーゼ群の方が対照群よりも高かった(11例[3%] vs. 2例[1%未満]、調整後OR 5.58[同1.22~25.50]、P=0.024)。 【解釈】DWI-FLAIR画像または灌流画像でミスマッチが認められた発症時刻不明の脳梗塞で、アルテプラーゼ静注によって、プラセボまたは標準治療と比べて90日時の良好な機能的転帰が得られた。症候性頭蓋内出血リスクが上昇したが、全機能的転帰で純便益が認められた。アルテプラーゼ群の方がプラセボ群よりも死亡が多かったが、重度の後遺症または死亡が少なかった。 第一人者の医師による解説 DWI-FLAIRミスマッチまたはCT/MRI灌流画像は アルテプラーゼ静注療法の適応判断に有用 秋山 武紀 慶應義塾大学医学部脳神経外科専任講師 MMJ. June 2021;17(3):77 アルテプラーゼ静注療法は、発症4.5時間以内の脳梗塞に対する重要な治療の1つとして普及している。しかし、起床時に症状を有することが確認されたものの正確な発症時刻を同定できない、いわゆるwake-up strokeも散見され、発症時刻不明であっても有効かつ安全な治療法が求められている。 本論文では、発症時刻不明または発症後4〜5時間を経過した脳梗塞に対し、画像診断により適応を判断しアルテプラーゼ静注療法を行った群とプラセボ群を比較した無作為化対照試験を系統的にレビューし、基準を満たした4試験(WAKEUP、EXTEND、THAWS、ECASS-4)から抽出した843人の個人データを用いてメタ解析を行った。患者背景は平均年齢68.5歳、女性38%、NIHSS中央値7点、治療判断のための画像診断は① DWIFLAIRミスマッチ(MRI拡散強調画像[DWI]で高信号域の領域はあるが、FLAIR画像で信号変化を認めない場合)または②灌流画像(MRI潅流画像またはCT灌流画像でのペナンブラ領域[灌流の低下はあるが、不可逆的な脳虚血に陥っていないと判断される領域]がある場合)が使用された。 結果は、90日後の予後良好(mRS 0-1)はアルテプラーゼ群47%、対照群39%と有意にアルテプラーゼ群で高かった(オッズ比,1.49)。有害事象では、症候性頭蓋内出血の発生率がアルテプラーゼ群3%と対照群1%未満に比べ有意に高かった。死亡率はアルテプラーゼ群の方が有意に高く(6%対3%)、アルテプラーゼ群の死亡の26%は症候性頭蓋内出血に起因した。しかし非自立・死亡であるmRS 3-6はアルテプラーゼ群の方が対照群よりも有意に少なく(35%対42%)、結論として、画像診断をガイドにアルテプラーゼ静注療法の適否を判断する方法の有効性が認められた。 本研究により発症時刻不明の脳梗塞に対し、より先進的な画像診断を追加することでアルテプラーゼ静注療法の適応を判断できることが明らかとなり、適応の範囲が広がった。脳卒中治療ガイドライン 2015(追補2019)では発症時刻不明の脳梗塞に対し、「頭部MRI拡散強調画像の虚血性変化がFLAIR画像で明瞭でない場合、アルテプラーゼ静注療法を行うことを考慮してもよい(グレードC1)」となっているが、近日改訂されるガイドラインでは灌流画像に関する追加記載、エビデンスレベルの変更が予想される。単純CT所見と発症時刻から治療適応を判断していた時代から、MRIやCTの灌流画像も求められる時代に突入したといえる。脳卒中診療体制の整備が進められる中、適切な治療を行うために、的確な画像診断を迅速に行える施設への転院搬送システムもより一層重要となることが予想される。 略号: NIHSS=National Institutes of Health Stroke Scale、DWI-FLAIR=diffusion weighted imaging-fluid attenuated inversion recovery、modified Rankin Scale=mRS.
小児および成人B細胞急性リンパ性白血病に用いるゲノム編集したドナー由来同種CD-19標的キメラ抗原受容体発現T細胞 第1相試験の結果
小児および成人B細胞急性リンパ性白血病に用いるゲノム編集したドナー由来同種CD-19標的キメラ抗原受容体発現T細胞 第1相試験の結果
Genome-edited, donor-derived allogeneic anti-CD19 chimeric antigen receptor T cells in paediatric and adult B-cell acute lymphoblastic leukaemia: results of two phase 1 studies Lancet. 2020 Dec 12;396(10266):1885-1894. doi: 10.1016/S0140-6736(20)32334-5. 原文をBibgraph(ビブグラフ)で読む 上記論文の日本語要約 【結果】ゲノム編集したドナー由来同種CD-19標的キメラ抗原受容体発現(CAR)T細胞によって、臨床現場での即時使用が可能なCAR-T細胞製品の新たな形態がもたらされ、利用のしやすさや適応性が広がることになる。UCART19はそのような製品の一つで、小児と成人の再発または治療抵抗性B細胞急性リンパ性白血病で検討されている。多施設共同第1相試験2件で、小児と成人の再発または治療抵抗性B細胞急性リンパ性白血病に用いるUCART19の実行可能性、安全性および抗白血病活性を調べることを目的とした。 【方法】UCART19の安全性および抗白血病活性を評価する進行中の多施設共同第1相試験2件に小児および成人患者を組み入れた。全例にフルダラビンとcyclophosphamideを投与して(一部にアレツムマブ併用)リンパ球を枯渇させてから、小児にUCART19を1.1~2.3×10^6個/kg、成人に6×10^6個/kg、または用量漸増試験で1.8~2.4×10^8個投与した。主要評価項目は、初回投与とデータカットオフ時点の間に認められる有害事象とした。この試験は、ClinicalTrials.govにNCT02808442とNCT02746952で登録されている。 【結果】2016年6月3日から2018年10月23日の間に、2試験に小児7例および成人14例を組み入れ、UCART19を投与した。サイトカイン放出症候群が最も発言頻度の高い有害事象で、19例(91%)に認められ、そのうち3例(14%)がグレード3~4のサイトカイン放出症候群を来した。他の有害事象として、8例(38%)にグレード1または2の神経毒性、2例(10%)にグレード1の急性移植片対宿主病、6例(32%)にグレード4の遷延する血球減少が発現した。2例が治療のため死亡し、1例はサイトカイン放出症候群と併発した好中球減少性敗血症によるもので、もう1例は血球減少が持続する症例で肺出血が原因であった。21例中14例(67%)が完全寛解または投与28日後の造血機能回復が不十分な完全寛解を得た。アレツムマブを投与しなかった患者(4例)では、UCART19の増殖または抗白血病活性が認められなかった。奏効期間中央値は、同種幹細胞移植前に奏効が認められた14例中10例(71%)で4.1カ月であった。6カ月無増悪生存率が27%、全生存率が55%であった。 【解釈】この2件の試験は、悪性度の高い白血病の治療に用いる同種のゲノム編集したCAR-T細胞の実行可能性を初めて示したものである。UCART19は、複数の治療歴がある小児および成人の再発または治療抵抗性B細胞急性リンパ球性白血病の生体内で増殖および抗白血病活性を示し、安全性も管理可能であった。本試験の結果によって同種CAR-T細胞の分野が一歩前進し、UCART19は、急速に進行する疾患や自家CAR-T細胞療法が利用できない場合にも治療の機会をもたらすものである。 第一人者の医師による解説 ドナーのTリンパ球を用いる同種 CAR-T細胞の開発で より多くの患者にCAR-T細胞療法の機会の増加を期待 金 裕花/森 鉄也 聖マリアンナ医科大学病院小児科 MMJ. June 2021;17(3):85 キメラ抗原受容体(chimeric antigen receptor; CAR)は、がん細胞が発現する抗原などを標的として作製された人工的受容体であり、CARを患者のTリンパ球に遺伝子導入したCAR-T細胞の輸注により、がん細胞に対する特異的な免疫応答による治療効果が期待される(自家CAR-T細胞療法)。 ほとんどのB細胞性急性リンパ芽球性白血病(acute lymphoblastic leukemia;ALL)患者のALL細胞に発現するCD19を標的とした自家CAR-T細胞は、再発・治療抵抗性のCD19陽性ALLに対し高い奏効割合(70〜90%)を示し、日本においても2019年5月に薬事承認された。一方で、自家CAR-T細胞の製造は複雑であり、時間を要し(少なくとも5〜6週)、1回の投与あたりの薬価は3,349万円(保険収載当時)と高額である。 本論文は、健常ドナーのTリンパ球を用いて製造されたCD19標的同種CAR-T細胞(UCART19)の再発・治療抵抗性のCD19陽性ALLに対する第1相試験の報告である。対象患者(小児7人、成人14人)には、リンパ球除去療法後にUCART19が輸注され、主要評価項目である有害事象などが評価された。最も頻度の高い有害事象はサイトカイン放出症候群であり、21人中19人(91%)に生じ、3人(14%)はグレード3以上であった。また、6人(32%)にグレード4の遷延する血球減少、2人(10%)にグレード1の急性移植片対宿主病を認めた。敗血症、肺出血により2人に治療関連死亡が生じた。輸注28日後に、14人(67%)に完全奏効、あるいは造血機能回復が不十分な完全奏効を認めた。6カ月無増悪生存率は27%、6カ月生存率は55%であった。リンパ球除去療法にアレムツズマブ*を使用しなかった4人では、UCART19の増殖および治療効果を認めなかった。UCART19は対応可能な安全性プロファイルのもとで、患者体内で増殖しALLに対する効果を発揮したことから、再発・治療抵抗性のCD19陽性ALL患者に対する同種CAR-T細胞療法の開発に有望な一歩を踏み出したと結論している。 患者ではなくドナーのTリンパ球を用いる同種CAR-T細胞の開発により、必要時に速やかに使用可能な既製(off-the-shelf)製剤として、より多くの患者にCAR-T細胞療法の機会が増加すると期待される。さらなる経験、評価の蓄積が待たれる。 *アレムツズマブ;リンパ球などが発現するCD52に対する抗体製剤であり、日本では同種造血幹細胞移植の前治療などに対する適応が薬事承認されている。
治療歴のない局所再発の切除不能または転移性トリプルネガティブ乳がんに用いるペムブロリズマブ+化学療法とプラセボ+化学療法の比較(KEYNOTE-355) 無作為化プラセボ対照二重盲検第3相試験
治療歴のない局所再発の切除不能または転移性トリプルネガティブ乳がんに用いるペムブロリズマブ+化学療法とプラセボ+化学療法の比較(KEYNOTE-355) 無作為化プラセボ対照二重盲検第3相試験
Pembrolizumab plus chemotherapy versus placebo plus chemotherapy for previously untreated locally recurrent inoperable or metastatic triple-negative breast cancer (KEYNOTE-355): a randomised, placebo-controlled, double-blind, phase 3 clinical trial Lancet. 2020 Dec 5;396(10265):1817-1828. doi: 10.1016/S0140-6736(20)32531-9. 原文をBibgraph(ビブグラフ)で読む 上記論文の日本語要約 【背景】転移性トリプルネガティブ乳がんで、ペムブロリズマブ単独療法による抗腫瘍活性の持続と管理可能な安全性が示された。今回、ペムブロリズマブの併用によって、転移性トリプルネガティブ乳がんに用いる化学療法の抗腫瘍活性が増幅するかを明らかにすることを目的とした。 【方法】29カ国209施設で実施したこの無作為化プラセボ対照二重盲検第3相試験では、ブロック法(ブロック数6)と統合ウェブ応答の自動音声応答システムを用いて、未治療の局所再発切除不能または転移性トリプルネガティブ乳がん患者をペムブロリズマブ(200mg)3週に1回+化学療法(ナブパクリタキセル、パクリタキセルまたはゲムシタビン+カルボプラチン)とプラセボ+化学療法に無作為に割り付けた。患者を化学療法の種類(タキサンまたはゲムシタビン・カルボプラチン)、試験開始時のPD-L1発現状態(統合陽性スコア[CPS]1点以上または1点未満)および術前または術後補助療法で用いた同等の化学療法による治療歴の有無で層別化した。適格基準を18歳以上、中央判定機関が確定したトリプルネガティブ乳がん、測定可能な腫瘍1個以上、中央検査機関でトリプルネガティブ乳がんの状態およびPD-L1発現を免疫組織学的に確認するために新たに採取した腫瘍検体の提供が可能、米国東海岸癌臨床試験グループの全身状態スコア0または1点、十分な臓器機能とした。スポンサー、治験担当医師、その他の施設職員(治療の割り付けを伏せなかった薬剤師を除く)および患者にペムブロリズマブと生理食塩水の投与の割り付けを伏せた。さらに、スポンサー、治験担当医師、その他の施設職員および患者に患者ごとの腫瘍PD-L1バイオマーカーの結果も知らせずにおいた。PD-L1 CPSが10以上、1以上およびITT集団のそれぞれで評価した無増悪生存期間および総生存期間を主要有効性評価項目とした。無増悪生存期間の最終的な評価はこの中間解析で実施し、総生存期間を評価すべく追跡を継続中である。無増悪生存期間に階層的検定手順を用いて、まずCPSが10以上の患者(この中間解析では事前に規定した統計学的基準がα=0.00111)、次にCPSが1以上の患者(この中間解析ではα=0.00111、CPSが10以上の患者の無増悪生存期間から得たpartial α)、最後にITT集団(この中間解析ではα=0.00111)を評価した。本試験はClinicalTrials.govにNCT02819518で登録されており、現在も進行中である。 【結果】2017年1月9日から2018年6月12日の間に1372例をふるいにかけ、847例を治療に割り付けることとし、566例をペムブロリズマブ+化学療法群、281例をプラセボ+化学療法に割り付けた。2回目の中間解析(2019年12月11日にデータカットオフ)では、追跡期間中央値がペムブロリズマブ+化学療法群25.9カ月(IQR 22.8~29.9)、プラセボ+化学療法群26.3カ月(22.7~29.7)であった。CPSが10以上の患者の無増悪生存期間中央値が、ペムブロリズマブ+化学療法群9.7カ月、プラセボ+化学療法群5.6カ月であった(進行または死亡のハザード比[HR]0.65、95%CI 0.49~0.86、片側のP=0.0012[主要目的達成]。CPSが1以上の患者の無増悪生存期間中央値がそれぞれ7.6カ月と5.6カ月(HR 0.74、0.61~0.90、片側のP=0.0014[有意差なし])、ITT集団で7.5カ月と5.6カ月(HR 0.82、0.69~0.97[検定未実施])であった。ペムブロリズマブの治療効果によってPD-L1発現が増加した。グレード3~5の治療関連有害事象発現率がペムブロリズマブ+化学療法群68%、プラセボ+化学療法群67%であり、そのうちの死亡率がペムブロリズマブ+化学療法群1%未満およびプラセボ+化学療法群0%であった。 【解釈】ペムブロリズマブ+化学療法で、CPSが10以上の転移性トリプルネガティブ乳がんの無増悪生存期間が、プラセボ+化学療法と比べて有意で臨床的意義のある改善が認められた。この結果は、転移性トリプルネガティブ乳がんの1次治療に用いる標準化学療法にペムブロリズマブを上乗せした場合の効果を示唆するものである。 第一人者の医師による解説 化学療法への上乗せ効果が示されたことで 新たな治療選択肢が増える 川端 英孝 虎の門病院乳腺内分泌外科部長 MMJ. June 2021;17(3):87 本論文は2020年に米国臨床腫瘍学会(ASCO)で発表された第3相KEYNOTE-355試験の中間解析結果の詳報である。PD-L1発現陽性(Combined Positive Scoreが10以上)の手術不能または転移性のトリプルネガティブ乳がんの1次治療として、抗PD-1モノクローナル抗体ペムブロリズマブと化学療法の併用が、化学療法のみの場合よりも有意に無増悪生存期間(PFS)を延長できるという内容である。なお、PFSはKEYNOTE-355試験の主要評価項目の1つであるが、もう1つの主要評価項目である全生存期間(OS)の評価は未発表で試験は継続されている。 ER陰性、PR陰性、HER2陰性を特徴とするトリプルネガティブ乳がんは他のサブタイプの乳がんに比べ治療ターゲットに欠けており、進行乳がんの治療戦略としては化学療法を主体としたものになるが、早晩治療抵抗性になってしまう。免疫チェックポイント阻害薬に分類されるペムブロリズマブは単剤でも抗腫瘍活性を示し、化学療法との併用が期待されていた。今回この薬剤の化学療法への上乗せ効果が示されたことで、我々は新たな治療選択肢を手に入れたことになる。 同じ免疫チェックポイント阻害薬として先行して2019年9月20日に適応拡大の承認を日本で受けたアテゾリズマブとの比較は重要である。アテゾリズマブは抗PD-L1モノクローナル抗体でIMpassion130試験(1)においてトリプルネガティブ進行乳がんにおける化学療法への上乗せ効果を示した。2つの薬剤の標的は異なっており、それぞれの臨床試験におけるPD-L1陽性例の評価もオーバーラップが多いが別個のコンパニオン診断を用いている。また臨床試験で用いられた化学療法がIMpassion130試験ではnab-パクリタキセルであるのに対してKEYNOTE355試験ではいくつかの化学療法レジメンが用いられている。 KEYNOTE-355試験には当院も含め日本の施設も参加しており、論文発表に先立ちMSD社は2020年10月12日、ペムブロリズマブについて、手術不能または転移性のトリプルネガティブ乳がんへの適応拡大申請を行ったと発表している。執筆時点(2021年4月17日)で審査中となっているが、承認されると、このセッティングの乳がん治療薬としてアテゾリズマブに加えて、新たな免疫チェックポイント阻害薬が加わることになる。なお、マイクロサテライト不安定性(MSI-High)を有する固形がんにおいてはがん種横断的にペムブロリズマブの使用が承認されており、この条件を満たせば現在でもペムブロリズマブを乳がんに使用することは可能である。 1. Schmid P, et al. N Engl J Med. 2018;379(22):2108-2121.
乳児に用いるビデオ喉頭鏡の初回成功率(VISI) 多施設共同無作為化対照試験
乳児に用いるビデオ喉頭鏡の初回成功率(VISI) 多施設共同無作為化対照試験
First-attempt success rate of video laryngoscopy in small infants (VISI): a multicentre, randomised controlled trial Lancet. 2020 Dec 12;396(10266):1905-1913. doi: 10.1016/S0140-6736(20)32532-0. 原文をBibgraph(ビブグラフ)で読む 上記論文の日本語要約 【背景】乳児に対する直接喉頭鏡を用いた気管挿管は困難である。今回、麻酔科医による標準ブレード型ビデオ喉頭鏡によって、直接喉頭鏡と比較して気管挿管初回成功率が改善し、合併症リスクが低下するかを明らかにすることを目的とした。著者らは、ビデオ喉頭鏡による初回成功率は直接喉頭鏡よりも高いという仮説を立てた。 【方法】この多施設共同並行群間無作為化対照試験では、米国の小児病院4施設とオーストラリアの小児病院1施設の手術室で気管挿管を要するが気道確保が困難でない乳児を組み入れた。ブロック数2、4、6の置換ブロック法を用いて、患児をビデオ喉頭鏡と直接喉頭鏡に(1対1の比率で)無作為に割り付け、施設と医師の役割で層別化した。保護者に処置の割り付けを伏せた。主要評価項目は、気管挿管時に初回で成功した乳児の割合とした。解析(修正intention-to-treat[mITT]集団およびper-protocol)に一般化推定方程式モデルを用いて、在胎期間、米国麻酔学会の術前全身状態分類、体重、医師の役割および施設で層別化した。試験は、ClinicalTrials.govにNCT03396432で登録されている。 【結果】2018年6月4日から2019年8月19日の間に乳児564例を組み入れ、282例(50%)をビデオ喉頭鏡、282例(50%)を直接喉頭鏡に割り付けた。乳児の平均年齢は5.5カ月(SD 3.3)であった。ビデオ喉頭鏡群の274例と直接喉頭鏡群の278群をmITT解析の対象とした。ビデオ喉頭鏡群では254例(93%)、直接喉頭鏡では244例(88%)が初回挿管に成功した(調整絶対リスク差5.5%[95%CI 0.7~10.3]、P=0.024])。ビデオ喉頭鏡群の4例(2%)、直接喉頭鏡群の15例(5%)に重度合併症が発生した(-3.7%[-6.5~-0.9]、P=0.0087)。ビデオ喉頭鏡群(1例[1%未満])の方が直接喉頭鏡群(7例[3%])よりも食道挿管が少なかった(同-2.3[-4.3~-0.3]、P=0.028)。 【解釈】麻酔下の乳児で、標準ブレード型ビデオ喉頭鏡を用いると初回成功率が改善し、合併症も減少した。 第一人者の医師による解説 乳児に対する気管挿管はビデオ喉頭鏡の使用が望ましい 大原 卓哉(助教)/清野 由輩(講師)/山下 拓(教授) 北里大学医学部耳鼻咽喉科・頭頸部外科 MMJ. June 2021;17(3):90 毎年、多くの乳児が気管挿管を必要とする全身麻酔手術を受けている。乳児は成人に比べ気管挿管時のリスクが高く、初回での気管挿管成功は重要であり、複数回の気管挿管は生命を脅かす合併症につながる可能性がある。ビデオ喉頭鏡は、気道確保が困難な乳児に対して直接喉頭鏡よりも初回成功率が高いことが報告されているが、構造的に正常な気道を持つ乳児におけるビデオ喉頭鏡の有用性については議論の余地があった。 本論文は、全身麻酔手術を受ける乳児に対する標準ブレード型ビデオ喉頭鏡を使用した気管挿管の有効性と安全性を検討した米国とオーストラリアの小児病院における多施設共同並行群間ランダム化比較試験の報告である。対象は、年齢12カ月齢未満、全身麻酔手術(30分以上の非心臓手術)、麻酔科医による気管挿管を受ける患者とされた。除外基準は、挿管困難の病歴、頭蓋顔面異常の病歴、または身体検査に基づく挿管困難が予測された患者であった。麻酔科医は、標準ブレード型ビデオ喉頭鏡(Storz C-Mac Miller Video Laryngoscope)または直接喉頭鏡のいずれかをランダムに割り当てられ、乳児に気管挿管を行った。気管チューブのサイズはガイドラインに基づいて選択され、気管挿管後24時間までの挿管関連有害事象が検討された。 最終的に274人(50%)がビデオ喉頭鏡、278人(50%)が直接喉頭鏡に割り付けられ、結果が解析された。ビデオ喉頭鏡の93%、直接喉頭鏡の88%で気管挿管に初回で成功し、特に体重6.5kg以下の乳児では、ビデオ喉頭鏡の初回成功率が直接喉頭鏡に比べ有意に高かった(92%対81%;P=0.003)。挿管試行回数も、ビデオ喉頭鏡の方が少なかった。重篤でない合併症(軽度喘鳴、喉頭痙攣[薬物 投与の必要性、緊急気管挿管を伴う]、気管支痙攣 、軽度気道外傷 、気道過敏化)の発生率は2群間で差はなかったが、ビデオ喉頭鏡では、重篤な合併症(中等度〜重度低酸素血症、食道挿管、心停止、咽頭出血)の発生が少なかった。特に、食道挿管は直接喉頭鏡では3%であったのに対し、ビデオ喉頭鏡では1%未満であった。また、ビデオ喉頭鏡では輪状軟骨圧迫の必要性が減少した。 毎年乳児への気管挿管が多く行われているが、 気管挿管中は重篤な有害事象が発生するリスクがあるため、初回成功率が5%向上することは非常に意味があると考えられる。費用的な課題はあるが、より高い初回成功率およびより少ない合併症のため標準ブレード型ビデオ喉頭鏡が広く使用されることが望まれる。
中等症ないし重症の尋常性乾癬に用いるbimekizumabとウステキヌマブの比較(BE VIVID) 52週間の多施設共同二重盲検実薬対照プラセボ対照第3相試験
中等症ないし重症の尋常性乾癬に用いるbimekizumabとウステキヌマブの比較(BE VIVID) 52週間の多施設共同二重盲検実薬対照プラセボ対照第3相試験
Bimekizumab versus ustekinumab for the treatment of moderate to severe plaque psoriasis (BE VIVID): efficacy and safety from a 52-week, multicentre, double-blind, active comparator and placebo controlled phase 3 trial Lancet. 2021 Feb 6;397(10273):487-498. doi: 10.1016/S0140-6736(21)00125-2. 原文をBibgraph(ビブグラフ)で読む 上記論文の日本語要約 【背景】乾癬の治療に、確実な即効性があり皮膚病変が完全に消失する結果をもたらすというアンメットニーズがある。bimekizumabは、IL-17Aに加えてIL-17Fを選択的に阻害するモノクローナルIgG1抗体である。中等症ないし重症の尋常性乾癬を対象に、bimekizumabの有効性および安全性を52週間にわたってプラセボおよびウステキヌマブと比較することを目的とした。 【方法】BE VIVIDは、アジア、オーストラリア、欧州および北米の11カ国で実施した多施設共同二重盲検実薬対照プラセボ対照第3相試験であった。18歳以上の中等症ないし重症の尋常性乾癬患者(乾癬の面積・重症度指数[PASI]スコア12点以下、体表面積に占める病変部位の割合10%以上および5点尺度の医師による全般的評価[IGA]スコア3点以上)を組み入れた。無作為化は、地理的地域、生物学的製剤投与歴で層別化し、患者、治験担当医師および資金提供者に治療の割り付けを伏せた。自動応答技術を用いて、患者をbimekizumab 320mgを4週に1回投与するグループ、ustekinumab 45mgまたは90mg(試験開始時の体重により決定)を0週時と4週時、その後12週に1回投与するグループ、プラセボを投与するグループに(4対2対1の割合で)割り付けた。16週時、プラセボを投与していた患者をbimekizumab 320mg 4週に1回投与に切り替えた。全試験薬を2回の皮下注射で投与した。主要評価項目は、16週時のPASIの90%改善(PASI 90)率およびIGAスコアで消失またはほぼ消失(スコア0または1点)が示されたIGA奏効率とした(データに欠損がある患者は非奏効例とした[non-responder imputation])。intention-to-treat集団を有効性解析の対象とし、試験薬を1回以上投与した患者を安全性解析の対象とした。この試験は、ClinicalTrials.govにNCT03370133で登録されている(終了)。 【結果】2017年12月6日から2019年12月13日の間に735例をふるいにかけ、567例を組み入れ、無作為に割り付けた(bimekizumab 320mg 4週に1回群321例、ustekinumab 45mgまたは90mg 12週に1回群163例、プラセボ群83例)。16週時、bimekizumab群321例中273例(85%)がPASI 90を達成したのに対して、ウステキヌマブ群は163例中81例(50%、リスク差35[95%CI 27~43]、P<0.0001)、プラセボ群は83例中4例(5%、同80[74~86]、P<0.0001)であった。16週時、bimekizumab群の270例(84%)がIGA奏効を達成したのに対して、ウステキヌマブ群は87例(53%、リスク差30[95%CI 22~39]、P<0.0001)、プラセボ群は4例(5%、同79[73~85]、P<0.0001)であった。52週間でbimekizumab群395例中24例(6%、16週時にプラセボから切り替えた患者を含む)、ウステキヌマブ群163例中13例(8%)から、治療下で発現した重篤な有害事象が報告された。 【解釈】中等症ないし重症の尋常性乾癬に用いるbimekizumabは、ウステキヌマブやプラセボより有効性が高い。bimekizumabの安全性に関するデータは、前回の試験で見られたものと同じであった。 第一人者の医師による解説 新たな作用機序を持つビメキズマブ 関節症状にも高い治療効果を期待 神谷 浩二(准教授)/大槻 マミ太郎(教授〈副学長〉) 自治医科大学医学部皮膚科学講座 MMJ. August 2021;17(4):126 日本での乾癬に対する生物学的製剤は、2010年に腫瘍壊死因子(TNF)阻害薬のインフリキシマブ、アダリムマブが承認され、11年にインターロイキン(IL)-12/23阻害薬のウステキヌマブが承認された。その後、IL-23阻害薬、IL-17阻害薬が開発、承認され、21年4月時点で10種類の治療選択肢がある。 ビメキズマブは、IL-17AとIL-17Fを選択的に阻害するヒト化モノクローナル IgG1抗体で、2021年2月26日に既存治療で効果不十分な尋常性乾癬、膿疱性乾癬、乾癬性紅皮症の効能または効果に係る製造販売承認申請が行われた。 本論文は、中等症~重症の18歳以上の乾癬患者におけるビメキズマブの有効性および安全性の評価を目的とし、ウステキヌマブ、プラセボを対照とした52週間の無作為化二重盲検試験(BEVIVID試験)の結果に関する報告である。主要評価項目は、16週時点での乾癬の皮疹面積・重症度指標(PASI)の90%以上の改善(PASI 90)の割合、医師による全般的評価(Investigators’ Global Assessment;IGA)で皮膚病変消失またはほぼ消失(IGA 0/1)の割合で、ビメキズマブはウステキヌマブよりも有意に優れた結果であった。また、臨床効果は52週時点まで維持され、安全性も確認された。速効性に関しては、4週時点でのPASI 75で評価され、ビメキズマブはウステキヌマブよりも有意に優れていた。乾癬に対するビメキズマブの有効性と安全性は、その他の第3相試験でも確認されている(1)。 乾癬に対するIL-17阻害薬では、IL-17Aを阻害するセクキヌマブ、イキセキズマブ、IL-17受容体Aを阻害するブロダルマブがすでに承認されているが、ビメキズマブはこれまでの薬剤とは異なった作用機序を有する薬剤であり、新たなIL-17阻害薬の治療選択肢として期待される。また、IL-17阻害薬は乾癬の皮膚症状だけでなく、関節症状に対しても高い治療効果が期待できる。今後は乾癬の関節症状に対するビメキズマブの有効性と安全性に関する試験結果が待たれる。 1. Gordon KB, et al. Lancet. 2021; 397(10273):475-486.
PD-L1発現率50%以上の進行非小細胞肺がんの1次治療に用いるcemiplimab単独療法 国際共同多施設非盲検第III相無作為化比較試験
PD-L1発現率50%以上の進行非小細胞肺がんの1次治療に用いるcemiplimab単独療法 国際共同多施設非盲検第III相無作為化比較試験
Cemiplimab monotherapy for first-line treatment of advanced non-small-cell lung cancer with PD-L1 of at least 50%: a multicentre, open-label, global, phase 3, randomised, controlled trial Lancet. 2021 Feb 13;397(10274):592-604. doi: 10.1016/S0140-6736(21)00228-2. 原文をBibgraph(ビブグラフ)で読む 上記論文の日本語要約 【背景】プログラム細胞死リガンド1(PD-L1)が50%以上発現している進行非小細胞肺がんの1次治療に用いるPD-L1阻害薬cemiplimabを検討することを目的とした。 【方法】国際共同多施設非盲検第III相試験、EMPOWER-Lung 1では、24カ国138試験で組み入れた適格患者(18歳以上で組織学的および細胞学的に確認した進行非小細胞肺がんがあり、ECOG全身状態0~1点、喫煙未経験者を適格とした)をcemiplimab 3週に1回350mg投与と白金製剤含む2剤併用化学療法に1対1の割合で無作為化した。病勢進行後に化学療法からcemiplimabへと交差(クロスオーバー)してもよいこととした。主要評価項目は、治療の割付を伏せられた独立審査委員会が評価する全生存期間および無増悪生存期間とした。intention-to-treat集団および使用説明書に従った22C3アッセイでPD-L1が50%以上発現した患者から成る事前に規定したPD-L1発現率50%以上の集団(FDAが試験依頼者に要請)を対象に、主要評価項目を評価した。割り付けた治療を1回以上実施した患者全例で有害事象を評価した。この試験は、ClinicalTrials.govに登録されており(NCT03088540)、進行中である。 【結果】2017年6月27日から2020年2月27日の間に、710例を無作為化した(intention-to-treat集団)。563例から成るPD-L1発現率50%以上の集団では、全生存期間中央値がcemiplimab群(283例)で未到達(95%CI 17.9~評価不能)、化学療法群(280例)で14.2カ月(11.2~17.5)であった(ハザード比[HR]0.57[0.42-0.77]、P=0.0002)。無増悪生存期間中央値はcemiplimab群8.2カ月(6.1~8.8)、化学療法群5.7カ月(4.5~6.2)であった(HR 0.54[0.43~0.68]、P<0.0001)。intention-to-treat集団でも、クロスオーバー率が高かった(74%)にも関わらず、cemiplimabで全生存期間および無増悪生存期間の有意な改善が認められた。cemiplimabで治療した355例中98例(28%)および化学療法で治療した342例中135例(39%)にグレード3~4の治療下発生有害事象が発生した。 【解釈】PD-L1発現率50%以上の進行非小細胞肺がんで検討した結果、化学療法と比べるとcemiplimab単独療法で全生存期間および無増悪生存期間が有意に改善した。この結果から、この患者集団に用いる新たな治療選択肢の可能性が示された。 第一人者の医師による解説 PD-L1 50%以上ではペムブロリズマブ、アテゾリズマブに続く選択肢 鹿毛 秀宣 東京大学大学院医学系研究科 次世代プリシジョンメディシン開発講座 特任准教授 MMJ. August 2021;17(4):112 免疫チェックポイント阻害薬は非小細胞肺がんの治療薬として重要な位置を占める。現在日本では抗 PD-1抗体であるニボルマブとペムブロリズマブ、抗 PD-L1抗体であるアテゾリズマブとデュルバルマブ、抗 CTLA-4抗体であるイピリムマブの5種類の免疫チェックポイント阻害薬が承認されている。そのうちペムブロリズマブとアテゾリズマブはPD-L1の免疫染色で高発現を示す非小細胞肺がんにおいて単剤で良好な成績を示している(1),(2)。 本研究は、根治治療の対象とならない局所進行あるいは転移を伴う非小細胞肺がん患者に、新たな抗 PD-1抗体であるセミプリマブを1次治療として単剤投与し、標準的な細胞障害性抗がん剤と比較した第3相試験である。ペムブロリズマブと同じ22C3アッセイにてPD-L1 50%以上の高発現を認めた非小細胞肺がん患者を対象とし、EGFR、ALK、ROS1遺伝子異常を認める患者および非喫煙者は除外された。喫煙者は非喫煙者よりも免疫チェックポイント阻害薬が効きやすいことはよく知られているが、治験から非喫煙者を除外する基準は新しい。 本試験でセミプリマブは主要評価項目である全生存期間(OS)、無増悪生存期間(PFS)ともに細胞障害性抗がん剤と比較して有意に延長した。奏効率は39%であり、健康関連 QOLの改善もみられた。PD-L1の発現率を50%以上~60%以下、60%超~90%未満、90%以上に分けたところ、PD-L1の発現が高い方がOS、PFS、奏効率すべてにおいて優れた結果であった。免疫関連有害事象は17%と過去の報告よりも少なく、治療関連死は3%と過去の報告と同等であった。 この結果をもって2021年2月に米食品医薬品局(FDA)はPD-L1 50%以上でEGFR・ALK・ROS1陰性の非小細胞肺がん患者を対象としてセミプリマブを承認した。日本における承認は未定であるが、ペムブロリズマブ、アテゾリズマブとの使い分けは難しい。PD-L1 50%以上で効果が高いのは新規性に乏しいものの、PD-L1発現と免疫チェックポイント阻害薬の効果の関係はアッセイや薬剤により多少の差異があるため、22C3アッセイでPD-L1の発現を評価し、高発現群に抗 PD-1抗体を使用すれば効果が高いことが再現されたことは意義がある。また、PD-L1 90%以上の群で有効性がさらに高いことが第3相試験の試験開始前から規定されていたサブグループ解析で示されたのは初めてであり、同じPD-L1 50%以上でも50%に近い群にはKEYNOTE-189試験(3)に準じてペムブロリズマブ+細胞障害性抗がん剤、100%に近い群にはセミプリマブを投与するという選択肢になるかもしれない。 1. Reck M, et al. New Engl J Med. 2016;375(19):1823-1833. 2. Herbst RS, et al. New Engl J Med. 2020;383(14):1328-1339. 3. Gandhi L, et al. N Engl J Med. 2018;378(22):2078-2092.
過体重または肥満の2型糖尿病成人患者に用いるセマグルチド週1回2.4mg投与(STEP 2試験) 無作為化二重盲検ダブルダミープラセボ対照第III相試験
過体重または肥満の2型糖尿病成人患者に用いるセマグルチド週1回2.4mg投与(STEP 2試験) 無作為化二重盲検ダブルダミープラセボ対照第III相試験
Semaglutide 2·4 mg once a week in adults with overweight or obesity, and type 2 diabetes (STEP 2): a randomised, double-blind, double-dummy, placebo-controlled, phase 3 trial Lancet. 2021 Mar 13;397(10278):971-984. doi: 10.1016/S0140-6736(21)00213-0. Epub 2021 Mar 2. 原文をBibgraph(ビブグラフ)で読む 上記論文の日本語要約 【背景】この試験では、過体重または肥満の2型糖尿病成人患者の体重管理を目的としたGLP-1アナログ製剤セマグルチド2.4mg、同1.0mg(糖尿病治療に承認された用量)またはプラセボの週1回皮下投与の有効性と安全性を評価した。 【方法】この第III相二重盲検ダブルダミー優越性試験では、スクリーニング180日以上前に2型糖尿病の診断を受けたBMI 27以上、糖化ヘモグロビン7~10%(53~86mmol/mol)の成人を登録した。欧州、北米、南米、中東、南アフリカおよびアジア12カ国の外来診療所149施設で患者を募集した。患者を自動ウェブ応答システム(IWRS)で無作為化し、基礎治療の血糖降下薬および糖化ヘモグロビンで層別化。セマグルチド2.4mg、同1.0mg、外見上見わけのつかないプラセボに(1対1対1の割合で)割り付け、週1回、68週間皮下投与し、生活習慣介入を実施した。患者、治験責任医師、結果の評価者に治療の割り付けを伏せた。主要評価項目は、治療意図に基づく評価で、プラセボと比較したセマグルチド2.4mg群の68週時の体重変化率と5%以上の減量達成の複合とした。試験薬を1回以上投与した患者全例で安全性を評価した。この試験は、ClinicalTrials.gov(NCT03552757)に登録されており、参加者の登録が終了している。 【結果】2018年6月4日から11月14日の間に1595例をスクリーニングし、そのうち1210例をセマグルチド2.4mg群(404例)、同1.0mg群(403例)、プラセボ群(403例)に割り付け、intention-to-treat解析の対象とした。ベースラインから68週時までの推定平均体重変化率は、セマグルチド2.4mg群-9.6%(SE 0.4)、プラセボ群-3.4%(同0.4)であった。プラセボ群と比較したセマグルチド2.4mg群の推定投与群間差は-6.2%ポイントだった(95%CI -7.3~-5.2、P<0.0001)。68週時、セマグルチド2.4mg群の方がプラセボ群よりも5%以上の減量を達成した患者が多かった(388例中267例[68.8%] vs. 376例中107例[28.5%]、オッズ比4.88、95%CI 3.58~6.64、P<0.0001)。セマグルチド2.4mg群(403例中353[87.6%])および1.0mg群(402例中329[81.8%])の方が、プラセボ群(402例中309[76.9%])よりも有害事象発生率が高かった。セマグルチド2.4mg群403例中256例(63.5%)、セマグルチド1.0mg群402例中231例(57.5%)、プラセボ群402例中138例(34.3%)に消化管系の有害事象が発現したが、ほとんどが軽度ないし中等度であった。 【解釈】過体重または肥満の2型糖尿病成人患者で、セマグルチド2.4mgを週1回投与によってプラセボと比較して効果的で臨床的に意義のある減量を達成した。 第一人者の医師による解説 セマグルチド 2.4mg/週投与は体重減少率および5%体重減の達成に有効 林 高則 医薬基盤・健康・栄養研究所 国立健康・栄養研究所臨床栄養研究部 栄養療法研究室室長/窪田 直人 東京大学医学部附属病院病態栄養治療部准教授 MMJ. August 2021;17(4):119 いくつかの糖尿病治療薬では体重増加をきたしやすいこともあり、血糖コントロールとともにいかに減量を達成していくかは、2型糖尿病治療において大きな課題である。 グルカゴン様ペプチド -1(GLP-1)受容体作動薬は血糖降下作用に加えて、減量効果も期待できる薬剤である。本論文は、肥満を有する2型糖尿病患者に対してGLP-1受容体作動薬であるセマグルチドを2.4mg週1回投与した際のプラセボまたはセマグルチド1.0mg投与(糖尿病治療として承認されている量)に対する有効性および安全性を検討したSTEP2試験の報告である。 対象は18歳以上、BMI 27kg/m2以上、HbA1c 7~10%の2型糖尿病患者1,210人で、上記3群に割り付けられ68週間の追跡が行われた。その結果、ベースラインからの体重減少率は2.4mg群で9.6%、1.0mg群で7.0%、プラセボ群で3.4%であり、また5%以上の減量を達成した割合は2.4mg群で68.8%、1.0mg群で57.1%、プラセボ群で28.5%と、いずれも2.4mg群で有意に大きかった。2.4mg群では、心血管危険因子(腹囲、収縮期血圧、脂質、尿中アルブミンなど)や身体機能評価スコア、QOL評価スコアの改善も認められた。有害事象の発現頻度は実薬群で多かったが(胃腸障害が最多)、そのほとんどは一過性かつ軽度~中等度であり、2.4mg群と1.0mg群で副作用による中止に差を認めなかった。 本試験ではセマグルチド 2.4mg投与により心血管危険因子の改善が認められたが、実際に心血管イベント発症を抑制するかは今後さらなる検証が必要である。この点に関しては、非糖尿病肥満者においてセマグルチド 2.4mgが心血管イベント発症を抑制するかどうかを検証するSELECT試験(1)が進行中であり、その結果も待たれる。 HbA1cに関しては、68週時点のベースラインからの変化量が2.4mg群で−1.6%、1.0mg群で−1.5%と差はわずかであったが、2.4mg群は1.0mg群と比べ併用薬が減った割合が高かったことも考慮して解釈する必要がある。 このSTPE2試験には日本を含む12カ国の施設が参加しており、研究参加者の26.2%がアジア人である。GLP-1受容体作動薬は非アジア人と比較してアジア人で血糖低下効果が高いこと(2)や、白人と比べアジア人でより主要心血管イベント発症抑制のベネフィットが大きいこと(3)が報告されており、日本人を含めたアジア人におけるセマグルチド 2.4mgの有効性が期待される。 1. Ryan Dh, et al. Am Heart J. 2020; 229: 61-69. 2. Kim YG, et al. Diabetes Obes Metab. 2014;16(10):900-909. 3. Matthew M Y Lee, et al. Diabetes Care. 2021; 44 (5) :1236-1241.
COVID-19患者の退院6カ月後の転帰 コホート研究
COVID-19患者の退院6カ月後の転帰 コホート研究
6-month consequences of COVID-19 in patients discharged from hospital: a cohort study Lancet. 2021 Jan 16;397(10270):220-232. doi: 10.1016/S0140-6736(20)32656-8. Epub 2021 Jan 8. 原文をBibgraph(ビブグラフ)で読む 上記論文の日本語要約 【背景】COVID-19の長期的な健康上の影響は、大部分が明らかになっていない。この研究の目的は、COVID-19患者の退院後の長期的な健康転帰を評価し、危険因子、特に疾患重症度関連のものを検討することであった。 【方法】2020年1月7日から5月29日の間に金銀潭病院(中国・武漢市)を退院したCOVID-19患者を対象に、前後両方向コホート研究を実施した。追跡調査前に死亡した患者、精神病性障害、認知症、再入院のため追跡調査ができなかった患者、骨関節症を合併し自由に動けない患者、脳卒中や肺塞栓症などの疾患により退院前後に動けなかった患者、試験への参加を拒否した患者、連絡が取れなかった患者、武漢市以外の地域に居住している患者や介護施設または福祉施設に居住している患者を除外した。全例に一連の質問票を用いた問診で症状および健康関連QOLを評価し、身体診察、6分間歩行テスト、血液検査を実施した。入院中に評価した7段階重症度尺度の最高点3点、4点、5~6点で層別化した手順を用いて、患者から検体を採取し、肺機能検査、胸部の高解像度CT、超音波検査を実施した。Lopinavir Trial for Suppression of SARS-CoV-2 in Chinaに参加した患者には、SARS-CoV-2抗体検査を実施した。多変量調整線形回帰モデルまたはロジスティック回帰モデルを用いて、重症度と長期的な健康転帰の関連を評価した。 【結果】退院したCOVID-19患者2469例から736例を除外し、計1733例を組み入れた。参加者の年齢中央値は57.0歳(IQR 47.0~65.0)であり、897例(52%)が男性であった。追跡調査は2020年6月16日から9月3日の間に実施し、発症後の追跡期間中央値は186.0日(175.0~199.0)日であった。よく見られた症状に、疲労または筋力低下(63%、1655例中1038例)、睡眠障害(26%、1655例中437例)があった。患者の23%(1617例中367例)から不安または抑うつが報告された。6分間歩行距離の中央値が正常範囲下限以下であった患者の割合は、重症度尺度3点の患者24%、4点の患者22%、5~6点の患者29%であった。これに対応する肺拡散障害の割合はそれぞれ22%、29%、56%であり、CTスコア中央値はそれぞれ3.0(IQR 2.0~5.0)、4.0(3.0~5.0)、5.0(4.0~6.0)であった。多変量調整後の重症度尺度3点と比較したオッズ比(OR)は、肺拡散障害で4点1.61(95%CI 0.80~3.25)、5~6点4.60(1.85~11.48)、不安または抑うつで4点0.88(0.66~1.17)、5~6点1.77(1.05~2.97)、筋力低下で4点0.74(0.58~0.96)、5~6点2.69(1.46~4.96)であった。追跡調査時に血中抗体検査を実施した94例の血清陽性率(96.2% vs. 58.5%)および中和抗体の力価中央値(19.0 vs. 10.0)は、急性期(入院時)に比べると著しく低かった。急性期に急性腎障害がなく推定糸球体濾過量(eGFR)が90mL/分/1.73m^2以上であった822例のうち、107例で追跡調査時のeGFRが90mL/分/1.73m^2未満であった。 【解釈】COVID-19生存者に、急性感染後6カ月時、主に疲労または筋力低下、睡眠障害、不安または抑うつ傾向が認められた。入院中の重症度が高かった患者は、肺拡散障害や胸部画像所見異常の重症度が高く、長期回復のための介入の主な対象者となる。 第一人者の医師による解説 6カ月後にも疲労・筋力低下、睡眠障害、不安・抑うつが残存 重症患者は回復後も介入が必要 葉 季久雄 平塚市民病院救急科・救急外科部長 MMJ. August 2021;17(4):105 2021年7月9日時点で、日本での新型コロナウイルス感染症(COVID-19)の感染者数は累計でおよそ81万人、回復者数は78万人、死亡者数は1万5,000人である(厚生労働省統計)。COVID-19回復後に続く後遺症はLong COVIDと称され、臨床家の注目を集めている。 本論文は、中国武漢市金銀潭病院を退院したCOVID-19患者1,733人を対象とした、発表時点で対象者数が最大かつ追跡期間が最長の前後両方向コホート研究の報告である。目的は退院患者の経過から残存症状を記述し危険因子を探ること、抗SARS-CoV-2抗体価の推移など肺外臓器機能の評価である。すべての対象患者に面接、診察、6分間徒歩テスト、血液検査、さらに7段階の重症度分類(#)で抽出した一部の患者に肺機能検査、高解像度胸部 CT、超音波検査が実施された。なお、抗体検査はロピナビルの臨床試験(LOTUS)参加者94人のみに行われた。 患者背景は、年齢中央値57歳、男性52%、観察期間中央値は発症後186日であった。患者のうち68%が入院中に酸素投与を必要とし、7%は高流量酸素療法(HFNC)、非侵襲的陽圧換気(NIV)または侵襲的機械換気(IMV)を必要とした。4%が集中治療室での治療を受けた。入院期間の中央値は14日であった。 発症6カ月後の主な残存症状は、疲労・筋力低下(63%)、睡眠障害(26%)、不安・抑うつ(23%)であった。1つ以上の症状を有した患者は76%で、女性の割合が高かった。重症度スケール 3の患者と比較し、重症度スケール 5~6の患者が発症6カ月後に何らかの症状を有するオッズ比は2.42(P<0.05)であった。肺拡散障害、不安・抑うつ、疲労・筋力低下のオッズ比は、スケール 5~6の患者はスケール 3の患者に比べ、それぞれ4.60(P=0.0011)、1.77(P=0.031)、2.69(P=0.0015)であった。女性は男性と比較し、それぞれ2.22(P=0.0071)、1.80(P<0.0001)、1.33(P=0.016)のオッズ比を示した。発症6カ月後、急性期と比較し、血清抗体陽性率は96.2%から58.5%へと低下し、中和抗体抗体価の中央値は19.0から10.0に低下していた。 本研究は、重症患者、女性患者は回復後も多彩な症状が残存し、治療介入を要することを示していた。退院が治療の終了ではなく、退院後も適切なフォローアップが必要である。中和抗体は発症6カ月後には減少しており、回復後の患者にも再感染のリスクがあることを念頭におく必要がある。COVID-19回復後の長期経過の全容解明には、より大規模で、より長期間に及ぶ追跡調査が求められる。 #脚注)重症度スケールの定義:スケール 6= 入院して ECMO、IMV もしくはその両方を要した。スケール 5= 入院して HFNC、NIV もしくはその両方を要した。スケール 3= 入院したが酸素投与は要さなかった。
早産予防に用いるプロゲステロンを評価する国際共同研究(EPPPIC) 無作為化試験から抽出した個別患者データのメタ解析
早産予防に用いるプロゲステロンを評価する国際共同研究(EPPPIC) 無作為化試験から抽出した個別患者データのメタ解析
Evaluating Progestogens for Preventing Preterm birth International Collaborative (EPPPIC): meta-analysis of individual participant data from randomised controlled trials Lancet. 2021 Mar 27;397(10280):1183-1194. doi: 10.1016/S0140-6736(21)00217-8. 原文をBibgraph(ビブグラフ)で読む 上記論文の日本語要約 【背景】早産は、優先して検討すべき世界的な健康問題である。早産リスクの高い妊娠中のプロゲステロン使用によって早産および新生児の有害転帰を抑制できると思われる。 【方法】早産リスクが高い無症候性の女性でプロゲステロン膣内投与、17-ヒドロキシプロゲステロンカプロン酸(17-OHPC)筋肉注射、経口プロゲステロンを対照またはそれぞれと比較した無作為化試験の系統的レビューを実施した。MEDLINE、Embase、CINAHL、Maternity and Infant Care Databaseの検索およびデータベース開始から2019年7月30日までの関連試験登録から、公開の有無を問わず2016年6月30日までに主要データの収集が完了した試験(データ収集開始12カ月前)を特定した。早期流産または差し迫った早産の危険を予防するプロゲステロンの試験を除外した。適格試験の担当医師に個別患者データの提供を依頼した。早産、早期早産および妊娠中期の出産を転帰とした。重篤な新生児合併症の複合および個別で早産による新生児有害転帰を評価した。複合および個別に有害妊娠転帰を調査した。研究者2人が個別患者データを確認し、バイアスリスクを評価した。主要メタ解析に、ランダム効果を統合した1段階の一般化線形混合モデルを用いて、試験間の異質性を考慮に入れた。このメタ解析は、PROSPERO(CRD42017068299)に登録されている。 【結果】初回検索で適格試験47件を特定した。このうち30件の個別患者データが得られた。対象とした更新があり、後日追加試験1件を組み入れた。従って、計31試験のデータが得られた(女性1万1644例および児1万6185例)。単胎妊娠を検討した試験に組み入れたのは、ほとんどが自然早産歴がある女性および子宮頸管長が短い女性であった。プロゲステロン膣内投与(9試験、女性3769例、相対リスク[RR]0.78、95%CI 0.68~0.90)、17-OHPC(5試験、女性3053例、0.83、0.68~1.01)および経口プロゲステロン(2試験、女性181例、0.60、0.40~0.90)を投与した女性で、34週未満の早産が減少した。その他の出産および新生児転帰の結果は一貫して良好であったが、信頼性が低かった。妊娠合併症リスク上昇の可能性が示唆されたが、不確かであった。治療の相互作用と患者背景に一貫した根拠は認められなかったが、下位集団の解析から、子宮頸管長が短くない女性では有効性がないことが疑われた。多胎妊娠を検討した試験に組み入れたのは、他の危険因子がない女性であった。プロゲステロン膣内投与(8試験、2046試験、RR 1.01、95%CI 0.84~1.20)で双胎妊娠女性の34週未満での早産が減少せず、17-OHPC(8試験、女性2253例、1.04、0.92~1.18)でも双胎および三胎妊娠で34週未満での早産が減少しなかった。多胎妊娠では、17-OHPC曝露で前期破水が増加した(34週未満の破水のRR 1.59、95%CI 1.15~2.22)が、プロゲステロン膣内投与でも17-OHPCでもその他の転帰に見られる便益や有害性の一貫した根拠は認められなかった。 【解釈】高リスクの単胎妊娠で、プロゲステロン膣内投与および17-OHPCによって34週未満での出産が減少した。潜在的リスクの上昇を考慮に入れると、子宮頸管長の短い女性で絶対リスクの低下度が大きいことから、このような女性では治療が有益であると思われる。経口プロゲステロンの使用を支持する根拠は不十分であった。高リスクの単胎妊娠女性との共同意思決定で、個別のリスク、考えられる便益と有害性および介入の実用性を話し合うべきである。この根拠から、任意に抽出した多胎妊娠でプロゲステロンによる治療は支持されない。 第一人者の医師による解説 日本で使用できるプロゲステロン製剤が課題 求められる国内での臨床研究 細谷聡史、左合治彦(副院長・周産期・母性診療センター長) 国立成育医療研究センター周産期・母性診療センター MMJ. August 2021;17(4):123 早産(妊娠22週以降37週未満の分娩)は妊娠中の最も頻度が高い最も重要な合併症である。早産児は呼吸障害や長期的な神経発達障害などの合併症をきたしやすく、早産を予防することは周産期医療の重要な課題である。内因性プロゲステロンは妊娠維持に関与し、低下すると陣痛発来することにより、1960年代から早産予防薬としてプロゲステロン製剤が用いられてきた。プロゲステロン製剤には天然型プロゲステロンを用いた腟製剤(錠剤とゲル)(VP)と合成化合物であるヒドロキシプロゲステロンカプロン酸エステル(17-OHPC)の筋肉注射製剤がある。2003年にはランダム化比較試験(RCT)で17-OHPCとVPの 早産予防効果が報告された(1),(2)。しかし、最近のOPPTIMUM試験とPROLONG試験という大規模 RCTでは、VPと17-OHPCの早産予防効果や児の予後改善効果は認められないと報告され(3),(4)、プロゲステロン製剤の効果に疑問が投げかけられた。 本論文は、早産リスクの高い妊婦(早産既往・子宮頸管長短縮)に対して早産予防効果を検証したRCTを対象とし、個別被験者データ(IPD)を集積してメタ解析を行い、VPと17-OHPCの早産予防効果を検証したものである。主要評価項目である単胎の34週未満の早産に関して、対照群と比較してVPは22%(相対リスク , 0.78;95%信頼区間 ,0.68~0.90)、17-OHPCは17%(0.83;0.68~1.01)のリスク低下を認め、早産予防効果の有効性を示した。ただし、頸管長が短縮していない(30mm超)妊婦では早産予防効果は認めなかった。母体合併症は増加する可能性が示唆された。また多胎妊娠に関して早産予防効果は認められなかった。以上の結果より、早産リスクが高く子宮頸管長短縮を認める単胎妊婦では、プロゲステロン製剤による早産予防効果が期待できるが、正確な情報を提供して個別のリスクとベネフィットを勘案した上での患者の意思決定に基づいて使用すべきとしている。 日本では17-OHPCは250mg/週筋注投与しているが、125mg/週のみが黄体機能不全に伴う切迫流早産を適応として保険収載されているだけである。またVPは生殖補助医療における黄体補充の適応で承認されているが薬価未収載で、切迫流早産は保険適応外である。日本ではVPの保険収載が大きな課題であり、そのための日本における質の高い臨床研究が求められている。 1. Meis PJ, et al. N Engl J Med. 2003;348(24):2379-2385. 2. da Fonseca EB, et al. Am J Obstet Gynecol 2003;188(2):419-424. 3. Norman JE, et al. Lancet. 2016;387(10033):2106-2116. 4. Blackwell SC, et al. Am J Perinatol. 2020;37(2):127-136.
思春期および若年成人期の1型糖尿病に用いるハイブリッド型クローズドループシステム2種の比較(FLAIR):多施設共同無作為化クロスオーバー試験
思春期および若年成人期の1型糖尿病に用いるハイブリッド型クローズドループシステム2種の比較(FLAIR):多施設共同無作為化クロスオーバー試験
A comparison of two hybrid closed-loop systems in adolescents and young adults with type 1 diabetes (FLAIR): a multicentre, randomised, crossover trial Lancet. 2021 Jan 16;397(10270):208-219. doi: 10.1016/S0140-6736(20)32514-9. 原文をBibgraph(ビブグラフ)で読む 上記論文の日本語要約 【背景】1型糖尿病の管理は困難である。著者らは、思春期および若年成人期の1型糖尿病患者を対象に、市販のハイブリッド型クローズドループシステムと開発中の新たなシステムを用いた結果を比較した。 【方法】この多施設共同無作為化クロスオーバー試験(Fuzzy Logic Automated Insulin Regulation[FLAIR])では、米国4施設、ドイツ、イスラエルおよびスロベニア各1施設の大学病院内分泌科で、1年以上前に1型糖尿病の臨床診断を受け、インスリンポンプまたは多数の1日1回インスリン注射を用いており、HbA1cが7.0~11.0%(53~97mmol/mol)の14~39歳の患者を募集した。試験に用いるポンプと持続グルコースモニタの使い方を指導する導入期間ののち、参加者をコンピュータが生成した数列を用いて、置換ブロックデザイン(ブロックの長さ2または4)で、治療前のHbA1cおよび登録時のMiniMed 670G system(Medtronic社)使用の有無で層別化した上で、最初の12週間をMiniMed 670G hybrid closed-loop system(670G)と開発中の高機能ハイブリッド型クローズドループシステム(Medtronic社)に(1対1の割合で)割り付け、その後の12週間をウォッシュアウト期間を設けずにもう一方のグループに交差させた。使用するシステムの性質上、遮蔽化は不可能であった。主要評価項目は、6時00分から23時59分(日中など)までの間に血糖値が180mg/dL(>10.0mmol/L)を超えた時間の割合および24時間のうち血糖値が54mg/dL(<3.0mmol/L)を下回った時間の割合とし、持続グルコースモニタで測定し、非劣性を評価した(非劣性のマージン2%)。intention to treatで解析することとした。治療を割り付けた患者全例で安全性を評価した。この試験はClinicalTrials.govにNCT03040414で登録されており、現在は終了している。 【結果】2019年6月3日から8月22日の間に113例を試験に組み入れた。平均年齢が19歳(SD 4)、70例(62%)が女性であった。日中の血糖値が180mg/dL(>10.0mmol/L)を超えた時間の平均割合が ベースラインで42%(SD 13)、670Gシステム使用中で37%(9)、高機能ハイブリッド型クローズドループシステム使用中で34%(9)であった(平均差[高機能ハイブリッド型クローズドループシステム-670Gシステム]-3.00%[95%CI -3.97~-2.04];P<0.0001)。24時間のうち血糖値が54mg/dL(<3.0mmol/L)を下回った時間の平均割合が試験開始前で0.46%(SD 42)、670Gシステム使用中で0.50%(0.35)、高機能ハイブリッド型クローズドループシステム使用中で0.46%(同0.33)であった(平均差[高機能ハイブリッド型クローズドループシステム-670Gシステム]-0.06%[95%CI ~0.11~-0.02];非劣性のP<0.0001)。高機能ハイブリッド型クローズドループシステム群で重篤な低血糖発作が1件発生したが、試験治療と関連がないと考えられ、670G群では1件もなかった。 【解釈】市販のMiniMed 670Gと比べると、開発中の高機能ハイブリッド型クローズドループシステムを用いた思春期および若年成人期の1型糖尿病患者で、低血糖発作が増えることなく高血糖が減少した。社会経済的因子のため十分なサービスを受けられていない集団や、妊婦、低血糖症状を自覚できない患者で高機能ハイブリッド型クローズドループシステムを検証すれば、この技術をさらに有効に活用することができるであろう。 第一人者の医師による解説 糖尿病合併症やQOLの改善など より長期の研究で検討する必要あり 長澤 薫 虎の門病院内分泌代謝科糖尿病・代謝部門特任医長 MMJ. October 2021;17(5):146 思春期や若年成人期の1型糖尿病患者の血糖コントロールは難易度が高く、やりがいのある課題である。本論文は、従来より使用されているハイブリッド型クローズドループシステム(HCLS;患者のグルコース値のアルゴリズムに基づき、ベーサルインスリンの投与量を調整するシステム)MiniMed670G(Medtronic社)と、現在開発中の次世代型のアドバンストハイブリッド型クローズドループシステム(AHCLS;従来の機能に加え、5分おきの自動修正ボーラスなど人工膵臓のアルゴリズムを用い、より強化されたインスリン調整機能が搭載されたシステム)(Medtronic社)の多施設共同無作為化クロスオーバー比較試験(FLAIR試験)の報告である。 米国、ドイツ、イスラエル、スロベニアの4カ国、計7つの専門施設で、診断後1年以上の14~29歳の1型糖尿病患者113人を対象とした。参加者のHbA1c値は7.0~11.0%で、ポンプの使用方法を習得するrun-in期間の後、初めにHCLSを使用する群とAHCLSを使用する群の2群に無作為に割り付け、12週間それぞれの機器を使用後、washout期間を設けずにクロスオーバーし、もう一方のインスリンポンプを12週間使用した。 主要評価項目は日中(6~24時)のグルコース値180mg/dL超、1日におけるグルコース値54mg/dL未満の時間の割合とされた(非劣性を検証、マージン 2%)。その結果、グルコース値180mg/dL超の時間はベースライン 42%であったが、HCLS使用期間は37%、AHCLS使用期間は34%と、HCLSに比べAHCLS使用期間では-3.0%(95%信頼区間[CI], -3.97~-2.04;P<0.0001)と高グルコース値の割合は有意に低下した。グルコース値54mg/dL未満の割合はベースライン 0.46%、HCLS使用期間は0.5%、AHCLS使用期間は0.46%と、AHCLS使用期間では-0.06%(95% CI, -0.11 ~-0.02;非劣性 P<0.0001)と有意な上昇は認められなかった。 AHCLS使用期間で1例の重症低血糖を認めたが、機器との関連はなかった。本試験はHCLSとAHCLSを直接無作為化クロスオーバーで比較した最初の論文で、AHCLSは従来型に比べ、低血糖を増やすことなく、有意に高血糖を減少させた。AHCLSのような進化したインスリン自動注入システムが高血糖、低血糖、自己管理の負担を減少させ、さらには糖尿病合併症、患者の生活の質(QOL)を改善するか否か、より長期の研究で検討する必要がある。実用化にあたっては適切なターゲット血糖値、アクティブインスリン(インスリンの作用時間)の設定など、さらなる議論も要するであろう。
中国・武漢の抗SARS-CoV-2抗体血清陽性率と体液性免疫の持続性:住民対象長期横断研究
中国・武漢の抗SARS-CoV-2抗体血清陽性率と体液性免疫の持続性:住民対象長期横断研究
Seroprevalence and humoral immune durability of anti-SARS-CoV-2 antibodies in Wuhan, China: a longitudinal, population-level, cross-sectional study Lancet. 2021 Mar 20;397(10279):1075-1084. doi: 10.1016/S0140-6736(21)00238-5. 原文をBibgraph(ビブグラフ)で読む 上記論文の日本語要約 【背景】武漢市は、中国で発生したCOVID-19大流行の中心地であった。著者らは、武漢市民の抗SARS-CoV-2抗体の血清陽性率と動態を明らかにし、ワクチン接種対策に役立てることを目的とした。 【方法】この長期横断研究では、多段階の人口層別型クラスター無作為標本抽出法を用いて、武漢市内13地区100地域を系統的に選択した。各地域から系統的に世帯を抽出し、全家族構成員に参加のため地域ヘルスケアセンターに来てもらった。2019年12月1日以降、武漢市に14日以上居住した住民を適格とした。参加に同意した全適格参加者が人口統計学および臨床的データに関するアンケートにオンラインで回答し、COVID-19に伴う症状やCOVID-19診断歴を自己申告した。2020年4月14~15日に免疫検査用に静脈血検体を採取した。血液検体でSARS-CoV-2ヌクレオカプシドタンパクに対する汎免疫グロブリン、IgM、IgA、IgG抗体の有無を検査し、中和抗体を評価した。2020年6月11日~13日、10月9日~12月5日に2回連続で追跡調査を実施し、その際に血液検体も採取した。 【結果】無作為に選択した4,600世帯のうち3,599世帯(78.2%)、計9,702例が初回評価のため来院した。3,556世帯9,542例から解析に十分な検体を得た。9,542例のうち532例(5.6%)がSARS-CoV-2に対する汎免疫グロブリン陽性で、この集団の調査開始時データで調整後の血清陽性率は6.92%(95%CI 6.41~7.43)であった。汎免疫グロブリンが陽性であった532例のうち437例(82.1%)が無症状であった。調査開始時、この532例のうち69例(13.0%)がIgM抗体陽性、84例(15.8%)がIgA抗体陽性、532例(100%)がIgG抗体陽性、212例(39.8%)が中和抗体陽性であった。汎免疫グロブリンが陽性で、4月に中和抗体が陽性を示した参加者の割合は、2回の経過観察の来院でも一定であった(2020年6月は363例中162例[44.6%]、2020年10~12月は454例中187例[41.2%])。全3回の調査に参加し汎免疫グロブリンが陽性であった335例のデータでは、調査期間中に中和抗体値の有意な減少は認められなかった(中央値:ベースライン1/5.6[IQR 1/2.0~1/14.0] vs 初回追跡調査1/5.6[1/4.0~1/11.2]、P=1.0、2回目追跡調査1/6.3[1/2.0~1/12.6]、P=0.29)。しかし、無症候性症例の方が確定症例や症候例症例よりも中和抗体価が低かった。時間の経過とともにIgG抗体価が低下したが、IgG抗体保有者の割合は大きく減少しなかった(確定症例でベースライン30例中30例[100%]から2回目追跡調査時29例中26例[89.7%]に減少、症候性症例で65例中65例[100%]から63例中58例[92.1%]に減少、無症候症例で437例中437例[100%]から362例中329例[90.9%]に減少)。 【解釈】武漢市の横断的標本の6.92%でSARS-CoV-2抗体が産生され、そのうち39.8%が中和抗体を獲得した。液性応答に関する耐久性データから、集団免疫を獲得し流行の再燃を防ぐには大規模ワクチン接種が必要であることが示唆される。 第一人者の医師による解説 流行収束後もワクチンによる集団免疫を付けることが 再流行を防ぐために必須 森内 浩幸 長崎大学大学院医歯薬学総合研究科小児科学教授 MMJ. October 2021;17(5):138 本論文の著者らは、中国における流行の中心であった武漢において経時的横断的研究を行い、多段階人口層化集落ランダム抽出法によって系統的に選ばれた世帯の成員に対して、人口統計学的データ、新型コロナウイルス感染症(COVID-19)関連症状の有無や診断歴を聴取するとともに、2020年4月、6月、および10~12月の間の3回にわたってSARS-CoV-2ヌクレオカプシド蛋白に対する抗体と中和抗体を測定した。 解析の対象となった3,556世帯の9,542人のうち532人(5.6%)が抗体陽性で、調整後の抗体保有率は6.92%(95%信頼区間 , 6.41~7.43)と推定された。陽性者の82.1%は無症状だった。また、抗体陽性者のうち4月の時点で39.8%、6月の時点で44.6%、最後の時点で41.2%が中和抗体も陽性で、その抗体価は期間中ほとんど減衰しなかった。無症状者は有症状者や診断確定者に比べ中和抗体価は低い傾向にあった。 この研究以前にも一般人口におけるSARSCoV-2抗体保有率の調査が行われている。例えばスイスの調査では、5歳以上の一般人口における診断確定例の11.6倍の抗体陽性者がいた(1)。米国の調査では1.0~6.9%の抗体保有率で、これは感染者の報告数の6~24倍に相当した(2)。アイスランドでは人口の0.9%が感染しており、抗体価は4カ月間で減衰しなかった(3)。武漢で以前行われた調査では成人の3.2%が抗体陽性だったが、統計解析のデザインは厳密なものではなかった。 どの地域のどのタイミングで調査が行われるかによって抗体保有率が異なるのは当然だが、一般人口における感染率を正しく捉えられる研究デザインだったか、無症状の感染者の割合がどれくらいだったか、そして抗体価の経時的推移がどうだったかについて、これまでの調査では十分に捉えられていなかった。今回の武漢における研究では、抽出法を工夫して一般人口を反映させ、かつ縦断的にフォローすることで武漢における流行が残した集団免疫の程度を明らかにすることができた。 この研究が意味することは、大きな流行が駆け抜けた地域においても住民の多くは感受性を持ったままであり、再び流行が起こるのを阻止するためにはワクチンによって集団免疫を構築すべきだということだ。また、不顕性感染の割合が非常に高かったことも、予防対策上重要な知見と思われる。 1. Stringhini S, et al. Lancet. 2020;396(10247):313-319. 2. Havers FP, et al. JAMA Intern Med. 2020.;180(12):1576-1586. 3. Gudbjartsson DF, et al. N Engl J Med. 2020;383(18):1724-1734.
重症急性腎障害に用いる2通りの腎代替療法開始遅延戦略の比較(AKIKI 2試験):多施設共同非盲検無作為化対照試験
重症急性腎障害に用いる2通りの腎代替療法開始遅延戦略の比較(AKIKI 2試験):多施設共同非盲検無作為化対照試験
Comparison of two delayed strategies for renal replacement therapy initiation for severe acute kidney injury (AKIKI 2): a multicentre, open-label, randomised, controlled trial Lancet. 2021 Apr 3;397(10281):1293-1300. doi: 10.1016/S0140-6736(21)00350-0. 原文をBibgraph(ビブグラフ)で読む 上記論文の日本語要約 【背景】重度の合併症がない重症急性腎障害で腎代替療法(RRT)開始をある程度遅らせることは安全であり、医療機器の使用が最適化できる。リスクがなくRRTを延期できる期間についてはいまだに明らかになっていない。本試験の目的は、開始をさらに遅らせることでRRTを受けない日数が長くなるという仮説を検証することであった。 【方法】本試験は、フランスの39の集中治療室で実施された多施設共同非遮蔽前向き非盲検無作為化対照試験であった。重症急性腎障害(Kidney Disease: Improving Global Outcomesステージ3と定義)の重篤患者を乏尿が72時間を超えるまで、または血中尿素窒素濃度が112mg/dLを超えるまでモニタリングした。その後、患者を直後にRRTを開始する戦略(遅延戦略)と長期遅延戦略に(1対1の割合で)割り付けた。長期遅延戦略では、必須の適応症(顕著な高カリウム血症、代謝性アシドーシスまたは肺水腫)が生じるか血中尿素窒素濃度が140mg/dLに達するまでRRTの開始を延期した。主要評価項目は、無作為化から第28日まで生存しRRTを受けなかった日数とし、intention-to-treat集団で評価した。本試験はClinicalTrial.govに登録され(NCT03396757)、終了している。 【結果】2018年5月7日から2019年10月11日までの間に、評価した5,336例のうち278例を無作為化し、137例を遅延群、141例を長期遅延群に割り付けれた。急性腎障害またはRRT関連の可能性がある合併症数は、両群同等であった。RRTを受けなかった日数中央値は、遅延群では12日(IQR 0~25)、長期遅延群では10日(IQR 0~24)であった(P=0.93)。多変量解析で、遅延群に対する長期遅延群の60日時の死亡のハザード比は1.65(95% CI 1.09~2.50、P=0.018)であった。急性腎障害またはRRTに関連する可能性のある合併症の数には両群間で差がなかった。 【解釈】乏尿が72時間を超えるか血中尿素窒素濃度が112mg/dLを超える重症急性腎障害患者で、即時RRTを要する重度合併症がない場合、RRT開始を長く延期しても便益は得られず、有害となる可能性がある。 第一人者の医師による解説 エビデンスの蓄積で透析開始基準のより明確化を期待 根本佳和(助教)/寺脇博之(教授) 帝京大学ちば総合医療センター第三内科(腎臓内科) MMJ. October 2021;17(5):147 急性腎障害(AKI)における緊急透析の適応基準は存在するものの、それに該当しない重症患者に対する腎代替療法(RRT)開始タイミングについては議論が分かれる。「透析はまだ待てる」、逆に「もう少し早くに連絡して欲しかった」とコンサルテーションを依頼した専門医に言われ、一体どのタイミングが正解だったのかと思い悩んだ医師も多いのではないだろうか。本論文はこの、「透析はどこまで待つか」というシンプルだがいまだ答えが曖昧な疑問に対する重要なエビデンスである。 著者らはフランスの39施設の集中治療室(ICU)において多施設共同ランダム化対照試験を実施した。重症 AKI(Kidney Disease: Improving Global Outcomes[KDIGO]分類のstage3)患者を、72時間以上の乏尿または尿素窒素(BUN)112 mg/dL超になるまでモニターした後、遅延群(delayed strategy:ランダム化直後にRRTを開始)と長期遅延群(more-delayed strategy:いわゆる緊急導入徴候[高カリウム血症・代謝性アシドーシス・肺水腫]の出現、またはBUN140mg/dL超でRRTを開始)の2群にランダムに割り付けた。評価対象となった患者は5,336人であり、そのうち278人がランダム化を受け、137人が遅延群、141人が長期遅延群に割り付けられた。主要評価項目のRRT-free daysは生存とRRTの期間の複合アウトカムであり、ランダム化から28日目までの期間において生存患者でRRTを実施しなかった日数がカウントされた。結果、RRT-free days中央値に関して遅延群と長期遅延群で有意差はなかった(12日対10日;P=0.93)。また、副次評価項目の1つである60日後の死亡率は、遅延群44%、長期遅延群55%と有意差はなかったが(P=0.071)、長期遅延群は60日後の死亡に関する有意な危険因子であることが多変量解析で示された(ハザード比,1.65;95%信頼区間,1.09〜2.50;P=0.018)。すなわち、透析開始を必要以上に遅延させることの有益性はなく、むしろ有害性と関連している、と著者らは結論づけている。 日本の臨床では、本試験の長期遅延群に該当するまでRRT開始を延期することは少ないと思われる。遅延群もしくはそれ以前の段階において緊急導入徴候が出現した際にRRTを開始することが多いのではないだろうか。著者のGaudryらは、AKIにおけるRRTの早期導入と晩期導入に関するメタ解析で、28日全死亡率に有意差がなかったとも報告している(1)。RRT開始は『早すぎず遅すぎず』のスタンスで良いことはわかってきたが、本論文のようなエビデンスがさらに蓄積されることで開始基準がより明確化されることに期待したい。 1. Gaudry S, et al. Lancet. 2020;395(10235):1506-1515. (MMJ 2020年12月号で紹介)
PCI実施患者に用いる個別化抗血小板療法と標準抗血小板療法の比較:システマティック・レビューとメタ解析
PCI実施患者に用いる個別化抗血小板療法と標準抗血小板療法の比較:システマティック・レビューとメタ解析
Guided versus standard antiplatelet therapy in patients undergoing percutaneous coronary intervention: a systematic review and meta-analysis Lancet. 2021 Apr 17;397(10283):1470-1483. doi: 10.1016/S0140-6736(21)00533-X. 原文をBibgraph(ビブグラフ)で読む 上記論文の日本語要約 【背景】経皮的冠インターベンション(PCI)を受ける患者に用いる個別化抗血小板療法は標準療法と比べて転帰が良好であるかは議論の余地がある。そこで、PCI実施患者で、個別化抗血小板療法と標準的抗血小板療法の安全性および有効性を評価した。 【方法】このシステマティック・レビューおよびメタ解析では、2020年8月20日から10月25日にかけて、MEDLINE(PubMed)、Cochrane、EmbaseおよびWeb of Scienceの各データベースで、PCIを受ける患者を対象に、血小板機能検査または遺伝子検査を用いる個別化抗血小板療法と標準的抗血小板療法を比較した無作為化比較試験および観察研究を言語を問わず検索した。レビューアー2名が個別に試験の適格性を評価し、データを抽出し、バイアスリスクを評価した。I2指標で評価した試験間の推定異質性によって、リスク比(RR)と95%CIにランダム効果または固定効果モデルを用いた。試験で定義した主要な主要有害心血管系事象および全出血を主要評価項目とした。全死因死亡、心血管系死、心筋梗塞、脳卒中、ステント血栓症(確定または疑い)、大出血および軽度出血を主な副次評価項目とした。この試験は、PROSPERO(CRD42021215901)で登録されている。 【結果】関連の可能性がある3,656報をスクリーニングした。患者2万743例のデータを含む無作為化比較試験11報および観察研究3報を解析の対象とした。標準療法と比べると、抗血小板療法の個別化選択で、統計的有意ではないものの(RR 0.88、0.77~1.01、P=0.069)、主要有害心血管事象(RR 0.78、95%CI 0.63~0.95、P=0.015)および出血が減少した。標準療法と比べると、個別化抗血小板療法では心血管死(RR 0.77、95%CI 0.59~1.00、P=0.049)、心筋梗塞(RR 0.76、0.60~0.96、P=0.021)、ステント血栓症(RR 0.64、0.46~0.89、P=0.011)、脳卒中(RR 0.66、0.48~0.91、P=0.010)、軽度出血(RR 0.78、0.67~0.92、P=0.0030)が少なかった。個別化抗血小板療法と標準的抗血小板療法で、全死因死亡と大出血のリスクに差はなかった。戦略によって転帰にばらつきが見られ、escalation法では安全性を損なうことなく虚血性事象が有意に減少し、de-escalation法では有効性を損なうことなく出血が有意に減少した。 【解釈】個別化抗血小板療法によって良好な安全性が保たれたまま複合評価項目および各有効性評価項目が改善し、軽度出血の出血も減少した。PCIを受ける患者の薬剤選択を最適化するため、血小板機検査または遺伝子検査を用いることが支持される。 第一人者の医師による解説 抗血小板療法のオーダーメード化には 日本でもRCTによる費用対効果分析が必要 河村 朗夫 国際医療福祉大学医学部循環器内科主任教授 MMJ. October 2021;17(5):143 冠動脈インターベンション(ステント留置など)における抗血小板療法は、死亡、心筋梗塞などの血栓性合併症を予防する優れた効果を示す一方、ときに重篤な出血性合併症を伴う。血栓性合併症と出血性合併症の双方を予防することはしばしば二律背反となる。これらの合併症の頻度は、抗血小板療法の内容のみならず、患者の体格、人種、性別などさまざまな要素により規定されるため、個々の患者における最適な抗血小板療法を決定することは重要な課題である。 本論文は、11件の無作為対照試験(RCT)と3件の観察研究から得た20,743人のデータを用いた、これまでで最大規模となるメタアナリシスの報告である。患者の血小板機能検査、あるいはクロピドグレルの代謝に影響を及ぼすことが知られているCYP2C19の遺伝子多型検査のいずれかを用いて、抗血小板療法を患者ごとに強化あるいは減弱することで、主要心血管イベントと出血を予防することができるかどうかが検証された。抗血小板療法の強化が行われた10件の研究では以下のいずれかが行われた。すなわち、クロピドグレルをプラスグレルかチカグレロルへ変更、クロピドグレルの倍量投与、あるいはシロスタゾールの追加である。一方、抗血小板療法を減弱した4件の研究では、プラスグレルやチカグレロルがクロピドグレルへ変更された。その結果、少なくとも6カ月以上の追跡期間において、個別化抗血小板療法は主要心血管イベント(相対リスク[RR],0.78;P=0.015)と、統計学的有意差には至らないものの出血も減少させた(RR, 0.88;P=0.069)。 今回のメタアナリシスには、アジア(中国、韓国)から発表されたRCTが4件、観察研究が2件含まれている。これらの報告ではいずれも抗血小板療法の強化が行われているのが興味深い。アジア人ではCYP2C19の活性が欠損する遺伝子多型の頻度が高く、クロピドグレルが活性体にならず効果が低下する可能性があることと関連していると思われるが、日本でも同様の結果が得られるのかどうかはいまだ議論の余地がある。本論文の対象研究で用いられた血小板機能測定や遺伝子多型検査は、日常診療では行われていない。最新の日米欧のガイドラインでもこれらの検査をルーチンに行うことは推奨されていない。今後さらなる知見が得られると見込まれるが、検査に要する時間や費用を考慮すると、費用対効果分析が欠かせない。
男性パートナーの総精子数および精子運動率が正常な不妊カップルに用いる卵細胞質内精子注入法と標準体外受精の比較:非盲検無作為化比較試験
男性パートナーの総精子数および精子運動率が正常な不妊カップルに用いる卵細胞質内精子注入法と標準体外受精の比較:非盲検無作為化比較試験
Intracytoplasmic sperm injection versus conventional in-vitro fertilisation in couples with infertility in whom the male partner has normal total sperm count and motility: an open-label, randomised controlled trial Lancet. 2021 Apr 24;397(10284):1554-1563. doi: 10.1016/S0140-6736(21)00535-3. 原文をBibgraph(ビブグラフ)で読む 上記論文の日本語要約 【背景】卵細胞質内精子注入法の使用は世界で大幅に増加している。しかし、このアプローチを標準体外受精(IVF)と比較した無作為化比較試験のデータが不足している。そこで、卵細胞質内精子注入法が標準IVFと比較して生産率が高いかを明らかにすることを目的とした。 【方法】この非盲検多施設共同無作為化試験は、ベトナム・ホーチミン市のIVFセンター2施設(IVFMD、My Duc HospitalおよびIVFAS、An Sinh Hospital)で実施された。男性パートナーの精子数および精子運動率(直進運動)が2010年のWHO基準から見て正常な18歳以上のカップルを適格とした。標準IVFまたは卵細胞質内精子注入法による治療歴が2回以下であり、卵巣刺激にアンタゴニスト法を用いており、胚移植数が2個以下であることとした。ブロックサイズが2、4または8のブロック置換法および電話による中央無作為化法を用いて、カップルを卵細胞質内精子注入法と標準IVFに(1対1の割合で)割り付けた。コンピュータ生成無作為化リストは、試験に関与していない独立の統計家が用意した。介入法および病院での支払いに差があるため、胚培養士およびカップルに試験群を伏せなかったが、胚移植を実施する臨床医には試験群の割り付けを伏せた。主要評価項目は、初回採卵周期で得た初回胚移植後の生産率とした。intention-to-treat集団で解析した。この試験はClinicalTrials.gov,にNCT03428919として登録されている。 【結果】2018年3月16日から2019年8月12日までの間に、1,064組を卵細胞質内精子注入法(532組)と標準IVF(532組)に割り付けた。卵細胞質内精子注入法に割り付けたカップル532組中284組(35%)および標準IVFに割り付けたカップル532組中166組(31%)が初回採卵周期で得た初回胚移植後に生児を出生した(絶対差3.4%、95%CI -2.4~9.2、リスク比[RR]1.11、95%CI 0.93~1.32;P=0.27)。卵細胞質内精子注入法群の29組(5%)と標準IVF群の34組(6%)で受精が失敗した(絶対差-0.9%、RR 0.85、95%CI 0.53~1.28;P=0.60)。 【解釈】男性パートナーの総精子数および運動率が正常な不妊カップルで、卵細胞質内精子注入法の生産率に標準IVFと比べて改善が見られなかった。この結果は、この集団に用いる生殖補助技術として卵細胞質内精子注入法のルーチンの使用を再考する必要性を示すものである。 第一人者の医師による解説 男性不妊因子のないカップルへの顕微授精は再考が必要 通常の体外受精で対応可 丸山 哲夫 慶應義塾大学医学部産婦人科学教室准教授 MMJ. October 2021;17(5):155 健常と思われる単一の精子を卵子に注入して受精卵を作成する顕微授精(ICSI)は、精液所見が不良のために通常の体外受精(cIVF)では妊娠が困難な不妊カップルを治療する目的で1990年代に開発された。本技術はこの約20年間世界中で広く用いられ、精液所見による男性不妊因子の割合はほぼ一定であるにもかかわらず、ICSI実施件数は増加の一途をたどっている(1)。この増加は、男性不妊因子のない不妊カップルに実施される割合が大幅に高まっていることに起因し、米国では1996年の15.4%から2012年には66.9%へと上昇した(2)。このような本来の目的以外でICSIが用いられる背景には、確実に受精させることで受精卵を効率的に増やし、生児が得られる確率を高めるという考えがある。しかし、その考えを裏付ける確かなエビデンスはこれまで得られていない。男性不妊因子のない不妊カップルを対象にICSIとcIVFを比較したランダム化試験(3)は報告されている。主要評価項目である着床率はcIVFの方が高かったが(30%対22%)、統計学的検出力が不十分であり、不妊カップルにとって最も重要な関心事である生児獲得率(生産率)のデータがないことから、これらの諸問題を解決する新たなランダム化試験が望まれていた。 今回のランダム化非盲検対照試験は、2018〜19年にベトナムのIVFセンター2施設で行われた。世界保健機関(WHO)2010基準で総精子数および精子運動率が正常、過去のcIVFまたはICSIの治療歴は2回以下などを組み入れ条件とし、卵巣刺激はアンタゴニスト法で移植胚数は2個以下と設定した。cIVFとICSIの介入方法と治療コストは両者で明らかに異なるので、胚培養士および対象カップルへの盲検化は不可のため非盲検となった。主要評価項目は、初回採卵周期で得られた最初の胚の移植での生産率とされた。6,440組の不妊カップルを絞り込んでいった結果、最終的にICSI群に532組、cIVF群に532組が割り当てられた。その結果、生産率は、ICSI群で35%、cIVF群で31%で、両群間に有意差は認められなかった。受精失敗率についても両群間で有意差はなかった(5%対6%)。 本研究の結果から、男性不妊因子のない不妊カップルにICSIを行っても生産率が向上することはなく、昨今の男性不妊因子を考慮しないICSIのルーチン的な使用については再考する必要性が示された。 1. Zagadailov P, et al. Obstet Gynecol. 2018;132(2):310-320. 2. Boulet SL, et al. JAMA. 2015;313(3):255-263. 3. Bhattacharya S, et al. Lancet. 2001;357(9274):2075-2079.
虚血性脳卒中後の上肢機能障害に用いるリハビリと迷走神経刺激(VNS-REHAB):無作為化盲検ピボタルデバイス試験
虚血性脳卒中後の上肢機能障害に用いるリハビリと迷走神経刺激(VNS-REHAB):無作為化盲検ピボタルデバイス試験
Vagus nerve stimulation paired with rehabilitation for upper limb motor function after ischaemic stroke (VNS-REHAB): a randomised, blinded, pivotal, device trial Lancet. 2021 Apr 24;397(10284):1545-1553. doi: 10.1016/S0140-6736(21)00475-X. 原文をBibgraph(ビブグラフ)で読む 上記論文の日本語要約 【背景】虚血性脳卒中後に長期的な上肢機能障害がよく起こるが、リハビリテーションと迷走神経刺激の組み合わせによって改善すると思われる。著者らは、この方法が脳卒中後の上肢障害改善に安全で有効な治療であることを明らかにすることを目的とした。 【方法】英国および米国の脳卒中リハビリテーション施設19箇所で実施されたこのピボタル無作為化三十盲検シャム対象試験は、虚血性脳卒中から9カ月以上経過し、中等度ないし重度の上肢機能障害が残る患者をリハビリテーション+迷走神経刺激(VNS群)とリハビリテーション+シャム刺激(対照群)に(1対1の割合で)割り付けた。ResearchPoint Global社(米テキサス州オースティン)がSAS PROC PLAN(米SAS Institute Software社)を用いて無作為化を実施し、地域(米国 vs 英国)、年齢(30歳以下 vs 30歳超)、治療開始前のFugl-Meyer Assessment-Upper Extremity(FMA-UE)スコア(20~35点 vs 36~50点)で層別化した。参加者、評価者および治療実施者に割り付けを伏せた。全参加者に迷走神経刺激装置を留置した。VNS群は、0.8mA、100μs、30Hzの刺激を0.5秒間受けた。対照群は、0mAの刺激を受けた。参加者は、6週間にわたり施設内で治療を受けた後(1週間に3回、計18回)、自宅で運動プログラムを継続した。主要評価項目は、施設内での治療完了初日の障害の変化とし、FMA-UEスコアで測定した。施設内治療完了から90日後にもFMA-UEの奏効率を評価した(副次評価項目)。全解析はintention-to-treatで実施した。この試験は、ClinicalTrials.govにNCT03131960として登録されている。 【結果】2017年10月2日から2019年9月12日までの間に、108例を無作為化により割り付けた(VNS群53例、対照群55例)。106例が試験を完遂した(各群1例が脱落)。施設内治療完了初日、平均FMA-UEスコアはVNS群では5.0点(SD 4.4)、対照群では2.4点(3.8)上昇した(群間差2.6点、95%CI 1.0~4.2、P=0.0014)。施設内での治療から90日後、VNS群53例中23例(47%)、対照群55例中13例(24%)がFDA-UEスコアの臨床的に意義のある効果を達成した(群間差24%、6~41、P=0.0098)。対照群に手術関連の重篤な有害事象が1件発生した(声帯麻痺)。 【解釈】リハビリテーションと組み合わせた迷走神経刺激は、虚血性脳卒中後の中等度ないし重度上肢機能障害の新たな治療選択肢となる可能性がある。 第一人者の医師による解説 対象患者の障害程度の見極めと 治療の侵襲性と介入時期等についての議論が必要 赤倉 奈穂実/早乙女 貴子(医長) 東京都立神経病院リハビリテーション科/髙橋 一司 東京都立神経病院院長 MMJ. October 2021;17(5):141 虚血性脳卒中後に多くの患者で上肢機能障害が残存することは知られているが、これまでに上肢機能障害に対する効果が報告された治療法はわずかである。 脳卒中後の脳神経細胞には可塑性があることが指摘されている。迷走神経刺激(VNS)は皮質全体でアセチルコリンやノルエピネフリンなどの可塑性を促進する神経調節物質の放出を引き起こす。VNSを運動と同期的に行うことでシナプス再編成と残存神経の動員を促し、上肢の運動機能を回復させることが、動物実験で示されている(1),(2)。 本論文は、脳卒中後遺症のある患者を対象にVNS治療を英国と米国の19の脳卒中リハビリテーション施設で実施した無作為化三重盲検比較試験の報告である。年齢22~80歳、発症後9カ月~10年、中等度~重度(Fugl-Meyer Assessment-Upper Extremity[FMA-UE]スコア,20〜50点[最高得点は66点])の上肢機能障害を有する片側テント上虚血性脳卒中患者108人にVNS装置の植込み術を行った後、VNS刺激+リハビリテーション(VNS群53人)または偽刺激+リハビリテーション(対照群55人)のいずれかを週3回・6週間施設内で実施、その後自宅での運動プログラムを継続した。リハビリテーションプログラムは、リーチと把握、物体の裏返し、食事動作などの患者ごとに個別化した難易度の課題を反復して行った。 プログラム終了時の評価では、FMA-UEスコアの平均値は、ベースラインに比べ、VNS群で5.0点、対照群2.4点改善し、2群間に有意差が認められた。プログラム終了後90日目にFMA-UEスコアで臨床的に意義がある6ポイント以上の改善が得られたのは、VNS群では23/53人(47%)、対照群では13/55人(24%)であり、2群間の差は有意であった。手術に関連した重篤な有害事象は、対照群で1件(声帯麻痺)であり、これはてんかんやうつ病に対するVNS治療でみられる頻度と相違なかった。著者らはVNSが脳卒中後遺症としての上肢機能障害を改善させる新しい戦略になりうると結論付けている。 この治療法の課題として、運動神経回路の回復には上肢の運動が必要であり、対象となる患者の障害の程度を見極める必要があることや、介入時期、治療の侵襲性についても、さらなる議論が必要である。脳卒中後の中等度〜重度の上肢機能障害に対するリハビリテーションとVNSの組み合わせは新規治療として可能性を秘めている。 1. Engineer ND, et al. Front Neurosci. 2019;13:280. 2. Meyers EC, et al. Stroke. 2018;49(3):710-717.
前立腺がんの救済放射線治療の決定に用いる18F-フルシクロビンPET/CT検査と従来の画像検査単独の比較:単一施設、非盲検、第II/III相無作為化比較試験
前立腺がんの救済放射線治療の決定に用いる18F-フルシクロビンPET/CT検査と従来の画像検査単独の比較:単一施設、非盲検、第II/III相無作為化比較試験
18 F-fluciclovine-PET/CT imaging versus conventional imaging alone to guide postprostatectomy salvage radiotherapy for prostate cancer (EMPIRE-1): a single centre, open-label, phase 2/3 randomised controlled trial Lancet. 2021 May 22;397(10288):1895-1904. doi: 10.1016/S0140-6736(21)00581-X. Epub 2021 May 7. 原文をBibgraph(ビブグラフ)で読む 上記論文の日本語要約 【背景】前立腺がんの治療の決定や計画分子イメージングを用いることが多くなっている。著者らは、救済放射線治療の癌制御率改善に果たす18F-フルシクロビンPET/CT検査の役割を従来の画像検査(骨シンチグラフィとCT検査またはMRI検査)を比較することを目的とした。 【方法】単一施設非盲検第II/III相無作為化試験、EMPIRE-1では、前立腺摘除後に前立腺特異抗原(PSA)が検出されたが従来の画像検査結果で陰性(骨盤外転移、骨転移なし)であった前立腺がん患者を放射線治療決定に従来の画像検査単独に用いるグループと放射線治療+18F-フルシクロビン-PET/CT検査を用いるグループに割り付けた。コンピュータが生成した無作為化をPSA濃度、異常が示唆される病理学的所見およびアンドロゲン除去療法の意図で層別化した。18F-フルシクロビン-PET/CT検査群では、標的の描写にも用いたPET画像で放射線治療を厳格に決定した。主要評価項目は3年無事象生存率とし(生化学的再発または進行、臨床的再発または進行、全身療法の開始を事象と定義)、放射線治療を受けた患者で単変量解析および多変量解析を実施した。この試験は、ClinicalTrials.govにNCT01666808として登録されており、患者の登録が終了している。 【結果】2012年9月18日から2019年3月14日にかけて165例を無作為化により割り付け、追跡期間が中央値3.52年(95%CI 2.98~3.95)となった。PET検査の結果から、18F-フルシクロビン-PET-CT検査群の4例が放射線治療を回避し、この4例は生存解析から除外した。生存期間中央値は、従来検査群(95% CI 35.2~未到達;81例中33%に事象発生)、18F-フルシクロビン-PET/CT検査群(95%未到達~未到達;76例中20%に事象発生)ともに未到達であり、3年無事象生存率が従来検査群63.0%(95%CI 49.2~74.0)、18F-フルシクロビン-PET-CT検査群75.5%(95%CI 62.5~84.6)であった(差difference 12.5; 95%CI 4.3~20.8;P=0.0028)。調整した解析で、試験群(ハザード比2.04[95%CI 1.06~3.93]、P=0.0327)に無事象生存との有意な関連が見られた。両群の毒性がほぼ同じであり、最も多い有害事象が遅発性頻尿および尿意切迫感(従来検査群81例中37例[46%]、PET群76例中31例[41%])および急性下痢(11例[14%]、16例[21%])であった。 【解釈】前立腺摘除後の救済放射線治療の方針決定や計画に18F-フルシクロビン-PET/CT検査を用いることによって生化学的再発や持続のない生存率が改善した。前立腺がん放射線治療の方針決定や計画に新たなPET放射性核種を組み込むことについて、新たな試験で検討する必要がある。 第一人者の医師による解説 新しいPET放射性核種を使用した治療決定や治療計画を期待 吉田 宗一郎 東京医科歯科大学大学院医歯学総合研究科腎泌尿器外科学講師 MMJ. October 2021;17(5):148 前立腺がんに対する前立腺全摘除後の放射線治療は、術後追加治療、または生化学的再発が認められた際の救済治療として行われることが多い。これらの放射線治療を行うかどうか、またいつ行うかの判断は、リスク群や病理所見、術後前立腺特異抗原(PSA)の推移などに応じて検討されている。18 F-フルシクロビン -PET/CTは、生化学的に再発した前立腺がんの再病期診断において、CTやMRIよりも優れた診断性能を有し、前立腺全摘除術後の生化学的再発に対し、3分の1以上の患者で、18 F-フルシクロビン -PET/CTにより救済放射線治療の方針が変更になることが報告されている(1)。 今回報告されたEMPIRE-1試験の目的は、前立腺全摘除後にPSA上昇が検知されるも、従来の画像検査が陰性である患者を対象とした単施設、非盲検、第2/3相無作為化試験により、18 F-フルシクロビン -PET/CTが、3年間の無イベント生存率を改善させるかどうかを明らかにすることである。対象患者は、PSA値、病理組織学的所見、ホルモン療法実施の意図で層別化され、従来の画像診断のみで行う放射線治療群、もしくは従来の画像診断に加え18 F-フルシクロビン -PET/CTを併用する放射線治療群に割り付けられた。主要評価項目は3年無イベント生存率で、イベントの定義は生化学的または臨床的な再発・進行、あるいは全身療法の開始とした。結果として、165人の患者が割り付けられ、追跡期間の中央値は3.52年であった。3年無イベント生存率は、従来の画像診断群の63.0%に対し、18 F-フルシクロビン -PET/CT群では75.5%と有意に高かった。調整後解析では、18 F-フルシクロビン -PET/CTの併用が無イベント生存率と有意に関連していた(ハザード比 , 2.04;95%信頼区間 ,1.06?3.93)。毒性は両群でほぼ同様であり、主な有害事象は遅発性の頻尿・尿意切迫感、急性下痢であった。 これまでも新規 PET放射性核種による診断精度や治療方針決定の変化についての検討が行われてきたが、今回の制がん効果を主要評価項目とした初めての前向き無作為化試験によって、前立腺全摘除術後の放射線治療の決定プロセスにおける18 Fフルシクロビン -PET/CTの導入が無イベント生存率を改善する可能性が示唆された。現在、多くの研究により前立腺特異的膜抗原を標的としたPSMAPETの良好な診断精度が示され、前立腺全摘除後の再発巣検知でもその有効性に大きな関心が寄せられている。今後、新しいPET放射性核種を使用した治療決定や治療計画についてさらなる研究が必要である。 1. Abiodun-Ojo OA, et al. J Nucl Med. 2021;62(8):1089-1096
抗がん剤治療に用いる中心静脈アクセスデバイス(CAVA試験):無作為化対照試験
抗がん剤治療に用いる中心静脈アクセスデバイス(CAVA試験):無作為化対照試験
Central venous access devices for the delivery of systemic anticancer therapy (CAVA): a randomised controlled trial Lancet. 2021 Jul 31;398(10298):403-415. doi: 10.1016/S0140-6736(21)00766-2. Epub 2021 Jul 21. 原文をBibgraph(ビブグラフ)で読む 上記論文の日本語要約 【背景】中心静脈を介した全身性抗がん剤治療(SACT)の送達に、Hickman型トンネルカテーテル(Hickman)、末梢挿入型中心静脈カテーテル(PICC)、完全埋め込み型ポート(PORT)が使用されている。この3つのデバイスで合併症の発生率とコストを比較し、SACTを受ける患者の受容性、臨床効果および費用効果を明らかにすることを目的とした。 【方法】3つの中心静脈アクセスデバイスを検討する非盲検多施設共同無作為化対照試験(Cancer and Venous Access[CAVA])を実施し、PICCとHickman(非劣性マージン10%)、PORTとHickman(優越性マージン15%)、PORTとPICC(優越性マージン15%)を比較した。英国内18カ所の腫瘍内科で、固形がんまたは血液がんのためSACT(12週以上)を受ける成人(18歳以上)を対象とした。無作為化の選択肢を4つ用意し、ヒックマン、PICC、PORT(2対2対1)、PICCとヒックマン(1対1)、PORTとヒックマン(1対1)、PORTとPICC(1対1)をそれぞれ比較することとした。最小化アルゴリズムで施設、BMI、がん種、デバイス使用歴、治療モードで層別化し、無作為化による割り付けを実施した。主要評価項目は、合併症の発生率(感染症、静脈血栓症、肺塞栓、血液吸引不能、機械的不具合、その他の複合)とし、デバイスの除去、試験からの離脱または1年後の追跡調査まで評価した。この試験はISRCTNに登録されている(ISRCTN44504648)。 【結果】2013年11月8日から2018年2月28日までに、適格性をスクリーニングした2,714例のうち1,061例を登録し、無作為化により該当する1つまたは複数の比較に割り付けた(PICCとHickmanの比較計424例、PICC 212例[50%]とHickman 212例[50%];PORTとHickmanの比較計556例、PORT 253例[46%]とHickman 303例[54%];PORTとPICCの比較計346例、PORT 147例[42%]、PICC 199例[58%])。PICC(212例中110例[52%])とHickman(212例中103例[49%])の合併症発生率がほぼ同じであった。観察された差は10%未満であったが、検出力が不十分であった可能性があるため、PICCの非劣性は確認されなかった(オッズ比[OR]1.15[95%CI 0.78~1.71])。PORTのHickmanに対する優越性が示され、合併症発生率が29%(253例中73例)と43%(303例中131例)であった(OR 0.54[95%CI 0.37~0.77])。PORTのPICCに対する優越性が示され、合併症発生率が32%(147例中47例)と47%(199例中93例)であった(OR 0.52[95%CI 0.33~0.83])。 【解釈】SACTを受ける患者のほとんどで、PORTがHickmanおよびPICCよりも有効かつ安全である。この結果から、英国National Health Serviceで、固形がんでSACTを受ける患者のほとんどが、PORTを使用すべきであることが示唆される。 第一人者の医師による解説 埋め込み型ポートが推奨されるが英国と日本では臨床背景が異なる点に留意 冲中 敬二 国立がん研究センター東病院感染症科科長・中央病院造血幹細胞移植科医員(併任) MMJ. February 2022;18(1):15 外来化学療法において、埋め込み型ポートは非常に重要な役割を果たしている。本論文は、比較的長期間使用する中心静脈アクセスデバイスである埋め込み型ポート(PORT)、トンネル型(Hickman)、末梢挿入型 (PICC)を3種類の2群間比較で評価した英国のCAVA試験の報告である(詳細版1)。これらデバイスを直接比較した無作為化試験はなく、どのアクセスを選択するか悩む際の参考となる。 CAVA試験には12週以上の抗がん剤治療(各群内訳:固形腫瘍87~97 %、血液腫瘍3~13 %)が予定されている18歳以上の患者1,061人が登録された。主要評価項目は合併症(デバイス関連感染症、静脈血栓症、ルートのつまり、故障など)の発生率であった。留置期間中央値はPICC 115日前後、Hickman 160日前後、PORT 380日前後であった。PORTの挿入は放射線科医によるものが多く(59~78%)、14~18%に予防的抗菌薬が使用された。一方 PICCの多く(67~73%)は看護師が挿入しており予防的抗菌薬の使用は2%以下であった。合併症の発生率はPICCとHickmanでは43~52%で同程度であったが、PORTでは30%前後と低かった。デバイス特異的な生活の質(QOL)はPORTが最も良好で、PICCとHickmanの間には差がなかった。総費用はPICC・PORT< Hickmanで、週当たりの費用でも同様であった。PICCとPORTの比較では総費用はPORTが高く、週当たりの費用はPICCが高かったが、後者に有意差はなかった。PORT(n=147)とPICC(n=199)の 直接比較では、挿入時の合併症はいずれも少なかった(PICC4%、PORT 0%)。PORTではカテーテル・週当たりの合併症件数が0.05であったがPICCでは0.13と多く、合併症による抜去(24% 対 38%)や静脈血栓症(2% 対 11%)の頻度もPICCが高かった。感染症の合併頻度はPORTの方が高かったが(12%vs 8%)、カテーテル・週当たりではいずれも0.02件と差はなかった。これらの結果を踏まえ、3カ月以上抗がん剤治療を受ける多くの固形がん患者ではPORTを利用すべきである、と著者らは結論している。 このような研究での課題として、問題が起きた場合に簡単に抜去できるか否かという点がある。また、今回の試験では挿入時合併症が非常に少なかったことから、処置に慣れた医療者が挿入したと考えられるが、特にPORT挿入に慣れていない医療者では動脈穿刺や気胸など挿入時の安全性も考慮する必要がある。なお、本試験ではPICCの90%以上が外来管理され、かつ大半を看護師が挿入している(研究期間中に英国で看護師によるPICCサービスが拡充された)など、日本の一般診療とは異なる点があり、今回のデータをそのまま日本に適用するのは難しい。 1. Wu O, et al. Health Technol Assess. 2021;25(47):1-126.
再発または難治性の多発性骨髄腫に用いるB細胞成熟抗原を標的とするキメラ抗原受容体T細胞療法、ciltacabtagene autoleucel(CARTITUDE-1):第Ib/II相非盲検試験
再発または難治性の多発性骨髄腫に用いるB細胞成熟抗原を標的とするキメラ抗原受容体T細胞療法、ciltacabtagene autoleucel(CARTITUDE-1):第Ib/II相非盲検試験
Ciltacabtagene autoleucel, a B-cell maturation antigen-directed chimeric antigen receptor T-cell therapy in patients with relapsed or refractory multiple myeloma (CARTITUDE-1): a phase 1b/2 open-label study Lancet. 2021 Jul 24;398(10297):314-324. doi: 10.1016/S0140-6736(21)00933-8. Epub 2021 Jun 24. 原文をBibgraph(ビブグラフ)で読む 上記論文の日本語要約 【背景】CARTITUDE-1試験では、予後が不良である再発または難治性の多発性骨髄腫患者で、2つのB細胞成熟抗原を標的とする単一ドメイン抗体、ciltacabtagene autoleucel(cilta-cel)を用いたキメラ抗原受容体T細胞療法の安全性と臨床活性を評価することを目的とした。 【方法】米国の16施設が参加したこの単群非盲検第Ib/II相試験では、ECOG全身状態スコア0または1であり、3回以上の治療歴またはプロテアソーム阻害薬および免疫調節薬に対する抵抗性があり、かつプロテアソーム阻害薬、免疫調節薬および抗CD38抗体の投与歴がある18歳以上の多発性骨髄腫患者を登録した。リンパ球枯渇開始から5~7日後にcilta-celの単回投与(目標用量CAR発現生T細胞数として0.75×106個/kg)を実施した。主要評価項目は、安全性、第II相試験の推奨用量の確認(第Ib相)および治療した全患者の奏効率(第II相)とした。奏効期間と無増悪生存期間を主な副次評価項目とした。この試験はClinicalTrials.govに登録されている(NCT03548207)。 【結果】2018年7月16日から2019年10月7日までに113例を登録した。97例(第Ib相29例、第II相68例)に、第II相試験の推奨用量となるCAR発現生T細胞として0.75×106個/kgのcilta-celを投与した。2020年9月1日の臨床カットオフ時点で、追跡期間が中央値で12.4カ月(IQR 10.6~15.2)であった。中央値で6カ月間の前治療歴のある97例にcilta-cel療法を実施した。総奏効率が97%(95%CI 91.2~99.4、97例中94例)であった。65例(67%)が厳格な完全寛解を達成し、最初の奏効までの期間は1カ月(IQR 0.9~1.0)であった。効果は経時的に深くなった。奏効期間(95%CI 15.9~評価不能)も無増悪生存期間(16.8~評価不能)も中央値に達しなかった。12ヵ月無増悪生存率は77%(95%CI 66.0~84.3)であり、総生存率が89%(80.2~93.5)であった。血液学的な有害事象が高頻度に発現した。グレード3~4の血液学的有害事象に、好中球減少症(97例中92例[95%])、貧血(66例[68%])、白血球減少症(59例[61%])、血小板減少症(58例[60%])、リンパ球減少症(48例[50%])があった。97例中92例(95%)にサイトカイン放出症候群が発現し(4%がグレード3または4)、発症までの期間が中央値で7.0日(IQR 5~8)、継続期間が中央値で4.0日(IQR 3~6)であった。サイトカイン放出症候群は、グレード5のサイトカイン放出症候群で血球貪食性リンパ組織球症を来した1例を除いて全例が回復した。20例(21%)にCAR T細胞の神経毒性が発現した(9%がグレード3または4)。試験中に14例が死亡した。6例が治療関連有害事象によるもの、5例が病勢進行によるもの、3例が治療と無関連の有害事象によるものであった。 【解釈】目標用量CAR発現生T細胞として0.75×106個/kgのcilta-cel単回投与は、治療歴の多い多発性骨髄腫患者に早期に持続的な深い寛解をもたらし、安全性は管理可能であった。本試験のデータは最近提出した薬事申請の根拠とした。 第一人者の医師による解説 現在は保険適応外 一般臨床では費用対効果を考慮した適正使用が課題 矢野 真吾 東京慈恵会医科大学腫瘍・血液内科教授 MMJ. February 2022;18(1):16 キメラ抗原受容体(CAR)は、腫瘍細胞の抗原を特異的に認識する受容体を人工的に作製したものであり、CAR-Tは患者末梢血由来のT細胞にCARを遺伝子導入させた再生医療等製品である。B細胞成熟抗原(BCMA)はB細胞の成熟と分化に働く膜貫通蛋白で、形質細胞や多発性骨髄腫細胞に発現している。BCMAを標的としたCAR-Tは数種類開発されており、ciltacabtagene autoleucel(cilta-cel)はBCMAの異なる2つのエピトープを認識し高い結合能を有する。 本論文は、米国で行われた再発・難治性多発性骨髄腫に対するcilta-celの第1b/2相試験の報告である。対象は、前治療数が3レジメン以上の患者、または免疫調整薬とプロテアソーム阻害薬の両方に抵抗性を示し抗 CD38抗体薬の投与を受けた患者とした。主要評価項目は第1b相試験では安全性(有害事象)、第2相試験では奏効率とされた。リンパ球除去療法後、97人(第1b相29人、第2相68人)にcilta-cel(推奨用量0.75X106 CAR-T細胞/kg)を投与した。年齢の中央値は61歳、前治療数の中央値は6レジメンであった。観察期間の中央値は12.4カ月で、cilta-celの全奏効は94人(97%)、厳格な完全奏効は65人(67%)で得られ、奏効は中央値1カ月で到達した。微小残存腫瘍は、評価可能な57人のうち53人(93%)で陰性化した。無増悪生存期間および全生存期間は中央値に未到達、12カ月の時点での無増悪生存率および全生存率はそれぞれ77%と89%であった。有害事象は97人(100%)に観察され、最も頻度が高かったのは血液毒性で、グレード 3以上の好中球減少が92人、貧血が66人、血小板減少は58人であった。感染症は56人(58%)に生じ、グレード 3以上の肺炎を8人、敗血症を4人に認めた。サイトカイン放出症候群は92人(95%)に発症したが、87人がグレード 2以下でグレード 3/4は4人であった。グレード5のサイトカイン放出症候群で1人が死亡したが、残りの91人は改善した。神経毒性は20人(21%)に生じ、グレード 3/4を9人に認めた。 Cilta-celは、濃厚な前治療歴を有する多発性骨髄腫に対して、迅速に深い寛解を得ることが期待できる。Cilta-celの投与細胞数が他のCAR-Tよりも少なく設定されているため、比較的安全に投与できる可能性がある。現在、長期的効果の評価、早期の治療ラインでの投与や外来での投与の検討が行われている。日本でもCAR-T細胞療法が普及してきている。現在 BCMAを標的としたCAR-T細胞療法は保険適応外であるが、近い将来一般臨床で用いられるようになる。費用対効果を考慮した適正使用が求められており、最新のエビデンスを蓄積していく必要がある。
2型糖尿病に用いる新規デュアルGIP/GLP受容体作動薬tirzepatideの有効性および安全性(SURPASS-1):二重盲検無作為化第III相試験
2型糖尿病に用いる新規デュアルGIP/GLP受容体作動薬tirzepatideの有効性および安全性(SURPASS-1):二重盲検無作為化第III相試験
Efficacy and safety of a novel dual GIP and GLP-1 receptor agonist tirzepatide in patients with type 2 diabetes (SURPASS-1): a double-blind, randomised, phase 3 trial Lancet. 2021 Jul 10;398(10295):143-155. doi: 10.1016/S0140-6736(21)01324-6. Epub 2021 Jun 27. 原文をBibgraph(ビブグラフ)で読む 上記論文の日本語要約 【背景】医療が進歩したが、2型糖尿病患者の多くが治療目標を達成していない。そのため、新たな治療法の開発が求められている。食事療法および運動療法のみではコントロール不十分な2型糖尿病患者を対象に、新たなグルコース依存性インスリン刺激性ポリペプチド(GIP)受容体およびGLP-1受容体作動薬、tirzepatide単剤療法のプラセボと比較した有効性、安全性および忍容性を評価することを目的とした。 【方法】インド、日本、メキシコおよび米国の医療研究センターおよび病院計52施設で40週間にわたる二重盲検無作為化プラセボ対照第III相試験を実施した(SURPASS-1試験)。食事療法と運動療法のみではコントロール不十分な2型糖尿病があり、糖尿病の注射薬治療歴がない成人患者(18歳以上)を対象とした。コンピュータで生成したランダム配列を用いて、患者をtirzepatide(5mg、10mgまたは15mg)週1回投与とプラセボに1対1対1対1の割合で割り付けた。患者、治験担当医師、治験依頼者に治療の割り付けを伏せた。主要評価項目は、40週時点の糖化ヘモグロビン(HbA1c)の治療前からの平均変化量とした。この試験はClinicalTrials.govにNCT03954834として登録されている。 【結果】2019年6月3日から2020年10月28日までに適格性を評価した705例のうち、478例(治療前の平均HbA1c値7.9%[63mmol/mol]、年齢54.1[SD 11.9]歳、女性231例[48%]、糖尿病罹病期間4.7年、BMI 31.9kg/m2)を無作為化によりtirzepatide 5 mg(121例[25%])、tirzepatide 10 mg(121例[25%])、tirzepatide 15 mg(121例[25%])、プラセボ(115例[24%])に割り付けた。66例(14%)が試験薬を中止し、50例(10%)が早期に試験を中止した。40週時点で、HbA1cの治療前からの変化量、空腹時血糖、体重、HbA1cの目標値7.0%未満(<53mmol/mol)達成および5.7%未満(<39mmol/mol)達成について、tirzepatide全用量のプラセボに対する優越性が示された。tirzepatide 5mg群に1.87%(20 mmol/mol)、10mg群に1.89%(21mmol/mol)、15mg群に2.07%(23mmol/mol)の治療前からの平均HbA1c値低下がみられたが、それに対してプラセボ群に0.04%の増加(+0.4mmol/mol)がみられた。その結果、プラセボと比較した治療差の推定値は、tirzepatide 5mg群が-1.91%(-21mmol/mol)、10mg群が-1.93%(-21mmol/mol)、15mgが-2.11%(-23 mmol/mol)であった(いずれもP<0.0001)。tirzepatide群の方がプラセボ群よりもHbA1c目標値7.0%未満(<53 mmol/mol;87~92% vs 20%)および6.5%未満(<48mmol/mol;81~86% vs 10%)を達成した患者の割合が多く、tirzepatide群の31–52%およびプラセボ群の1%が目標値5.7%未満(<39mmol/mol)達成した。tirzepatideにより、用量依存性に7.0~9.5kgの体重減少がみられた。tirzepatide群に最も多くみられた有害事象は、軽度ないし中等度で一過性の消化管事象であり、悪心(12~18% vs 6%)、下痢(12~14% vs 8%)、嘔吐(2~6% vs 2%)などがあった。tirzepatide群には、臨床的に重要ではない低血糖(54mg/dL[3mmol/L]未満)または重度の低血糖は報告されなかった。プラセボ群に死亡が1件発生した。 【解釈】tirzepatideにより、低血糖リスクが上昇することなく、血糖コントロールおよび体重に強固な改善が認められた。安全性はGLP-1受容体作動薬のものと一致しており、2型糖尿病の治療にtirzepatide単剤療法を用いうる可能性を示唆するものである。 第一人者の医師による解説 低血糖を伴わずに血糖を正常化させることで心血管イベント低減を期待 篁 俊成 金沢大学大学院医学系研究科内分泌・代謝内科学教授 MMJ. February 2022;18(1):13 現在臨床応用されているグルカゴン様ペプチド-1受容体作動薬(GLP-1RA)は血糖降下作用と体重減少作用を有する。同じインクレチンであるグルコース依存性インスリン分泌刺激ポリペプチド(GIP)もインスリン分泌を促進し、視床下部の受容体を介して体重を減少させるが、2型糖尿病におけるインスリン分泌促進作用は明確でない。食事誘発性肥満モデルマウスでは、GLP-1とGIPの併用により、満腹感が増大し、甘みへの嗜好が減少した。 チルゼパチドはGIPおよびGLP-1の両受容体を刺激する合成ペプチドで、第2B相試験ではチルゼパチド(15 mg週1回皮下投与)はプラセボやGLP-1RAのデュラグルチドに比べ血糖と体重のコントロールで優位だったが、悪心など消化器系副作用の頻度も高かった。本論文は、消化器症状の忍容性を高める目的で、より低用量のチルゼパチドを2型糖尿病患者に投与し、血糖コントロール効果と安全性を検討した第3相 SURPASS-1試験の報告である。対象患者は5、10、15 mgのチルゼパチド群またはプラセボ群に割り付けられた。 その結果、40週の時点においてチルゼパチドの全群で、プラセボ群に比べ、HbA1c、空腹時血糖、体重、HbA1c 7.0%未満 お よ び5.7%未満達成率が有意に改善した。主要評価項目である40週でのベースラインからのHbA1c平均変化量は、チルゼパチド 5mg群-1.87%、10mg群-1.89%、15mg群-2.07%に対し、プラセボ群は+0.04%であった。チルゼパチドの全群で、プラセボ群に比べ、体重もより減少した。チルゼパチド群におけるHbA1c降下は20週でプラトーに達したが、体重減少は40週まで続いた。すべてのチルゼパチド用量群で、プラセボ群と比較し、総コレステロール、中性脂肪、HOMA-Rが有意に低下し、高比重リポ蛋白コレステロールが有意に上昇した。一方、有害事象に起因する試験薬中止率、有害事象が1件以上発現した患者の割合、総有害事象数に有意な群間差はなかった。チルゼパチドの主な有害事象は軽度~中等症の嘔気・下痢・嘔吐などの消化器症状であり、経過中に軽快した。消化器症状による試験薬中止率はチルゼパチド群で2~7%、プラセボ群で1%だった。両群ともに重度低血糖と膵炎の報告はなかった。 チルゼパチド群のHbA1c 7.0%未満達成率は87~92%で、他のGLP-1RA単独療法の既報値(週1回エキセナチド 63%、デュラグルチド 61~63%、セマグルチド 72~74%)を上回る。今回チルゼパチドの用量依存性がみられなかったのは、ほぼ正常レベルまで血糖降下が得られたためと思われる。GIP受容体作動薬の心血管への作用は検討段階であるが、チルゼパチドにより低血糖を伴わずに血糖を正常化させることで、心血管イベント低減が期待される。
低温や高温による死亡 2019年には全世界で169万と推定
低温や高温による死亡 2019年には全世界で169万と推定
Estimating the cause-specific relative risks of non-optimal temperature on daily mortality: a two-part modelling approach applied to the Global Burden of Disease Study Lancet. 2021 Aug 21;398(10301):685-697. doi: 10.1016/S0140-6736(21)01700-1. 上記論文のアブストラクト日本語訳 ※ヒポクラ×マイナビ 論文検索(Bibgraph)による機械翻訳です。 【背景】気温の高低と死亡率や罹患率の増加との関連は以前から報告されているが、疾病負担の包括的な評価は行われていない。そこで、最適でない気温への曝露による世界的及び地域的な負担を推定することを目的とした。 【方法】本研究の第1部では、ERA5再解析データセットからの毎日の気温推定値と死亡を関連付けた。我々は、ベイズメタ回帰の枠組みの中で、二次元スプラインを用いて、日温と23の平均気温ゾーンに沿った176の個々の死因の原因特異的相対リスクをモデル化した。そして、日ごとの死亡データが入手可能な国について、気温に起因する原因別負担と総負担を計算した。第2部では、第1部の原因別相対リスクを世界の全地域に適用した。我々は、曝露反応曲線と日次グリッドの気温を組み合わせ、1990年から2019年までの世界疾病負担、傷害、リスク要因研究からの疾病負担に基づいて、原因別負担を算出した。リスク,気温への曝露,及び含まれる全ての原因における最小死亡率の温度として定義される理論的最小リスク曝露レベルを含むモデリングチェーンの全ての構成要素からの不確実性は,1000ドローの事後シミュレーションを用いて伝播させた。 【調査結果】1980年1月1日から2016年12月31日までに発生した,9か国からの国際疾病分類でコードされた個人死亡64~900万件を対象とした。17の死因が包含基準を満たした。虚血性心疾患,脳卒中,心筋症・心筋炎,高血圧性心疾患,糖尿病,慢性腎臓病,下気道感染症,慢性閉塞性肺疾患は日気温とJ型の関係を示したが,外因(例:殺人,自殺,溺死,災害・機械・輸送・その他の不慮の負傷に関連)のリスクは気温とともに単調に増加した。理論的な最小リスク暴露レベルは、死因の構成に基づく機能として、場所や年によって異なる。最適でない気温の推定値は,ブラジルの 10 万人当たりの死亡数 7-98 人(95%不確実性区間 7-10-8-85 ),人口起因率(PAF)1-2%(1-1-1-4)から中国の 10 万人当たりの死亡数 35-1 人(同 29-9-40-3 ),PAF4-7%(4-3-5-1 )までであった。2019年、データが入手できたすべての国で、寒さ起因の平均死亡率が暑さ起因の死亡率を上回った。寒さの影響は中国で最も顕著で、PAFは4-3%(3-9-4-7)、10万人当たりの帰属率は32-0人(27-2-36-8)、ニュージーランドでは3-4%(2-9-3-9)、26-4人(22-1-30-2)であった。熱の影響は中国で最も顕著で、PAFは0-4%(0-3-0-6)、10万人当たりの帰属率は3-25人(2-39-4-24)、ブラジルでは0-4%(0-3-0-5)、死亡数は2-71人(2-15-3-37)であった。我々の枠組みを世界のすべての国に適用すると、2019年には世界で1-6900万人(1-52-1-83)の死亡が最適でない気温に起因すると推定されました。熱に起因する負担が最も高いのは南・東南アジア、サハラ以南のアフリカ、北アフリカ・中東で、寒さに起因する負担が最も高いのは東・中央ヨーロッパ、中央アジアでした。 【解釈】急な暑さや寒さへの暴露は、多様な死因の死亡リスクを増加または減少させる可能性があります。ほとんどの地域では寒冷の影響が支配的であるが、実勢気温が高い場所では寒冷起因の負担をはるかに上回る実質的な暑熱の影響を示すことがある。特に、外的死因の負荷が高いことが暑さの影響を強くしているが、心肺疾患や代謝性疾患も大きく寄与している可能性がある。曝露と死因の構成の両方における変化が、時間の経過に伴うリスクの変化を促した。高温のリスクへの曝露が着実に増加していることは、健康への懸念が高まっている。 【FUNDING】ビル&メリンダ・ゲイツ財団。 第一人者の医師による解説 高温地域では高温による負荷も大きく 温暖化の影響にも今後注目 山口 聡子(特任准教授)/岡田 啓(特任助教) 東京大学大学院医学系研究科糖尿病・生活習慣病予防講座 MMJ. April 2022;18(2):54 気温と死亡についてはこれまでも関連性が報告されてきたが、特定の地域に関する報告が多く、また、詳細な死因別の報告は少なかった。本論文ではまず気候情報と死亡統計が得られた地域で予測モデルを作成し、全世界に適用して非至適気温による死亡者数の推計を行った。 モデル作成では、9カ国(ブラジル、チリ、中国、コロンビア、グアテマラ、メキシコ、ニュージーランド、南アフリカ、米国)における1980年〜2016年の死亡(6490万人)と気温の関係を解析した。全地域をERA5のデータセットから算出した、この期間の平均気温によって23温度帯に分類した。日平均気温を曝露要因、死亡の相対リスクを応答変数として、176の死因のうち気温と有意な関連性がみられた17の死因について、23温度帯とそのそれぞれの年平均気温を考慮し、ベイズモデルで結果を統合し、2次元スプラインモデルを作成した。循環器・呼吸器疾患(虚血性心疾患、脳卒中、下気道感染症、慢性閉塞性肺疾患[COPD]など)、代謝疾患(糖尿病、慢性腎臓病)などの非外因性の死因では、低温と高温で相対リスクが上昇するJ字型の曲線がみられたのに対し、外因性死因(他殺、自殺、溺水、損傷など)では、気温が上がるにつれてリスクが単調上昇する傾向がみられた。全9カ国で、低温による負荷は高温による負荷を上回っていた。高温と低温を合わせた非至適気温による影響は、ブラジルで最小、中国で最大で、10万人あたりの死亡数および人口寄与割合はそれぞれ7.98(95%不確実性区間 7.10〜8.85)と1.2%(1.1〜1.4)、35.1(29.9〜40.3)と4.7%(4.3〜5.1)だった。 次に、上記のように作成されたモデルを全世界の204の国・地域に適用した。この結果、2019年の非至適気温による死亡は169万人(152〜183万人)と推定された。高温による負荷は、南アジア、東南アジア、サハラ以南のアフリカ、北アフリカ、中東で特に大きく、低温による負荷は、東ヨーロッパ、中央ヨーロッパ、中央アジアで特に大きかった。以上から、多くの地域では低温による影響が優位であるが、高温の地域では、高温の影響が低温による負荷を上回ることがわかった。著者らは今後地球温暖化による影響も懸念されると考察した。 本研究では、現在限られた地域で入手可能な気候情報と死亡統計を元に予測モデルを作成し、データが入手できない地域も含めた全世界に適用することにより、低温と高温がもたらす死因別死亡を推定した。本研究では死亡当日の気温のみを見ており、死亡前の一定期間の気温を用いた先行研究(1),(2)に比べ気温による影響が過小評価されている可能性があることに留意する必要がある。 1. Gasparrini A, et al. Lancet. 2015;386(9991):369-375. 2. Guo Y, et al. Epidemiology. 2014;25(6):781-789.
世界の高血圧有病率、治療率、コントロール率の傾向(地域住民研究1201件のプール解析 ; 1990-2019年)
世界の高血圧有病率、治療率、コントロール率の傾向(地域住民研究1201件のプール解析 ; 1990-2019年)
Worldwide trends in hypertension prevalence and progress in treatment and control from 1990 to 2019: a pooled analysis of 1201 population-representative studies with 104 million participants Lancet. 2021 Sep 11;398(10304):957-980. doi: 10.1016/S0140-6736(21)01330-1. Epub 2021 Aug 24. 上記論文のアブストラクト日本語訳 ※ヒポクラ×マイナビ 論文検索(Bibgraph)による機械翻訳です。 【背景】高血圧はプライマリーヘルスケアレベルで発見することができ、低コストの治療で効果的に高血圧をコントロールすることができる。我々は,200の国と地域について,1990年から2019年までの高血圧の有病率とその発見,治療,コントロールの進展を測定することを目的とした。 【方法】我々は,血圧の測定と血圧治療に関するデータを有する人口代表研究からの30~79歳の人々に関する1990年から2019年のデータを使用した。収縮期血圧140mmHg以上、拡張期血圧90mmHg以上、または高血圧の薬を服用していることを高血圧と定義しました。ベイズ型階層モデルを適用して,高血圧の有病率と,高血圧の診断歴があり(検出),高血圧の治療を受けており(治療),高血圧が140/90 mm Hg未満にコントロールされている(コントロール)人の比率を推定した.モデルは,経時的な傾向が非線形であり,年齢によって異なることを許容した。 発 見】高血圧を有する30~79歳の人々の数は,世界の年齢標準化有病率が安定しているにもかかわらず,1990年の女性331(95%信頼区間306~359)万人と男性317(292~344)万人から2019年には女性626(584~668)万人と男性652(604~698)万人に倍加している。2019 年の年齢標準化高血圧有病率は,男女ともカナダとペルーで最も低く,女性は台湾,韓国,日本,スイス,スペイン,英国など西ヨーロッパの一部の国で,男性はエリトリア,バングラデシュ,エチオピア,ソロモン諸島などいくつかの低所得国および中所得国で低かった.高血圧の有病率は、女性では2カ国、男性では中・東欧、中央アジア、オセアニア、ラテンアメリカの9カ国で50%を超えています。世界的に見ると、2019年に高血圧の女性の59%(55~62人)と男性の49%(46~52人)が高血圧の診断歴を報告し、女性の47%(43~51人)と男性の38%(35~41人)が治療を受けていた。2019年の高血圧の人のコントロール率は、女性が23%(20-27)、男性が18%(16-21)でした。2019年の治療率およびコントロール率は、韓国、カナダ、アイスランドが最も高く(治療率70%以上、コントロール率50%以上)、米国、コスタリカ、ドイツ、ポルトガル、台湾がそれに続いていた。ネパール,インドネシア,サハラ以南のアフリカとオセアニアの一部の国では,治療率は女性で25%未満,男性で20%未満であった。これらの国の女性と男性、および北アフリカ、中央・南アジア、東ヨーロッパの一部の国の男性では、コントロール率が10%を下回っていました。1990年以降,ほとんどの国で治療率とコントロール率が向上しているが,サハラ以南のアフリカとオセアニアのほとんどの国では,ほとんど変化がないことがわかった.高所得国,中央ヨーロッパ,およびコスタリカ,台湾,カザフスタン,南アフリカ,ブラジル,チリ,トルコ,イランなどの一部の上位中所得国や最近の高所得国での改善が最も大きかった。 【解釈】高血圧の発見,治療,管理における改善は国によって大きく異なり,一部の中所得国は現在ほとんどの高所得国を凌駕している。一次予防による高血圧の有病率の低下と治療およびコントロールの強化という二重のアプローチは、高所得国だけでなく低所得国や中所得国の環境でも達成可能である。 第一人者の医師による解説 世界の高血圧人口はこの30年でほぼ倍増し、国ごとのばらつきが極めて大きい 苅尾 七臣 自治医科大学循環器内科学部門教授 MMJ. April 2022;18(2):41 本論文は、世界184カ国の地域住民データをプール解析したNCD Risk Factor Collaborationの報告である。今回の報告でまず驚くことは、高血圧人口(30 ~ 79歳)はこの30年でほぼ倍増したことである。次に、国ごとのばらつきが極めて大きい点である。人口増加の偏りもあり、2019年の高血圧人口の82%は低・中所得国の住民が占めていた。高血圧有病率も、低・中所得国で上昇、高所得国で低下する傾向にあるが、状況は国ごとにばらつきが大きく、高所得国でも有病率を抑制できていない国がある一方で、低・中所得国の中にも高血圧有病率の低い国がみられた。有病率に関与する因子としては、地域の風土、食文化、高血圧制圧に向けた国家の取り組みなどが考えられるが、これらを積極的に同定し、各国でいっそう取り組む必要がある。 また、高血圧コントロール率(降圧薬服用下 で140/90 mmHg未満)の上昇が高血圧人口の増加に追い付いていないことも明らかになった。2019年の世界のコントロール率は女性23%、男性18%であった。アジアでは韓国で治療率・コントロール率が高く、ネパールやインドネシアでは低かった。アジア・パシフィックで高所得国に分類される3カ国に比べ、その他の東アジア・東南アジア地域の国(低・中所得国)では血圧が高いにもかかわらず高血圧と診断されていない住民の割合が高く(29 ~ 34% vs 46 ~ 55%)、コントロール率は低かった(31 ~ 38% vs 13 ~ 17%)。これは南アジアでも同様であった(高血圧だが診断歴なし55 ~ 67%、コントロール率11 ~ 17%)。アジアの高血圧治療・コントロール状況を改善するには、まず血圧測定機会を増やし、高血圧を早期に発見し、適切に治療を行うことが重要である。ちなみに日本の高血圧有病率、治療率、コントロール率は女性でそれぞれ23、51、30%、男性でそれぞれ40、46、24%であり、アジア・パシフィック地域で高所得国に分類される3カ国中、治療率・コントロール率ともに最下位であり、決して優等生とは言い難い。 これまで、我々はアジア各国の高血圧専門家有志で作るHOPE Asia Networkで同一の家庭血圧計を用いて、アジア 11カ国・地域、15施設で家庭血圧コントロール状況を調査するAsiaBP@Home研究を行った。家庭血圧コントロール状況は36〜84%とばらつきが大きいものの、コントロール状況は比較的良好であった(1)。したがって、欧米と比較し、日本を含むアジアには早朝・夜間高血圧が多いという特徴があるが(2)、コントロール率は改善できると思われる。 1. Kario K, et al. J Clin Hypertens. 2018;20(12):1686-1695. 2. Kario K. Essential Manual of 24-Hour Blood Pressure Management: From Morning to Nocturnal Hypertension, 2nd Edition. pp. 1-384. 2022 (WileyBlackwell; ISBN 978-1-119-79936-8400)
植込み型ループレコーダーによる心房細動スクリーニングは予後を改善せず
植込み型ループレコーダーによる心房細動スクリーニングは予後を改善せず
Implantable loop recorder detection of atrial fibrillation to prevent stroke (The LOOP Study): a randomised controlled trial Lancet. 2021 Oct 23;398(10310):1507-1516. doi: 10.1016/S0140-6736(21)01698-6. Epub 2021 Aug 29. 上記論文のアブストラクト日本語訳 ※ヒポクラ×マイナビ 論文検索(Bibgraph)による機械翻訳です。 【背景】 心房細動のスクリーニングと、心房細動が検出された場合のその後の抗凝固剤による治療が、脳卒中を予防できるかどうかは不明である。植え込み型ループレコーダー(ILR)を用いて心電図を継続的に監視することで,無症状の心房細動エピソードの検出が容易になる。我々は,心房細動のスクリーニングと抗凝固薬の使用が,高リスクの人の脳卒中を予防できるかどうかを調べることを目的とした。 【方法】我々は,デンマークの4施設で無作為化対照試験を行った。対象は心房細動のない70~90歳の人で,少なくとも1つの脳卒中リスク因子(すなわち,高血圧,糖尿病,脳卒中の既往,心不全)を追加した人であった。参加者は、オンラインシステムを介して、ILRモニタリング群と通常のケア(コントロール)群に1:3の割合で無作為に割り付けられ、センターに応じて4人または8人のブロックサイズで層別された順列ブロックで行われた。ILR群では、心房細動が6分以上続いた場合、抗凝固療法が推奨された。主要評価項目は,最初の脳卒中または全身性動脈塞栓症を発症するまでの時間であった。本試験はClinicalTrials. gov, NCT02036450に登録されている。 【結果】2014年1月31日から2016年5月17日までに、6205人が対象としてスクリーニングされ、そのうち6004人が対象となり、無作為に割り付けられた。1501人(25-0%)をILRモニタリングに、4503人(75-0%)を通常ケアに割り付けた。平均年齢は74~7歳(SD 4~1)、2837人(47~3%)が女性で、5444人(90~7%)が高血圧症だった。追跡調査で失われた参加者はいなかった。中央値64-5カ月(IQR 59-3-69-8)の追跡期間中、1027人の参加者に心房細動が診断された。ILR群1501人中477人(31-8%)に対し,対照群4503人中550人(12-2%)であった(ハザード比[HR]3-17[95%CI 2-81-3-59],p<0-0001)。経口抗凝固療法が開始されたのは1036名であった。経口抗凝固療法が開始されたのは1036名で,ILR群445名(29-7%)対対照群591名(13-1%)(HR 2-72 [95%CI 2-41-3-08]; p<0-0001),主要転帰が発生したのは318名(脳卒中315名,全身性動脈塞栓症3名)で,ILR群67名(4-5%)対対照群251名(5-6%)(HR 0-80 [95%CI 0-61-1-05]; p=0-11)であった。大出血は221名に発生しました。脳卒中の危険因子を有する人において,ILRスクリーニングにより心房細動の検出および抗凝固療法の開始が3倍に増加したが,脳卒中および全身性動脈塞栓症のリスクは有意に減少しなかった。これらの知見は,すべての心房細動がスクリーニングに値するわけではなく,また,スクリーニングで検出されたすべての心房細動が抗凝固療法に値するわけではないことを示唆しているかもしれない。 【FUNDING】Innovation Fund Denmark,The Research Foundation for the Capital Region of Denmark,The Danish Heart Foundation,Aalborg University Talent Management Program,Arvid Nilssons Fond,Skibsreder Per Henriksen,R og Hustrus Fond,The AFFECT-EU Consortium (EU Horizon 2020),Lage Sophus Carl Emil Friis og hustru Olga Doris Friis’ Legat,Medtronic。 第一人者の医師による解説 短時間の心房細動発作に対する抗凝固療法の意義は再検討が必要 香坂 俊 慶應義塾大学医学部循環器内科専任講師 MMJ. April 2022;18(2):39 本論文は、脳卒中危険因子(いわゆるCHADSスコアでカウントされる項目のうち心不全、高血圧、糖尿病、脳卒中の既往のいずれか1つ以上)を有するが、心房細動(AF)とは診断されていない70〜90歳の患者を対象に行われたランダム化対照試験(LOOP試験)の報告である。登録患者は植込み型ループレコーダー(ILR)群もしくは通常ケア(対照)群に割り付けられ、ILR群でAFが検出され、発作が6分以上持続する場合には抗凝固療法が推奨された。 両群の患者は中央値で64.5カ月間追跡され、その期間内にILR群の31.8%、対照群の12.2%がAFと診断された(ハザード比[HR], 3.17;95%信頼区間[CI], 2.81〜3.59;P<0.0001)。そして、各群の29.7%と13.1%に経口抗凝固薬が開始された(HR, 2.72;95% CI, 2.41〜3.08;P<0.0001)。しかし、主要評価項目である脳卒中または全身性動脈塞栓症の発生率は、ILR群が4.5%、対照群が5.6%(HR, 0.80;95% CI, 0.61〜1.05;P=0.11)と有意な差は認めなかった。さらに、ILR群の4.3%、対照群の3.5%に出血イベントが発生していた(HR, 1.26;95% CI, 0.95〜1.69;P=0.11)。 AF発作が「6分以上持続した場合に抗凝固療法を推奨する」という規定の時間枠が緩すぎたのではないかとの批判がなされているが、おそらくは的を射ているように感じられる。実際、このLOOP試験と同じ時期に発表された、高齢者を対象とした14日間にわたる心電図スクリーニング検査の効果を検討したランダム化対照試験(STOPSTROKE試験)では、介入に関して比較的有利な結果が得られている(HR, 0.96;95% CI, 0.92〜1.00;P=0.045)(1)。このほかに周術期のAFに関しては、診療ガイドラインなどでは「30分」というカットオフが用いられ、それ以下であれば一過性の可逆的なAF、それ以上ならばその後も抗凝固療法などのケアが必要なAF、といった形で線引きがなされることが多いようである。 今後 Apple Watchなどのスマートデバイスを用いたAFの検出が広く行われていくようになることが予想される(2)。本研究の結果に鑑みても、こうしたスマートデバイスを用いたAFの診断を行う際にも、時間的なAF burden(負荷)を考慮することが大切であると考えられる。抗凝固療法は高リスク AF患者において塞栓系のイベントを抑制するために極めて有効な治療法であるが、そのリスクに対する考え方は診断の進歩と共に変化していくものと予想される。 1. Svennberg E, et al. Lancet. 2021;398(10310):1498-1506. 2. Perez MV, et al. N Engl J Med. 2019;381(20):1909-1917.
GIP/GLP-1デュアルアゴニストのチルゼパチド 2型糖尿病で糖代謝改善を示す
GIP/GLP-1デュアルアゴニストのチルゼパチド 2型糖尿病で糖代謝改善を示す
Tirzepatide versus insulin glargine in type 2 diabetes and increased cardiovascular risk (SURPASS-4): a randomised, open-label, parallel-group, multicentre, phase 3 trial Lancet. 2021 Nov 13;398(10313):1811-1824. doi: 10.1016/S0140-6736(21)02188-7. Epub 2021 Oct 18. 上記論文のアブストラクト日本語訳 ※ヒポクラ×マイナビ 論文検索(Bibgraph)による機械翻訳です。 【背景】 経口血糖降下薬で十分にコントロールされていない心血管リスクの高い成人2型糖尿病患者を対象に、新規GIP・GLP-1受容体デュアルアゴニストtirzepatideのインスリン グラルギンに対する有効性と安全性を、特に心血管安全に焦点を当てて評価した。 【方法】 この非盲検並行群間第3相試験は5大陸14か国187施設で実施された。対象者は18歳以上で、メトホルミン、スルホニル尿素、Na-グルコース共輸送体-2阻害剤のいずれかの併用療法を受けている2型糖尿病患者、ベースラインの糖化ヘモグロビン(HbA1c)が7-5~10-5%(58~91mmol/mol)、体格指数25kg/m2以上、心血管疾患の確立または心血管イベントの高リスクを持つ者とされました。参加者は、インタラクティブなWeb応答システムにより、週1回投与のtirzepatide(5 mg、10 mg、15 mg)またはグラルギン(100 U/mL)のいずれかの皮下注射を、空腹時血糖値が100 mg/dL未満になるように滴定して、1:1:1:3方式でランダムに割り付けられた。主要評価項目は、ベースラインから52週後までのHbA1c変化におけるtirzepatide 10 mgまたは15 mg、あるいはその両方のグラルギンに対する非劣性(非劣性境界0~3%)であった。すべての参加者は少なくとも52週間治療を受け、最大104週間または試験終了まで治療を継続し、主要有害心血管イベント(MACE)の収集と判定を行いました。安全性については、全試験期間にわたって評価されました。本試験はClinicalTrials. gov、NCT03730662に登録されました。 【FINDINGS】2018年11月20日から2019年12月30日の間に患者を募集しました。3045人がスクリーニングされ、2002人がティルゼパチドまたはグラルギンに無作為に割り付けられた。1995人はtirzepatide 5mg(n=329、17%)、10mg(n=328、16%)、15mg(n=338、17%)、またはグラルギン(n=1000、50%)を少なくとも1回投与し、修正intention to treat集団に含まれることとなった。52週時点におけるTirzepatideの平均HbA1c変化率は、10mgで-2-43%(SD 0-05)、15mgで-2-58%(0-05)であり、Glargineでは-1-44%(0-03)であった。グラルギンに対する推定治療差は、ティルゼパチド10 mgで-0~99%(多重度調整後97~5%CI -1-13~-0-86)、15 mgで-1~14%(-1-28~-1-00)であり、両用量の非劣性マージンは0~3%であった。吐き気(12-23%)、下痢(13-22%)、食欲減退(9-11%)、嘔吐(5-9%)は、グラルギンよりtirzepatideでより頻繁に発生し(それぞれ吐き気2%、下痢4%、食欲減退1%未満、嘔吐2%)ほとんどのケースは軽度から中等度で用量漸減期に発生しました。低血糖(グルコース<54mg/dLまたは重度)の割合は、グラルギン(19%)に対してTirzepatide(6-9%)で低く、特にスルホニルウレア剤を使用していない参加者(Tirzepatide 1-3% vs Glargine 16%)で低くなっています。判定されたMACE-4イベント(心血管死、心筋梗塞、脳卒中、不安定狭心症による入院)は109人に発生し、ティルゼパチドはグラルギンに比べて増加しなかった(ハザード比0-74、95%CI 0-51-1-08)。死亡は60例(tirzepatide:25例[3%]、グラルギン:35例[4%])。 【解釈】2型糖尿病で心血管リスクが高い患者において、tirzepatideはグラルギンに比べて52週目に低血糖の発生率が低く、臨床的に意味のあるHbA1c減少が示されました。Tirzepatideの投与は、過剰な心血管リスクと関連しなかった。 第一人者の医師による解説 新規薬剤チルゼパチド 消化器症状の懸念はあるが実地臨床での有効性・安全性に期待 河合 俊英 東京都済生会中央病院 糖尿病・内分泌内科部長 MMJ. April 2022;18(2):43 下部小腸のL細胞から分泌されるペプチドホルモンであるグルカゴン様ペプチド -1(GLP-1)は、糖尿病薬として血糖コントロールのみならず、心血管イベント抑制効果も示され(1),(2)、臨床で広く使用されてきた。上部小腸に存在するK細胞から分泌されるグルコース依存性インスリン分泌刺激ポリペプチド(GIP)は、GLP-1と共にインスリン分泌を強力に増強させることから、生理的に重要なインクレチンと考えられてきた。しかし、GIPのもつ脂肪蓄積作用、高血糖時のグルコース濃度依存性インスリン分泌反応の減弱、グルカゴン分泌促進などの作用が、臨床応用への障壁であった。今回、GIP/GLP-1デュアルアゴニストのチルゼパチドが糖尿病治療薬として有用である可能性が報告された。 本論文は、2型糖尿病患者を対象に14カ国で実施された無作為化非盲検第3相試験(SURPASS-4)の報告である。HbA1c 7.5%超〜10.5%未満、体格指数(BMI)25kg/m2以上でメトホルミン、スルホニル尿素(SU)薬、Na+/グルコース共役輸送担体2(SGLT2)阻害薬を単独または併用で使用している心血管リスクの高い2型糖尿病患者約2,000人が3用量のチルゼパチド群(週1回投与)、または持効型インスリンのグラルギン群(1日1回投与)に割り付けられた。その結果、主要評価項目である52週時点のベースラインからのHbA1c変化量は、チルゼパチド 5mg群−2.24%、10mg群−2.43%、15mg群−2.58%、グラルギン群−1.44%、副次評価項目である同期間の体重変化量はチルゼパチド群でそれぞれ−7.1、−9.5、−11.7kg、グラルギン群で+1.9kgであった。チルゼパチドのグラルギンに対するHbA1cに基づく推定治療差は、10mg群で−0.99%、15mg群で−1.14%となり、両群を含めた非劣性が検証された。安全性解析では、MACE-4(心血管死、心筋梗塞、不安定狭心症による入院、脳卒中)について、チルゼパチド群全体でグラルギン群と比較し、ハザード比が0.74であった。 今回の検討から、チルゼパチドは血糖コントロールが不十分な心血管リスクの高い2型糖尿病患者において強力なHbA1c低下、減量効果を有することが示唆された。消化器症状の懸念はあるものの、実地臨床での有効性・安全性が期待される。 1. Marso SP, et al. N Engl J Med. 2016;375(4):311-322. 2. Marso SP, et al. N Engl J Med. 2016;375(19):1834-1844.
切除不能進行・再発胃がんの1次治療へのニボルマブ上乗せ OSとPFSを有意に延長
切除不能進行・再発胃がんの1次治療へのニボルマブ上乗せ OSとPFSを有意に延長
First-line nivolumab plus chemotherapy versus chemotherapy alone for advanced gastric, gastro-oesophageal junction, and oesophageal adenocarcinoma (CheckMate 649): a randomised, open-label, phase 3 trial Lancet. 2021 Jul 3;398(10294):27-40. doi: 10.1016/S0140-6736(21)00797-2. Epub 2021 Jun 5. 上記論文のアブストラクト日本語訳 ※ヒポクラ×マイナビ 論文検索(Bibgraph)による機械翻訳です。 【背景】進行性または転移性のヒト上皮成長因子受容体2(HER2)陰性の胃または胃食道接合部腺癌に対する第一選択の化学療法は、全生存期間(OS)の中央値が1年未満である。我々は、胃、胃食道接合部、および食道腺がんにおいて、プログラム細胞死(PD)-1阻害剤を用いたファーストライン治療を評価することを目的としました。この多施設共同無作為化非盲検第3相試験(CheckMate 649)では、29カ国の175の病院およびがんセンターから、PD-リガンド1(PD-L1)の発現状況にかかわらず、前治療歴のある切除不能な非HER2陽性の胃・胃食道接合部・食道腺がんの成人患者(18歳以上)を登録しました。患者は、インタラクティブなウェブ応答技術を用いて、ニボルマブ(360mgを3週に1回または240mgを2週に1回)と化学療法(カペシタビンとオキサリプラチンを3週に1回またはロイコボリンとフルオロウラシルとオキサリプラチンを2週に1回)、ニボルマブとイピリムマブ、または化学療法単独に無作為に割り付けられました(ブロックサイズは6)。ニボルマブと化学療法の併用療法と化学療法単独療法の主要評価項目は、PD-L1複合陽性スコア(CPS)が5以上の腫瘍を有する患者を対象に、盲検化された独立中央審査によるOSまたは無増悪生存期間(PFS)とした。安全性は、割り当てられた治療を少なくとも1回受けたすべての患者で評価しました。本試験はClinicalTrials. gov, NCT02872116に登録されています。 【結果】2017年3月27日から2019年4月24日までに、適格性を評価した2687名の患者のうち、1581名の患者を治療(ニボルマブ+化学療法[n=789、50%]または化学療法単独[n=792、50%])に同時無作為に割り付けました。OSの追跡期間中央値は、ニボルマブと化学療法の併用で13-1カ月(IQR 6-7-19-1)、化学療法単独で11-1カ月(5-8-16-1)であった。PD-L1 CPSが5以上の患者(最低追跡期間12-1カ月)において、ニボルマブと化学療法の併用により、OS(ハザード比[HR]0-71[98-4%CI 0-59-0-86]、p<0-0001)およびPFS(HR 0-68[98%CI 0-56-0-81]、p<0-0001)が化学療法単独に比べて有意に改善しました。さらに、PD-L1 CPSが1以上の患者および無作為に割り付けられたすべての患者において、PFSの有益性とともに、OSの有意な改善が認められました。治療を受けた全患者のうち、ニボルマブと化学療法の併用群では782人中462人(59%)、化学療法単独群では767人中341人(44%)にグレード3~4の治療関連有害事象が認められました。両群で最も多かった(25%以上)グレードの異なる治療関連有害事象は、吐き気、下痢、末梢神経障害でした。ニボルマブと化学療法併用群で16名(2%)、化学療法単独群で4名(1%)の死亡が治療関連死とされた。 【解釈】ニボルマブは、前治療歴のない進行性胃癌、胃食道接合部癌、食道腺癌患者において、化学療法との併用により、化学療法単独と比較して、優れたOS、PFSのベネフィット、許容できる安全性プロファイルを示した初めてのPD-1阻害剤です。ニボルマブと化学療法の併用は、これらの患者さんに対する新しい標準的な第一選択治療となります 【資金提供】ブリストル・マイヤーズ スクイブ社、小野薬品工業株式会社との共同研究。 第一人者の医師による解説 新たな標準1次治療としてニボルマブ+化学療法を支持する結果 堀江 沙良、浜本 康夫(准教授) 慶應義塾大学医学部腫瘍センター MMJ. April 2022;18(2):45 ヒト上皮増殖因子受容体(HER2)陰性の切除不能・再発進行胃がんに対する化学療法による1次治療後の全生存期間(OS)は1年以下であり、予後不良である。CheckMate(CM)649試験は日本を含む29カ国で行われた切除不能な進行・再発胃がんを対象にニボルマブ(抗 PD-1抗体)+化学療法群の化学療法群に対する優越性を検討した第3相非盲検無作為化対照試験である。主要評価項目はPD-L1発現状況の指標 CPS(combined positive score)5以上の集団におけるOSと無増悪生存期間(PFS)であった。 CPS 5以上の集団のOS中央値はニボルマブ+化学療法群14.4カ月、化学療法群11.1カ月(ハザード比[HR], 0.71;P<0.0001)であり、ニボルマブ+化学療法群はOSで統計学的に有意な延長を示した。PFS中央値はニボルマブ+化学療法群7.7カ月、化学療法群6.0カ月(HR, 0.68;P<0.0001)であり、ニボルマブ+化学療法群で有意に長かった。CPS 1以上の集団および全体集団でもニボルマブ+化学療法群と化学療法群のOS中央値はそれぞれ14.0カ月と11.3カ月(HR, 0.77;P<0.0001)、13.8カ月と11.6カ月(HR, 0.80;P=0.0002)であり、いずれの集団でもニボルマブ+化学療法群の優越性が示された。PFSに関しても、両集団ともニボルマブ+化学療法群でPFS延長効果が示された。全体集団におけるグレード 3/4治療関連有害事象はニボルマブ+化学療法群では59%、化学療法群では44%に認められた。 CM 649試験の結果ではHER2陰性の進行・再発胃がん患者において、ニボルマブと化学療法の併用によってCPSに関わらずOSとPFSの延長が示された。本試験と同様、進行・再発胃がんの1次治療においてニボルマブ+化学療法を化学療法単独と比較したアジアのATTRACTION-4(AT-4)試験では、ニボルマブ併用によりPFSは有意に延長したが、OSの有意な延長は示されなかった。CM649試験との結果の違いは主に後治療の差によると考えられている。後治療の実施率はCM649試験の39%に対し、AT-4試験では66%と高く、特に日本人サブセットでは3次治療以降でニボルマブが広く使用されていた。CM649試験の結果は新たな標準1次治療としてニボルマブを支持するものであるが、高齢者や全身状態不良例における併用療法の適性、長期使用に伴う毒性についてはさらに検証が必要である。なお、PD-L1発現によりニボルマブの上乗せ効果が異なる傾向が示唆されていることから、日本胃癌学会の速報では可能な限りPD-L1検査を行うことが望ましいとし、CPS5未満では3次治療以降での使用機会の可能性を考慮し、治療戦略を慎重に検討する必要があるとしている。
無症候性頸動脈高度狭窄症へのCEAとCAS 有用性に差はない
無症候性頸動脈高度狭窄症へのCEAとCAS 有用性に差はない
Second asymptomatic carotid surgery trial (ACST-2): a randomised comparison of carotid artery stenting versus carotid endarterectomy Lancet. 2021 Sep 18;398(10305):1065-1073. doi: 10.1016/S0140-6736(21)01910-3. Epub 2021 Aug 29. 上記論文のアブストラクト日本語訳 ※ヒポクラ×マイナビ 論文検索(Bibgraph)による機械翻訳です。 【背景】重度の頸動脈狭窄を有し、最近脳卒中や一過性脳虚血を発症していない無症状の患者においては、頸動脈ステント留置術(CAS)や頸動脈内膜剥離術(CEA)は、開存性を回復し長期脳卒中リスクを軽減することができる。しかし、最近の全国的な登録データでは、いずれの治療法も約1%の脳卒中や死亡のリスクをもたらすとされている。ACST-2はインターベンションが必要と考えられる重度の狭窄を有する無症候性患者を対象に,CASとCEAを比較する国際多施設共同無作為化試験であり,他のすべての関連試験と比較して解釈されたものである。対象は、片側または両側の重度の頸動脈狭窄があり、医師と患者の両方が頸動脈の処置を行うべきであることに同意しているが、どちらを選択するかはかなり不確かな患者であった。患者はCASとCEAに無作為に割り付けられ、1ヵ月後とその後毎年、平均5年間フォローアップされた。手続き上のイベントは、手術後30日以内のものを対象とした。Intention-to-treatの解析が行われた。手技の危険性を含む解析は表形式を使用。非手術的な脳卒中の解析およびメタ解析にはKaplan-Meier法およびlog-rank法を用いた。本試験はISRCTNレジストリ、ISRCTN21144362に登録されている。 【所見】2008年1月15日から2020年12月31日までに、130施設で3625例がランダムに割り付けられ、CASに1811例、CEAに1814例、コンプライアンスも良好、内科治療も良好で平均5年の追跡調査である。全体として、手術中に障害を伴う脳卒中または死亡が1%(CAS群15例、CEA群18例)、手術中に障害を伴わない脳卒中が2%(CAS群48例、CEA群29例)であった。5年間の非手術的脳卒中のKaplan-Meier推定値は、致死的脳卒中と障害性脳卒中が各群2~5%、あらゆる脳卒中がCASで5~3%、CEAで4~5%(率比[RR]1~16、95%CI 0-86~1-57、p=0~33)であった。CASとCEAのすべての試験で、あらゆる非手術的脳卒中のRRを合わせると、症状のある患者と症状のない患者でRRは同等であった(全体RR 1-11, 95% CI 0-91-1-32; p=0-21)。 【解釈】能力のあるCASとCEAでは重大な合併症は同様に少なく、これら二つの頸動脈手術が致命的または障害のある脳卒中に及ぼす長期効果は同等である【FUNDING】英国医学研究評議会と健康技術評価計画.頸動脈の手術は、頸動脈の手術と同様に、頸動脈の手術と同様に致命的または障害のある脳卒中を引き起こす。 第一人者の医師による解説 患者ごとのリスク評価の重要性は揺るがない 髙橋 淳 近畿大学医学部脳神経外科主任教授 MMJ. April 2022;18(2):38 頚部頚動脈狭窄症は脳梗塞の原因の1つである。これに対する頚動脈内膜剥離術(CEA)は複数のランダム化試験(1991 ~ 95年)を経て、症候性、無症候性のいずれも単独内科的治療に対する優位性が確立された。一方、1990年代以降普及した頚動脈ステント留置術(CAS)については、CEAを対照とする非劣性試験が実施されてきた。SAPPHIRE(北米、2004年、症候性 + 無症候性)はCEA高リスク群でCASの非劣性を証明したが、CEA低リスク群では欧州の2試験でこれを証明できず、CREST(北米、2010年、症候性 + 無症候性)でようやく非劣性が示された。無症候性限定の試験としては、ACT-1(米国、2016年)がCEA低リスク群におけるCASの非劣性を示した(1)。 本論文は、「無症候性高度狭窄に対するCEAとCASの効果を十分な症例数で比較すること」を目的とした、英国を拠点とする多施設共同ランダム化試験(ASCT-2)の報告である。「頚動脈高度狭窄に対して介入の適応がありCEAとCASの選択に迷う例」を対象とした。33カ国130施設から患者3,625人を登録、CASまたはCEAにランダムに割り付け、平均5年間観察した。主要評価項目は①30日以内の手術関連死 +脳卒中、②その後の手術非関連脳卒中である。 その結果、術後30日以内の手術関連死 +重度脳卒中(6カ月後 modified Rankin Scale[mRS]スコア 3〜5)はCAS群0.9%、CEA群1.0%(P=0.77)、手術関連死 +全脳卒中 はCAS群3.7 %、CEA群2.7%(P=0.12)で、いずれも有意差なし。観察期間中(手術非関連)においては、致死的 / 重度脳卒中が両群ともに2.5%、全脳卒中がCAS群5.3%、CEA群4.5%(P=0.33)で有意差なし。手術非関連脳卒中について過去の比較研究と合算しても、CASのリスク比は1.11でCEAと差がなかった。 重篤な術後合併症は両群ともに僅少で、長期の脳卒中発生率にも差がなく、有用性は同等とされた。しかし本試験は最新・最良の内科的治療との比較試験ではない。医療経済学的検討もなされていない(CASは材料費が高額)。また何よりも、「患者状態や病変特性から見て、明らかにどちらかに適する例は対象に含まない」ことに注意が必要であり、個々の患者におけるリスク評価の重要性が軽視されてはならない。今回の結果は、「どちらを選択するかが同等の局面において、自由な選択に一定のお墨付きを与える」と言うべきものである。 1. Rosenfield K, et al. N Engl J Med. 2016;374(11):1011-1020.
尿中Na排泄、血圧、心血管疾患、死亡:地域単位の前向き疫学的コホート研究
尿中Na排泄、血圧、心血管疾患、死亡:地域単位の前向き疫学的コホート研究
Urinary sodium excretion, blood pressure, cardiovascular disease, and mortality: a community-level prospective epidemiological cohort study LANCET 2018 Aug 11;392(10146):496-506. 上記論文のアブストラクト日本語訳 ※ヒポクラ×マイナビ 論文検索(Bibgraph)による機械翻訳です。 【背景】WHOは、心血管疾患の予防対策として、国民のナトリウム摂取量を2g/日未満とすることを推奨しているが、この目標はどの国でも達成されていない。この勧告は主に血圧(BP)の短期試験による個人レベルのデータに基づいており、無作為化試験や観察研究による低ナトリウム摂取と心血管イベントの減少との関連データはない。我々は、地域レベルの平均ナトリウムおよびカリウム摂取量と心血管疾患および死亡率との関連を調査した。 【方法】Prospective Urban Rural Epidemiology研究は、21カ国で進行中である。ここでは、18カ国で行われた臨床転帰のデータを用いた解析を報告する。対象者は、一般住民から抽出された35~70歳の心血管疾患のない成人である。摂取量の代用として、朝の空腹時尿を用いて24時間のナトリウムとカリウムの排泄量を推定した。369地域(すべて50人以上)でナトリウムとカリウムの摂取量と血圧との地域レベルの関連を,255地域(すべて100人以上)で心血管疾患と死亡率との関連を評価し,個人レベルのデータを用いて既知の交絡因子について調整した。 【所見】369地域の95 767人が血圧について,255地域の82 544人が心血管予後の評価を行い,中央値は8~1年間フォローアップされた。中国では103地域中82地域(80%)が平均ナトリウム摂取量が5g/日を超えていたが,その他の国では266地域中224地域(84%)が平均3~5g/日であった。全体として、平均収縮期血圧は平均ナトリウム摂取量が1g増加するごとに2~86mmHg上昇したが、正の関連はナトリウム摂取量の最高三分位の地域でのみ見られた(異質性についてはp<0-0001)。平均ナトリウム摂取量と主要な心血管イベントとの関連は、ナトリウム摂取量の最低三分位で有意な逆相関が見られ(最低三分位<4-43 g/日、平均摂取量4-04 g/日、範囲3-42-4-43;1000年当たりの変化-1-00イベント、95%CI -2-00~-0-01, p=0497)、中間三分位では関連がなく(中三分位 4-43-5-08 g/日、平均取得4-70 g/日, 4-44-5.05;1000年当たりの変化0-24イベント、-2-12〜2-61、p=0-8391)、最高三分位では正の関連があるが有意ではない(最高三分位>5-08g/日、平均摂取量5-75g/日、>5-08〜7-49;1000年当たりの変化0-37イベント、-0-03〜0-78、p=0-0712)。中国(平均ナトリウム摂取量5-58 g/日,1000年当たり0-42イベント,95%CI 0-16~0-67,p=0-0020) では,他の国(4-49 g/日,-0-26イベント,-0-46~-0-06,p=0-0124;異質性についてはp<0-0001) に比べ脳卒中に強い関連が見られた.すべての主要な心血管アウトカムは,すべての国でカリウム摂取量の増加とともに減少した。 【解釈】ナトリウム摂取は,平均摂取量が5 g/日を超える地域でのみ心血管疾患および脳卒中と関連していた。これらの地域や国ではナトリウムを減らすが、他の地域では減らさないという戦略が適切かもしれない。 【FUNDING】人口健康研究所、カナダ保健研究所、カナダ保健省患者指向研究戦略、オンタリオ保健長期ケア省、オンタリオ心臓・脳卒中財団、欧州研究評議会。 第一人者の医師による解説 厳格な減塩と心血管疾患の関連 ランダム化試験での検証が必要 桑島 巖 NPO 法人臨床研究適正評価教育機構理事長 MMJ.February 2019;15(1):13 本論文は21カ国を対象として8.1年間追跡した大 規模疫学コホート研究 PURE(Prospective Urban Rural Epidemiology)研究の結果報告である。Na摂取量が1日摂取量5g(食塩12.7g)を超える地域(おもに中国)では、食塩摂取量増加につれて心血管イベントが上昇するが、摂取量平均4.04g(食塩10.3g)以下の低摂取群でもリスクが上昇する、いわゆる“Jカーブ現象”がみられたというのが趣旨である。厚生労働省や健康日本21が目標とする食塩摂取量1日 8g未満ではむしろリスクが上昇するということになる。本論文の特徴は、中国の地域社会でのデータが約40%を占め、そのうち80%の地域でNa摂取量 が1日5g(食塩12.7g)以上であり、平均摂取量が他の地域よりも際だって高い点である(Na 5.58g 対 4.45g;食塩14.2g 対 11.3g)。すなわち食塩摂取量とリスクの関係には地域性(community-based)が関連していることを示している。地域差が大きいことは人種による食塩感受性の違いも関連している可能性もあり、このcommunity-basedの結果を日本人 一般社会に適用することには慎重でなければならない。食塩摂取量と心血管イベントのJカーブ現象の報告は2011年のEPOCH研究(1)やO’Donnellらの観 察研究(2)でも報告されており、疫学的には真実なのかもしれない。しかし重要なことは、このような疫学観察研究の結果は、リスクと食塩摂取の因果関係を示すものではなく、減塩によるリスク低減効果の有無は高血圧や心血管疾患既往の有無といった個人的背景でも異なることを理解しておく必要があろう。食塩の低摂取群で心血管合併症が多い理由の1 つとして減塩によるレニン-アンジオテンシン系の亢進が心血管イベントを増やす可能性は否定できない。さらに低Na血症自体が慢性疾患や栄養障害を反映した結果である可能性もある。ただしK摂取と心血管イベントは逆相関するとの結果は、世界のガイドラインと一致する。 食塩摂取量と血圧あるいは心血管イベントとの関連の研究間で異なった結果がでる要因の1つは、尿中Na排泄量の測定方法である。24時間蓄尿から測定するのが標準であるが、PURE研究では早朝スポット尿を用いて1日Na排泄量を推定しているが、 信頼性の限界がある。 2013年に発表されたランダム化試験のメタアナリシス結果(3)では人種、高血圧の有無にかかわらず 食塩摂取量と血圧の関連は直線的であり、1日6g未満の減塩で収縮期血圧5.8mmHgが低下すると報告しており、やはり世界基準と矛盾しない結果を示している。しかしこの論文は、心血管イベントとの関係を示したメタアナリシスではない。日本人は食塩 摂取が過度な人種であり、減塩が心血管イベントや死亡を増やすか否かに関してはより高いエビデンスによる検証に期待したい。 1. Stolarz-Skrzypek K, et al., JAMA. 2011;305(17):1777-1785. 2. O'Donnell MJ, et al., JAMA. 2011;306(20):2229-2238. 3. He FJ, et al., BMJ. 2013;346:f1325.
心血管イベントの予測を目的としたベースラインおよびスタチン療法中のリポ蛋白(a) 値:スタチンに関する心血管アウトカム試験の患者個別データを用いたメタアナリシス
心血管イベントの予測を目的としたベースラインおよびスタチン療法中のリポ蛋白(a) 値:スタチンに関する心血管アウトカム試験の患者個別データを用いたメタアナリシス
Baseline and on-statin treatment lipoprotein(a) levels for prediction of cardiovascular events: individual patientdata meta-analysis of statin outcome trials LANCET 2018 Oct 13;392(10155):1311-1320. 上記論文のアブストラクト日本語訳 ※ヒポクラ×マイナビ 論文検索(Bibgraph)による機械翻訳です。 【背景】リポ蛋白(a)の上昇は、一般人口調査において、心血管疾患の遺伝的危険因子である。 【方法】7つの無作為化プラセボ対照スタチンアウトカム試験から得られた患者レベルのデータを照合し,調和させて,致死的または非致死的冠動脈心疾患,脳卒中,血行再建術と定義した心血管イベントのハザード比(HR)を算出した。心血管イベントのHRは、あらかじめ定義されたリポ蛋白(a)群(15~<30mg/dL、30~<50mg/dL、≧50mg/dL、<15mg/dL)ごとに各試験内で推定され、多変量ランダム効果メタ解析により推定値がプールされた。 【結果】解析には、リポ蛋白(a)測定を繰り返し行った患者29069人(平均年齢62歳[SD 8]、女性8064人[28%]、リスク95 576人年中に5751件のイベント)のデータが含まれた。)スタチン治療の開始により、LDLコレステロールは減少したが(平均変化率 -39% [95% CI -43 to -35])、リポ蛋白(a)には有意な変化がなかった。ベースラインおよびスタチン治療中のリポ蛋白(a)と心血管疾患リスクとの関連はほぼ線形であり、ベースラインのリポ蛋白(a)は30 mg/dL以上、スタチン治療中のリポ蛋白(a)は50 mg/dL以上でリスクが増加した。ベースラインのリポ蛋白(a)について、年齢と性別で調整したHR(対15mg/dL)は、15mg/dL以上30mg/dL未満で1-04(95%CI 0-91-1-18)、30mg/dL以上50mg/dL未満で1-11(1-00-1-22)、50mg/dL以上では1-31(1-08-1-58)であった。オンスタチンリポ蛋白(a)のHRは0-94(0-81-1-10)、1-06(0-94-1-21)、1-43(1-15-1-76)であった。心血管疾患の既往、糖尿病、喫煙、収縮期血圧、LDLコレステロール、HDLコレステロールでさらに調整しても、HRはほぼ同じであった。スタチン投与によるリポ蛋白(a)と心血管疾患リスクとの関連は、プラセボ投与によるリポ蛋白(a)よりも強く(相互作用p=0-010)、若年でより顕著であり(相互作用p=0-008)、他の患者レベルまたは試験レベルの特性による効果修飾はなかった。 【解釈】このスタチン投与患者の個人-患者データのメタ解析では、ベースラインおよびスタチン投与によるリポ蛋白(a)の上昇により、独立に、心血管疾患リスクとほぼ直線関係が示される。本研究は、心血管疾患アウトカム試験において、リポ蛋白(a)低下仮説を検証する根拠を提供する。 第一人者の医師による解説 リポ蛋白(a)高値の残余リスク示し 低下療法の妥当性を裏付け 田中 正巳(特任講師)/伊藤 裕(教授) 慶應義塾大学医学部腎臓内分泌代謝内科 MMJ.February 2019;15(1):15 スタチンによるLDLコレステロール低下療法が 心血管疾患を抑制することには今や議論の余地はない。しかしその抑制率は50%にも満たず、多くの残余リスクの存在が想定されている。本論文は、高リポ蛋白(a)血症がスタチン療法中の残余リスクであることを明らかにするとともに、リポ蛋白(a)低下療法の妥当性に言及している。 高リポ蛋白(a)血症が心血管疾患の危険因子であることは以前より知られていたが、この知見は主に 1次予防におけるエビデンスに基づいており、2次予防やスタチン療法中の患者でも危険因子であるのかは不明であった。本研究は、このような患者でもリポ蛋白(a)値が高まると直線的に心血管リスクが 高まることを明確に示した。さらに、スタチンはリ ポ蛋白(a)値を変化させず、プラセボよりもスタチン療法中の患者においてリポ蛋白(a)はより重要な危険因子である可能性を示した。本研究の最大の強みは、豊富な患者数(n = 29,069)、イベント数(n = 5,751)に基づく統計学的パワーである。従来の研究とは異なり、リポ蛋白(a)> 50 mg/dLの患者を多く含んでいる。したがって、スタチン療法中の高リポ蛋白(a)血症が心血管疾患の残余リスクであることは確からしいと言えるだろう。 本研究は無作為化対照試験のメタアナリシス、すなわちpost-hoc解析であるため、その結果の解釈には注意が必要である。日本の「動脈硬化性疾患予防ガイドライン2017年版」をひもとくと、「高リポ蛋白(a)血症が動脈硬化性疾患の危険因子である」とのステートメントには「E-1a(コホート研究のメタアナリシス)」という高いエビデンスレベルを付与し ている。またリポ蛋白(a)の遺伝子異常(1塩基多型) を有する患者では、血中リポ蛋白(a)値と冠動脈疾 患リスクが高いことが研究によって示されている(1) 。 これらの知見と本研究の結果を併せて考えると、高リポ蛋白(a)血症が原因、心血管疾患が結果という 因果関係の成り立つ可能性が一層高まったと言えよう。 そうなると次のステップは本論文に書かれているとおり、高値のリポ蛋白(a)を低下させた場合に心 血管疾患を実際に予防できるか証明することであ る。スタチン、エゼチミブ、フィブラートはリポ蛋 白(a)値を下げることはできず、低下作用が報告されていたPCSK9阻害薬も最近の臨床試験では低下作用を認めなかった(2) 。一方、アポリポ蛋白(a)のア ンチセンス薬を用いた第1相、第2相試験の結果が報告されている(3),(4)。リポ蛋白(a)値は用量依存的に大きく低下し、安全性には問題を認めなかったことより、このアンチセンス療法は有望視されている。 本論文は、リポ蛋白(a)低下療法に関する前向き臨床試験を行う「お墨付き」を与えたと言えよう。安全で効果的な薬剤が開発されれば、スタチン療法にもかかわらず依然として心血管疾患の残余リスクが高い患者にとって大きな福音となる。 1. Clarke R, et al. N Engl J Med. 2009;361(26):2518-2528. 2. Roth EM, et al. Int J Cardiol. 2014;176(1):55-61. 3. Tsimikas S, et al. Lancet. 2015;386(10002):1472-1483. 4. Viney NJ, et al. Lancet. 2016;388(10057):2239-2253.
侵襲的治療を受けた急性冠症候群患者における橈骨動脈と大腿動脈アクセス、ビバリルジ ンと未分画ヘパリンの比較(MATRIX):多施設共同、無作為化対照試験1年目の最終結果
侵襲的治療を受けた急性冠症候群患者における橈骨動脈と大腿動脈アクセス、ビバリルジ ンと未分画ヘパリンの比較(MATRIX):多施設共同、無作為化対照試験1年目の最終結果
Radial versus femoral access and bivalirudin versus unfractionated heparin in invasively managed patients with acute coronary syndrome (MATRIX): final 1-year results of a multicentre, randomised controlled trial LANCET 2018 Sep 8;392(10150):835-848. 上記論文のアブストラクト日本語訳 ※ヒポクラ×マイナビ 論文検索(Bibgraph)による機械翻訳です。 【背景】Minimizing Adverse Haemorrhagic Events by Transradial Access Site and Systemic Implementation of Angiox(MATRIX)プログラムは、侵襲的治療を受ける急性冠症候群の患者において、径方向アクセス法と大腿動脈アクセス法の比較、および糖タンパク質IIb/IIIa阻害剤を選択した二価イルジンと未分画ヘパリンとの安全性と有効性を評価するために計画されました。MATRIXは,イタリア,オランダ,スペイン,スウェーデンの78施設で急性冠症候群患者を対象とした3つのネステッド無作為化多施設共同非盲検優越試験からなるプログラムであった。ST上昇型心筋梗塞患者を対象に、冠動脈造影前に橈骨または大腿動脈アクセス、および経皮的冠動脈インターベンション後の輸液または未分画ヘパリン投与(1段階選択)ありまたはなしのビバリルジンに同時に無作為(1:1)割り付けを行った。非ST上昇型急性冠症候群の患者を冠動脈造影前に橈骨または大腿動脈アクセスに無作為に(1:1に)割り付け,造影後に経皮的冠動脈インターベンションが可能と判断された場合のみ(2段階選択),アンチトロンビンの種類と治療期間のプログラムに参加させた。無作為化配列はコンピュータで作成され、ブロックされ、P2Y12阻害薬の新規または現在の使用目的(clopidogrel vs ticagrelor or prasugrel)、急性冠症候群のタイプ(ST上昇型心筋梗塞、トロポニン陽性、トロポニン陰性の非ST上昇型急性冠症候群)により層別化された。Bivalirudinは0〜75mg/kgのボーラス投与後,経皮的冠動脈インターベンション終了まで1時間あたり1〜75mg/kgの点滴を行った。ヘパリンは糖蛋白IIb/IIIa阻害剤非投与例では70〜100単位/kg,糖蛋白IIb/IIIa阻害剤投与例では50〜70単位/kgで投与された。臨床的なフォローアップは30日後と1年後に行われた。MATRIXアクセスおよびMATRIXアンチトロンビン型に関する共同主要アウトカムは、30日までの全死亡、心筋梗塞、脳卒中の複合と定義した主要有害心血管イベント、および非冠動脈バイパス移植関連大出血、30日までの主要有害心血管イベントの複合と定義した臨床有害事象であった。MATRIXの治療期間に関する主要アウトカムは、緊急標的血管再血行再建術、明確なステント血栓症、または30日までの純有害臨床イベントの複合としました。解析はintention-to-treatの原則に従って行われました。本試験はClinicalTrials. gov(番号NCT01433627)に登録されている。 【所見】2011年10月11日から2014年11月7日の間に、8404人の患者を橈骨アクセス(4197人)または大腿アクセス(4207人)にランダムに割付た。この8404人のうち,7213人がMATRIXアンチトロンビン型研究に含まれ,ビバリルジン(3610人)またはヘパリン(3603人)に無作為に割り付けられた。ビバリルジンに割り付けられた患者はMATRIX治療期間試験に組み入れられ、無作為に処置後輸液あり(1799例)または処置後輸液なし(1811例)に割り付けられた。1年後、主要な有害心血管イベントは、橈骨アクセスに割り付けられた患者と大腿骨アクセスに割り付けられた患者で差がなかったが(14-2% vs 15-7%、率比0-89、95%CI 0-80-1-00、p=0-0526)、正味の有害臨床イベントは、大腿骨アクセスよりも橈骨アクセスの方が少なかった(15-2% vs 17-2%、 0-87、 0-78-0-97; p=0-0128)。ヘパリンと比較して、ビバリルジンは主要な心血管有害事象(15-8%対16-8%; 0-94, 0-83-1-05; p=0-28)および臨床上の純有害事象(17-0%対18-4%; 0-91, 0-81-1-02; p=0-10)とは関連がなかった。緊急標的血管再血行再建術,ステント血栓症,臨床的有害事象の複合は,術後のバイバルジン点滴の有無にかかわらず差がなかった(17-4% vs 17-4%;0-99, 0-84-1-16;p=0-90). 【解釈】急性冠症候群患者において,径方向アクセスは1年後の主要有害心臓イベントではなく,大腿動脈アクセスと比較して,純有害事象率が低いとされた.ビバリルジンの投与と投与後の点滴の有無は、主要な有害心血管イベントや臨床的な有害事象の発生率の低下と関連はなかった。イタリア侵襲的心臓病学会,The Medicines Company,Terumo,Canada Research Chairs Programme. 第一人者の医師による解説 出血リスク軽減も 実臨床では患者に応じた対応が必要 古賀 聖士(病院講師)/前村 浩二(教授) 長崎大学病院循環器内科 MMJ.February 2019;15(1):17 近年、PCI時の橈骨動脈アクセスと大腿動脈アクセスを比較した多数のランダム化比較試験が行われ、橈骨動脈アクセスでは出血性イベント、特にアクセスサイト関連の出血が有意に少ないことが報告されている(1) 。その結果に基づき、現在欧州のPCI ガイドラインでは、橈骨動脈は第1選択のアクセス サイトとして推奨されている(推奨度I、エビデンス レベルA)(2) 。しかし、ほとんどの研究が30日程度の 短期予後をみたものであり、橈骨動脈アクセスと長期予後の関連については明らかとなっていなかった。 本試験は、大腿動脈アクセスに対する橈骨動脈アクセスの長期的な安全性と有効性を検証するために実施された。なお、本試験は未分画ヘパリンに対するビバリルジン(日本未承認の抗トロンビン薬)の安全性と有効性も検証するデザインになっている。 結果であるが、1年の追跡で、橈骨動脈アクセス 群と大腿動脈アクセス群の間で主要心血管イベント (全死亡、心筋梗塞または脳卒中)発生率に有意差はなかったが、総臨床的有害事象(NACE;主要心血管イベントおよびnon-CABG関連大出血)は橈骨動脈 アクセス群の方が有意に少なかった。これには、橈骨動脈アクセス群では大出血、特にアクセスサイト 関連の出血(0.4% 対 1.1%)が有意に少なかったことが影響していた。また、この差は30日までに認 められ、31日から1年までのNACE発生率に有意差はなかった。なお、ビバリルジン群とヘパリン群で1年の主要心血管イベントおよびNACEに有意差はなかったが、ビバリルジン群は大出血(2.2% 対 3.3%)が有意に少なかった。 以上より、本研究はACS患者のPCIにおいて橈骨動脈は最も望ましいアクセスサイトであると結論づけている。また、ビバリルジンを使用した橈骨動 脈アクセスのPCIは、より出血性イベントを軽減で きる可能性があることについても触れられている。 日本においても、可能であれば橈骨動脈アクセスを第1選択にすることに、ほぼ異議はないであろう。 しかしACS患者では、ショックバイタルのため橈 骨動脈の触知が弱くアクセスが困難な患者があり、 また高齢の患者で鎖骨下動脈や腕頭動脈の蛇行のためカテーテル操作が困難な場合、大腿動脈アクセス の方が迅速にPCIを行える患者も経験する。また複 雑病変へのPCIで大口径カテーテルが必要な場合には橈骨動脈は不向きである。ガイドラインや臨床研 究の結果から橈骨動脈アクセスに固執するのではなく、患者に応じてアクセス部位を使い分ける臨機応変な対応が実臨床の場では求められるであろう。 1. Ferrante G, et al. JACC Cardiovasc Interv. 2016;9(14):1419- 1434. 2. Neumann FJ, et al. Eur Heart J 2018. Aug 25. doi:10.1093/ eurheartj/ehy394.
コントロール不十分な1型糖尿病におけるハイブリッドクローズドループ型インスリンデ リバリー:多施設共同、12週間無作為化試験
コントロール不十分な1型糖尿病におけるハイブリッドクローズドループ型インスリンデ リバリー:多施設共同、12週間無作為化試験
Closed-loop insulin delivery in suboptimally controlled type 1 diabetes: a multicentre, 12-week randomised trial LANCET 2018 Oct 13;392(10155):1321-1329. 上記論文のアブストラクト日本語訳 ※ヒポクラ×マイナビ 論文検索(Bibgraph)による機械翻訳です。 【背景】1型糖尿病患者にとって、血糖コントロールの達成は依然として困難である。6歳以上の最適にコントロールされていない1型糖尿病患者を対象に、センサー付きポンプ療法と比較した昼夜ハイブリッド閉ループインスリン投与の有効性を評価した。 【方法】このオープンラベル、多施設、多国籍、単一期間、並行無作為化対照試験では、英国の4病院と米国の2施設の糖尿病外来クリニックから参加者を募集した。インスリンポンプで治療を受けている6歳以上の1型糖尿病で,血糖コントロールが最適でない(糖化ヘモグロビン[HbA1c] 7-5-10-0%) 参加者を12週間の自由生活でハイブリッド閉ループ療法またはセンサー補強型ポンプ療法にランダムに割付けた。インスリンポンプと持続的グルコースモニタリングのトレーニングは4週間のランイン期間中に行われた。対象者は中央無作為化ソフトウェアにより無作為に割り付けられた。2群への割り付けは非盲検下で行われ、無作為化は施設内でHbA1cが低い(8~5%未満)か高い(8~5%以上)かで層別化された。主要評価項目は、無作為化後12週目にグルコース濃度が3-9-10-0mmol/Lの目標範囲内に収まった時間の割合とした。主要評価項目と安全性評価項目の解析は、無作為化されたすべての患者さんで行われました。本試験はClinicalTrials. govに登録されており、番号はNCT02523131、募集は終了している。 【所見】2016年5月12日から2017年11月17日まで、114名がスクリーニングされ、適格患者86名がハイブリッド閉ループ療法(n=46)またはセンサー補強ポンプ療法(n=40;対照群)にランダムに割り当てられた。グルコース濃度が目標範囲内にあった時間の割合は、コントロール群(54%、SD9、平均変化率差10-8%ポイント、95%CI 8-2~13-5、p<0-0001)と比較して、クローズドループ群で有意に高かった。クローズドループ群では,HbA1cがスクリーニング値の8-3%(SD 0-6)から4週間のランイン後に8-0%(SD 0-6)へ,12週間の介入期間後に7-4%(SD 0-6)へ低下した.対照群では,HbA1c値はスクリーニング時に8-2%(SD 0-5),ランイン後に7-8%(SD 0-6),介入後に7-7%(SD 0-5)であり,HbA1c割合の減少は対照群と比較して閉ループ群では有意により大きかった(変化の平均差 0-36%,95%CI 0-19 ~ 0-53;p<0-0001)。グルコース濃度が3-9mmol/L未満(変化の平均差-0-83%ポイント、-1-40から-0-16;p=0-0013)および10-0mmol/L以上(変化の平均差-10-3%ポイント、-13-2から-7-5;p<0-0001)で過ごした時間は、コントロール群と比較してクローズドループ群で短くなっていました。センサーで測定したグルコースの変動係数は,介入間で差がなかった(変化の平均差 -0-4%,95% CI -1-4%~0-7%;p=0-50).同様に,1日の総インスリン量にも差はなく(変化の平均差0-031 U/kg/日,95% CI -0-005~0-067;p=0-09), 体重にも差はなかった(変化の平均差0-68 kg,95% CI -0-34~1-69;p=0-19 ).重篤な低血糖は発生しなかった。クローズドループ群では輸液セットの不具合により糖尿病性ケトアシドーシスが1件発生した。その他の有害事象は、クローズドループ群で13件、対照群で3件であった。 【解釈】ハイブリッドクローズドループインスリン投与は、最適にコントロールされていない1型糖尿病患者の幅広い年齢層で低血糖のリスクを低減しながら血糖コントロールを改善する。 【FUNDING】JDRF、NIHR、Wellcome Trust。 第一人者の医師による解説 さらなる改良が望まれるインスリンデリバリーシステム 松久 宗英 徳島大学先端酵素学研究所糖尿病・臨床研究開発センターセンター長・教授 MMJ.February 2019;15(1):19 本研究はハイブリッドクローズドループインスリンポンプの有用性と安全性を示した研究である。 Cambridge 大学が開発した Day-and-night hybrid closed-loop(FlorenceD2A closed-loop)システムに、 ミニメド620G(Medtronic)とスマートフォンを連係させたシステムを使用している。先進的インスリン機器の検証は、入院患者を対象とした完全監視下 の閉鎖的環境から、目の行き届くサマーキャンプやホテル生活、そして最後に自由生活での実証へと段階的に進められる。本研究はその最終段階に相当し、 かつ最も良い適応と考えられる血糖管理不十分な幅広い年齢の1型糖尿病患者を対象に12週間行われた研究である。 今回使用されたクローズドループインスリンポンプはハイブリッド型と呼ばれ、安定した状態では CGMによる皮下間質ブドウ糖濃度に応じたインスリン持続注入が行われ、夜間に優れた血糖管理が得られる。しかし、現状のインスリン注入アルゴリズ ムでは、食事による急激な血糖上昇を十分に抑制できないことが示されている。この理由として、皮下 インスリンの作用時間の遅れ、皮下間質と血漿のブドウ糖濃度のずれ、食事量評価の自動化の困難さなど複数の要因がある。また、血糖管理手段がインスリンによる血糖降下ベクトルしかないことも大きな要因である。そこで、カーボカウントに基づく追加インスリン量を患者が自己注入する従来の方法を併用するため、ハイブリッド型とされる。 本研究では血糖管理不十分の1型糖尿病患者に対し、これまでのSAPと比較して、HbA1cを0.36% 低下させ、平均センサーブドウ糖濃度、目標ブドウ 糖濃度達成時間、SDでみた血糖変動の改善もみられ、安定した血糖改善効果が示された。一方、低血糖の頻度や重症低血糖は両群間で同等であった。また、注入トラブルで糖尿病ケトアシドーシスが発症したことから、安全性の面では従来のSAPを凌駕するには至っていない。 これからも、より作用発現の早いインスリン製剤 の開発や、血糖上昇ベクトルを持つグルカゴンの併 用注入が可能なバイオニックインスリンポンプ(1)などの開発が進められており、1型糖尿病のより良い 管理が実現し、合併症のかなりの抑制が期待される。 ただし、これら先進機器も一定の機器トラブルを起 こすリスク、医療コスト、操作の煩雑さなど最後まで課題の克服に向けた研究開発が必要とされる。 1. El-Khatib FH, et al. Lancet. 2017;389(10067):369-380.
肥満症患者の体重減少に対するセマグルチドの有効性と安全性をリラグルチドおよびプラセボと比較:無作為化、二重盲検、プラセボおよびアクティブコントロール、用量設定、第 2 相試験
肥満症患者の体重減少に対するセマグルチドの有効性と安全性をリラグルチドおよびプラセボと比較:無作為化、二重盲検、プラセボおよびアクティブコントロール、用量設定、第 2 相試験
Efficacy and safety of semaglutide compared with liraglutide and placebo for weight loss in patients with obesity: a randomised, double-blind, placebo and active controlled, dose-ranging, phase 2 trial Lancet 2018 Aug 25 ;392 (10148 ):637 -649 . 上記論文のアブストラクト日本語訳 ※ヒポクラ×マイナビ 論文検索(Bibgraph)による機械翻訳です。 【背景】 肥満は公衆衛生上の大きな問題であり、体重管理のための新しい医薬品が必要とされている。そこで、グルカゴン様ペプチド-1(GLP-1)アナログのセマグルチドについて、リラグルチド及びプラセボと比較して、体重減少を促進する有効性と安全性を評価しました。 【方法】無作為化、二重盲検、プラセボ及び活性対照、多施設、用量設定、第2相試験を実施しました。本試験は、71の臨床施設を含む8カ国で行われました。対象者は、糖尿病のない、肥満度(BMI)30kg/m2以上の成人(18歳以上)です。56のブロックサイズで、参加者を各活性治療群(セマグルチド[0-05 mg、0-1 mg、0-2 mg、0-3 mg、または0-4 mg;1日0-05 mgで開始し4週間ごとに段階的に増加]またはリラグルチド[3-0 mg;1日0-6 mgで開始し週0-6 mgごとに増加])または適合プラセボ群(活性治療群と同じ注入量と増加スケジュール)に6対1の比率でランダムに割り付けました。すべての投与量は1日1回、皮下注射で投与されました。参加者と治験責任医師は、割り付けられた治療法についてマスキングされたが、目標投与量についてはマスキングされなかった。主要評価項目は、52週目の体重減少率であった。主要解析はintention-to-treat ANCOVA法により行われ、欠損データはプラセボ群から抽出された。本試験はClinicalTrials. govに登録されており、番号はNCT02453711。 【FINDINGS】2015年10月1日から2016年2月11日の間に、957人がランダムに割り付けられた(有効治療群ごとに102~103人、プラセボプール群に136人が割り付けられた)。平均ベースライン特性は、年齢47歳、体重111-5kg、BMI39-3kg/m2などであった。体重データは、957人中891人(93%)から52週目に入手できた。推定平均体重減少率は、プラセボ群-2-3%に対し、セマグルチド群-6-0%(0-05mg)、-8-6%(0-1mg)、-11-6%(0-2mg)、-11-2%(0-3mg)、-13-8%(0-4mg)であった。セマグルチド群はプラセボ群に対してすべて有意であり(未調整p≦0-0010)、多重試験で調整後も有意であった(p≦0-0055)。セマグルチド0~2mg以上とリラグルチドの平均体重減少率は、いずれも有意であった(-13~8% vs -11~2% vs -7~8%)。推定体重減少率10%以上は、プラセボ投与群では10%、セマグルチド0-1mg以上投与群では37~65%に認められました(p<0-0001 vs プラセボ)。セマグルチドの全用量において、新たな安全性の懸念はなく、概ね良好な忍容性を示しました。最も一般的な有害事象は、GLP-1受容体作動薬で以前に見られたように、吐き気を主とする用量に関連した消化器症状であった。 【解釈】食事療法および身体活動に関するカウンセリングとの併用において、セマグルチドは52週間にわたり忍容性が高く、すべての用量でプラセボと比較して臨床的に適切な体重減少が認められた。 【FUNDING】Novo Nordisk A/S. 第一人者の医師による解説 糖尿病含む肥満関連疾患の発症抑制にも期待 脇 裕典 東京大学大学院医学系研究科糖尿病・代謝内科講師 MMJ.February 2019;15(1):21 新しいグルカゴン様ペプチド -1(GLP-1)受容体アゴニストであるセマグルチドは現在、週1回 注射薬が日本で2型糖尿病治療薬として承認されている。他のGLP-1受容体アゴニストと同様、セマグルチドは2型糖尿病の改善のみならず、体重減少効果が知られている。本研究は、オーストラリ ア、欧州、北米の2型糖尿病非合併の成人肥満(BMI 30kg/m2以上)患者957人を対象にセマグルチド 1日1回52週間投与による体重減少率を主要評 価項目に定め、体重減少効果と安全性に関してリラグルチドおよびプラセボと比較した多施設共同無 作為化二重盲検用量反応第2相試験である。 対象者の平均年齢 は47歳、平均体重 は111.5 kg、BMIは39.3kg/m2。プ ラ セ ボ 群 で は 体重 減少(減少率)は-2.48kg(-2.3%)であるの に 対 し、セ マ グ ル チ ド 0.05、0.1、0.2、0.3、 0.4mg群 で は そ れ ぞ れ -6.66kg(-6.0%)、 -9.34kg(-8.6%)、-12.30kg(-11.6%)、 -12.45kg( -11.2 %)、-15.15kg( -13.8 %) で あ り、そ の 差 は 有意 で あ っ た(P<0.001)。 10%以上の体重減少率がみられた患者はプラセ ボ 群10%に 対してセ マグ ル チド 群(0.1mg以 上)37~65%で差はいずれも有意であった(P< 0.0001)。セマグルチドの体重減少効果は52週 間にわたって観察された。 リラグルチド 3mg群との比較では、セマグル チ ド 群(0.2mg以上 )の 体重減少率( -13.8 ~ -11.2 %)は リ ラ グ ル チ ド 群 の 体重減少率 (-7.8%)よりも有意に大きかった(P≦0.05)。 主な副作用である嘔気の頻度はプラセボ群18%に 対し、セマグルチド群(0.05~0.4mg)で31~ 48%と高いが、リラグルチド 3mg群では45%で 忍容性は同程度であると考えられた。 本試験では、摂取カロリーで500 kcalの減少と 最低週150分の運動を指導されており、食事・運 動療法の併用が重要である可能性には留意したい。 セマグルチドは2型糖尿病の臨床試験(SUSTAIN 6)において心血管イベント(3 point‒MACE)を 有意に減少させたことが注目されている(1)。今回、 糖尿病非合併肥満患者の体重コントロールに有効な結果が示されたことから、糖尿病を発症していない肥満者において糖尿病を含む関連疾患の発症 抑制効果を有するかもしれない。現在、肥満症治療 薬としての第3相試験が進行中で、今後の結果が 期待される。 1. Marso SP et al. N Engl J Med. 2016;375(19):1834-1844.
世界、地域、国別の年齢・性別死亡率と平均余命、1950~2017年:Global Burden of Disease Study 2017の系統的分析
世界、地域、国別の年齢・性別死亡率と平均余命、1950~2017年:Global Burden of Disease Study 2017の系統的分析
Global, regional, and national age-sex-specific mortality and life expectancy, 1950-2017: a systematic analysis for the Global Burden of Disease Study 2017 LANCET 2018 Nov 10;392(10159):1684-1735. 上記論文のアブストラクト日本語訳 ※ヒポクラ×マイナビ 論文検索(Bibgraph)による機械翻訳です。 【背景】年齢別死亡率と平均余命の評価は、国連経済社会局人口部(UNPOP)、米国国勢調査局、WHO、および世界疾病・傷害・リスク要因調査(GBD)の過去の反復調査の一部として行われてきた。)GBD の以前の版では、UNPOP による人口推定値が使用されたが、これは GBD の死亡者数の推定値と内部的に一貫性のある方法で導き出されたものではない。GBDの現在の反復、GBD 2017は、以前の評価を改善し、グローバルな集団の死亡率の経験のタイムリーな推定値を提供します。 【方法】GBDは、1950年から2017年の間の死亡率の推定値を作成するために、23年齢グループ、両性、および195カ国と領土、16カ国のサブナショナルな場所を含む918の場所について、利用できるすべてのデータを使用しています。使用したデータは、バイタル登録システム、サンプル登録システム、家庭調査(完全出生歴、要約出生歴、兄弟姉妹歴)、センサス(要約出生歴、世帯死亡)、人口動態監視サイトなどである。合計で、この分析には8259のデータソースが使用された。出生から5歳までと15歳から60歳までの死亡確率の推定値を作成し、モデル生命表システムに入力して、すべての場所と年について完全な生命表を作成した。致命的な不連続性とHIV/AIDSによる死亡率は別々に分析され、その後、推定に組み込まれる。年齢別死亡率と開発状況との関係は、25歳以下の出生率、教育、所得に基づく複合指標である社会人口統計指数を用いて分析する。GBD 2017では、GBD 2016と比較して、主に4つの方法論的改善がなされている。622の追加データソースが組み込まれたこと、GBD研究によって生成された新しい推定人口が使用されたこと、分析の異なる要素で使用される統計手法がさらに標準化・改善されたこと、分析が20年遡って1950年から始まるように拡張されたこと。 【FINDINGS】Global, 18-7% (95% uncertainty interval 18-4-19-0) of deaths were registered in 1950 and that proportion has steadily increasing, in 58-8% (58-2-59-3) of all deaths are registered in 2015.世界レベルでは、1950年から2017年の間に、男性の平均寿命は48-1歳(46-5-49-6)から70-5歳(70-1-70-8)へ、女性は52-9歳(51-7-54-0)から75-6歳(75-3-75-9)へ伸びています。このような全体的な進展にもかかわらず、2017年の出生時平均寿命にはかなりのばらつきが残っており、中央アフリカ共和国の男性の49-1歳(46-5-51-7)からシンガポールの女性の87-6歳(86-9-88-1)まで幅がある。年齢層を超えて最も進歩したのは5歳未満の子どもで、5歳未満の死亡率は1950年の出生1000人あたり216-0人(196-3-238-1)から2017年の出生1000人あたり38-9人(35-6-42-83)まで減少し、国によって大きな減少がみられた。それでも、2017年の世界の5歳未満の子どもの死亡者数は500万~400万人(5-2-5-6)でした。成人、特に成人男性については、進展はあまり顕著ではなく、変動が大きく、いくつかの国で死亡率が停滞または上昇していた。1950年から2017年までの男女の平均寿命の差は、世界レベルでは比較的安定しているものの、超地域間で特徴的なパターンを示し、一貫して中央ヨーロッパ、東ヨーロッパ、中央アジアで最も大きく、南アジアで最も小さくなっている。また、開発に基づいて予想される死亡率と比較して観測された死亡率では、国や時間によってパフォーマンスが変化していた 【解釈】年齢性別死亡率のこの分析は、国によって人口死亡率に驚くほど複雑なパターンがあることを示すものである。この研究結果は、5歳未満児の死亡率の大幅な低下など、世界的な成功を強調している。これは、数十年にわたる地域、国、世界の重要なコミットメントと投資を反映したものである。しかし、同時に懸念される死亡率パターン、特に成人男性や、女性においては、本調査の期間中、多くの国で死亡率が停滞し、場合によっては上昇していることにも注意を喚起している。 第一人者の医師による解説 5歳未満児の劇的な死亡率低下と、停滞する成人期前期の健康増進 野村 周平 東京大学大学院医学系研究科国際保健政策学教室助教 MMJ.April 2019;15(2) 保健政策立案や保健介入における優先順位決定のためには、その基礎データとして、疾患別の死亡や障害、それらの原因となりうる危険因子に関する比較可能なエビデンスは必須である。その先駆けとして1991年に開始された世界の疾病負荷研究(Global Burden of Disease:GBD)は、複数の疾患や危険因子をすべて同時にかつ包括的に解析した野心的プロ ジェクトであった。その初期の成果は、「世界開発 報告1993年度版:健康への投資(世界銀行)」などで 発表され(1)、大きな反響を得た。 時は巡り、GBD研究は米国ワシントン大学保健指標・保健評価研究所(IHME)を事務局とし、GBD 2010プロジェクトが2007年に始動。多くの議論や方法論的発展を経て、1990~2010年の世界の疾病負荷が推計され、その成果は2012年に7編の原著論文としてLancetで発表された。プロジェクトはその後、GBD2013、GBD2015、GBD2016と続き、 データ収集の拡大、最新の統計技術の適用、方法論の改善を繰り返してきた。本論文は、これまでにない量・種類のデータを収集、包括的で多面的な新手法で解析された最新のGBD2017プロジェクトの研究成果からの1編である(2) 。 本研究の重要な発見の1つは、過去半世紀にわたる5歳未満児の死亡率の劇的な低下だ。1950年時の 約5分の1程度にまで低下した。このすばらしい成果は、世界、地域、国レベルの数十年にわたるコミットメントと投資、とりわけ2000年以降の全世界的なワクチンキャンペーンの果たした役割が大きい(3) 。しかしながら、依然として毎年520万人以上 の5歳未満児が死亡していることが本研究で認められており、やるべきことはまだ多く残っている。子どもの死亡率の急激な低下以外にも、GBD 2017の結果は、5歳を過ぎて生き延びる子どもが青年期や壮年期に死亡する可能性は依然高いことを示している。特に20~45歳のグループでは、死亡率の改善 は1990年以降停滞しており、2000年ごろには増加 もみられた。 GBD2017 では、 社会人口学的インデックス (Socio-Demographic Index:SDI)と呼ばれる、収入、 教育レベル、出生率という3つの側面に関して、ある国における平均達成度を測るための指標を使用し ている。例えば本研究では、各国の死亡率をSDIに基づき分類することで、同程度の開発レベルにある 国々の中で、具体的にどの国が低い死亡率を達成しているか、または相対的に達成が遅れているかを確認できる。 世界的に健康増進が進む中、地域や年齢によってその進展は大きく異なる。本研究成果は、世界の健康指標(死亡率)における課題を、国別(国によっては地方別)、年齢別、性別レベルで評価し、それら に対応する最善の方法を見つけるための新たなデータを提示するものだ。顕著な死亡率低下を実現してきた国々の成功の原動力を学び、各国レベルで新たな研究プロジェクト遂行や政策立案、自国の国民を対象とした詳細な疾病負荷研究などに活かされることが期待される(4) 。一方で、達成の遅れている国々には緊急の注意を払う必要がある。 1.The World Bank. World Development Report 1993:Investing in Health. Washington, D.C.:World Bank;1993. 2. The Lancet. Lancet. 2018;392(10159):1683. 3. Haakenstad A, et al. Health Aff(Millwood). 2016;35(2):242- 249. 4. Murray CJL, Lopez AD. Lancet. 2017;390(10100):1460-1464.
2型糖尿病および心血管疾患患者におけるアルビグルチドおよび心血管アウトカム(Harmony Outcomes):二重盲検無作為化プラセボ対照試験
2型糖尿病および心血管疾患患者におけるアルビグルチドおよび心血管アウトカム(Harmony Outcomes):二重盲検無作為化プラセボ対照試験
Albiglutide and cardiovascular outcomes in patients with type 2 diabetes and cardiovascular disease (Harmony Outcomes): a double-blind, randomised placebo-controlled trial Lancet 2018 Oct 27 ;392 (10157 ):1519 -1529 . 上記論文のアブストラクト日本語訳 ※ヒポクラ×マイナビ 論文検索(Bibgraph)による機械翻訳です。 【背景】 グルカゴン様ペプチド1受容体作動薬は、化学構造、作用時間、臨床転帰への影響に違いがある。2型糖尿病における週1回投与のアルビグルチドの心血管効果については不明である。アルビグルチドの心血管死、心筋梗塞、脳卒中の予防に関する安全性と有効性を明らかにすることを目的とした。 【方法】28か国610施設で二重盲検無作為化プラセボ対照試験を実施した。40歳以上の2型糖尿病と心血管疾患を有する患者(1:1の割合)を、標準治療に加え、アルビグルチドの皮下注射(30~50mg、血糖反応と忍容性に基づく)または同量のプラセボを週1回受ける群に無作為に割り付けました。治験責任医師は、対話型音声応答システムまたはウェブ応答システムを用いて治療割り付けを行い、患者およびすべての治験責任医師は治療割り付けをマスキングされました。主要評価項目である心血管死、心筋梗塞、脳卒中の初発について、アルビグルチドはプラセボに対して非劣性であると仮定し、intention to treatの集団で評価した。ハザード比の95%信頼区間の上限が1-30未満で非劣性が確認された場合、優越性に関するクローズドテストが事前に指定された。本試験はClinicalTrials. govに登録されており、番号はNCT02465515である。 【FINDINGS】2015年7月1日から2016年11月24日の間に患者をスクリーニングした。10 793名がスクリーニングされ、9463名が登録され、4731名がアルビグルチド投与群、4732名がプラセボ投与群に無作為に割り付けられた。2017年11月8日、611の主要評価項目と少なくとも1~5年のフォローアップ中央値が発生したと判断され、参加者は最終診察と試験治療の中止のために戻り、最後の患者の診察は2018年3月12日に行われた。これらintention-to-treat集団である9463人の患者は、中央値1~6年の期間、主要評価項目を評価された。主要複合転帰は、アルビグルチド群では4731例中338例(7%)に100人年当たり4~6件の発生率で、プラセボ群では4732例中428例(9%)に100人年当たり5~9件の発生率(ハザード比0-78、95%CI 0-68~0-90)で、アルビグルチドがプラセボに対して優位であるとした(非劣性のp<0-0001;優位のp=0-0006)。急性膵炎(アルビグルチド群10例、プラセボ群7例)、膵臓がん(アルビグルチド群6例、プラセボ群5例)、甲状腺髄様がん(両群0例)、その他の重篤な有害事象発生率は両群間に差はなかった。また、試験薬の割り付けをマスキングした治験責任医師が治療に関連すると評価した死亡例はプラセボ群で3例(1%未満)、アルビグルチド群で2例(1%未満)であった。 【解釈】2型糖尿病と心血管疾患を有する患者において、アルビグルチドはプラセボに比べ主要有害心疾患に関して優れていることが示された。したがって、エビデンスに基づくグルカゴン様ペプチド1受容体作動薬は、2型糖尿病患者における心血管イベントのリスクを低減するための包括的戦略の一部として考慮されるべきである。 資金提供】グラクソ・スミスクライン社。 第一人者の医師による解説 GLP-1受容体作動薬の間でも抑制効果には差 要因ははっきりせず 山田 祐一郎 秋田大学大学院医学系研究科内分泌・代謝・老年内科学教授 MMJ.April 2019;15(2) 米食品医薬品局(FDA)が新規糖尿病治療薬の承認要件として心血管イベント(MACE)を増やさないエビデンスも要求しているため、シタグリプチンなどのDPP-4阻害薬、エンパグリフロジンなど のSGLT2阻害薬、リキシセナチドなどのGLP-1 受容体作動薬など新しい糖尿病治療薬では、MACE を増やさないことを明らかにするため、大規模臨床試験が実施されてきた。その結果、DPP-4阻害薬 がMACEを増やさないこと(非劣性)やSGLT2阻害薬がMACEを減らすこと(優越性)をクラスエフェクトと考えていいような結果が出ている。 GLP-1受容体作動薬については、リキシセナチ ドやエキセナチドはMACEを増やさないが、減らすこともないのに対し、リラグルチド (1)やセマグ ルチド (2)はMACEを有意に減らすことが発表され、 GLP-1受容体作動薬がクラスエフェクトとして MACEを減らすかどうかはっきりしていない。新たなGLP-1受容体作動薬アルビグルチドを用いた 大規模臨床試験(Harmony Outcomes)の結果が昨年の欧州糖尿病学会(EASD)で発表され、本論文で報告された。 アルビグルチドは、ヒト GLP-1をベースにし、 アルブミンとの遺伝子融合によって、1週間1回の 投与で、持続的な効果が期待される薬剤である。 40歳以上、虚血性心疾患・脳血管疾患・末梢動脈疾 患を有する2型糖尿病患者に、アルビグルチドあるいはプラセボを投与した上で、各国ガイドラインに設定された血糖コントロール目標値に達しない場合、適宜他の糖尿病治療薬の追加や調整を可としている。中央値1.6年の研究期間で、主要評価項 目である最初のMACE(心血管死、心筋梗塞、脳卒中) 発生率は、アルビグルチド群で100人・年あたり4.6 件とプラセボ群の5.9件と比較して有意に低下していることが示された。特に、心筋梗塞への効果があったことは、リラグルチドなどの結果に合致する。 これらの結果から、ある種のGLP-1受容体作動薬を心血管イベントを有する2型糖尿病患者に投与すると、新たな心血管イベント発症を抑制する効果のあることがあらためて示されたのである。薬剤間で抑制効果に差がある要因として、対象集団の 特性の違い、GLP-1受容体作動薬の基本骨格(GLP-1 対 exendin-4)、作用時間(短時間 対 長時間)などが想定されるが、いまだはっきりしていない。 最後に、アルビグルチドは、一部の国ではすでに上市されていたが、2018年までに経営上の理由で 販売を中止すると報じられた。日本では未販売であり、血糖コントロールや心血管イベントの抑制 にエビデンスがあるアルビグルチドが使えないのは残念である。 1. Marso, SP, et al. N Engl J Med. 2016;375(4):311-322. 2. Marso, SP, et al. N Engl J Med. 2016;375(19):1834-1844.
195カ国・地域の282の死因に対する世界・地域・国の年齢性別死亡率、1980-2017年:世界疾病負担調査2017のための系統的分析。
195カ国・地域の282の死因に対する世界・地域・国の年齢性別死亡率、1980-2017年:世界疾病負担調査2017のための系統的分析。
Global, regional, and national age-sex-specific mortality for 282 causes of death in 195 countries and territories, 1980-2017: a systematic analysis for the Global Burden of Disease Study 2017 Lancet 2018 Nov 10 ;392 (10159):1736 -1788. 上記論文のアブストラクト日本語訳 ※ヒポクラ×マイナビ 論文検索(Bibgraph)による機械翻訳です。 【背景】世界的な開発目標では、国の進捗をベンチマークするために、国別の推定値に頼ることが多くなっています。この必要性を満たすために、Global Burden of Diseases, Injuries, and Risk Factors Study (GBD) 2016は、1980年から世界、地域、国、そして特定の場所については、国未満の原因別死亡率を推定している。ここでは、新たに入手可能となったデータと改善された手法を活用し、その研究の更新を報告する。GBD 2017は,1980年から2017年までの195の国と地域における282の原因別死亡率の包括的な評価を提供する。 【方法】死因データベースは,生命登録(VR),口頭検死(VA),登録,調査,警察,監視データから構成されている。GBD 2017では,10件のVA調査,127国年のVRデータ,502国年のがん登録,1国年のサーベイランスが追加された。GBDの死因階層の拡張により、GBD 2017では18の死因が追加推定された。新たに入手可能となったデータにより、エチオピア、イラン、ニュージーランド、ノルウェー、ロシアの5カ国が追加され、国別推定値が算出された。国際疾病分類(ICD)コードが非特異的、ありえない、または中間的な死因に割り当てられた死亡は、不確実性推定に組み込まれた再分配アルゴリズムによって基礎的な死因に再割り当てされた。死因アンサンブルモデル(CODEm)を含むGBDのために開発された統計モデリングツールを用いて、地域、年、年齢、性別ごとに死因割合と死因別死亡率を算出した。GBD 2017は、旧版のように国連の推定値を使用する代わりに、すべての場所の人口サイズと出生率を独自に推定しました。そして、各死亡の合計に各年齢の標準余命を乗じることで損失年数(YLL)を算出した。ここで報告されたすべての率は年齢標準化されている。 発見]死因の最も広いグループ分け(レベル1)では、非伝染性疾患(NCD)が死因の最も大きな割合を占め、2017年の総死因の73-4%(95%不確実性区間[UI] 72-5-74-1 )に寄与し、伝染病、母親、新生児、栄養(CMNN)原因は18-6%(17-9~19-6)、負傷8-0%(7-7~8-2)であった。NCDの原因による総死亡者数は2007年から2017年にかけて22-7%(21-5-23-9)増加し、2007年と比較して2017年には7-6100万(7-20-8-01)の追加死亡者数が推定されたことになる。NCDによる死亡率は、世界的に7-9%(7-0-8-8)減少した。CMNNの原因による死亡者数は22-2%(20-0-24-0)、死亡率は31-8%(30-1-33-3)減少した。傷害による総死亡者数は2007年から2017年にかけて2-3%(0-5-4-0)増加し、傷害による死亡率は13-7%(12-2-15-1)減少して2017年には10万人あたり57-9人(55-9-59-2)であった。物質使用障害による死亡も増加し、2007年に世界で284 000人(268 000-289000)だったのが、2017年には352 000人(334 000-363000)に増加しました。2007年から2017年の間に、紛争とテロによる死亡者総数は118-0%(88-8-148-6)増加した。5歳未満の子どもの下気道感染症による死亡が36-4%(32-2-40-6)減少したのに対し、70歳以上の成人では33-6%(31-2-36-1)増加するなど、一部のCMNN原因では高齢者よりも死亡総数および死亡率の減少が大きく見受けられた。世界的に、2017年の死亡者数は、85歳以上の高齢者を除くほとんどの年齢で、女性より男性の方が多かった。世界のYLLの動向は疫学的な変遷を反映しており、1990年から2017年にかけて腸管感染症、呼吸器感染症および結核、母体および新生児障害によるYLL総数が減少し、これらは社会人口統計指数(SDI)の最低レベルにおいて概して大きさが増している。同時に、新生物および心血管疾患によるYLLが大きく増加した。すべてのSDI五分位において、レベル2の主要な5つの死因でYLL率が減少した。1990年にYLLの主要原因であった新生児障害、下気道感染症、下痢性疾患は、2017年には2位、4位、5位となった。一方、虚血性心疾患(2017年1位)と脳卒中(3位)では、YLL率は低下したものの、推定YLLは増加した。人口増加は、2007年から2017年にかけて、レベル2の主要な20の死因における総死亡者数の増加に寄与した。原因別死亡率の減少は、3つの原因(物質使用障害、神経疾患、皮膚・皮下疾患)を除くすべての原因について、人口増加の影響を軽減した。 【解釈】グローバルヘルスの改善は、集団間で不均一に分布している。傷害、物質使用障害、武力紛争とテロ、新生物、心血管系疾患による死亡は、世界の健康に対する脅威を拡大している。下気道感染症や腸管感染症などの死因については、高齢の成人よりも子どもの方が急速に進歩しており、年齢層による性差で死亡率に格差がある状態が続いている。一般的な疾患の死亡率の減少は、NCDを中心に減速または停止しており、特定の原因による死亡率は過去10年間で増加している 【FUNDING】ビル&メリンダ・ゲイツ財団 第一人者の医師による解説 死因の半数が10種の疾病 これらへの対処で健康改善が進展 野村 周平 東京大学大学院医学系研究科国際保健政策学教室助教 MMJ.June 2019;15(3) 世界の疾病負荷研究(Global Burden of Disease:GBD)は米国ワシントン大学のInstitute of Health Metrics and Evaluation(IHME)を主軸として、多くの研究機関が連携して行っている。本論文は、これまでにない量・種類のデータを収集、包括的で多面的な新手法で解析した最新のGBD 2017プロジェクトの研究成果からの1編であり(1)、 世界195の国・地域における死亡の原因を性・年 齢階級別に詳細に分析したものである。 本研究により、非感染症が2017年における世界の死因の4分の3(73.4%)を占めていたことがわかった(感染症・母体および新生児の病気・栄養障害が18.6%、傷害が8.0%)。近年においては、 非感染症は死亡数・率ともに増加を続け、一方で感染症や傷害は数・率ともにそれぞれ減少、停滞傾向にある。 本研究の重要な発見の1つは、282種類の疾病・ 傷害を調べたところ、世界の死亡の半数以上がそのうちわずか10種類の疾病が原因となっていたことだ。これらの少数の疾病に対処すれば、健康改善において進展を遂げることができる。本研究は死亡の原因となる疾病・傷害のタイプも大きく変わりつつあることを示した。2000~17年の 約20年間で、10大死因のうち、虚血性心疾患と卒中は2大主要死因にとどまったが、他の8死因は 入れ替わった。アルツハイマー病、糖尿病、肺がん、慢性閉塞性肺疾患および肝硬変は上位に上がり、下 痢性疾患や下気道感染症、新生児障害、HIV/AIDS、 結核は順位を下げた。がん全体でみると死亡数・率ともに増加し続けている。 上述の傾向が当てはまらない地域が、サハラ以南のアフリカである。HIV/AIDS、マラリア、結核といった三大感染症や下痢性疾患、新生児障害が南アジアやオセアニア地域で死因の5分の1、その他の地域で2~10%以下を占める一方、サハラ以南のアフリカでは、これらの疾病が死因の約半数の48%を占める。これらの疾病による死亡率はこの20年で大きく低下し(50%減)、重要な前進を遂げたが、依然として死亡率はとても高く、このような状況はこの地域特有である。加えて、虚血性心疾患や脳卒中といった心血管疾患も今ではアフリカにおいても脅威となっている。 世界的に健康増進が進む中、地域や年齢によって その進展は大きく異なる。本研究成果は、世界の死亡における課題を、国や地方別、年齢別、性別レベルで評価し、それらに対応する最善の方法を見つけるための新たなデータを提示するものだ。 1. The Lancet. Lancet. 2018;392(10159):1683.
英国の中等学校におけるいじめと攻撃性に対するLearning Together介入の効果(INCLUSIVE):クラスター無作為化対照試験。
英国の中等学校におけるいじめと攻撃性に対するLearning Together介入の効果(INCLUSIVE):クラスター無作為化対照試験。
Effects of the Learning Together intervention on bullying and aggression in English secondary schools (INCLUSIVE): a cluster randomised controlled trial Lancet 2018 Dec 8 ;392 (10163 ):2452 -2464 . 上記論文のアブストラクト日本語訳 ※ヒポクラ×マイナビ 論文検索(Bibgraph)による機械翻訳です。 【背景】子どもや若者のいじめ、攻撃性、暴力は、公共のメンタルヘルス問題の中で最も大きな影響を及ぼしている。我々は,修復的実践を用い,社会的・感情的スキルを開発することによって学校環境を修正する取り組みに生徒を参加させるLearning Together介入を検証した。 【方法】イングランド南東部の中学校で3年間,経済的評価とプロセス評価を行い,標準的実践(コントロール)と比較したLearning Together介入のクラスター無作為化試験を行った。Learning Togetherは、修復的実践に関する職員研修、学校活動グループの招集と進行、および生徒の社会的・感情的スキルのカリキュラムで構成されていた。主要アウトカムは,36カ月目に測定された自己申告によるいじめ被害経験(Gatehouse Bullying Scale; GBS)と攻撃行為(Edinburgh Study of Youth Transitions and Crime; ESYTC)の学校不品行副尺度であった。意図的治療縦断混合効果モデルを用いてデータを分析した。本試験はISRCTN登録(10751359)された。 【調査結果】40校(各群20校)を対象とし、脱落校はなかった。ベースライン時には7121人中6667人(93-6%)、36か月時には7154人中5960人(83-3%)の生徒が参加した。36か月時点のGBSいじめスコアの平均は、対照群0-34(SE 0-02)に対して介入群0-29(SE 0-02)であり、有意な調整平均差(-0-03、95%CI -0-06~-0-001; 調整効果サイズ -0-08)であった。36ヶ月時点のESYTCスコアの平均は、対照群4-33(SE 0-20)に対して介入群4-04(0-21)であり、群間差は認められなかった(調整済み差-0-13、95%CI -0-43~0-18; 調整済み効果量 -0-03)。費用は介入校では対照校よりも生徒一人当たり58ポンド追加された。 【解釈】Learning Togetherは公衆衛生上重要な可能性のあるいじめに対して小さいが有意な効果を示したが,攻撃性に対しては効果がなかった。学校全体の環境を修正することによって生徒の健康を促進する介入は、子どもや若者の密接に関連したリスクや健康上の結果に対処する最も実行可能かつ効率的な方法の一つであると考えられる。 第一人者の医師による解説 低コストで効果的なポピュレーションアプローチ 日本での導入に期待 矢澤 亜季(ハーバード大学公衆衛生大学院リサーチフェロー)/友田 明美(福井大学子どものこころの発達研究センター教授) MMJ.June 2019;15(3) いじめや暴力は、青少年における主要な精神保 健上の問題であり、その対策として学校レベルでの介入が期待されている。本論文の著者らは、英国においてLearning Togetherという、学校の社会環境改善によるいじめ防止を目的としたプログラムを開発し、その効果検証を行った。具体的には、 ①学校のポリシーやシステムの見直しによる、特に社会経済的に恵まれない子どものエンゲージメントの向上②修復的実践(いじめの当事者が一堂に会し、集団的に問題を解決するプロセス)によって 生徒や教員の間にある問題を予防・解決するシス テムづくり③各個人の社会性や情動のスキルの向上を図るためのカリキュラムの導入̶̶の3つで構成される。特徴として、学校レベルでの介入、すなわちポピュレーションアプローチでありながら、 低コスト(生徒1人あたり58ポンド[約8,500円]) であることが挙げられる。 本研究では、40の中等学校に在籍する6,667 人の生徒を対象とし、介入群(20校)に対して11 ~12歳時に介入を行い、3年間にわたっていじめや攻撃性の低減につながったかどうかクラターランダム化比較試験により効果が検証された。いじめはGatehouse Bullying Scale、攻撃性はEdinburgh Study of Youth Transitions and Crimeを用いて、それぞれ自己評価の経験をアウトカムとした。 結果、3年後のいじめスコアは対照群の0.34に 対して介入群で0.29と有意に低かった。攻撃性に関しては、全体としては有意な差はみられなかったものの、もともと攻撃性が高かった集団ではその低下がみられた。また、介入群では3年後の生活の質(QOL)や心理的健康度が対照群より高く、飲酒や喫煙を含む問題行動も総じて少なかった。興味深いのは、こうした効果は介入から2年後ではみられなかったことである。学校環境の変化にはそれなりに時間がかかることを示唆する結果だと著者らは考察している。 こうしたプログラムの効果検証にランダム化比較試験が用いられるのは初の試みであり、効果検証において現状最も理想的な手法であると言える。 認められた効果は決して大きなものではなかったが、低コストのポピュレーションアプローチとして その有用性が示唆されたことは意義深い。 日本でもいじめは増加の一途を辿っており、2017年度には過去最多の41万件を記録している。 学校および教師への負担が増加する中、各世帯や子どもに個別の対応を十分に提供するのは難しいのが現状であり、このようなプログラムを学校教育全体に組み込むことで、いじめが予防できるのであれば理想的だろう。こうしたエビデンスを踏まえて、日本のコンテクストに合ったプログラムを作成し、取り入れていく機運が高まることを期待する。
急性期脳卒中後の機能的転帰に対するfluoxetineの効果(FOCUS):実用的な二重盲検無作為化対照試験。
急性期脳卒中後の機能的転帰に対するfluoxetineの効果(FOCUS):実用的な二重盲検無作為化対照試験。
Effects of fluoxetine on functional outcomes after acute stroke (FOCUS): a pragmatic, double-blind, randomised, controlled trial Lancet 2019 Jan 19 ;393 (10168):265 -274 . 上記論文のアブストラクト日本語訳 ※ヒポクラ×マイナビ 論文検索(Bibgraph)による機械翻訳です。 【背景】小規模試験の結果から、フルオキセチンが脳卒中後の機能的転帰を改善する可能性があることが示された。FOCUS試験は、これらの効果を正確に推定することを目的とした試験である。対象は、18歳以上で、臨床的に脳卒中と診断され、発症後2日から15日の間に登録され無作為に割り付けられた、局所神経障害がある患者であった。患者はfluoxetine 20mgまたはマッチングプラセボを1日1回6ヵ月間、Webベースのシステムで最小化アルゴリズムを用いて無作為に割り付けられた。主要評価項目は,6 ヵ月後の修正 Rankin スケール(mRS)で測定された機能状態であった.患者,介護者,医療スタッフ,および試験チームは治療割り付けをマスクされた.機能状態は,無作為化後 6 ヵ月および 12 ヵ月で評価された.患者さんは、治療割り付けに従って分析されました。本試験はISRCTN登録番号ISRCTN83290762に登録されている。 【所見】2012年9月10日から2017年3月31日の間に、3127人の患者が募集された。1564人の患者にフルオキセチンが、1563人の患者にプラセボが割り当てられた。 各治療群の1553人(99-3%)の患者について、6ヶ月後のmRSデータが入手可能であった。6ヵ月後のmRSカテゴリー間の分布は、fluoxetine群とプラセボ群で類似していた(最小化変数で調整した共通オッズ比0-951[95%CI 0-839-1-079];p=0-439 )。fluoxetineを投与された患者は、プラセボを投与された患者よりも6ヵ月までに新たにうつ病を発症する可能性が低かったが(210人[13-43%]対269人[17-21%];差3-78%[95%CI 1-26-6-30];p=0-0033)、骨折がより多かった(45人[2-88%]対23人[1-47%];差1-41%[95%CI 0-38-2-43];p=0-0070 )。その他のイベントについては,6か月,12か月とも有意差はなかった。 【解釈】フルオキセチン20mgを急性脳梗塞後6か月間毎日投与しても,機能的転帰は改善しないようである。この治療はうつ病の発生を減少させるが,骨折の頻度を増加させた。これらの結果は、脳卒中後のうつ病の予防や機能回復の促進のためにfluoxetineをルーチンに使用することを支持しない。 第一人者の医師による解説 他の集団やサブ集団で、副作用を含めた有用性の検証がさらに必要 山口 修平 島根県立中央病院 島根県病院事業管理者 MMJ.June 2019;15(3) 選択的セロトニン再取り込み阻害薬のフルオキセチンは、脳卒中後のうつ病発症を抑制するとともに、90日後の運動機能を有意に改善し、自立生活の割合を有意に上昇させたとの報告がある(FLAME 試験)(1)。コクランレビューでも脳卒中後の運動障害 を改善する可能性が示唆されているが、副作用を含めて大規模研究による検証が必要とされていた。 本論文で報告されたFOCUS試験の主要アウトカムはフルオキセチンによる脳卒中発症6カ月後の 機能予後変化(Modifi ed Rankin Scale[mRS]に よる評価)であり、副次アウトカムは他の評価スケールによる運動機能、情動機能、QOLに関する6カ月後 と12カ月後の機能予後および有害事象などである。 本試験では18歳以上の急性期脳卒中患者を対象として、発症2~15日後よりフルオキセチン 20mg/日またはプラセボを開始し、6カ月間投 与した。試験デザインは多施設共同ランダム化プラセボ対照二重盲検試験である。患者数は実薬群 1,564人、プラセボ群1,563人で、英国の103 病院で実施された。 その結果、主要アウトカムである6カ月後の mRSによる機能評価では、実薬群とプラセボ群で差を認めなかった(オッズ比 , 0.951;95% CI, 0.839~1.079)。病型、臨床症状、年齢、発症~ 服薬開始間隔などによるサブグループ解析でも差は認めなかった。副次アウトカムに関しては、6カ 月後のうつ病の発症が実薬群で有意に抑制され た(13.4% 対 17.2%;P=0.0033)。一方、実薬群で骨折の頻度が有意に上昇した(2.88% 対 1.47%;P=0.0070)。また12カ月後には、うつ病の発症頻度に群間差はなくなり、6カ月評価で差が認められたMental Health Inventory-5の結果も両群間に差は認めなかった。12カ月後の生存 率にも差はなかった。以上の結果から、脳卒中後早期からの機能予後の改善目的あるいはうつ病発症予防のためのフルオキセチンのルーチン投与は支持されないと結論づけている。 今回のFOCUS試験の結果はFLAME試験の結果を否定するものであった。本試験は、患者割り付けのバイアスがないこと、組み入れ患者数が多いこと、 脱落患者が少ないこと、intention-to-treat解析が行えたことなど、従来の試験に比べて優れた点があり信頼性は高い。一方、6カ月後のうつ病発症を抑制したことは、FLAME試験の結果と一致しており、他のSSRIを含むメタアナリシスでもうつ病の発症を63%減少させることが示されている。しかし、今回の試験で認められた骨折頻度の上昇を考慮すると、その有用性は低くなる。同様のデザインの研究が独立して現在進行中であり、他の患者集団での検討や本剤が有効なサブ集団の有無に関する検討が待たれる。 1. Chollet F, et al. Lancet Neurol. 2011;10(2):123-130.
頭部CTスキャンにおける重要な所見の検出のためのディープラーニングアルゴリズム:レトロスペクティブスタディ。
頭部CTスキャンにおける重要な所見の検出のためのディープラーニングアルゴリズム:レトロスペクティブスタディ。
Deep learning algorithms for detection of critical findings in head CT scans: a retrospective study Lancet 2018 Dec 1 ;392 (10162):2388 -2396. 上記論文のアブストラクト日本語訳 ※ヒポクラ×マイナビ 論文検索(Bibgraph)による機械翻訳です。 【背景】非造影頭部CTスキャンは、頭部外傷や脳卒中の症状を持つ患者の初期画像診断の現在の標準である。我々は,これらのスキャンから以下の重要な所見を自動検出するための一連の深層学習アルゴリズムの開発と検証を目的とした:頭蓋内出血とその種類(すなわち,胸膜内,脳室内,硬膜下,硬膜外,くも膜下),頭蓋底骨折,正中線移動,および腫瘤効果。 【方法】2011年1月1日から2017年6月1日の間に,インドの約20の施設から,頭部CTスキャン313件とその臨床報告書を含むデータセットをレトロスペクティブに収集した。このデータセットのランダムに選択された部分(Qure25kデータセット)を検証に使用し、残りはアルゴリズムを開発するために使用された。追加の検証用データセット(CQ500データセット)は、開発およびQure25kデータセットに使用した施設とは異なる施設から2つのバッチで収集した。術後スキャンと7歳未満の患者のスキャンは除外した。Qure25k と CQ500 のデータセットでは,それぞれオリジナルの臨床放射線報告書と 3 人の独立した放射線科医のコンセンサスをゴールドスタンダードとみなした.アルゴリズムの評価には,主に受信者動作特性曲線下面積(AUC)が用いられた。Qure25kデータセットにおいて,アルゴリズムは頭蓋内出血の検出で0-92(95%CI 0-91-0-93)のAUCを達成した(胸膜内は0-90 [0-89-0-91], 脳室内は0-96 [0-94-0-97], 硬膜下は0-92 [0-90-0-93], 硬膜外は0-93 [0-91-0-95] そして くも膜下は0-90 [0-89-0-92]).CQ500データセットでは,頭蓋内出血のAUCは0-94(0-92-0-97)であった(それぞれ,0-95 [0-93-0-98], 0-93 [0-87-1-00], 0-95 [0-91-0-99], 0-97 [0-91-1-00], 0-96 [0-92-0-99]).Qure25kデータセットにおけるAUCは、頭蓋底骨折が0-92(0-91-0-94)、正中線移動が0-93(0-91-0-94)、mass effectが0-86(0-85-0-87)、CQ500データセットにおけるAUCはそれぞれ0-96(0-92-1-00), 0-97 (0-94-1-00) および 0-92(0-89-0-95)であった。 【解釈】我々の結果は、深層学習アルゴリズムが緊急の注意を要する頭部CTスキャンの異常を正確に特定できることを示しており、これらのアルゴリズムを使用してトリアージプロセスを自動化する可能性を開いている。 【FUNDING】Qure .ai . 第一人者の医師による解説 頭部外傷や脳卒中の自動トリアージに道を開く研究成果 井上 優介 北里大学医学部放射線科学画像診断学主任教授 MMJ.June 2019;15(3) 人工知能(AI)が社会のさまざまな分野で注目を集めており、画像診断を含めた医療分野も例外でない。近年のAIブームを牽引しているのは深層学習であり、本研究では、頭部単純 CTから危機的所見を検出する深層学習アルゴリズムの開発と評価を後ろ向きに行っている。このアルゴリズムは頭蓋内出血、頭蓋冠骨折、中心構造偏位、占拠性効果 (mass effect)の有無を判定し、頭蓋内出血についてはそのタイプを脳実質内、脳室内、硬膜下、硬 膜外、くも膜下に分類するものである。インドの約 20施設から313,318件の頭部 CTとその画像診断報告書を収集し、この中から290,055件をア ルゴリズム開発に、21,095件を性能評価に使用した。さらに、別の6施設から集めた491検査で も性能評価を行った。検討の結果、いずれの判定項 目についても良好な診断能が示され、本アルゴリズムが頭部 CTにおける急性所見検出の補助技術として期待されると述べられている。 深層学習アルゴリズムの開発では質の高い大量の教師データの集積が鍵になる。本研究では多数の頭部 CTとその診断結果を教師データとしてアルゴリズムを開発し、有効性も多数例で示している。 CT画像は装置や使用施設によって異なり、さまざまな施設からデータを集めていることも本研究の長所である。 診断結果については、6施設の491検査では3人の放射線科医が合議で判定した結果を ゴールドスタンダードとしているが、その他の検査では日常臨床で作成された画像診断報告書を用い、報告書の記載から注目所見の有無を自動判定してスタンダードとしている。効率的なスタンダード決定によって大量のデータの使用を実現しており、 今後の深層学習アルゴリズム研究にも参考になると思われる。 しかし、忙しい臨床の中で1人が作成した報告書からスタンダードを決定することには、 見落としや過剰診断の可能性、主所見だけを記載して副所見が十分記載されない可能性による限界がある。また、画像診断報告書は臨床情報や過去の画像検査結果なども踏まえて作成されていることも考慮する必要がある。報告書は必ずしも画像情報 を忠実に反映したものではない。 AI技術を実用化するには、臨床状況の中での位置付けを具体化することが望まれる。この論文では、開発したアルゴリズムを頭部外傷や脳卒中患者の自動トリアージに使用して放射線科医の業務効率を改善することを提案しており、現実的で有益な役割と考えられる。一方、トリアージ結果が過剰に信頼されて誤診につながる危険性も指摘しており、 自動診断技術全般に適用される戒めとして尊重したい。
回復した拡張型心筋症患者における心不全の薬物療法の中止(TRED-HF):非盲検試験、パイロット試験、無作為化試験。
回復した拡張型心筋症患者における心不全の薬物療法の中止(TRED-HF):非盲検試験、パイロット試験、無作為化試験。
Withdrawal of pharmacological treatment for heart failure in patients with recovered dilated cardiomyopathy (TRED-HF): an open-label, pilot, randomised trial Lancet 2019 Jan 5 ;393 (10166 ):61 -73 . 上記論文のアブストラクト日本語訳 ※ヒポクラ×マイナビ 論文検索(Bibgraph)による機械翻訳です。 【背景】 症状と心機能が回復した拡張型心筋症患者は、しばしば薬剤を中止してよいかどうかを尋ねる。このような状況での治療中止の安全性は不明である。 【方法】拡張型心筋症の既往があり、現在は無症状で、左室駆出率(LVEF)が40%未満から50%以上に改善し、左室拡張末期容積(LVEDV)が正常化し、N末端プロB型ナトリウム利尿ペプチド(NTプロBNP)濃度が250ng/L未満である患者において、心不全薬の段階的中止の影響を調べるためにオープンラベル、パイロット、ランダム化試験を実施した。英国内の病院のネットワークから患者を募集し、1つのセンター(ロイヤルブロンプトン・アンド・ハレフィールドNHS財団トラスト、英国ロンドン)で評価を行い、段階的な治療中止または治療継続に1対1の割合で無作為に割り付けた。6ヵ月後、治療継続群の患者さんは、同じ方法で治療が中止されました。主要評価項目は、6ヶ月以内の拡張型心筋症の再発とし、LVEFが10%以上低下し50%未満となる、LVEDVが10%以上上昇し正常範囲より高くなる、NT-pro-BNP濃度が2倍上昇し400ng/L以上、または心不全の臨床症状で定義し、その時点で治療を再確立させた。主要解析はintention to treat。本試験はClinicalTrials. govに登録されており、番号はNCT02859311。 【FINDINGS】2016年4月21日から2017年8月22日の間に、51人の患者が登録された。25名が治療中止群に、26名が治療継続群に無作為に割り付けられた。最初の6カ月間で、治療中止群に無作為に割り付けられた11人(44%)が主要評価項目である再発を示したのに対し、治療継続群に割り付けられた患者は1人もいなかった(カプラン・マイヤー推定イベント率45-7%[95%CI 28-5-67-2];p=0-0001 )。6ヵ月後、最初に治療継続と割り付けられた患者26人のうち25人(96%)が治療中止を試みた。その後の6ヵ月間に、9人の患者が主要評価項目である再発を示した(Kaplan-Meier推定イベント発生率36-0%[95%CI 20-6-57-8])。死亡はいずれの群でも報告されず,治療中止群では3つの重篤な有害事象が報告された:非心臓性胸痛による入院,敗血症,待機的手術。 【解釈】拡張型心筋症から回復したとみなされた患者の多くは,治療中止後に再発する。再発の確実な予測因子が明らかになるまでは、治療を無期限に継続すべきである。 資金提供】英国心臓財団、Alexander Jansons財団、ロイヤルブロンプトン病院とインペリアルカレッジ・ロンドン、インペリアルカレッジ生物医学研究センター、Wellcome Trust、およびRosetrees Trust。 第一人者の医師による解説 心機能の改善 回復か軽快かを鑑別する臨床指標の抽出を期待 猪又 孝元 北里大学北里研究所病院循環器内科教授 MMJ.June 2019;15(3) 今から遡ること30年ほど前、私が医学生だったころ、心筋細胞は再生能力がないから、心不全を起こす悪くなった心臓は決して良くならない、そう教わった。しかし、医師になり、循環器診療へと専門の道を進めたころ、その教えは必ずしも正しくないことを知った。致死の病とみなされていた拡張型心筋症による重症心不全患者に、当時禁忌として扱われていたβ遮断薬を導入することで、心機能が劇的に改善する経験をするようになったからである。現在、左室逆リモデリングと呼ばれるこの現象は、後に加わった心臓再同期療法とともに、なかでも拡張型心筋症で容易に観察できる。当施設からの報告でも、β遮断薬を含む至適薬物療法のみで、約半数の拡張型心筋症に左室逆リモデリングがもたらされる(1)。今や若い医師たちにとって拡張型心筋症は、安全パイとすら思われている節すらある。 喉元過ぎれば熱さを忘れる。治療が奏効し、症状も消え失せ、心機能の劇的改善のデータに歓喜し、その状態が何年も持続すると、「こんなに元気なのに、いったいいつまでこのクスリを飲み続ける必要があるのか」と思うのが患者心理である。一方、服薬の自己中断により、改善した心機能が再度悪化をきたす拡張型心筋症を経験した医師は少なくなく、原則として「一生飲み続ける」ことを患者に勧めている。しかし、系統立ててこれを証明した報告 は皆無であった。 本研究では、51人の拡張型心筋症患者を登録し、 β遮断薬などの心不全治療を継続する群または漸減後に中止する群にランダム化し、服薬中止群の4 割に及ぶ患者において半年間で心機能が有意に悪化する現象を見いだした。より長期に観察すれば、死亡や心不全入院などのイベント発生も観察されたであろう。これまで多くの医師が経験則で患者指導していた内容は、パイロット研究とはいえ正しいことがはじめて実証された。 拡張型心筋症において心筋生検や心臓 MRIで心筋の障害や線維化が高度と評価されても、β遮断薬の導入によってときに心機能データは正常値すらたたき出す(2)。ただし、心筋が傷んでいることは事実である。recovery(回復)とremission(軽快)とはときに混同されがちだが、左室逆リモデリングはあくまでremissionに過ぎないと解釈すべきである(3)。一方、自然回復のような臨床経過を辿る拡張型心筋症も経験され、このような症例では薬物療法の終了が期待できる。同じく心機能が改善した症例間で、recoveryとremissionを鑑別する臨床指標の抽出が望まれる。 1. Ikeda Y, et al. Heart Vessels. 2016;31(4):545-554. 2. Ishii S, et al. Heart Vessels. 2016;31(12):1960-1968. 3. Nabeta T, et al. Heart Vessels. 2019;34(1):95-103.
Eicosapentaenoic acid と aspirin の単独および併用による大腸腺腫の予防(seAFOod Polyp Prevention trial):多施設、無作為化、二重盲検、プラセボ対照、2×2要因試験。
Eicosapentaenoic acid と aspirin の単独および併用による大腸腺腫の予防(seAFOod Polyp Prevention trial):多施設、無作為化、二重盲検、プラセボ対照、2×2要因試験。
Eicosapentaenoic acid and aspirin, alone and in combination, for the prevention of colorectal adenomas (seAFOod Polyp Prevention trial): a multicentre, randomised, double-blind, placebo-controlled, 2 × 2 factorial trial Lancet 2018 Dec 15 ;392 (10164 ):2583 -2594 . 上記論文のアブストラクト日本語訳 ※ヒポクラ×マイナビ 論文検索(Bibgraph)による機械翻訳です。 【背景】 オメガ3系多価不飽和脂肪酸であるエイコサペンタエン酸(EPA)とアスピリンは、ともに優れた安全性プロファイルとともに、大腸がん化学予防の概念実証がなされている。そこで、大腸内視鏡検査で散発性の大腸新生物が検出された人を対象に、EPAとアスピリンの単独および併用、プラセボとの比較で有効性を検証することを目的とした。 【方法】多施設共同無作為化二重盲検プラセボ対照2×2要因試験において、英国Bowel Cancer Screening Programme(BCSP、少なくとも1つが直径10mm以上の腺腫3個以上、または直径10mm未満の腺腫5個以上)で高リスクと結腸鏡検査で確認された55~73歳の患者が、英国イングランドのBCSP内視鏡ユニット53施設から募集された。患者は、安全なウェブベースのサーバーを使用して、1日2gのEPA遊離脂肪酸(FFA)(FFAまたはトリグリセリドとして)、1日300mgのアスピリン、両方の治療の併用、またはプラセボを12ヵ月間受けるように、無作為に大きさを変えた順列付きブロックを使ってBCSP施設によって層別化され、1:1:1:1:1で無作為に割り当てられた。研究スタッフと参加者はグループ分けについてマスクされていた。主要評価項目は、1年後の監視下結腸鏡検査における腺腫検出率(ADR:腺腫を有する参加者の割合)で、観察可能なフォローアップデータを有する参加者全員について、いわゆるアット・ザ・マージン法を用いて分析し、BCSPサイトおよびベースラインの反復内視鏡検査で調整されたものでした。安全性集団には、少なくとも1回の試験薬投与を受けたすべての参加者が含まれます。本試験は、国際標準ランダム化比較試験番号登録、番号ISRCTN05926847に登録されている。 【所見】2011年11月11日から2016年6月10日の間に、709人の参加者が4つの治療群(プラセボ176人、EPA179人、アスピリン177人、EPA+アスピリン177人)に無作為に割り付けられた。腺腫のアウトカムデータは、プラセボ群163例(93%)、EPA群153例(85%)、アスピリン群163例(92%)、EPA+アスピリン群161例(91%)で入手可能であった。ADRはプラセボ群61%(163例中100例)、EPA群63%(153例中97例)、アスピリン群61%(163例中100例)、EPA+アスピリン群61%(161例中98例)で、EPAの効果は認められなかった(リスク比[RR]0-98、95%CI 0-87~1-12;リスク差 -0-9%,-8-8-6-9;p=0-81) またはアスピリン(RR 0-99(0-87~1-12; リスク差 -0-6%,-8-5~7-2;p=0-88)) の効果は認められなかった.EPAおよびアスピリンの忍容性は良好であった(176例中78例[44%]に1件以上の有害事象が発生したのに対し、プラセボ群ではEPA群82例[46%]、アスピリン群68例[39%]、EPA+アスピリン群76例[45%])、消化器の有害事象数はEPA単独群では146件で増加した(一方プラセボ群では85件、アスピリン群86件、アスピリン+プラセボ群68件)が、この有害事象数はEPA群では1件のみであった。上部消化管出血の事象は、治療群全体で6件報告された(EPA群2件、アスピリン群3件、プラセボ群1件)。 【解釈】EPAおよびアスピリン治療のいずれも、大腸腺腫を少なくとも1つ有する患者の割合の低下と関連しなかった。腺腫の種類や部位による大腸腺腫数への影響については、さらなる研究が必要である。EPAとアスピリンの最適な使用には、腺腫の再発に対する精密医療的なアプローチが必要かもしれない。 第一人者の医師による解説 サブ解析では抑制傾向示唆 腺腫の性質に基づく予防薬選択の検討を 上野 雅資 がん研有明病院大腸外科部長 MMJ.June 2019;15(3) エイコサペンタエン酸(EPA)は、血小板凝集抑 制などの薬効があり、閉塞性動脈硬化症や高脂血症の治療薬としてすでに承認されている。また、ニシンやサケ、サバなどの魚類に多く含まれており、サプリメントとしても広く販売されている。EPAの大腸がんに対する予防効果については、家族性大 腸腺腫症患者55人を対象とした小規模な無作為化二重盲検プラセボ対照試験で、投与後6カ月の内視 鏡検査で、プラセボ群に比べて直腸ポリープの数および大きさを有意に減少させることが報告されている(1)。 本論文は、複数の比較試験で、ポリープの発生を抑制することが認められているアスピリン(2)と EPAを併用することにより、大腸ポリープ(腺腫)の発生を抑制することを証明し、腺腫由来の発がんを長期にコントロールできる可能性を示すことを意図した多施設共同無作為化二重盲検プラセボ対照(seAFOod Polyp Prevention)試験の結果報告である。同試験では高リスクの大腸腺腫(ポリープ)を有する患者709人をEPA+アスピリン併用 群、EPA単独群、アスピリン単独群、プラセボ群に割り付け、主要評価項目である1年目の大腸腺腫発生率が比較された。 その結果、残念ながら、意図した成果は得られなかった。すなわち、EPAが大腸腺腫の発生を抑制する効果を認めなかった。疫学研究のメタアナリシスでも、EPA摂取は、大腸がんの発生を抑制していないとの報告があり(3)、本論文の結果からも、大腸がんの発がんに関しては、EPAの関与は少ないと考えるべきかもしれない。 ただし、サブ解析では、EPAは、通常型の腺腫と左側大腸腺腫を抑制する傾向があり、アスピリンはこれに加えて鋸歯状腺腫と右側大腸腺腫を抑制する傾向があることが示唆されたとし、今後の研究では、発生した腺腫の性質や、背景粘膜のバイオマーカーなどに基づいて、化学予防薬を選択するとも検討すべきかもしれないと述べている。 1. West NJ, et al. Gut. 2010;59(7):918-925. 2. Cole BF, et al. J Natl Cancer Inst. 2009;101(4):256-266. 3. Geelen A, et al. Am J Epidemiol. 2007;166(10):1116-1125.
1990年から2017年の195の国と地域の354の病気と傷害の世界、地域、国の発生率、有病率、障害をもって生きた年数:世界疾病負担調査2017のための系統的分析。
1990年から2017年の195の国と地域の354の病気と傷害の世界、地域、国の発生率、有病率、障害をもって生きた年数:世界疾病負担調査2017のための系統的分析。
Global, regional, and national incidence, prevalence, and years lived with disability for 354 diseases and injuries for 195 countries and territories, 1990-2017: a systematic analysis for the Global Burden of Disease Study 2017 Lancet 2018 Nov 10 ;392 (10159):1789 -1858. 上記論文のアブストラクト日本語訳 ※ヒポクラ×マイナビ 論文検索(Bibgraph)による機械翻訳です。 【背景】Global Burden of Diseases, Injuries, and Risk Factors Study 2017(GBD 2017)は、1990年から2017年までの195の国と地域における354の原因について、発生率、有病率、障害とともに生きた年(YLDs)を包括的に評価するものである。これまでのGBD研究は、1990年から2016年にかけての死亡率の低下が、平均寿命の伸び、世界人口の高齢化、病気やけがの非致死的負担の拡大につながったことを示してきました。これらの研究はまた、世界人口のかなりの部分が、異なる原因、場所、年齢、性別の間でかなりの不均質性をもって、非致死的な健康損失を経験していることを示した。GBD研究の継続的な目標は、推計の詳細レベルを上げ、分析戦略を改善し、高品質なデータ量を増やすことである。 【方法】354の疾病と傷害、3484の後遺症について、発生率と有病率を推計した。最新の広範な文献研究,調査データ,サーベイランスデータ,入院記録,外来受診記録,健康保険請求書を用い,さらに死因モデルの結果を用いて,合計68 781のデータソースから推定を行った。インド、イラン、日本、ヨルダン、ネパール、中国、ブラジル、ノルウェー、イタリアから新たに入手した臨床データ、米国からの最新の請求データ、台湾(中国省)およびシンガポールからの新しい請求データを取り入れた。推定には主にベイズ型メタ回帰ツール DisMod-MR 2.1 を用い、各病態の発生率、有病率、寛解率、死因の間に一貫性を持たせた。YLDは、各相互排他的後遺症の健康状態に対する有病率推定値と障害ウエイトの積として推定し、併存症で調整した。一人当たりの所得,学校教育年数,合計特殊出生率からなる要約開発指標である社会人口統計指数(SDI)を更新した.さらに、男女のYLDの差を算出し、男女間の乖離した傾向を確認しました。GBD 2017は,「正確で透明性のある健康推定報告のためのガイドライン」に準拠している。 【調査結果】世界的に,女性では,1990年と2017年の両方で,年齢標準化有病率が最も高かった原因は,口腔障害,頭痛障害,ヘモグロビン異常症および溶血性貧血症であった。男性では、1990年、2017年ともに、年齢標準化有病率が最も高かった原因は、口腔障害、頭痛障害、潜在性結核感染を含む結核であった。YLD数では、1990年では腰痛、頭痛障害、食事性鉄欠乏がレベル3原因のトップであったが、2017年では男女合わせて腰痛、頭痛障害、うつ病性障害がトップであった。全原因年齢標準化YLD率は1990年から2017年にかけて3~9%(95%不確実性区間[UI]3-1~4-6)減少したが,全年齢YLD率は7~2%(6-0~8-4)増加し,世界のYLDの総和は562万(421~723)人から8億5300万(642~ 1100)人へと上昇した。男女の増加率はほぼ同じであり、全年齢のYLD率は男性で7-9%(6-6-9-2)、女性で6-5%(5-4-7-7)の増加であった。複数の原因による年齢標準化有病率推定値では、男女間に有意差が認められた。2017年に男女間の相対的な差が最も大きかった原因には、物質使用障害(男性10万人当たり3018件[95% UI 2782-3252]対女性10万人当たりs1400[1279-1524])、交通外傷(3322[3082-3583]対 2336[2154-2535] )、自傷および対人暴力(3265[2943-3630]対5643[5057-6302])などがあった。 【解釈】世界の全原因年齢標準化YLD率は、ほぼ30年にわたる期間にわずかながら改善されただけである。しかし、非致死的疾患の負担の大きさは世界的に拡大しており、幅広い疾患を持つ人々が増加している。1990年以降、一部の疾患は世界的に広まり続けていますが、他の疾患はよりダイナミックな傾向を示しており、世界中の異なる年齢、性別、地域がさまざまな負担と健康喪失の傾向を経験しています。本研究では、特定の疾患における早死率の世界的な改善が、複雑で費用のかかる疾患を持つ高齢者集団につながったことを強調するとともに、疾患や傷害の特定の領域における世界的な成果も明らかにしている。 第一人者の医師による解説 腰痛・頭痛、運動機能低下、視聴覚障害 必ずしも死に至らない健康問題が課題 野村 周平 東京大学大学院医学系研究科国際保健政策学教室助教 MMJ.August 2019;15(4) 本論文は最新の世界の疾病負荷研究(Global Burden of Disease:GBD)プロジェクト(GBD 2017)の研究成果からの1編であり(1)、世界195 の国・地域における障害(disability)を詳細に分析したものである。本研究では、障害生存年数(Years lived with disability:YLD)で存命中の疾病負荷を評価している。YLDは障害を抱えて過ごす年数であり、障害の程度によって重み付けされる。 GBDでは過去に、仮想人物2人の健康状態の比較などの質問調査を行い、0~1点の障害度の重み付け評価を実施している(2)。GBD2017では、欧州諸国で再現された同様の調査結果を加えて、各疾病の障害度の重み付けを再評価している(3),(4)。 これまでのGBD結果で、完全に健康な状態で生活している人はほとんどおらず、人々は年齢を重 ねるにつれて健康問題を蓄積することがわかっている。本研究結果では、1990~2017年の間に、年齢調整 YLD率(人口増加や高齢化を考慮した場合)は3.9%の低下を見せた。しかしながら、世界の総 YLDは同期間で51.8%の増加が認められた。 医療の進歩や開発の進展によって、世界の人口の大半が早死にしなくなったものの、人口増加や高齢 化に伴い病気を抱えながら長生きするようになった人が増えていることを示している。 世界ではこれまで感染症などによる致死的な疾患との闘いが繰り広げられてきた。一方現在では、 腰痛など筋骨格系の痛みや運動機能の低下、頭痛、見聞き・思考する力の低下など、必ずしも死に至らない障害が大きな課題である。 本研究の重要な発見の1つは、354種類の疾病・ 傷害に伴う障害を調べたところ、世界の総 YLDの半数以上がそのうちわずか12種類の少数の疾病が原因となっていたことだ。腰痛、頭痛、うつ病はこの20年、YLDのトップ 3の原因だ。鉄欠乏性貧血は2000年以降、YLDの原因の4位から7位へ 脱落した(人口当たりのYLD率で24.7%減)。一方で同期間中、糖尿病はYLD率で38.8%増と顕著な伸びを示し、6位から4位へと順位を上げた。また老年性難聴や失明・視力障害も、それぞれ18.3%、 14.3%と増加していることがわかった。 昨今の高齢社会では、健康上の問題(障害)がない期間の延伸が医療政策の大きな柱となっている。世界経済が低迷している現代において、GBD 2017は、優先順位決定のための1つのベンチマークとして、疾患別の障害に関する比較可能なエビデンスを提供している。GBDは公益を目的としてデータのビジュアル化も行っている(https://vizhub.healthdata.org/gbd-compare/)。 1. The Lancet. Lancet. 2018;392(10159):1683. 2. Salomon JA, et al. Lancet. 2012;380(9859):2129-2143. 3. Haagsma JA, et al. Popul Health Metr. 2015;13:10. 4. Salomon JA, et al. Lancet Glob Health. 2015;3(11):e712-723.
2型糖尿病および慢性腎臓病患者におけるアトラセンタンと腎イベント(SONAR):二重盲検、無作為化、プラセボ対照試験。
2型糖尿病および慢性腎臓病患者におけるアトラセンタンと腎イベント(SONAR):二重盲検、無作為化、プラセボ対照試験。
Atrasentan and renal events in patients with type 2 diabetes and chronic kidney disease (SONAR): a double-blind, randomised, placebo-controlled trial Lancet 2019 May 11 ;393 (10184):1937 -1947. 上記論文のアブストラクト日本語訳 ※ヒポクラ×マイナビ 論文検索(Bibgraph)による機械翻訳です。 【背景】2型糖尿病患者に対して、選択的エンドセリンA受容体拮抗薬であるアトラセンタンを低用量で短期投与すると、有意なナトリウム貯留を引き起こすことなくアルブミン尿が減少する。我々は、主要な腎アウトカムに対するアトラセンタン治療の長期的効果を報告する 【METHODS】我々は、41カ国の689施設で二重盲検無作為化プラセボ対照試験を実施した。対象は18~85歳の成人で、2型糖尿病、体表面積1~73m2あたりの推定糸球体濾過率(eGFR)25~75mL/min、尿中アルブミン/クレアチニン比(UACR)300~5000mg/gで、レニン-アンジオテンシン系の最大投与期間または忍容性のあるレニン-アンジオテンシン系阻害薬を4週間以上投与された患者であった。無作為群に割り付けられる前の濃縮期間に、参加者にはアトラセンタン0-75mgを1日1回経口投与した。濃縮期間中にUACRが30%以上低下し、実質的な体液貯留が認められなかった者(反応者)を二重盲検治療期間に含めた。反応者は、アトラセンタン0-75mgを1日1回経口投与する群とプラセボ投与する群のいずれかに無作為に割り付けられた。すべての患者と治験責任医師は、治療の割り付けをマスクした。主要エンドポイントは、すべての反応者の意図的治療集団における血清クレアチニンの倍増(30日以上持続)または末期腎疾患(1-73m2あたりのeGFRが15mL/min未満、90日以上持続、90日以上の慢性透析、腎移植、または腎不全による死亡)の複合値としました。安全性は、割り当てられた試験治療を少なくとも1回投与されたすべての患者さんで評価されました。本試験はClinicalTrials. gov、番号NCT01858532に登録されている。 【FINDINGS】2013年5月17日から2017年7月13日までの間に、11人の患者がスクリーニングされた;5117人が濃縮期間に入り、4711人が濃縮期間を終了した。このうち、2648人の患者が奏効し、アトセンタン群(n=1325)またはプラセボ群(n=1323)に無作為に割り付けられた。追跡期間中央値は2-2年(IQR 1-4-2-9)であった。アトラセンタン群1325例中79例(6-0%)、プラセボ群1323例中105例(7-9%)に主要複合腎エンドポイントイベントが認められた(ハザード比[HR]0-65[95%CI 0-49-0-88]、p=0-0047)。これまでエンドセリン受容体拮抗薬に起因するとされてきた体液貯留と貧血の有害事象は、プラセボ群よりもアトセンタン群の方が頻度が高かった。心不全による入院は、アトラセンタン群では1325人中47人(3-5%)、プラセボ群では1323人中34人(2-6%)に認められました(HR 1-33 [95%CI 0-85-2-07]; p=0-208)。アトラセンタン群58例(4-4%)、プラセボ群52例(3-9%)が死亡した(HR 1-09 [95%CI 0-75-1-59]; p=0-65)。 【INTERPRETATION】アトラセンタンは、有効性と安全性を最適化するために選択された糖尿病および慢性腎臓病患者において、腎イベントのリスクを低下させた。これらのデータは、末期腎疾患を発症するリスクの高い2型糖尿病患者の腎機能を保護するための選択的エンドセリン受容体拮抗薬の潜在的な役割を支持している 【FUNDING】AbbVie. 第一人者の医師による解説 有害事象を最小限にし 有効性維持する治療法開発を期待 南学 正臣 東京大学医学部附属病院腎臓・内分泌内科教授 MMJ.August 2019;15(4) 2019年4月に、筆者がプログラム委員長を務めた 国際腎臓学会総会(WCN 2019)が、オーストラリアのメルボルンで開催された。学会の目玉は、late-breaking clinical trial sessionで発表されたSONAR研究とCREDENCE研究で、このセッションは全世界にライブストリーミングで中継し、SONAR研究はLancetに、CREDENCE研究は New England Journal of Medicineに発表とともに掲載され、大きな反響を呼んだ(1)。 SONAR研究は、AbbVie社がスポンサーとなって行った選択的エンドセリン A受容体拮抗薬アトラセンタンの二重盲検多施設ランダム化比較試験である。本試験では、アトラセンタンに短期的に反応した患者(体液貯留なくアルブミン尿が30%以上減少)における長期的な有効性と安全性をみる enrichment designが採用され、eGFR 25~75 mL/分 /1.73m2 , 尿 ア ル ブ ミ ン /Cr比 300~ 5,000mg/g Crの2型糖尿病患者が組み入れられた。主要エンドポイントは血清クレアチニン倍化と末期腎不全の複合エンドポイントとした。 Enrichment phaseを完遂した患者4,711人のうち2,648人がresponderであった。本研究は主要アウトカムイベント数が当初の予想より少ないということでスポンサー企業が研究期間の途中で中止を決定した。アトラセンタン群(n=1,325) では6.0%(79人)、プラセボ群(n=1,323)で は7.9%(105人)が主要エンドポイントに到達した(相対リスク , 0.65;95%信頼区間[CI], 0.49 ~0.88;P=0.0047)。しかし、体液貯留 および貧血はアトラセンタン群に多く認められた(体液貯留:36.6% 対 32.3%[P=0.022]、貧血: 18.5%対10.3%[P<0.0001])。 有害事象を理由にスポンサー企業は本薬物の糖尿病性腎臓病(DKD)をターゲットとした開発を中止したが、イベント数が予想より少なかったにもかかわらず統計学的に有意な効果が認められており、有害事象を最小限にして有効性を維持できるような治療法の開発が期待される。 1. Nangaku M. Kidney Int. 2019;96(1):2-4.
2型糖尿病における心血管および腎疾患の一次および二次予防のためのSGLT2阻害剤:心血管アウトカム試験のシステマティックレビューとメタアナリシス。
2型糖尿病における心血管および腎疾患の一次および二次予防のためのSGLT2阻害剤:心血管アウトカム試験のシステマティックレビューとメタアナリシス。
SGLT2 inhibitors for primary and secondary prevention of cardiovascular and renal outcomes in type 2 diabetes: a systematic review and meta-analysis of cardiovascular outcome trials Lancet 2019 Jan 5 ;393 (10166):31 -39. 上記論文のアブストラクト日本語訳 ※ヒポクラ×マイナビ 論文検索(Bibgraph)による機械翻訳です。 【背景】ナトリウム-グルコースコトランスポーター-2阻害剤(SGLT2i)の特定の心血管および腎アウトカムに対する効果の大きさ、および主要なベースライン特性に基づく異質性の有無については、依然として未定義である。 【METHODS】2型糖尿病患者を対象としたSGLT2iの無作為化プラセボ対照心血管アウトカム試験のシステマティックレビューおよびメタアナリシスを行った。2018年9月24日までに発表された試験をPubMedおよびEmbaseで検索した。データ検索と抽出は、標準化されたデータフォームを用いて行い、不一致はコンセンサスで解決した。有効性のアウトカムは、主要な心血管有害事象(心筋梗塞、脳卒中、心血管死亡)、心血管死亡または心不全による入院の複合、および腎疾患の進行であった。ハザード比(HR)と95%CIは試験間でプールされ、有効性の結果はベースラインでの動脈硬化性心血管系疾患の存在、心不全、および腎機能の程度によって層別化された。 【結果】同定された3つの試験から得られたデータ、34322人の患者(60-2%が動脈硬化性心血管系疾患を有する)、3342件の主要な心血管系有害事象、2028件の心血管系死亡または心不全による入院、766件の腎複合事象が含まれていた。SGLT2iは、主要な心血管イベントを11%減少させたが(HR 0-89 [95% CI 0-83-0-96]、p=0-0014)、動脈硬化性心血管系疾患のある患者でのみ有効性が認められ(0-86 [0-80-0-93])、動脈硬化性心血管系疾患のない患者では有効性が認められなかった(1-00 [0-87-1-16]、p for interaction=0-0501)。SGLT2iは、心血管死または心不全による入院のリスクを23%減少させ(0-77 [0-71-0-84]、p<0-0001)、動脈硬化性心血管疾患の有無や心不全の既往歴の有無にかかわらず同様の効果を示した。SGLT2iは腎疾患の進行リスクを45%減少させ(0-55 [0-48-0-64]、p<0-0001)、動脈硬化性心血管系疾患の有無にかかわらず同様の効果を示した。SGLT2iのベネフィットの大きさは、ベースラインの腎機能によって異なり、ベースラインの腎疾患が重度の患者では、心不全による入院の減少が大きく(p for interaction=0-0073)、腎疾患の進行の減少が小さかった(p for interaction=0-0258)。 【解釈】SGLT2iは、動脈硬化性の主要な有害心血管系イベントに対するベネフィットは中程度で、動脈硬化性の心血管系疾患が確立している患者に限られていると思われる。しかし、SGLT2iは、既存の動脈硬化性心血管系疾患や心不全の病歴にかかわらず、心不全による入院や腎疾患の進行を抑制するという点では、しっかりとした効果を発揮します。 第一人者の医師による解説 心血管病既往の有無にかかわらず 2型糖尿病治療の目標達成に寄与 辰巳 文則 川崎医科大学糖尿病・代謝・内分泌内科/加来 浩平 川崎医科大学・川崎医療福祉大学 MMJ.August 2019;15(4) 近年のSGLT2阻害薬に関する一連の心血管疾患アウトカム検証試験(CVOT)は、本クラス薬剤が主要心血管イベント(MACE)だけでなく、心不全や腎機能悪化のリスク抑制効果を発揮する可能性 を強く示唆している。本研究は背景が異なる2型 糖尿病患者を対象とした3つのCVOT(EMPA-REG OUTCOME、CANVAS Program、DECLARE-TIMI 58)のデータを統合し、試験開始時のアテローム 性心血管疾患(ASCVD)、心不全の既往や腎機能の程度により層別化して、MACE、心不全入院、全死亡、腎アウトカムの評価項目ごとに交互作用の有無を検討したメタアナリシスである。 対象 は 合計34,322人(60.2 % にASCVD、 11.3 % に 心不全 の 既往 )で、平均年齢 は63.5 歳(35.1%が 女性)であった。SGLT2阻害薬は MACEを11%有意に減少(ハザード比[HR], 0.89; 95% CI, 0.83~0.96;P=0.0014)させたが、 ASCVD既往なしの患者では有意ではなかった(P =0.0501)。ただし、心血管死・心不全入院については、ASCVDや心不全の既往の有無にかかわらずリスクを23%低下(HR, 0.77;95% CI, 0.71 ~0.84;P<0.0001)させた。脳卒中リスクには影響しなかった。 腎アウトカムについては、SGLT2阻害薬が腎機 能悪化、末期腎不全、腎死のリスクを45%低下(HR, 0.55;95% CI, 0.48~0.64;P<0.0001)させ、 その腎保護効果はASCVDの存在には左右されず、 開始時の腎機能が保たれているほど強い効果を示す傾向があっ た(eGFR 60mL/分 /1.73m2未満で33%、60以上~90未満で44%、90以上で 56%の減少)。逆に、心不全による入院リスクは開始時のeGFRが低いほど減少した(それぞれ40%、 31%、12%の減少)。 下肢切断と骨折のリスク上昇が1つの研究でのみ示された(1)が、異質性が高く、後に同薬で行われたCREDENCE試験では明らかな上昇がなかった(2)。 また、糖尿病ケトアシドーシスのリスクはSGLT2 阻害薬で約2倍であったが、発生そのものがまれであった。 今回の結果は、SGLT2阻害薬がMACEに対する 2次予防として有用である一方、心不全入院および腎機能悪化のリスク低下は心血管疾患既往の有無にかかわらず高い有益性が期待できるが、開始時の腎機能によって効果が影響される可能性を示唆するものである。 本メタアナリシスによってSGLT2阻害薬は心 血管疾患や心不全の既往の有無にかかわらず2型 糖尿病治療の目標達成に寄与しうる薬剤であることがより明確になったといえよう。 1. Neal B, et al. N Engl J Med. 2017;377(7):644-657. 2. Perkovic V, et al. N Engl J Med. 2019 Apr 14. doi:10.1056/NEJMoa1811744.
急性虚血性脳卒中に対する静脈内血栓溶解療法と血圧の集中的な低下(ENCHANTED):国際無作為化、非盲検、盲検エンドポイント、第3相試験。
急性虚血性脳卒中に対する静脈内血栓溶解療法と血圧の集中的な低下(ENCHANTED):国際無作為化、非盲検、盲検エンドポイント、第3相試験。
Intensive blood pressure reduction with intravenous thrombolysis therapy for acute ischaemic stroke (ENCHANTED): an international, randomised, open-label, blinded-endpoint, phase 3 trial Lancet 2019 Mar 2 ;393 (10174):877 -888. 上記論文のアブストラクト日本語訳 ※ヒポクラ×マイナビ 論文検索(Bibgraph)による機械翻訳です。 【背景】急性虚血性脳卒中患者において、収縮期血圧185mmHg以上はアルテプラーゼ静注による血栓溶解療法の禁忌であるが、最適な転帰のための収縮期血圧の目標値は不明である。我々は,急性虚血性脳卒中に対してアルテプラーゼを投与した患者において,ガイドラインで推奨されている血圧低下と比較して,集中的な血圧低下を評価した。 【方法】15か国110施設でスクリーニングされた,急性虚血性脳卒中で収縮期血圧150mmHg以上の血栓溶解療法の適用患者(年齢18歳以上)を対象に,国際部分要因・オープンラベル・ブラインドエンドポイント試験を行った。脳卒中発症後6時間以内に,対象患者を72時間かけて血圧を下げる集中治療(目標収縮期血圧130~140mmHg,1時間以内)またはガイドライン治療(目標収縮期血圧180mmHg未満)に1対1でランダムに割り付けた.主要アウトカムは,90日後の機能状態を修正ランキン尺度得点で測定し,無調整順序ロジスティック回帰で分析した.安全性の主要評価項目は,頭蓋内出血の有無とした.主要評価項目と安全性評価項目は,盲検下で実施された.解析は intention-to-treat ベースで行われた。本試験はClinicalTrials. gov(番号NCT01422616)に登録されている。 【所見】2012年3月3日から2018年4月30日の間に、2227例が治療群にランダムに割付られた。同意の欠如、無作為化の誤りまたは重複により31名の患者を除外した後、急性虚血性脳卒中のアルテプラーゼ適用患者2196名を対象とした。集中治療群1081例,ガイドライン治療群1115例で,実際にalteplaseを静脈内投与した2175例のうち1466例(67~4%)に標準用量が投与された。脳卒中発症から無作為化までの時間の中央値は3〜3時間(IQR 2-6-4-1)であった。24時間の平均収縮期血圧は,集中治療群で144-3 mm Hg(SD 10-2),ガイドライン群で149-8 mm Hg(12-0) であった(p<0-0001).集中治療群1072例、ガイドライン群1108例で主要評価項目データを得ることができた。90 日後の機能状態(mRS スコア分布)に群間差はなかった(未調整オッズ比 [OR] 1-01、95% CI 0-87-1-17、p=0-8702)。頭蓋内出血は,集中治療群(1081 例中 160 例[14-8%])の方がガイドライン群(1115 例中 209 例[18-7%])より少なかった(OR 0-75,0-60-0-94,p=0-0137 ).重篤な有害事象が発生した患者数は、集中治療群(1081例中210例[19-4%])とガイドライン群(1115例中245例[22-0%]、OR 0-86, 0-70-1-05, p=0-1412)で有意差はなかった。主要アウトカムに関して、集中的な血圧低下とアルテプラーゼの用量(低用量と標準用量)の相互作用のエビデンスはなかった。この結果は、軽度から中等度の急性虚血性脳卒中にアルテプラーゼを投与された患者に対して、この治療法を適用することに大きくシフトすることを支持しないかもしれない。この患者群における早期の集中的な血圧低下による有益性と有害性の基礎的なメカニズムを明らかにするために、さらなる研究が必要である。 【出典】National Health and Medical Research Council of Australia; UK Stroke Association; Ministry of Health and the National Council for Scientific and Technological Development of Brazil; Ministry for Health, Welfare, and Family Affairs of South Korea; Takeda.など。 第一人者の医師による解説 両群間の血圧差がわずかで 有意差の証明に至らず 内山 真一郎 国際医療福祉大学教授/山王病院・山王メディカルセンター脳血管センター長 MMJ.August 2019;15(4) 急性虚血性脳卒中(脳梗塞)患者において収縮期 血圧185mmHg以上はアルテプラーゼによる血 栓溶解療法の禁忌とされているが、転帰改善をもたらす至適血圧目標値は不明である。 今回報告されたENCHANTED試験は、アルテプラーゼを投与された 発症後6時間以内 の 急性虚血性脳卒中患者において、72時間以上の、ガイドラインが推奨する降圧療法と、収縮期血圧のアルテプラーゼ投与後1 時間以内の降圧目標が130~140mmHgの積極的降圧療法を比較したprospective randomized open blinded end-point(PROBE)デザインによる国際共同試験であった。 有効性の1次評価項目は 90日後の改変ランキンスコアの分布、安全性の1 次評価項目は全頭蓋内出血であった。患者1,081 人が積極降圧療法群、1,115人がガイドライン降 圧療法群に無作為に割り付けられた。90日後の転帰は両群間で差がなかったが、頭蓋内出血は積極的 降圧療法群で有意に少なかった。 積極的降圧療法は 安全であることは証明されたが、頭蓋内出血の減少が転帰の改善に結びつかなかったので、現行のガイドラインの降圧目標値を下げる変更を支持するものではなかった。ただし、積極的降圧療法群が 達成した 平均収縮期血圧 は144.3mmHgと目標に達しておらず、ガイドライン降圧療法群の平均 収縮期血圧も149.8mmHgと予想以上に低く、目標としていた両群間の血圧差15mmHgには遠く及ばず、両群間の血圧差がわずか5.5mmHgと少なすぎたため有意差を証明する検出力が弱くなり、 このような結果がもたらされたとも考えられ、この試験結果から至適血圧レベルに結論を下すことはできない。 これまでの観察研究では、脳梗塞急性期の収縮期血圧は140~150mmHgが最も良好な転帰をもたらすことが報告されており、両群ともに平均収縮期血圧はこの範囲に入っているので 両群間で転帰に差がなかったのは当然かもしれな い。脳梗塞急性期患者では、血圧の自動調節能が低下するため過度の降圧は梗塞巣の拡大をもたらす懸念があり、過度の血圧上昇を放置することも頭蓋内出血のリスクを高めることが危惧されるので、 試験参加医師が両方のリスクを考慮したことが両群間の血圧差を少なくさせた心理的要因であったように思われる。
250 の死因に対する平均寿命、失われた生命年、全死因および死因別死亡率の予測:195 の国と地域に対する 2016-40 年の参照シナリオと代替シナリオ。
250 の死因に対する平均寿命、失われた生命年、全死因および死因別死亡率の予測:195 の国と地域に対する 2016-40 年の参照シナリオと代替シナリオ。
Forecasting life expectancy, years of life lost, and all-cause and cause-specific mortality for 250 causes of death: reference and alternative scenarios for 2016-40 for 195 countries and territories Lancet 2018 ;392 (10159):2052 -2090. 上記論文のアブストラクト日本語訳 ※ヒポクラ×マイナビ 論文検索(Bibgraph)による機械翻訳です。 【背景】健康における潜在的な軌道と健康の推進要因を理解することは、長期的な投資と政策の実施を導く上で極めて重要である。予測に関する過去の研究は、将来の健康シナリオの不完全な風景を提供しており、政策オプションと潜在的な健康の軌道を評価することができる、より堅牢なモデリングプラットフォームの必要性を強調している。本研究は、195の国と地域における2016年から2040年までの250の死因について、平均寿命、全死因死亡率及び死因の予測-及び代替的な将来シナリオをモデル化する新しいアプローチを提供する。 【方法】我々は、2017年から40年の予測を生成するために、1990年から2016年のGBD 2016推定を用いて、世界疾病、負傷及びリスク要因研究(GBD)階層的原因構造によって編成された250の原因及び原因群をモデル化した。我々のモデリングフレームワークは、GBD 2016研究のデータを用いて、健康の79の独立したドライバーについて、リスク要因と健康アウトカムとの関係を系統的に説明した。我々は、原因別死亡率の3成分モデルを開発した:リスク要因の変化と選択的介入による成分、一人当たり所得、教育達成度、25歳未満の合計特殊出生率と時間の関数である各原因の基礎的死亡率、時間に相関した説明できない変化に対する自己回帰統合移動平均モデルである。1990年から2006年のデータでモデルを当てはめ、これを用いて2007年から16年の予測を行うことで、性能を評価した。予測と代替シナリオの生成に使用した最終モデルは、1990年から2016年のデータに適合させた。このモデルを195の国と地域について使用し、場所ごとの各指標について2040年までの参照シナリオまたは予測を作成した。さらに、すべてのGBDリスク因子、一人当たり所得、教育達成度、選択的介入率、過去25年未満の合計出生率について、場所年ごとの年率変化率のそれぞれ85%および15%に基づき、より健康なシナリオおよびより悪い健康シナリオを生成した。このモデルを用いて、250の原因について、全死因年齢性別死亡率、平均余命、損失年数(YLL)を算出した。出生率に関するシナリオも作成し、人口シナリオを作成するためのコホート成分モデルで使用した。基準予測,健康増進シナリオ,健康悪化シナリオのそれぞれについて,将来における各リスク要因に起因する死亡率とYLLの推定値を作成した。 【調査結果】世界的に,健康の独立したドライバーのほとんどが2040年までに改善すると予測されたが,36は悪化すると予測された。より良い健康シナリオが示すように、より大きな進歩が可能かもしれないが、高い体格指数(BMI)のようないくつかのドライバーについては、介入がない場合、その犠牲者は増加するだろう。我々は、2040年までに世界の平均寿命が男性で4-4年(95% UI 2-2 to 6-4)、女性で4-4年(2-1 to 6-4)伸びると予測したが、健康状態の良いシナリオと悪いシナリオに基づくと、男性では7-8年の増加(5-9 to 9-8)から有意ではない0-4年の減少(-2-8 to 2-2)、女性では7-2年の増加(5-3 to 9-1)から本質的に変化なし(0-1年 [-2-7 to 2-5])までのトラジェクトリーが可能であった。2040年には、日本、シンガポール、スペイン、スイスが男女とも85歳を超え、中国を含む59カ国が80歳を超えると予測された。一方、中央アフリカ共和国、レソト、ソマリア、ジンバブエは2040年の平均寿命が65歳未満と予測され、現在の傾向が続くとすれば、生存率の世界的な格差が継続する可能性があることが示された。予測されたYLLは、人口増加と高齢化により、いくつかの非伝染性疾患(NCDs)による犠牲者の増加を示している。基準予測と代替シナリオの違いは、HIV/AIDSにおいて最も顕著であり、健康悪化シナリオの下では、2016年から40年にかけてYLLが120-2%(95% UI 67-2-190-3)増加する可能性があると予測された(約1億1800万人)。2016 年と比較して、NCD は 2040 年までにすべての GBD 地域で YLL に占める割合が大きくなると予測された(世界の YLL の 67-3% [95% UI 61-9-72-3] )。それでも、多くの低所得国では、2040 年の YLL に占める感染症、妊婦、新生児、栄養(CMNN)の割合はまだ大きい(例えば、サブサハラ・アフリカでは YLLの 53-5% [95% UI 48-3-58-5] など)。多くの健康リスクにおいて、帰属する YLL の基準予測とより良い健康シナリオの間に大きなギャップがあった。ほとんどの国において,ヘルスケアに従順な代謝リスク(例:高血圧,高血漿空腹時血糖),および集団レベルまたはセクター間介入によって最もターゲットとなるリスク(例:タバコ,高 BMI,環境中粒子状物質汚染)は,基準予測とより良い健康シナリオの間で最大の差がある.主な例外はサハラ以南のアフリカで、貧困と低開発レベルに関連する多くのリスク(例えば、安全でない水と衛生、家庭大気汚染、子供の栄養不良)が、2040年においても基準シナリオとより良い健康シナリオの間の実質的格差を占めると予測された。我々の参照予測は、ほとんどの国で2040年まで全体的な改善を指摘しているが、健康シナリオの改善と悪化に見られる幅は、不安定な未来像を示している-技術革新による加速的な進歩を持つ世界だが、意図的な政策行動がない場合は健康アウトカムが悪化する可能性を持つ。YLLsの原因によっては、基準予測と代替シナリオの間に大きな差があるため、各国がより良い健康シナリオに向かって軌道修正すれば、利益を加速させる機会が得られ、基準予測に遅れをとれば憂慮すべき課題が発生する。一般的に、意思決定者は NCDs への移行が続くことを想定し、早期の死亡率を大幅に押し上げる修正可能なリスク に資源を集中させるべきです。そのような修正可能なリスクを今日優先させれば、将来的に回避可能な死亡率を減少させる機会 がある。しかし、CMNNの原因とそれに関連するリスクは、低所得国において引き続き保健上の優先事項である。我々の2040年の健康悪化シナリオに基づけば、各国がHIVの流行に対する勢いを失い、この病気に対する数十年の進歩を危うくすれば、HIV死亡率が回復する現実的なリスクが存在する。技術革新の継続と、世界の最貧困層を対象とした保健分野の開発援助を含む保健支出の増加は、すべての国民が健康で充実した生活を送ることができる未来を描くために、今後も不可欠な要素であると考えられます。 第一人者の医師による解説 今後の政策の選択 各国の将来の健康に大きな影響 野村 周平 慶應義塾大学医学部医療政策管理学特任准教授 MMJ.February 2020;16(1) 本論文は世界の疾病負荷研究(Global Burden of Disease ;GBD)プロジェクトの研究成果からの1編であり、GBD 2016の枠組みに基づき(1),(2)、世界195の国・地域における2016~40年の250 疾患別の死亡率、および平均寿命や損失生存年数 (YLL)を分析したものである。 死亡率の予測モデルは1990~2016年のデータに基づき、大きく以下の3要素からなる:(1)疾患との関連が認められる危険因子の年次変化(2)社会人口指数(SDI:1人当たりの収入レベル、教育レベル、25歳以下の出生率の混合指標)(3)これらで説明できない時間的変動(自己回帰和分移動平均モデル[ARIMA]を適用)。特別なシナリオを想定しない将来予測(基準シナリオ)に加え、仮想的な健康増進のシナリオを2つ設定している。すなわち、危険因子保有率とSDIの3要素について年率 換算した変動率の85および15パーセンタイルを、 将来起こりうる変動と仮定した“良い(better)”シ ナリオと“悪い(worse)”シナリオである。 結果、世界の平均寿命は向上し、男女ともに 2040年までに4.4歳延伸すると予測された。日本、シンガポール、スペイン、スイスでは男女ともに85歳を超える。一方、中央アフリカ共和国、レソト、ソマリア、ジンバブエは依然65歳未満と予測され、世界の寿命格差は大きいままである。良い シナリオで最大7歳以上の平均寿命の向上が予測 される一方で、悪いシナリオでは現在比較で有意な変化は認められなかった。 2040年、非感染症が世界的 にYLLの大部分 (67.3%)を占めるが、サハラ以南アフリカをはじめ多くの低所得国においては、感染症、妊婦・新生児の疾患、栄養関連疾患が依然 YLLの大部分を占めると予測された。 世界的に高血圧、高血糖、高BMIなどの代謝系リスク、また喫煙や微小粒子状物質汚染のYLLへの寄与が、基準̶良いシナリオ間で差が大きかった。 つまりこれら危険因子によるYLL回避は健康増進に大きく寄与しうると解釈できる。一方、サハラ以南アフリカは例外で、安全ではない水・衛生、家庭内空気汚染、小児栄養失調など、貧困関連の危険因 子でシナリオ間の差が大きかった。 多くの国で死亡率の継続的な低下、平均寿命の向上が予測される一方で、仮想シナリオ間で大きな差が認められ、将来の展望は不確実であることも示唆された。世界の最貧層への開発援助を含め、継続的な技術革新と保健支出の増加が、すべての人が豊かで健康な生活を送る上で、引き続き極めて重要な要素である。 1. GBD 2016 Risk Factors Collaborators. Lancet. 2017;390(10100):1345-1422. 2. GBD 2016 Causes of Death Collaborators. Lancet. 2017;390(10100):1151- 1210.
195の国と地域における健康関連の持続可能な開発目標の1990年から2017年までの進捗状況の測定と2030年までの達成度の予測:世界疾病負担調査2017のための系統的分析。
195の国と地域における健康関連の持続可能な開発目標の1990年から2017年までの進捗状況の測定と2030年までの達成度の予測:世界疾病負担調査2017のための系統的分析。
Measuring progress from 1990 to 2017 and projecting attainment to 2030 of the health-related Sustainable Development Goals for 195 countries and territories: a systematic analysis for the Global Burden of Disease Study 2017 Lancet 2018; 392: 2091 –2138 上記論文のアブストラクト日本語訳 ※ヒポクラ×マイナビ 論文検索(Bibgraph)による機械翻訳です。 【背景】国連の持続可能な開発目標(SDGs)の2015年の基準値を設定し、早期実施を監視する取り組みは、2030年までに健康を改善するための大きな可能性と脅威の両方を浮き彫りにしています。誰一人取り残さない」というSDGsの目標を完全に実現するためには、国レベルの推計を超えて健康関連のSDGsを検証することがますます重要となっています。Global Burden of Diseases, Injuries, and Risk Factors Study 2017(GBD 2017)の一環として、1990年から2017年までの195の国・地域の健康関連SDGs指標52項目のうち41項目の進捗を測定し、健康関連SDGs指標を推定、2030年までの指標を予測、世界の達成状況を分析しました。 【方法】1990年から2017年にかけて41の健康関連SDGs指標の進捗を測定し、GBD2016から4つの指標が増加しました(新しい指標は、保健師密度、非親密なパートナーによる性的暴力、人口調査状況、身体的・性的暴力の蔓延[別途報告]です)。また、以前に報告されたいくつかの指標の測定値を改善した。国レベルの推定値を作成し、健康関連のSDGsのサブセットについては、性別や社会人口統計指数(SDI)五分位による指標レベルの差異を調査しました。また、特定の国については、サブナショナルなパフォーマンス評価も行いました。健康関連SDGsの指標を構築するために、1990年から2030年まで算出した1000のドローのうち、0を2-5パーセンタイル、100を97-5パーセンタイルとして各指標の値を0-100で変換し、目標別に尺度化した指標の幾何平均を取りました。2030 年までの予測を行うため、より広範な GBD 研究から推定値を抽出し、1990 年から 2017 年までの指標別および国別の年率換算変化率の加重平均を使用した予測フレームワークを使用し、将来の推定値に反映させた。まず、2030年に予測される平均値を用い、次に1000回の抽選から算出される2030年の達成確率を用いて、目標が設定された指標の達成度を2つの方法で評価した。また、過去の傾向からSDGsの目標達成の可能性について、グローバルな達成度分析を行いました。SDGsの目標が設定された指標の2015年の世界平均を用い、これらの目標を達成するために2015年から2030年までに必要な世界の年率変化率を算出し、1990年から2015年の国レベルの変化率分布において、必要な世界の年率変化率がどのパーセンテージに該当するかを確認した。指標間でこれらの世界的パーセンタイル値の平均を取り、この平均世界的パーセンタイルにおける過去の変化率を、目標の定義にかかわらずすべての健康関連SDG指標に適用し、各指標の2030年相当の世界平均値と2015年から2030年の変化率を推定した。 【調査結果】2017年の健康関連SDG指標の世界中央値は59-4(IQR 35-4-67-3)、低い11-6(95%不確実性間隔 9-6-14-0) から高い 84-9(83-1-86-7) まで幅がありました。サブナショナル・レベルで評価された国々のSDGs指標の値は、日本や英国ではより均質であったが、特に中国やインドでは大きく変動した。また、SDI五分位値や性別によっても指標は異なり、特に非感染性疾患(NCD)死亡率、アルコール使用、喫煙などでは、男性の方が女性よりも悪い結果となった。ほとんどの国が2030年には2017年よりも健康関連のSDGs指数が高くなると予測されたが、2030年までに達成する国レベルの確率は指標によって大きく異なっていた。5歳未満死亡率、新生児死亡率、妊産婦死亡率、マラリアの指標は、目標達成確率が95%以上の国が最も多かった。NCD死亡率や自殺死亡率を含むその他の指標は、2030年の予測平均値に基づいて対応するSDGs目標を達成すると予測される国はなかったが、2030年までに達成する可能性があることが示された。子どもの栄養不良、いくつかの感染症、ほとんどの暴力対策など、いくつかの指標については、SDGsの目標達成に必要な年率換算の変化率は、過去にどの国も達成した進歩のペースをはるかに超えています。2030 年までに、喫煙とアルコール摂取をそれぞれ 19%と 22%削減すること、思春期の出生率を 47%低下させること、人口 1000 人当たりの保健ワーカー密度を 85%以上増加させることに相当する。 GBD 研究は、人口動態や地理的次元を超えて健康関連の SDGs を監視するためのユニークで強固なプラ ットフォームを提供している。我々の発見は、細分化されたデータの収集と分析の重要性を強調し、より意図的なデザインまたは介入策のターゲティングがSDGs達成の進捗を加速させる可能性がある場所を強調するものである。現在の予測では、健康に関連するSDGsの指標、NCD、NCD関連のリスク、暴力関連の指標の多くは、過去の成果を推進したもの(NCDの場合は治療的介入)から、SDGsの目標を達成するための多部門にわたる予防指向の政策行動や投資への協調的シフトを必要としています。注目すべきは、いくつかのターゲットが 2030 年までに達成されるためには、どの国も過去に達成したことのない 進捗ペースが要求されることです。未来は基本的に不確実であり、どのようなブレークスルーや出来事がSDGsの行方を変えるかを完全に予測できるモデルはありません。しかし、今日の私たちの行動や不作為が、2030年までに「誰一人取り残さない」ことにどれだけ近づけるかを左右することは明らかです。 【FUNDING】ビル&メリンダ・ゲイツ財団 第一人者の医師による解説 多分野にわたる予防志向の政策行動と さらなる投資が達成に必要 野村 周平 慶應義塾大学医学部医療政策管理学特任准教授 MMJ.February 2020;16(1) 持続可能な開発目標(SDG)は、「誰一人取り残さない」という世界的な掛け声のもとに、2015年9月の国連サミットで採択された、2030年までの 17のゴール・169のターゲットで構成される世界全体の目標である。健康目標においては、これまでさまざまな研究が健康改善の大きな可能性と課題の両方を浮き彫りにしており(1)、国レベルでの健康 関連のSDGの進捗評価がますます重要になっている。 本論文は世界の疾病負荷研究(Global Burden of Disease ;GBD)プロジェクトの研究成果からの 1編であり、GBD 2017の枠組みに基づき(2)、世界195の国・地域における1990~2017年の52 の健康関連 SDG指標のうち41の指標の進捗を0 ~100スケール(達成度の低~高)で測定し、さらに2030年までの予測指標を分析したものである。 2017年の41のSDG指数の平均値を国別にみると、中央値は59.4(四分位範囲35.4~67.3)で、最低が中央アフリカ共和国の11.6(95%不確実 性区間9.6~14.0)、最高がシンガポールの84.9 (83.1~86.7)であった。国内地方レベルで評価 したSDG指数平均値は、日本や英国ではより均一 的で地域差は小さかったが、中国とインドでは大きく地域差が認められた。また男女間、社会人口指 数間でも、SDG指数平均値に格差がみられた。「誰も取り残さない」ためには、細分化された評価が必要であることを示唆している。 指標別にみると、5歳未満児死亡率、新生児死亡率、妊産婦死亡率、マラリア死亡率はほとんどの国が達成する見込みである。しかし、その他の指標、例えば非感染症による死亡や自殺などの指標では、 特段の治療や予防の政策的介入なしに、2030年までにSDG目標を達成すると予測された国はなかった。子どもの栄養不良や暴力、結核などその他の感染症は特に、達成のために必要な指標値の年間改善率と実際の改善率のギャップが大きく、達成のためには甚大な努力が求められる。 大部分の国はすでにミレニアム開発目標(MDG)のための国家行動計画を策定しており、MDGに由来する指標目標(乳幼児、妊産婦、エイズ、マラリア、 結核など)を達成する上では良い状況にある。しかし、SDGは多くの国の政策においてカバーされていない。本研究は、SDG時代の残された数年間、 SDG達成のための継続的なモニタリング、多分野にわたる予防志向の政策行動とさらなる投資への協調的アプローチの必要性を示している。 1. Bertelsmann Stiftung, Sustainable Development Solutions Network. Sustainable Development Report 2019. URL: https://bit.ly/2sayJmJ 2. The Lancet. Lancet. 2018;392(10159):1683.
非制御喘息における単回吸入エキストラファイン3剤併用療法(TRIMARAN、TRIGGER):2つの二重盲検並行群間無作為化比較第3相試験。
非制御喘息における単回吸入エキストラファイン3剤併用療法(TRIMARAN、TRIGGER):2つの二重盲検並行群間無作為化比較第3相試験。
Single inhaler extrafine triple therapy in uncontrolled asthma (TRIMARAN and TRIGGER): two double-blind, parallel-group, randomised, controlled phase 3 trials Lancet 2019 ;394 (10210):1737 -1749. 上記論文のアブストラクト日本語訳 ※ヒポクラ×マイナビ 論文検索(Bibgraph)による機械翻訳です。 【背景】 現在までに、喘息における単回吸入3剤併用療法の有効性を評価した研究はない。ここでは、ベクロメタゾンジプロピオネート(BDP:吸入コルチコステロイド)、ホルモテロールフマル酸塩(FF:長時間作用型β2アゴニスト)、グリコピロニウム(G:長時間作用型ムスカリン拮抗薬)の単一吸入超微粒子配合剤とBDP+FF配合剤を比較した2試験結果を報告します。 【方法】2つの並行群間二重盲検無作為化能動比較第3相試験(Triple in Asthma With Uncontrolled Patients on Medium Strength of ICS + LABA[TRIMARAN]とTriple in Asthma High Strength Versus ICS/LABA HS and Tiotropium[TRIGGER])は、16か国、171施設(TRIMARAN)から、17か国、221施設から患者を募集した(TRIGGER)。対象施設は、二次医療施設と三次医療施設、および専門的な調査部門が混在しています。対象は、コントロール不良の喘息患者で、過去1年間に1回以上の増悪の既往があり、吸入コルチコステロイド(TRIMARAN:中用量、TRIGGER:高用量)と長時間作用型β2アゴニストによる治療経験のある成人(18~75歳)であった。登録された患者は、まずBDP/FF(TRIMARAN:BDP 100μgとFF 6μg、TRIGGER:BDP 200μgとFF 6μg)で2週間治療した後、国によって層別されたバランスブロック無作為化スキームのインタラクティブ応答技術システムを使ってランダムに治療が割り当てられました。患者、治験責任医師、施設スタッフ、スポンサースタッフは、BDP/FF/GおよびBDP/FFの割り付けをマスクされた。TRIMARANでは、患者は52週間のBDP/FF/G(BDP 100μg、FF 6μg、G 10μg)またはBDP/FF(BDP 100μg、FF 6μg)、1日2回吸入に(1:1)無作為に割り付けられました。TRIGGERでは、患者を52週間にわたり、BDP/FF/G(200μg BDP、6μg FF、10μg G)またはBDP/FF(200 BDP、6μg FF)を1日2回、あるいはオープンラベルでBDP/FF(200μg BDP、6μg FF)2吸入とTiotropium 2-5 μg2吸入の1日1回併用投与に無作為(2:2:1)割付けました。両試験(BDP/FF/G vs BDP/FF)の主要評価項目は、投与前の26週目の強制呼気1秒量(FEV1)と52週目の中等度および重度の増悪率でした。安全性は、少なくとも1回の投与が行われたすべての患者さんで評価されました。これらの試験はClinicalTrials. govに登録され、NCT02676076(TRIMARAN)、NCT02676089(TRIGGER)。 【所見】2016年2月17日から2018年5月17日の間に、TRIMARANの1155人の患者にBDP/FF/G(n=579)またはBDP/FF(n=576)が投与された。2016年4月6日から2018年5月28日の間に、TRIGGERの1437人の患者にBDP/FF/G(n=573)、BDP/FF(n=576)、またはBDP/FF+チオトロピウム(n=288)が投与された。BDP/FF群と比較して、投与26週目の投与前FEV1は、TRIMARANでは57mL(95%CI 15-99、p=0-0080)、TRIGでは73mL(26-120;また、中等度および重度の増悪の割合は、TRIMARANでは15%(率比0-85、95%CI 0-73-0-99、p=0-033)、TRIGGERでは12%(0-88、0-75-1-03、p=0-11)減少しています。治療関連の重篤な有害事象は4例で、TRIMARANではBDP/FF/G群に1例、TRIGGERではBDP/FF/G群に1例、BDP/FF群に2例の計3例であった。TRIMARANではBDP/FF/G群に3例、TRIGGERではBDP/FF/G群に1例、BDP/FF群に1例の計2例に死亡に至る有害事象が認められました。また,死亡例はいずれも治療との関連は認められなかった。 【解釈】コントロールされていない喘息において,吸入コルチコステロイド+長時間作用性β2-agonist療法に長時間作用性ムスカリン拮抗薬を追加すると,肺機能の改善と増悪の抑制が認められる。 【FUNDING】Chiesi Farmaceutici. 第一人者の医師による解説 単一デバイスに3剤配合 吸入アドヒアランス改善につながる可能性も 長瀬 洋之 帝京大学医学部内科学講座呼吸器・アレルギー学教授 MMJ.February 2020;16(1) 本論文 は、吸入 ステロイド 薬 (ICS)/長時間作 用性β2刺激薬(LABA)/長時間作用性抗コリン薬 (LAMA)を単一デバイスに配合したトリプル製剤の喘息に対する有効性を検討した、Chiesi 社主導 による2件の第3相試験の報告である。トリプル製剤は、慢性閉塞性肺疾患(COPD)に対してはすでに日本でも2剤が承認されているが、喘息に対し ては未承認である。本論文では喘息に対するトリプル製剤の長期効果が初めて報告されている。トリプル製剤のICS/LABA配合薬に対する優位性を52週にわたって検討しており、中用量 ICSを用いたTRIMARAN試験 (n=1,155)と高用量 ICSを 用いたTRIGGER試験(n=1,437)の2試験をまとめて報告している。 超微粒子の加圧式定量噴霧吸入器(pMDI)として、 ベクロメタゾン(ICS)、ホルモテロール (LABA)、 グリコピロニウム(LAMA)が配合されており、ベクロメタゾンの用量は中用量製剤では400 μ g/ 日、高用量製剤では800μ g/日である。主要評価項目である26週後の1秒量と52週間の増悪は、 ICS/LABAと比較し、トリプル製剤ではそれぞれ +57 mLと -15 % (TRIMARAN試験 )、+73 mLと-12%(TRIGGER試験)と有意に優れており (TRIGGER試験の増悪のみ P=0.11)、主要評価項目をほぼ満たした。   ICS/LABAへのLAMA追加効果については、チオトロピウムのICS/LABAへの追加によって呼吸機能改善と増悪抑制効果が先行試験で示されており(1)、今回の結果は想定の範囲内ではある。しかし、 単一デバイスに3剤が配合されると、喘息治療で常に問題となる吸入アドヒアランスの改善につながる可能性がある。また、今回の評価項目ではないが、高用量 ICS/LABAと中用量トリプル製剤の効果はほぼ同等であり、副作用で高用量 ICSを用いることができない場合、中用量トリプル製剤を用いる選択肢も出てくるかもしれない。   2019年の米国胸部疾患学会(ATS)では、ドライパウダー吸入器を用いるモメタゾン(ICS)/イン ダカテロール(LABA)/グリコピロニウム(LAMA) 配合薬 QVM149(Novartis社 )が、ICS/LABAに 比べ、呼吸機能改善の点で優れていることが第2 相試験で示された。COPDでは、すでにトリプル製剤として、テリルジー®やビレーズトリ®が用いられている。今後、複数のトリプル製剤が喘息に対して使用可能となり、剤型や吸入回数などの選択肢が 増えることが予想される。 1. Kerstjens HA et al. N Engl J Med. 2012;367(13):1198-1207.
成人の多発性分裂病の急性期治療における32種類の経口抗精神病薬の有効性と忍容性の比較:系統的レビューとネットワークメタ解析。
成人の多発性分裂病の急性期治療における32種類の経口抗精神病薬の有効性と忍容性の比較:系統的レビューとネットワークメタ解析。
Comparative efficacy and tolerability of 32 oral antipsychotics for the acute treatment of adults with multi-episode schizophrenia: a systematic review and network meta-analysis Lancet 2019 ;394 (10202 ):939 -951 . 上記論文のアブストラクト日本語訳 ※ヒポクラ×マイナビ 論文検索(Bibgraph)による機械翻訳です。 【背景】統合失調症は、世界中の成人に最も多く見られ、負担が大きく、費用のかかる精神疾患の1つである。抗精神病薬はその治療法として選択されているが、どの薬剤を使用すべきかについては論争がある。無作為化比較試験の情報を定量化することにより,抗精神病薬の比較とランク付けを行うことを目的とした。データベース開設から2019年1月8日までに,Embase,MEDLINE,PsycINFO,PubMed,BIOSIS,Cochrane Central Register of Controlled Trials(CENTRAL),WHO International Clinical Trials Registry Platform,ClinicalTrials. gov で検索を行った。2名の著者が独立して研究を選択し、データを抽出した。統合失調症または関連疾患の急性症状を有する成人を対象とした無作為化対照試験を対象とした。治療抵抗性、初回エピソード、陰性症状または抑うつ症状が優位な患者、内科的疾患の併存、再発予防の研究は除外した。主要アウトカムは、標準化された評価尺度を用いて測定された全体的な症状の変化とした。また、8つの有効性と8つの安全性についてのデータも抽出した。研究結果の違いは、メタ回帰分析および感度分析で検討した。効果量の測定は、標準化平均差、平均差、または95%信頼区間(CrI)付きのリスク比とした。エビデンスの信頼度は、CINeMA(Confidence in Network Meta-Analysis)を用いて評価した。研究プロトコルはPROSPEROに登録されており、番号はCRD42014014919である。 【所見】54417件の引用を確認し、53463人の参加者のデータを持つ402件の研究を組み込んだ。効果量の推定では、すべての抗精神病薬がプラセボよりも全症状を軽減することが示唆され(6剤では統計的に有意ではなかったが)、標準化平均差はクロザピンの-0-89(95%CI -1-08 to -0-71)からレボメプロマジンの-0-03(-0-59 to 0-52)まであった(40 815 名参加)。陽性症状の軽減に関するプラセボとの標準化平均差(31 名)は、アミスルプリドで -0-69(95% CrI -0-86~-0-52) 、ブレクスピプラゾールで -0-17(-0-31~-0-04) 、陰性症状(32 名)で -0-62(-0-84~-0-39) と変動していた。から-0-10(-0-45 から 0-25、フルペンチオール)、抑うつ症状(19 683 名)では-0-90(-1-36 から-0-44、スルピリド)から 0-04(-0-39から0-47、フルペンチオール)であった。)プラセボとの比較では、全死亡(42 672名)のリスク比は0-52(0-12~0-95;clopenthixol)~1-15(0-36~1-47;pimozide)、鎮静(30 770名)は0-92(0-17~2-03;pimozide)となっている。から 10-20(4-72 から 29-41、ズクロペンチキソール)、抗パーキンソン薬の使用(24 911 名) から 6-14(4-81 から 6-55、ピモジド)。体重増加(28 名)についてはプラセボとの平均差は-0-16kg(-0-73~0-40;ジプラシドン)~3-21kg(2-10~4-31;ゾテピン)、プロラクチン上昇(21 名)については-77~05ng/ml (-120-23~33-54; クロザピン)~48~59ng/ml (-80-24; ゾルテピネ)であり、プラセボとの差は-0-16kg(-100-100;ゾルテピネ)となっている。から 48-51 ng/mL(43-52 から 53-51;パリペリドン)、QTc 延長(15467 名) から -2-21ms(-4-54 から 0-15;ルラシドン)から 23-90ms(20-56 から 27-33;セルチンドール)でした。)主要アウトカムに関する結論は、考えられる効果調整因子で調整しても、感度分析(例えば、プラセボ対照試験を除外した場合)でも、実質的に変わらなかった。また、エビデンスの信頼度は低いか非常に低いことが多かった。 【解釈】抗精神病薬間の有効性の違いはあるが、そのほとんどは個別的というよりは緩やかである。副作用の差はより顕著である。これらの知見は、臨床医が自国で入手可能な薬剤のリスクとベネフィットのバランスをとるのに役立つであろう。臨床医は、それぞれの結果の重要性、患者の医学的問題、そして嗜好を考慮する必要がある。 第一人者の医師による解説 統合失調症の急性期治療 安全性ファーストの治療が望ましい 三宅 誕実 聖マリアンナ医科大学神経精神科講師 MMJ.February 2020;16(1) 抗精神病薬同士の有効性と安全性を比較する際、 直接比較した無作為化対照試験(RCT)が少ないため、ランキングで比較するネットワークメタ解析 (NMA)の手法は、その限界点を考慮してもなお有益である。本論文は、急性期の成人統合失調症に対する抗精神病薬の有効性と安全性/忍容性を比較した系統的レビューとNMAであり、2013年に行われた同様のNMA(1)のアップデート版である。抗精 神病薬は15から32種類に、RCTは212から402 件に、対象者は43,049から53,463人に、評価 項目は7から17(有効性8、安全性/忍容性9)へ と大幅に増えている(2)。治療抵抗例、初回エピソー ド例、陰性症状や抑うつ優位例、身体合併症などは 除外されている。主要評価項目は全般的症状変化で、 副次評価項目として陽性症状や抑うつへの有効性、 体重増加や錐体外路症状の安全性などを評価して いる。 日本 で 上市されている抗精神病薬に結果を絞ると、全般的症状変化(効果量として標準化平均差 [SMD]:プラセボは0)ではクロザピン(-0.89)、 ゾテピン(-0.61)、オランザピン(-0.56)、リ スペリドン(-0.55)が他の抗精神病薬より優れ ていたが、薬剤差はわずかであった。 一方、安全性においては、体重増加(ゾテピン、オランザピンで 多く、アリピプラゾールで少ないなど)、抗パーキ ンソン病薬の使用やアカシジア(定型抗精神病薬で 多く、クロザピンで少ないなど)、プロラクチン値上昇(パリペリドン、リスペリドンなどで多く、クロザピンで少ないなど)やQTc延長(ブレクスピプラゾールで少ない等)などに関して、統計学的にも有意な薬剤差があった。 副作用に明確な薬剤差 があるのは前回のNMA(1)と同じ結果ではあるが、 shared decision-makingの時代において、このようなランキング比較が治療選択肢に及ぼす影響は 大きい。したがって、統合失調症治療においては、 有効性を重視した抗精神病薬の選択よりも、副作用 を最小限に抑える安全性ファーストの治療アプロー チが重要であることが再確認されたと言える。 1. Leucht S et al. Lancet. 2013 Sep 14;382(9896):951-962. 2. Correll CU et al. JAMA Psychiatry. 2019 Nov 6. [Epub ahead of print]
片頭痛予防薬4種類までが無効であることが証明された患者さんにおける片頭痛予防のためのFremanezumab対プラセボ(FOCUS):無作為化二重盲検プラセボ対照第3b相試験。
片頭痛予防薬4種類までが無効であることが証明された患者さんにおける片頭痛予防のためのFremanezumab対プラセボ(FOCUS):無作為化二重盲検プラセボ対照第3b相試験。
Fremanezumab versus placebo for migraine prevention in patients with documented failure to up to four migraine preventive medication classes (FOCUS): a randomised, double-blind, placebo-controlled, phase 3b trial Lancet 2019 Sep 21 ;394 (10203):1030 −1040. 上記論文のアブストラクト日本語訳 ※ヒポクラ×マイナビ 論文検索(Bibgraph)による機械翻訳です。 【背景】 カルシトニン遺伝子関連ペプチド(CGRP)またはその受容体を標的とする抗体は、片頭痛発作の予防に有効であることが示されている。我々は、2~4種類の片頭痛予防薬が効かない片頭痛患者を対象に、完全ヒト化CGRP抗体であるfremanezumabの有効性と忍容性を検討した。 【方法】無作為化二重盲検プラセボ対照並行群間第3b相FOCUS試験は、ベルギー、チェコ、デンマーク、フィンランド、フランス、ドイツ、イタリア、オランダ、ポーランド、スペイン、スウェーデン、スイス、イギリス、アメリカにわたる104施設(病院、医療センター、研究機関、グループ診療クリニックなど)で行われました。過去10年間に2〜4種類の片頭痛予防薬が無効であったことが記録されている18〜70歳の片頭痛患者を登録しました。失敗の定義は、担当医師の判断により、安定した用量で少なくとも3ヶ月間治療しても臨床的に意味のある改善が見られない場合、治療に耐えられない有害事象のために中止した場合、または患者にとって片頭痛の予防治療に禁忌または適さない治療である場合とされました。参加者は,電子対話型応答技術により,フリーマネマブを四半期ごとに皮下投与(1ヶ月目は675 mg,2,3ヶ月目はプラセボ),フリーマネマブを毎月投与(1ヶ月目は片頭痛のエピソード型と慢性片頭痛の675 mg,2,3ヶ月目は両片頭痛サブグループの225 mg),またはマッチさせたプラセボの毎月投与に12週間ランダムに割り当てられた.主要評価項目は、12週間の治療期間中の月平均片頭痛日数のベースラインからの平均変化とした。本試験はClinicalTrials. govに登録されており、番号NCT03308968、現在終了しています。 【FINDINGS】2017年11月10日から2018年7月6日の間に、エピソード性(329[39%])または慢性(509[61%])片頭痛の患者838人を、プラセボ(n=279)、四半期フレマネズマーブ(n=276)、毎月フレマネズマーブ(n=283)へランダムに割り付けました。12週間における月平均片頭痛日数のベースラインからの減少は、プラセボに対して四半期毎のフリーマネマブ投与で大きく(最小二乗平均[LSM]変化量-0-6[SE 0-3])、LSM変化量はプラセボに対して-3-1[95%CI -3-8~-2-4]; p<0-0001)、月毎のフリーマネマブで大きかった(LSM変化量 -4-1 [0-34]; LSM差 -3-5[-4-2-8];p < 0-0001 )。有害事象は、プラセボとフレマネズマブで同程度であった。重篤な有害事象は,プラセボ投与群では277例中4例(1%),フレマネズマブ投与群では276例中2例(1%未満),フレマネズマブ投与群では285例中4例(1%)で報告された。 【解釈】最大4クラスの片頭痛予防薬が効かない治療困難な片頭痛患者に対してフレマネズマブは有効かつ忍容性が高かった。 【FUNDING】Teva Pharmaceuticals. 第一人者の医師による解説 片頭痛日数半減は30% 難治性患者にはまだ残るunmet needs 柴田 護 慶應義塾大学医学部神経内科准教授 MMJ.February 2020;16(1) カルシトニン遺伝子関連ペプチド(CGRP)は 37個のアミノ酸からなる神経ペプチドで、三叉神経節ニューロンなどに発現し、片頭痛の病態に深く関与している。近年、CGRPおよびCGRP受容体に対するモノクローナル抗体が片頭痛予防薬として開発され、欧米では既に臨床応用されている(1)。 本論文は、欧州・北米104施設において2 ~ 4 種類の片頭痛予防薬を用いても治療が奏効しなかった患者838人を対象に抗 CGRP完全ヒト化モノクローナル抗体であるフレマネズマブ (FRN)の 片頭痛予防における有効性と安全性を評価した多施設共同ランダム化二重盲検プラセボ対照第3b相 (FOCUS)試験の報告である。対象は反復性片頭痛 (月に6日以上かつ15日以下の頭痛があり、そのうち4日以上で片頭痛の特徴を認める)または慢性片 頭痛(月に15日以上の頭痛があり、そのうち8日 以上で片頭痛の特徴を認める)を有し、過去10年間に3カ月以上の片頭痛予防薬投与で臨床的に意味のある改善が得られなかった、または有害事象で治療中止に至ったなど、2 ~ 4種類の予防薬が奏効しなかった患者である。 片頭痛の内訳は慢性片頭痛 61%、反復性片頭痛39%であった。その結果、主 要評価項目である12週間(治療期間)における1カ月あたりの平均片頭痛日数のベースラインからの変化量は、プラセボ群で-0.6日、FRN 675 mg 単回投与群で-3.7(プラセボ群との差 , -3.1日)、 FRN 225 mg毎月投与群で-4.1(プラセボ群との差 , -3.5日)であり、FRN群で有意な治療効果が認められた。副次評価項目である治療期間全体あるいは治療開始後4週目で片頭痛日数が50%以上減少した患者の割合に関してもFRN群でプラセボ 群と比較し有意な効果が確認された。一方、有害事象に関してはプラセボ群とFRN群の間に有意差はなかった。 以上より、既存の予防薬に対して抵抗性を示した患者や有害事象のために治療続行が不可能であった患者においてもFRNは有効かつ忍容性の良好な治療になりうることが示された。ただし、本試験ではFRN両群ともに50%以上の片頭痛日数減少が得られた割合は、プラセボ群に比べ有意に高かったとはいえ、およそ30%にとどまっていた。これは、他の抗 CGRP抗体であるガルカネズマブの臨床試験でもほぼ同等の成績であった(2)。したがって、 抗 CGRP抗体が登場しても、難治性片頭痛患者には“unmet needs”が存在し続けることも示唆されたといえよう。 1. Edvinsson L et al. Nat Rev Neurol. 2018;14(6):338-350. 2. Ru DD, et al. Eur J Neurol. 2019 Nov 6. Doi:10.1111/ene.14114. [Epub]
非心臓手術を受ける患者における周術期のcovert stroke(NeuroVISION):前向きコホート研究。
非心臓手術を受ける患者における周術期のcovert stroke(NeuroVISION):前向きコホート研究。
Perioperative covert stroke in patients undergoing non-cardiac surgery (NeuroVISION): a prospective cohort study Lancet 2019 ;394 (10203):1022 -1029. 上記論文のアブストラクト日本語訳 ※ヒポクラ×マイナビ 論文検索(Bibgraph)による機械翻訳です。 【背景】手術以外の環境では、covert strokeはovert strokeよりも一般的であり、認知機能の低下と関連している。顕性脳卒中は非心臓手術後の成人の1%未満に発生し、かなりの罹患率と関連しているが、周術期の隠性脳卒中についてはほとんどわかっていない。したがって,我々の主な目的は,周術期のcovert stroke(すなわち,臨床的な脳卒中症状を持たない患者の非心臓手術後のMRIで検出された急性脳梗塞)と術後1年の認知機能低下の関係を調べることである。 【方法】NeuroVISIONは9か国12の学術施設で行われた前向きコホート研究で,入院で非心臓手術を受けた65歳以上の患者で,術後に脳のMRIが得られた患者の評価を行うものであった。臨床データをマスクした独立した2人の神経放射線学の専門家が、それぞれのMRIについて急性脳梗塞の有無を評価した。多変量回帰法を用いて,covert stroke と主要転帰である認知機能低下(術前ベースラインから 1 年後のフォローアップまで,モントリオール認知機能評価で 2 ポイント以上の低下と定義)との関連を検討した.患者,医療従事者,アウトカム判定者はMRIの結果をマスキングした。 【FINDINGS】2014年3月24日から2017年7月21日の間に,本研究に募集した1114名のうち,78名(7%;95%CI 6-9)が周術期の隠微な脳卒中を発症していた。1年間の追跡調査を完了した患者のうち,術後1年の認知機能低下は,周術期のcovert strokeがあった69名中29名(42%),周術期のcovert strokeがなかった932名中274名(29%)で生じた(調整オッズ比1-98,95%CI 1-22-3-20,絶対リスク増加13%;p=0-0055)。また,covert strokeは周術期せん妄のリスク上昇(ハザード比[HR]2-24,95%CI 1-06-4-73,絶対リスク上昇6%;p=0-030),1年フォローアップ時のovert strokeまたは一過性虚血発作(HR 4-13,1-14-14-99, 絶対リスク上昇3%;p=0-019)にも関連していた. 【解釈】周術期のcovert strokeは、非心臓手術の1年後の認知機能低下のリスク上昇と関連しており、周術期のcovert strokeは非心臓手術を受けた65歳以上の患者の14人に1人で発生していた。周術期のcovert strokeの予防・管理戦略を確立するための研究が必要である。 【FUNDING】Canadian Institutes of Health Research; The Ontario Strategy for Patient Oriented Research support unit; The Ontario Ministry of Health and Long-Term Care; Health and Medical Research Fund, Government of Hong Kong Special Administrative Region, China; and The Neurological Foundation of New Zealand. 第一人者の医師による解説 周術期の潜在性脳梗塞 原因の特定と病態の解明が喫緊の課題 田中 亮太 自治医科大学附属病院脳卒中センター・センター長 教授 MMJ.February 2020;16(1) 非心臓手術例の周術期における症状の明らかな脳梗塞の発症率は0.1~1%程度と報告されており、その後の機能障害や生命予後に大きく影響する(1)。 一方、症状は明らかでないが頭部 MRI拡散強調画 像などで初めて診断される潜在性脳梗塞は実臨床でしばしば観察され、その後の認知機能低下、認知症発症、精神運動速度の低下、そして新たな脳卒中発症のリスクになる。 全身麻酔を要する手術では、 術後の脳障害や記憶障害などが最も懸念される要 因であることが患者調査でも報告されている(2)。このような背景から、非心臓手術後に診断される潜在性脳梗塞(無症状だが、頭部 MRIで診断される急性期脳梗塞)と術後1年時の認知機能低下の関係を調べる多施設共同前向きコホート研究が実施され、 その結果が本論文で報告された。 アジアを含む9カ国、12施設から65歳以上の定 時非心臓手術症例1,116人が登録され、1,114人 が解析対象とされた。平均年齢は73歳、56%が男 性、術前の既往症は高血圧64%、糖尿病27%、心房細動6%、脳卒中5%などであった。整形外科、泌尿器科、婦人科、一般外科の手術症例が多かった。 MRI検査は術後中央値5日に実施され、そのうち 78人(7%)に潜在性脳梗塞が認められた。 潜在性脳梗塞は術後1年時の認知機能低下のリスクを 1.98倍(95% CI, 1.22~3.20)有意に上昇させ たが、麻酔の種類や施設間の影響はなかった。また潜在性脳梗塞は術後3日以内のせん妄(HR, 2.24; 95% CI, 1.06~4.73)、1年以内の有症候性脳梗 塞または一過性脳虚血発作の発症(HR, 4.13;1.14 ~14.99)に有意に影響した。死亡は潜在性脳梗塞群の8%、非脳梗塞群の5%に認められたが、統計学的有意差はなかった(HR, 1.66;95%CI, 0.71 ~3.88)。 本研究から、高齢者(65歳以上)の定時非心臓手術周術期において14人に1人が潜在性脳梗塞を合併し、1年後の認知機能低下の危険因子になることが示された。潜在性脳梗塞の45%は皮質梗塞、 13%は多発性梗塞であり、卵円孔開存や心房細動などの塞栓性の機序が疑われるが、大多数の脳梗塞の原因は不明である。 一方、脳梗塞の原因として悪性腫瘍に伴う過凝固状態、周術期の発作性心房細 動、大量出血に伴う血栓形成傾向などの影響は不明である。また、認知機能低下に影響する因子として術前からある認知機能低下やうつ状態などについても明らかにされていない。手術を受ける高齢者の増加が予想される中で、非心臓手術後の認知機能 低下の予防や長期的予後改善のためには、潜在性脳梗塞の原因、病態、そして関連する要因の解明が喫緊の課題である。 1. Mashour GA et al. Anesthesiology. 2011 Jun;114(6):1289-1296. 2. Matthey P et al. Can J Anaesth. 2001 Apr;48(4):333-339.
洞調律中の心房細動患者を識別するための人工知能を用いた心電図アルゴリズム:転帰予測のレトロスペクティブな分析。
洞調律中の心房細動患者を識別するための人工知能を用いた心電図アルゴリズム:転帰予測のレトロスペクティブな分析。
An artificial intelligence-enabled ECG algorithm for the identification of patients with atrial fibrillation during sinus rhythm: a retrospective analysis of outcome prediction Lancet 2019 ;394 (10201):861 -867. 上記論文のアブストラクト日本語訳 ※ヒポクラ×マイナビ 論文検索(Bibgraph)による機械翻訳です。 【背景】心房細動は無症状であることが多く、そのため発見が遅れているが、脳卒中、心不全、死亡と関連している。既存のスクリーニング法は長時間のモニタリングを必要とし、コストと低い収率によって制限されている。我々は,機械学習を利用して心房細動患者を迅速かつ安価に特定するポイントオブケア手段を開発することを目的とした。 【方法】我々は,10秒間の標準的な12誘導心電図を用いて正常洞調律中に存在する心房細動の心電図シグネチャを検出するために,畳み込みニューラルネットワークを用いた人工知能(AI)対応の心電計(ECG)を開発した。1993年12月31日から2017年7月21日の間にMayo Clinic ECG研究所で仰臥位で取得したデジタルで正常洞調律の標準10秒12誘導心電図を少なくとも1枚有する18歳以上の患者を対象とし、循環器医の監督下で訓練を受けた担当者によってリズムラベルが検証されたものを使用した。心房細動または心房粗動のリズムを持つ心電図が少なくとも1つある患者を心房細動陽性と分類した。心電図をトレーニング、内部検証、テストデータセットに7:1:2の割合で割り付けた。内部検証データセットの受信者操作特性曲線の曲線下面積(AUC)を算出して確率の閾値を選択し,これをテストデータセットに適用した.AUCと精度,感度,特異度,F1スコアを両側95%CIで算出し,テストデータセットにおけるモデル性能を評価した. 【所見】解析対象として正常洞調律心電図を持つ患者180 922人,649 931件を含む.テストデータセットの3051人(8-4%)の患者は、モデルによってテストされた正常洞調律ECGの前に心房細動が確認された。1回のAI対応ECGで心房細動を識別した場合のAUCは0-87(95% CI 0-86-0-88)、感度79-0%(77-5-80-4)、特異度79-5%(79-0-79-9)、F1スコア39-2%(38-1-40-3)、全体精度79-4%(79-0-79-9)であった.各患者の関心領域(すなわち、研究開始日または最初に記録された心房細動ECGの31日前)の最初の月に取得されたすべてのECGを含めると、AUCは0-90(0-90-0-91)、感度は82-3%(80-9-83-6)、特異度は83-4%(83-0-83-8)、F1スコアは45-4%(44-2-46-5)、全体の精度は83-3%(83-0-83-7)へと上昇しました. 【解説】正常洞調律時に取得されたAI対応心電図は、心房細動患者のポイントオブケアでの同定を可能にする。 第一人者の医師による解説 リスクの根拠は説明できず 現状の深層学習の問題点 津本 周作 島根大学医学部医学科医療情報学講座教授 MMJ.February 2020;16(1) 本論文で著者は、心房細動の発症1カ月前に構造 的変化が起こっているという仮説に基づき、心房細 動が発症した患者の1カ月前の正常洞性リズムから、心房細動が発症するかどうかを予測する人工知能(AI)分類器の性能を調べた。彼らは、これ以前に左室不全のリスクを心電図から推測する分類器を作成、曲線下面積(AUC)0.93, 感度86.3% , 特異度85.7%という好成績を収め、このAI分類器が陽性と診断した患者はオッズ比4.1のリスクを持っていることを示している(1)。 今回の対象は、メイヨークリニックで1993年 12月~2017年7月 に10秒 の12誘導心電図検 査(500Hz)を受けた患者210,414人(心電図数 1,000,000)から、洞性でない患者、データが不 完全な患者、心房細動より前に洞性リズムが観測 されていない患者を除外した180,992人(心電 図数649,931)からなり、これを70%の訓練標本、 10%の検証標本、20%のテスト標本にランダムに 割り付け、深層学習のアルゴリズムに適用した。分 類器はTensorfl owを用いたKerasパッケージで設 計、R3.4.2で解析した(2) (Appendix参照)。 訓練標本(正常例では最初に記録された心電図を indexとし、それ以降のすべての心電図、心房細動 例では心房細動が初めて観測された波形をindex とし、31日前以降のすべての心電図)で学習が行われた。訓練によってできあがった分類器で用いられるパラメータ(確率の閾値など)を最適化するため、検証標本を用いた。ただし、検証標本は、正常例は最初の波形、心房細動の症例は発症31日前以降の最初の洞性リズムのみである。最適なパラ メータを設定後、完成した分類器に対して、テスト標本(こちらも検証標本と同様の波形)のみで、心 房細動が1カ月後に発症するかどうかを判定した。 結果では2つの評価方法が示されている。第1 に、対象 と な る1つ 目 の 心電図 の み で の 評価 で AUC 0.87、感度79.0%、特異度79.5%、正答率79.4%。第2の複数の心電図(洞性のみを選んで)を用いた評価では、AUC 0.90、感度 82.3%、 特異度83.4%、正答率83.3%に改善している。 精度は他の検査、例えば心不全での脳性ナトリウム利尿ペプチドよりも高く、スクリーニングとしては有効ではないかと論じた。 また、メイヨーの別 の研究チームが心電図のさまざまな指標では高い分類精度が得られないことを示していることから、 何らかの心電図上の微妙な変化を深層学習で捉えられたかもしれないと論じている。ただし、今回どうしてこのような精度が得られたかは説明できていない。分類器に波形を与えれば、心房細動のリス クを予想するが、その理由が説明できない。これが 現状の深層学習の方法の問題点でもある。 1. Attia ZI et al. Nat Med. 2019;25(1):70-74. 2. François Chollet、J. J. Allaire(瀬戸山 雅人監訳、長尾 高弘訳). R と Keras によるディープラーニング . オライリージャパン、2018.
血圧降下治療の4つの戦略の適格性とその後の心血管疾患の負担:レトロスペクティブ・コホート研究。
血圧降下治療の4つの戦略の適格性とその後の心血管疾患の負担:レトロスペクティブ・コホート研究。
Eligibility and subsequent burden of cardiovascular disease of four strategies for blood pressure-lowering treatment: a retrospective cohort study Lancet 2019 ;394 (10199):663 -671. 上記論文のアブストラクト日本語訳 ※ヒポクラ×マイナビ 論文検索(Bibgraph)による機械翻訳です。 【背景】血圧を下げるための世界的な治療勧告は,血圧の閾値によって主に導かれ続けているが,血圧を下げることの利点は血圧スペクトル全体にわたる患者に観察されるという強い証拠があるにもかかわらず,このようなことはない。本研究では,英国を例に,血圧治療の代替戦略の意味を検討することを目的とした。 【方法】心血管疾患を持たない30~79歳のプライマリケア患者を対象に,Hospital Episode StatisticsとOffice for National Statistics死亡率にリンクした英国のClinical Practice Research Datalinkからのデータを用いて,レトロスペクティブ・コホート研究を実施した。治療対象者を決定するための4つの異なる戦略を評価・比較した:2011年英国国立医療技術評価機構(NICE)ガイドライン、または2019年NICEガイドライン案、または血圧のみ(閾値≧140/90mmHg)、または10年予測心血管リスクのみ(QRISK2スコア≧10%)、を使用した。患者は、心血管疾患の診断、死亡、またはフォローアップ期間の終了(2016年3月31日)のうち最も早い発生までフォローアップされた。各戦略について、治療対象となる患者の割合と、治療により予防できる心血管イベント数を推定した。次に,英国の一般集団における10年間に発生するであろう適格性とイベント数を推定した。 【FINDINGS】2011年1月1日から2016年3月31日の間に,コホート内の1 222 670例が中央値4-3年(IQR 2-5-5-2)フォローアップされた。2011年のNICEガイドラインでは271 963例(22-2%)、2019年のNICEガイドライン案では327 429例(26-8%)、血圧閾値140/90mmHg以上基準では481 859例(39-4%)、QRISK2閾値10%以上基準では357 840例(29-3%)が治療対象であった。追跡期間中に32 183人の患者が心血管疾患と診断された(全体の割合:1000人年当たり7-1、95%CI:7-0-7-2)。各戦略の対象患者における心血管イベント発生率は、2011年NICEガイドラインでは1000人年当たり15-2(95%CI 15-0-15-5)、2019年NICEガイドライン案では14-9(14-7-15-1)、血圧閾値のみでは11-4(11-3-11-6)、QRISK2閾値のみでは16-9(16-7-17-1)であった。英国人口に換算すると、2011年NICEガイドラインでは233 152イベント(1イベントを回避するために10年間治療する必要がある患者は28人)、2019年NICEガイドラインでは270 233(29人)、血圧閾値を用いた場合は301 523(38人)、QRISK2閾値を用いた場合は322 921(27人)が回避できると推測されました。 【INTERPRETATION】心血管リスクに基づく戦略(QRISK2≧10%)は、2011年NICEガイドラインより3分の1以上、2019年NICEガイドラインより5分の1以上、イベント回避あたりの治療数に関して同様の効率で心血管疾患イベントを予防できる。 【FUNDING】National Institute for Health Research【FUNDING】国立健康研究所。 第一人者の医師による解説 高血圧ポピュレーション戦略において 絶対リスクは血圧値よりも重要な可能性 田中 正巳(特任講師)/伊藤 裕(教授) 慶應義塾大学医学部腎臓内分泌代謝内科 MMJ.February 2020;16(1) 国民の脳心血管病リスクを低下させるためには、 国民全体の血圧を低い方向へシフトさせるポピュ レーション戦略と同時に、高リスク者に積極的に介入する高リスク戦略も重要である。降圧治療が必要な患者を選定する際の指標として、脳心血管病の絶対リスクと血圧値が一般に用いられ、英国の NICEガイドラインは両者を組み合わせている。 本論文は、英国 で 心血管疾患の 既往 が な い30 ~79歳 の 患者1,222,670人 を 中央値4.3年 間追跡した後ろ向きコホート研究の報告である。 2011年 と19年 のNICEガ イ ド ラ イ ン、血圧 の み(140/90 mmHg以上)、10年間の心血管リス ク(QRISK2スコア 10%以上)のみの4つの戦略 が、降圧治療の適格性、降圧治療で回避しうる心血 管イベント数、治療効率性の観点から比較された。 QRISK2スコアは、年齢、性、民族性、貧困度、BMI、 血圧、脂質、糖尿病、慢性腎臓病、心房細動、関節リ ウマチ、冠動脈疾患の家族歴、喫煙などの要素から算出される。治療効率性は、1件の心血管イベン トを防ぐために10年間治療する必要がある患者数(NNT)に基づいて評価した。その結果、英国全 体では2011NICE、2019NICE、血圧のみ、リス クのみの患者選定によって予防できるイベント数 は そ れ ぞ れ233,152、270,233、301,523、 322,921件、NNTは28、29、38、27と推定され た。したがって、絶対リスクのみによる患者選定は 効率が高く、最も多くの心血管イベントを予防で きると考えられた。 本研究は後ろ向き研究であるため、この結果をもって「絶対リスクのみに基づく戦略が、リスクと血圧を組み合わせた戦略よりも優れている」と結論づけるのは時期尚早であろう。絶対リスク評価 の重要性が新たな手法で示された、と解釈するのが現時点では妥当である。   日本 の 高血圧治療 ガ イ ド ラ イ ン 2019で は、 JALSおよび久山町研究に基づくリスクスコアを用いて脳心血管イベントの絶対リスクを算出し、 診察室血圧を組み合わせてリスク層別化を行った。 リスクの高低に応じて血圧再評価や薬物療法の開始時期を変えている。動脈硬化性疾患予防ガイドラ イン 2019では「吹田スコアによる冠動脈疾患発症予測モデル」から予測されたリスクの高低に応 じて、脂質の管理目標値を変えている。このように 個々の患者に合わせた治療が可能になるため、絶対リスクと血圧、脂質値を組み合わせた日本のガイドラインは高リスク戦略の点では優れている。 一方、ポピュレーション戦略の観点からは、絶対リスクのみに基づく戦略によって脳心血管イベントをどれだけ減らせるのか、本研究と同様の研究を日本で検討する必要もある。
晩期早産の子癇症(PHOENIX)に対する計画的な早期分娩または妊婦管理:ランダム化比較試験。
晩期早産の子癇症(PHOENIX)に対する計画的な早期分娩または妊婦管理:ランダム化比較試験。
Planned early delivery or expectant management for late preterm pre-eclampsia (PHOENIX): a randomised controlled trial Lancet 2019 Sep 28 ;394 (10204):1181 -1190. 上記論文のアブストラクト日本語訳 ※ヒポクラ×マイナビ 論文検索(Bibgraph)による機械翻訳です。 【背景】 晩期早産の女性では、母体の疾患進行の制限と乳児の合併症とのバランスをとる必要があるため、分娩開始の最適な時期は不明である。この試験の目的は、計画的に分娩を早期に開始することで、後期早産の女性における妊婦の有害転帰が、新生児または乳児の転帰を実質的に悪化させることなく減少するかどうかを、予想される管理(通常のケア)と比較して明らかにすることであった。 【METHODS】イングランドとウェールズの46の産科ユニットで行われたこの並行群、ノンマスキング、多施設、無作為化比較試験では、妊娠34週から37週未満の後期早産の子癇前症で、単胎妊娠または二卵性双胎妊娠の女性を対象に、計画分娩と妊婦管理(通常のケア)を個別に無作為化して比較した。共起母体転帰は、母体の罹患率または記録された収縮期血圧160mmHg以上の複合値とし、優越性仮説を設定した。共一次周産期転帰は、非劣性仮説(発生率の10%差の非劣性マージン)を用いて、周産期死亡または新生児退院までの新生児ユニット入院の複合体とした。解析は治療意図別に行われ、周産期アウトカムのプロトコル別解析も併せて行われた。この試験はISRCTN登録(ISRCTN01879376)にプロスペクティブに登録された。本試験は募集を終了しているが、フォローアップは継続中である 【FINDINGS】2014年9月29日から2018年12月10日までの間に、901名の女性が募集された。450人の女性(分析対象は448人の女性と471人の乳児)が計画分娩に、451人の女性(分析対象は451人の女性と475人の乳児)が期待管理に割り付けられた。計画分娩群(289人[65%]女性)と妊婦管理群(338人[75%]女性;調整後相対リスク0-86、95%CI 0-79-0-94、p=0-0005)と比較して、共起母体転帰の発生率は有意に低かった。治療意図別の共一次周産期転帰の発生率は,計画分娩群(196[42%]の乳児)が予想管理群(159[34%]の乳児;1-26,1-08-1-47;p=0-0034)と比較して有意に高かった。プロトコールごとの解析結果も同様であった。重篤な有害事象は計画分娩群で9件、妊婦管理群で12件であった。 【解説】計画分娩は妊婦管理に比べて母体の罹患率と重度の高血圧症を減少させることを示唆する強いエビデンスがあり、未熟児に関連した新生児ユニットの入院は多いが、新生児の罹患率の増加を示す指標はない。このトレードオフは、分娩のタイミングについての意思決定を共有できるように、後期早産前子癇症の女性と議論されるべきである。 第一人者の医師による解説 新生児には早産分娩リスク 妊娠高血圧腎症発症の際はすみやかな母体搬送を 田中 守 慶應義塾大学医学部産婦人科(産科)教授 MMJ.February 2020;16(1) 妊娠高血圧腎症は、主として妊娠後期に発症し、 高血圧、蛋白尿などの全身性の障害をもたらし、母 体脳出血、腎障害、肝障害のみならず胎児発育遅延 など母児ともに重篤な後遺症をもたらす疾患である。全妊婦のおよそ10%が妊娠高血圧を発症し、さらに2~3%が妊娠高血圧腎症を発症するとされている。また、その原因は不明であるため、根本的な治療法はなく、分娩が最大の治療となる。妊娠 37週以降は、直ちに分娩することが母児の安全の ために推奨されているが、妊娠34週から37週未満の後期早産期での分娩の効果については明らか とされていなかった。 本試験は、後期早産期の妊娠高血圧腎症に対して 経過観察または積極的な人工早産のどちらが母児の予後に良いのかを明らかにするために実施された無作為化対照試験(RCT)である。妊婦4,498人 を対象として検討し、1,606人が適格条件に適合した。このうち901人が計画に同意し、450人が人工早産群、451人が経過観察群に無作為に割り付けられ、母体および新生児予後が評価された。 結果では、経過観察群では人工早産群に比べて5 日間の妊娠期間の延長が認められた。人工早産群において統計学的有意差を持って母体予後が良好で あり、一方、人工早産群においては有意に児の周産期合併症が増加した。また、母児にかかる医療費(英国)は、明らかに人工早産群で少なかった。妊娠34 週から37週未満の後期早産期の妊娠高血圧腎症においては、積極的に分娩を図ることによって母体 予後は明らかに改善するが、新生児に対しては明 らかに早産分娩のリスクが上昇するため、そのメ リット、デメリットを常に考えて分娩時期を決定する必要がある。 本試験は、早産後期に発症した妊娠高血圧腎症患者に対して、積極的に早産分娩とする医学的な根拠 を検討するために行われた英国における多施設共同 RCTである。日本でもLate Preterm Delivery の問題は議論されているものの、少なくとも新生児集中治療室(NICU)を有している周産期施設で は積極的な早産分娩を議論するべきである。したがって、産科診療の1次施設では、妊娠高血圧腎症 と診断されたら直ちにNICUを備えた周産期センターに母体搬送すべきであり、受け入れた周産期センターでは積極的分娩を含めた周産期管理を検討すべきであるというエビデンスが明らかにされたと言えよう。
心臓手術を受けた患者における尿中dickkopf-3、急性腎障害、およびその後の腎機能喪失の関連性:観察的コホート研究
心臓手術を受けた患者における尿中dickkopf-3、急性腎障害、およびその後の腎機能喪失の関連性:観察的コホート研究
Association between urinary dickkopf-3, acute kidney injury, and subsequent loss of kidney function in patients undergoing cardiac surgery: an observational cohort study Lancet 2019 ;394 (10197):488 -496 . 上記論文のアブストラクト日本語訳 ※ヒポクラ×マイナビ 論文検索(Bibgraph)による機械翻訳です。 【背景】心臓手術は、術後の急性腎障害(AKI)とその後の腎機能低下の高いリスクと関連している。我々は、AKIとその後の腎機能低下のリスクのある患者の術前識別のための腎尿細管ストレスマーカーである尿中dickkopf-3(DKK3)の臨床的有用性を検討した。 【方法】この観察的コホート研究では、派生コホートの心臓手術を受けた患者と検証コホートの心臓手術を受けた患者(RenalRIP試験)を対象とした。本研究は,ザールランド大学医療センター(ドイツ・ホンブルグ,派生コホート)で待機的心臓手術を受けた連続患者と,前向きRenalRIP多施設試験(検証コホート)に登録された待機的心臓手術を受けた患者(Cleveland Clinical Foundationスコア6以上に基づいて選択),および遠隔虚血性前処置または偽手術にランダムに割り付けられた患者で構成されている。術前のDKK3とクレアチニンの尿中濃度の比(DKK3:クレアチニン)と、Kidney Disease Improving Global Outcomes基準に従って定義された術後AKI、および推定糸球体濾過量によって決定されるその後の腎機能低下との関連性を評価した。 【結果】派生コホートの733例において、クレアチニンに対するDKK3の尿中濃度が471pg/mgより高いことは、ベースラインの腎機能とは無関係に、AKIの有意なリスク上昇と関連していた(オッズ比[OR]1-65、95%CI 1-10-2-47、p=0-015)。臨床検査値や他の検査値と比較して、尿中DKK3:クレアチニン濃度はAKI予測を有意に改善した(net reclassification improvement 0-32, 95% CI 0-23-0-42, p<0-0001)。尿中DKK3:クレアチニン濃度の高値は、退院時および中央値820日(IQR733~910)の追跡調査後の腎機能の有意な低下と独立して関連していた。RenalRIP試験において、術前の尿中DKK3:471pg/mgより高いクレアチニン濃度は、471pg/mg以下のDKK3と比較して、90日後のAKI(OR 1-94, 95% CI 1-08-3-47, p=0-026)、持続性腎機能障害(OR 6-67, 1-67-26-61, p=0-0072 )、透析依存(OR 13-57, 1-50-122-77, p=0-020)に対するリスクが有意に高くなることと関連しました。471pg/mgより高い尿中DKK3:クレアチニン濃度は、偽手術を受けた患者のみAKI(OR 2-79, 95% CI 1-45-5-37)および持続的腎機能障害(OR 3-82, 1-32-11-05)の有意に高いリスクと関連しており、遠隔虚血性前処理(AKI OR 1-35, 0-76-2-39 and persistent renal dysfunction OR 1-05, 0-12-9-45)には関連していないことが明らかになった。 【解釈】術前の尿中DKK3は、術後AKIとその後の腎機能低下の独立した予測因子である。尿中DKK3は予防的治療戦略が有効な患者を特定するのに役立つかもしれない。 第一人者の医師による解説 ヨーロッパ系人種対象の研究だが 尿中 DKK3はAKI予防に有用なツール 新田 孝作 東京女子医科大学腎臓内科教授 MMJ.February 2020;16(1) 心臓手術後の急性腎障害(AKI)発症率は30%弱と報告されている(1)。近年、心臓手術患者の高齢化と併存疾患割合の上昇に伴い、AKI発症率も上昇しつつある。一部の患者では、AKI後も腎機能の低下が認められ、AKIから慢性腎臓病(CKD)への移行 (AKI-CKD transition)として知られている。 今回報告されたドイツの観察コホート研究では、 ストレス下の腎尿細管細胞で発現されるWnt/β - カテニンシグナル調節糖蛋白 dickkopf-3(DKK3) に 着目し、心臓手術患者 に お け るAKI発症 とAKI 後の腎機能低下の予測因子として尿中 DKK3を検討した。対象は導出コホート 733人、検証コホー ト 216人(RenalRIP試験参加者)で、導出コホー トでは待機的心臓手術を受けた患者を退院後に中 央値820日間追跡 した。 そ の 結果、AKIは 導出 コ ホートの患者733人中193人(26%)に発症した。術前の尿中 DKK3高値は術後 AKI発症と有意に関連していた。この関連は、登録時の推算糸球 体濾過量(eGFR)などの交絡因子で補正後も維持 され、機械学習アプローチにより裏付けられた。術前の尿中 DKK3値が471pg/mg Cr超の患者では、471pg/mg Cr以下 の 患者 よりもAKI発症リ スクが有意に高かった。完全補正後のモデルでは、 登録時 のeGFR値 が 正常(90mL/分 /1.73m2 超)または低値(90mL/分 /1.73m2以下)の患 者 に お い て、尿中 DKK3高値 はAKI発症 リ ス ク 上昇と関連していた。 また、術前の尿中 DKK3値 が471pg/mg Cr以下 の 患者 に お け る 退院時 の eGFR値 は76.6mL/分 /1.73m2で あ っ た。対 照的 に、術前 の 尿中 DKK3値 が471pg/mg Cr 超 の 患者 で は 補正後 のeGFRが 有意 に 低 かった (72.3mL/分 /1.73m2)。中央値820日 の 追跡 後、登録時の尿中 DKK3値が471pg/mg Cr以下 の患者では、eGFRは67.0mL/分 /1.73m2であったが、尿中 DKK3値が471pg/mg Cr超の患者では、eGFRは63.1mL/分 /1.73m2であった。術前 の尿中 DKK3値が高いと、AKI後の中等度・重度の eGFR低下およびCKDに進行するリスクが高まっていた。 対象集団がヨーロッパ系人種であり、他の人種に適用できるかどうかは今後確認する必要はあるが、 今回の結果から、尿中 DKK3は心臓手術後にAKI を発症するリスクの高い患者を特定し、特に入院中のAKI発症を予測するのみならず、その後の腎機能低下も予測することが示された。尿中DKK3 は予防が有用な患者を特定するための新しいツー ルとなり得ると言える。 1. Neugarten J. et al. Clin J Am Soc Nephrol. 2016; 11(12): 2113–2122.
未熟児網膜症を有する超低出生体重児の治療に対するラニビズマブ対レーザー治療(RAINBOW):非盲検ランダム化比較試験。
未熟児網膜症を有する超低出生体重児の治療に対するラニビズマブ対レーザー治療(RAINBOW):非盲検ランダム化比較試験。
Ranibizumab versus laser therapy for the treatment of very low birthweight infants with retinopathy of prematurity (RAINBOW): an open-label randomised controlled trial Lancet 2019 ;394 (10208 ):1551 -1559 . 上記論文のアブストラクト日本語訳 ※ヒポクラ×マイナビ 論文検索(Bibgraph)による機械翻訳です。 【背景】未熟児網膜症(ROP)治療における抗血管内皮増殖因子製剤の使用は世界的に増加しているが、その眼に対する効果、適切な薬剤と用量、再投与の必要性、長期の全身作用の可能性に関するデータはほとんどない。ROPの治療における硝子体内ラニビズマブの有効性と安全性をレーザー治療と比較して評価した 【方法】この無作為化、非盲検、優越性多施設、3群並行群間試験は26カ国の87新生児センターと眼科センターで行われた。網膜症の治療基準を満たした出生体重1500g未満の乳児をスクリーニングし、ラニビズマブ0-2mgまたはラニビズマブ0-1mgの両側静脈内単回投与、またはレーザー治療を受けるよう平等に(1:1:1)患者を無作為に割り付けました。コンピュータ対話型応答技術により,疾患領域と地理的地域によって層別化された.主要評価項目は,24 週間以内に活動性網膜症,好ましくない構造的転帰,または別の治療手段を必要としない生存率とした(レーザー療法に対するラニビズマブ 0-2 mg の優位性は両側 α=0-05 であった).解析は intention to treat で行った。本試験はClinicalTrials. gov、NCT02375971に登録されています。 【解釈】2015年12月31日から2017年6月29日の間に、225名の参加者(ラニビズマブ0-2mg n=74、ラニビズマブ0-1mg n=77、レーザー療法 n=74)を無作為に割り付けました。治療前に7名が離脱し(それぞれn=1、n=1、n=5)、各群4名の死亡を含む17名が24週までのフォローアップを完了しなかった。214人の乳児が主要アウトカムについて評価された(それぞれn=70, n=76, n=68)。治療成功は,ラニビズマブ 0-2 mg 投与群では 70 例中 56 例(80%)であったのに対し,ラニビズマブ 0-1 mg 投与群では 76 例中 57 例(75%),レーザー治療後では 68 例中 45 例(66%)で発生した.階層的検定戦略により、レーザー治療と比較して、ラニビズマブ0-2 mg投与後の治療成功のオッズ比(OR)は2-19(95% Cl 0-99-4-82、p=0-051)、ラニビズマブ0-1 mg投与後は1-57(95% Cl 0-76-3-26)、0-1 mgと比較して、ラニビズマブ 0-2 mgでは1-35(95% Cl 0-61-2-98)であった。ラニビズマブ0-2 mg投与群では構造的に好ましくない転帰をたどった乳児が1人いたが,ラニビズマブ0-1 mg投与群では5人,レーザー療法では7人であった.死亡、重篤および非重篤な全身性有害事象、および眼の有害事象は3群間で均等に分布した。 ROPの治療において、ラニビズマブ0-2mgはレーザー治療より優れており、レーザー治療より眼の有害事象が少なく、24週間の安全プロファイルが許容できるかもしれない。 第一人者の医師による解説 適切な適応の検討が必要だが 未熟児網膜症に新たな治療選択肢 東 範行 国立成育医療研究センター眼科診療部長・視覚科学研究室長 MMJ.February 2020;16(1) 未熟児網膜症は失明につながる疾患で、日本における発症数は年間およそ4,300人と推定されている。治療としては、これまでに網膜凝固(冷凍凝固、光凝固 )、硝子体手術、抗血管内皮増殖因子(VEGF)療法が行われてきた。本論文で報告されたRAINBOW試験は、未熟児網膜症に対する抗 VEGF抗体薬ラニビズマブ(RBZ)硝子体内注射の有効性と安全性をレーザー光凝固療法との比較で評価した初めての国際共同治験である。RBZはすでに成人における加齢黄斑変性や糖尿病黄斑浮腫、 網膜静脈変性など眼底の病的血管新生疾患に使用されている。 対象は出生体重1,500g未満の両眼性の治療を要する未熟児網膜症で、225人が登録された。参加国は26カ国で登録数は日本が最多であった。患者はRBZ 0.2mg(成人用量の40%)群、RBZ 0.1mg 群、またはレーザー光凝固群にランダム化され、初回治療が両眼同時に行われた。初回治療後に網膜症が悪化した場合は、治療後28日以降であれば同じ治療の追加が各眼で2回まで許されるが、28日未満であればレスキューとして別の治療が行われた。 主要評価項目は、治療の成功であり、治療開始24 週後 , 両眼とも活動性の未熟児網膜症がなく、網膜の牽引や剥離など不良な形態学的転帰もないことと定義された。各群ともほぼ90%以上の患者で試験が完了した。 結果は、治療成功率がRBZ 0.2mg群で80%、 RBZ 0.1mg群で75%、レーザー群で66%であったが、有意差はなかった。しかしRBZ 0.2mg群 で治療後20週に脱落した1人の結果を加えると、 RBZ 0.2mg群とレーザー群の間に有意差が認められた。0.1 mg群では有意差がなく、0.2mgの使用が推奨された。治療成功率において、患者の性、人種、年齢(出生週数、治療週数)、未熟児網膜症の病期分布による差はなかった。 眼合併症は結膜出血、網膜出血などで、成人の硝 子体注射でみられるものと差はなく、いずれも軽微であった。全身合併症は、未熟児特有の全身の問 題が中心で、治療によって惹起されたと思われる ものはなかった。VEGFは発生や成長に関連するため、眼内投与のRBZが血中に回って全身性に影響することが危惧されたが、RBZ投与直後以降の血中 VEGF値に低下はみられなかった。 本試験では、その後5年間にわたって眼底の変化や視力、全身への影響が追跡されるが、今回の結果を踏まえて、日本でも未熟児網膜症におけるRBZ の使用が最近承認された。RBZに関しては血管増 殖組織の収縮や網膜症の再燃などの問題も報告されており、適切な適応についてはさらなる検討を要するが、未熟児網膜症は新たな治療選択肢を得たことになる。
自然災害後の経済的弱者における社会サービスの改善と心的外傷後ストレス障害の負担:モデル化研究.
自然災害後の経済的弱者における社会サービスの改善と心的外傷後ストレス障害の負担:モデル化研究.
Improved social services and the burden of post-traumatic stress disorder among economically vulnerable people after a natural disaster: a modelling study Lancet Planet Health 2019 ;3: e93 –101 上記論文のアブストラクト日本語訳 ※ヒポクラ×マイナビ 論文検索(Bibgraph)による機械翻訳です。 【背景】ハリケーンなどの自然災害は、公衆衛生や経済的な影響をもたらし、その直後よりもずっと長く続く。資源の喪失は、大規模な心的外傷イベント後の心的外傷後ストレス障害(PTSD)の中核的な推進力である。我々は、災害後の文脈における住宅および住宅関連財源の回復がPTSDからの回復に及ぼす効果を調べた。 【方法】我々は、有病率と持続性によって測定されるPTSD回復に対する異なる医療サービスアプローチの効果を検証するために、観察および実験データで経験化したエージェントベースモデルを構築した。心理的ファーストエイドに類似した社会サービスケースマネジメント(SSCM)アプローチを検証し、避難所ベースの社会サービス提供と、避難し所得を失った人々へのメンタルヘルス治療への連携を特徴とし、通常のケアのみ、SSCM付き通常のケア、ステップケアのみ、SSCM付きステップケアの治療効果を比較した。 【結果】大規模自然災害後に住居を失い、所得を失った人々に対して、住居を回復し、メンタルヘルスサービスへの連携を提供するSSCMアプローチは、1年目終了時に、SSCMを行わない状態に比べて1-56(95%CI 1-55-1-57)倍から5-73(5-04-6-91)倍の寛解PTSD症例をもたらし、2年目終了時には1-16(1-16-1-17)倍から2-28(2-25-2-32)倍の寛解症例をもたらすことが示された。 【解釈】自然災害の影響を受けた集団に経済的・住宅的資源を回復させることは、災害後の集団、特に資源喪失者の精神衛生負担を著しく減少させるであろう。 【資金】米国保健社会福祉省. 第一人者の医師による解説 災害後のPTSDの予防と回復 心理療法・精神療法だけでなく住居確保の支援が重要 岩井 圭司 兵庫教育大学大学院人間発達教育専攻教授 MMJ.April 2020;16(2) 人間は生きていくうえで常にさまざまな要求 (demanding)に対応しなければならない。資源維持(Conservation of Resources;COR)理論(1)によると、我々はふだん多様な資源を駆使してそれらに応じているが、そういった資源が減少ないし喪失すると要求に応えることができなくなり、ストレスが生じる。さしずめ自然災害の被災者における住居の喪失と収入の減少は、資源喪失の最たるものであろう。 本研究は、社会福祉的ケースマネジメント(social services care management;SSCM)が、米国のハリケーン災害被災者の精神健康に及ぼす効果を検討したものである。なお、ケースマネジメントとは、生活に困難さを抱える人々の支援にあたり、さまざまな資源を組み合わせ結びつけて提供することであり、ケアマネジメントやケースコーディネーションとほぼ同義である。ここでは、恒久的住居への帰住の援助と精神保健機関に直接接続することを目指して、SSCMが実施された。また、被災者の精神健康の指標としては、心的外傷後ストレス障害(PTSD)の発生率と持続期間が用いられた。 まず、ハリケーン・サンディ(2012年)によって住居をなくし減収を余儀なくされた被災者は、「通常ケア群」と「段階的ケア群」に分けられた。前者にはサイコロジカル・リカバリースキル(skills for psychological recovery;SPR)(2)が適用され、 後者はスクリーニングによってさらに非 PTSDと PTSDに分けられ、それぞれSPRと認知行動療法が施行された。 そこにSSCMを適用したところ、「通常ケア群」においても「段階的ケア群」においてもPTSDの発生率は低くなり、PTSDを発症したケースでも罹病期間が短くなることが明らかになった。SSCMを適用した場合のPTSD の寛解率は、しなかった場合に比べて1年後では1.56 ~ 5.73倍、2年後では1.16 ~ 2.28倍であった。 以上から、住居資源ないし経済的資源の回復は、自然災害の被災者、特に災害によって資源を喪失した者の精神健康上の負担を有意に軽減すると考えられた。 災害後の被災者に対する心のケアは、狭義の心 理療法・精神療法に限ることなく、住居確保などの経済的支援を織り込んでいくことが必要であり有効であると、本研究は実証したといえる。 1. Hobfoll SE. Am Psychol. 1989;44(3):513-524. 2. 兵庫県こころのケアセンター.http://www.j-hits.org/spr/
症候性重症大動脈弁狭窄症患者に対する経カテーテル大動脈弁置換術における自己拡張型人工弁とバルーン拡張型人工弁の安全性と有効性:無作為化非劣性試験。
症候性重症大動脈弁狭窄症患者に対する経カテーテル大動脈弁置換術における自己拡張型人工弁とバルーン拡張型人工弁の安全性と有効性:無作為化非劣性試験。
Safety and efficacy of a self-expanding versus a balloon-expandable bioprosthesis for transcatheter aortic valve replacement in patients with symptomatic severe aortic stenosis: a randomised non-inferiority trial Lancet 2019 ;394 (10209):1619 -1628. 上記論文のアブストラクト日本語訳 ※ヒポクラ×マイナビ 論文検索(Bibgraph)による機械翻訳です。 【背景】経カテーテル大動脈弁置換術(TAVR)は、症候性重症大動脈弁狭窄症の高齢患者に対する好ましい治療選択肢である。利用可能なTAVRシステムの特性の違いは、臨床転帰に影響を与える可能性がある。TAVRを受ける患者において,自己拡張型ACURATE neo TAVRシステムとバルーン拡張型SAPIEN 3 TAVRシステムを,初期の安全性と有効性について比較した。 【方法】この無作為化非劣性試験において,症候性の高度大動脈狭窄症の治療で経大腿TAVRを受けており,手術リスクが高いと考えられる患者(75歳以上)をドイツ,オランダ,スイス,イギリスの20の三次心臓弁センターで募集した。参加者は、コンピュータベースの無作為並べ替えブロック方式により、ACURATE neoまたはSAPIEN 3による治療を受ける群に1対1で無作為に割り付けられ、試験施設と胸部外科学会予測死亡リスク(STS-PROM)カテゴリーにより層別化された。安全性と有効性の主要複合エンドポイントは、全死亡、あらゆる脳卒中、生命を脅かすまたは障害をもたらす出血、主要血管合併症、治療を必要とする冠動脈閉塞、急性腎障害(ステージ2または3)、弁関連症状または鬱血性心不全による再入院、再手術を必要とする弁関連機能不全、中程度または重度の人工弁逆流、手術後30日以内の人工弁狭窄で構成されました。エンドポイント評価者は治療割り付けに対してマスクされていた。ACURATE neo の SAPIEN 3 に対する非劣性は intention-to-treat 集団において、主要複合エンドポイントのリスク差マージンを 7-7% とし、片側 α を 0-05 とすることで評価されました。本試験はClinicalTrials. govに登録されており(番号NCT03011346)、継続中ですが募集はしていません。 【所見】2017年2月8日から2019年2月2日までに、最大5132人の患者をスクリーニングし、739人(平均年齢82-8歳[SD 4-1]、STS-PROMスコア中央値3-5%[IQR 2-6-5-0])が登録されました。ACURATE neo群に割り付けられた372例中367例(99%)、SAPIEN 3群に割り付けられた367例中364例(99%)で30日の追跡調査が可能であった。30日以内に主要評価項目はACURATE neo群87例(24%)、SAPIEN 3群60例(16%)で発生し、ACURATE neoの非劣性は満たされなかった(絶対リスク差7-1%[95%信頼限界上12-0%]、P=0-42)。主要評価項目の二次解析では、SAPIEN 3デバイスのACURATE neoデバイスに対する優越性が示唆された(リスク差の95%CI:-1-3~-12-9、p=0-0156)。ACURATE neo群とSAPIEN 3群では,全死亡(9例[2%] vs 3例[1%]),脳卒中(7例[2%] vs 11例[3%])の発生率は変わらなかったが,急性腎障害(11例[3%] vs 3例[1%]),中度または重度の人工大動脈逆流(34例[9%] vs 10例[3%])はACURATE neo群に多く見られた.【解説】自己拡張型ACURATE neoを用いたTAVRは、バルーン拡張型SAPIEN 3デバイスと比較して、初期の安全性と臨床効果のアウトカムにおいて非劣性を満たさないことが示された。早期の安全性と有効性の複合エンドポイントは、異なるTAVRシステムの性能を識別するのに有用であった。 【FUNDING】Boston Scientific(アメリカ)。 第一人者の医師による解説 SAPIEN 3の安定性とACURATE neoの課題が明確に 小山 裕 岐阜ハートセンター心臓血管外科部長 MMJ.April 2020;16(2) 経カテーテル的大動脈弁置換術(TAVR)は、大動脈弁狭窄症に対する治療として、日本でも大きな役割を果たしている。バルーン拡張型弁である SAPIENは、外科手術不能・高リスク患者を無作為化したPARTNER Ⅰ試験から、その改良とともに手術低リスク患者に適応拡大したPARTNER III試験により外科手術に対する優位性が示されるまでになった。バルーン拡張型弁と自己拡張型弁の比較に関しては、バルーン拡張型弁の方が良好なデバ イス成功率を示したCHOICE試験、複合エンドポイントで同等な早期成績を示したSOLVE-TAVI試験がある。 本研究(SCOPE Ⅰ試験)は、欧州4カ国20施設 で75歳以上の手術リスクのある症候性大動脈弁狭 窄症患者739人( 平均年齢82.8歳、STS-PROM スコア中央値3.5%)を経大腿動脈アプローチによるTAVRにおいて、新しい自己拡張型弁である ACURATE neo(日本未承認)群とバルーン拡張型弁であるSAPIEN 3群に無作為化し、早期安全性と臨床的有効性の非劣性を検証した。 治療目標比較 (intention to treat)において、ACURATE neo群 はSAPIEN 3群と比較し、1次安全性・有効性複合 エンドポイント(全死亡、脳卒中、重篤な出血、血管 合併症、治療を要する冠動脈閉塞、急性腎障害など)で非劣性を示せなかった(エンドポイント発生率: 24% 対 16%;Pnoninferiority=0.42)。また2次解 析で、急性腎障害、弁機能不全においてSAPIEN 3 の優位性が示唆された(Psuperiority=0.0156)。心臓超音波評価 では、ACURATE neo群 はSAPIEN 3群と比較し、中等度以上の弁周囲逆流が多かったが(9.4% 対 2.8%;P<0.0001)、弁平均圧 較差は低く(中央値7 mmHg 対 11 mmHg;P< 0.0001)、有効弁口面積は大きかった(中央値1.73 cm2 対 1.47 cm2:P<0.0001)。両群ともに全死亡、脳卒中、新規ペースメーカー植え込みの頻度は低く、良好な成績であった。 本研究では、日本未導入のACURATE neoが SAPIEN 3に対する非劣性を示せず、SAPIEN 3の安定した成績が示された一方で、ACURATE neo の課題も明らかになった。急性腎障害の発生は造影剤使用量や手技時間の影響を受けると考えられることから、手技や症例選択による改善の余地があり、 弁周囲逆流もデバイスの改良で軽減されうる。今回の結果では自己拡張型弁の方がより大きい有効弁口面積を得られることが示されており、体格の小さい日本人や狭小弁輪には、さらに改良された ACURATE neoの導入が期待される
集団ベースの心血管系リスク層別化に対する非HDLコレステロールの適用:Multinational Cardiovascular Risk Consortiumの結果。
集団ベースの心血管系リスク層別化に対する非HDLコレステロールの適用:Multinational Cardiovascular Risk Consortiumの結果。
Application of non-HDL cholesterol for population-based cardiovascular risk stratification: results from the Multinational Cardiovascular Risk Consortium Lancet 2019 ;394 (10215):2173 -2183. 上記論文のアブストラクト日本語訳 ※ヒポクラ×マイナビ 論文検索(Bibgraph)による機械翻訳です。 【背景】血中脂質濃度と心血管疾患の長期発症との関連性,および脂質低下療法と心血管疾患の転帰との関連性は不明である。我々は,血中非HDLコレステロール濃度の全領域に関連する心血管疾患リスクについて調査した。また,non-HDLコレステロールに関連する心血管疾患イベントの長期確率を推定する使いやすいツールを作成し,脂質低下治療によるリスク低減をモデル化した。 【方法】このリスク評価およびリスクモデル化研究では,欧州,オーストラリア,北米の19か国から得たMultinational Cardiovascular Risk Consortiumのデータを使用した。ベースライン時に心血管疾患の有病率がなく,心血管疾患の転帰に関する確実なデータが利用可能な個人を対象とした。動脈硬化性心血管病の主要複合エンドポイントは、冠動脈性心疾患イベントまたは虚血性脳卒中の発生と定義された。欧州ガイドラインの閾値に従った非HDLコレステロールのカテゴリーを用いて、年齢、性別、コホート、古典的な修正可能な心血管危険因子で調整した性特異的多変量解析が計算された。導出と検証のデザインにおいて,年齢,性,危険因子に依存する75歳までに心血管疾患イベントが発生する確率と,非HDLコレステロールが50%減少すると仮定した場合の関連するモデルリスク減少を推定するツールを作成した。 【調査結果】コンソーシアムのデータベースにおける44コホートの524 444人のうち,38コホートに属する398 846人(184 055 [48-7%] 女性;年齢の中央値 51-0 歳 [IQR 40-7-59-7] )が同定された。派生コホートには199 415人(女性91 786人[48-4%])、検証コホートには199 431人(女性92 269人[49-1%])が含まれた。最大43~6年の追跡期間(中央値13~5年,IQR7~0~20~1)において,54 542件の心血管エンドポイントが発生した.発生率曲線解析では、非HDLコレステロールのカテゴリーが増えるにつれて、30年間の心血管疾患イベント率が徐々に高くなることが示された(女性では非HDLコレステロール<2〜6mmol/Lの7〜7%から≧5〜7mmol/Lの33〜7%、男性では12〜8%から43〜6%、p<0〜0001)。非HDLコレステロールが2-6 mmol/L未満を基準とした多変量調整Coxモデルでは、男女ともに非HDLコレステロール濃度と心血管疾患の関連性が増加した(非HDLコレステロール2-6~<3-7 mmol/Lのハザード比1-1、95% CI 1-0-1-3から、女性では5-7 mmol/L 以上の1-9、 1-6-2-2 、男性では 1-1, 1-0-1-3 から 2-3, 2-0-2-5 )。このツールにより,non-HDLコレステロールに特異的な心血管疾患イベント確率の推定が可能となり,滑らかなキャリブレーション曲線解析と心血管疾患推定確率の二乗平均誤差が1%未満であることから,派生集団と検証集団の間の高い比較可能性が反映されていることが示された.非HDLコレステロール濃度の50%低下は、75歳までの心血管疾患イベントのリスク低下と関連し、このリスク低下は、コレステロール濃度の低下が早ければ早いほど大きくなった。我々は、個人の長期的なリスク評価と、早期の脂質低下介入の潜在的な利益のための簡単なツールを提供する。これらのデータは,一次予防戦略に関する医師と患者のコミュニケーションに有用であると考えられる。 【FUNDING】EU Framework Programme,UK Medical Research Council,German Centre for Cardiovascular Research. 第一人者の医師による解説 臨床医として高リスクの患者の素早い把握につながる指標を期待 山田 悟 北里大学北里研究所病院糖尿病センター長 MMJ.April 2020;16(2) 日本の「動脈硬化性疾患予防ガイドライン 2017 年版」では、LDLコレステロール(LDL-C)および中性脂肪(TG)が高いほど、またHDLコレステロール(HDL-C)が低いほど冠動脈疾患の発症頻度は高いとされ、non-HDL-Cについては、「食後採血の場合やTGが400mg/dL以上の時にはFriedwald式でLDL-Cを求めることができないため、LDL-Cの 代用として用いる」という扱いである。 一方、欧米のガイドラインでは、LDL-Cのほかに non-HDL-C測定がすでに推奨されていると著者らは本論文の緒言で述べている。本文では、そこまで明確にnon-HDL-CとLDL-Cの心血管リスクとして の意義を比較していないが、論文 appendixには両者の心血管疾患に対するハザード比が記載されている。全体ではほとんど差はないが、45歳未満で は若干 non-HDL-Cの方が優れているようであった(男性ハザード比:non-HDL-C ①100mg/dL未 満;1基準、②100~145mg/dL;1.4、③145 ~185mg/dL;1.9、④185~220mg/dL;3.0、 ⑤220mg/dL以上;4.2:LDL-C ①70mg/dL未 満;1基準、②70~115mg/dL;1.1、③115~ 155mg/dL;1.5、④155~190mg/dL;2.3、 ⑤190mg/dL以上;3.6)。 既存 の 研究も、冠動脈疾患 の予測因子 と し て LDL-Cよりnon-HDL-Cの方が優れる(1)。しかし、この既報が1番良い予測因子として推しているのはApoB(HDL以外のすべてのリポタンパク質に包含されるアポ蛋白)である。実際に欧州心臓病学会(ESC)/欧州動脈硬化学会(EAS)ガイドライン 2019をみると、やはりnon-HDL-CはLDL-Cの 代替とされている(2)。さらに、ガイドライン 2016 年版との新旧比較表は、ApoBもLDL-Cの代替として測定可能で、高 TG、糖尿病、肥満などの人では、non-HDL-Cより優先されるかもしれないと記載されている(2)。よって、今後、すぐにnon-HDL-Cが LDL-Cに取って代わるということはなかろう。 ただ、30年間の脂質異常症 による動脈硬化症の影響は10年間の脂質異常症の3倍ではないことから(3)、45歳未満での薬物療法の適応を検討するの にnon-HDL-Cが 有用 で ある可能性はある。 Friedwald式でLDLコレステロールを測定している施設では、新たな測定が不要となる点は、ApoB を越えたnon-HDL-Cのメリットであろう。 いずれにせよ、LDL-C、non-HDL-C、ApoBの心 血管リスクとしての差異は小さかろう。Lp(a)、 small dense LDLも含め、どれが最善の指標であるかはその道の研究者に委ね、いずれの指標を用いてもよいので高リスクの患者をいかに早く把握し、いかに早く(薬物)治療するかに臨床医はこだ わるべきだと私は思う。 1. Pischon T et al. Circulation. 2005;112(22):3375-3383. 2. Mach F et al. Eur Heart J. 2020;41(1):111-188. 3. Pencina MJ et al. Circulation. 2009;119(24):3078-3084.
新規に診断された2型糖尿病患者におけるビルダグリプチンとメトホルミンの早期併用療法とメトホルミン単剤逐次投与の血糖値耐久性(VERIFY):5年間の多施設共同無作為化二重盲検比較試験。
新規に診断された2型糖尿病患者におけるビルダグリプチンとメトホルミンの早期併用療法とメトホルミン単剤逐次投与の血糖値耐久性(VERIFY):5年間の多施設共同無作為化二重盲検比較試験。
Glycaemic durability of an early combination therapy with vildagliptin and metformin versus sequential metformin monotherapy in newly diagnosed type 2 diabetes (VERIFY): a 5-year, multicentre, randomised, double-blind trial Lancet 2019; 394: 1519 -1529. 上記論文のアブストラクト日本語訳 ※ヒポクラ×マイナビ 論文検索(Bibgraph)による機械翻訳です。 【背景】糖尿病合併症を遅らせるためには、良好な血糖コントロールを持続させる早期の治療強化が不可欠である。Vildagliptin Efficacy in combination with metfoRmIn For early treatment of type 2 diabetes(VERIFY)は、新たに2型糖尿病と診断された患者を対象に、34か国254施設で行われた無作為化二重盲検並行群間試験である。試験は、2週間のスクリーニング検査、3週間のメトホルミン単剤でのランイン期間、5年間の治療期間からなり、さらに試験期間1、2、3に分割されました。対象は、登録前2年以内に2型糖尿病と診断され、中心静脈血糖値(HbA1c)が48〜58mmol/mol(6-5〜7-5%)、体格指数22〜40kg/m2の18〜70才の患者さんでした。患者さんは1:1の割合で、早期併用療法群と初期メトホルミン単剤療法群に、対話型応答技術システムを用いて、層別化なしの単純無作為化により、ランダムに割り付けられました。患者、治験責任医師、評価を行う臨床スタッフ、データ解析者は、治療割り付けについてマスクされていた。試験期間1では、メトホルミン(1日安定量1000mg、1500mg、2000mg)とビルダグリプチン(50mg)1日2回の早期併用療法、または標準治療のメトホルミン単独療法(1日安定量1000mg、1500mg、2000mg)およびプラセボ1日2回投与が患者さんに実施されました。初回治療でHbA1cを53mmol/mol(7-0%)以下に維持できなかった場合、13週間間隔で連続した2回の定期診察で確認され、メトホルミン単独療法群の患者にはプラセボの代わりにビルダグリプチン50mg1日2回投与が行われ、すべての患者が併用治療を受ける第2期試験に移行した。主要評価項目は、無作為化から初回治療失敗までの期間とし、期間1を通じて無作為化から13週間隔で2回連続した定期診察時にHbA1c測定値が53mmol/mol(7-0%)以上と定義されました。全解析セットには、少なくとも1つの無作為化された試験薬の投与を受け、少なくとも1つの無作為化後の有効性パラメータが評価された患者さんが含まれています。安全性解析セットには、無作為化された試験薬を少なくとも1回投与されたすべての患者さんが含まれます。本試験はClinicalTrials. gov、NCT01528254に登録されている。 【所見】試験登録は2012年3月30日に開始し、2014年4月10日に完了した。スクリーニングされた4524人のうち、2001人の適格者が早期併用療法群(n=998)または初期メトホルミン単剤療法群(n=1003)に無作為に割り付けられた。合計1598名(79-9%)の患者が5年間の研究を完了した。早期併用療法群811例(81-3%)、単剤療法群787例(78-5%)であった。第1期の初回治療失敗の発生率は、併用療法群429例(43-6%)、単剤療法群614例(62-1%)であった。単剤治療群で観察された治療失敗までの期間の中央値は36-1(IQR 15-3-未到達[NR])カ月であり、早期併用療法を受けた患者の治療失敗までの期間の中央値は61-9(29-9-NR)カ月で試験期間を超えているとしか推定できない。5年間の試験期間中、早期併用療法群では単剤療法群と比較して初回治療失敗までの期間の相対リスクの有意な減少が認められました(ハザード比0-51[95%CI 0-45-0-58]、p<0-0001)。いずれの治療法も安全で忍容性が高く、予期せぬ新たな安全性所見はなく、試験治療に関連する死亡例もなかった。 【解釈】新たに2型糖尿病と診断された患者に対して、ビルダグリプチン+メトホルミン併用療法による早期介入は、現在の標準治療である初期メトホルミン単独療法と比較して長期的に大きく持続する利益をもたらす。【助成】ノバルティス。 第一人者の医師による解説 アジア人で効果高いDPP-4阻害薬 日本人の併用はより効果的な可能性 林 高則 医薬基盤・健康・栄養研究所臨床栄養研究部室長/窪田 直人 東京大学医学部附属病院病態栄養治療部准教授 MMJ.April 2020;16(2) 2型糖尿病の初期治療としてはメトホルミンの 使用が推奨されており、その後段階的に治療強化がなされるが、この治療強化はしばしば遅れ(臨床 的な惰性;clinical inertia)、推奨される血糖管理目標を達成できていない患者も多い。近年、初期からの併用療法がメトホルミン単剤療法より有用であるとの報告もあるが、長期の有用性や段階的な併用療法に対する優位性などについては十分なエビデンスがない。今回報告されたVERIFY試験は治療 歴のない2型糖尿病患者を対象に、メトホルミンとビルダグリプチンの早期併用療法による長期の血糖コントロール持続性や安全性について、メトホ ルミンによる単剤療法との比較を行った初めての研究である。 対象は新たに診断された2型糖尿病患者約2,000 人で、5年間の追跡が行われた。結果、早期併用群では単剤群と比較して、初期治療失敗までの期間の相対リスクが有意に低下した(ハザード比[HR], 0.51)。本研究では初期治療失敗後に単剤群はビルダグリプチンを併用する2次治療に移行し、この2次治療失敗までの期間を副次項目として評価して いるが、注目すべきことに早期併用群では2次治療 失敗までの期間の相対リスクも有意に低下している(HR,0.74)。つまり単に2剤併用の有用性を示しているのではなく、早期より併用療法を行うことで長期間の良好な血糖コントロール維持が可能であることが示された。    また、探索的項目として評価された心血管イベント発症も早期併用群において少ないことが示されたが、本研究は心血管イベント発症を評価するためにデザインされておらず、十分な検出力はなく、 結論を出すにはさらなる研究が必要である。 DPP-4阻害薬は非アジア人と比較してアジア人 で血糖低下効果が高いことが示されている(1)。本研究における東アジア地域でのサブ解析においても、早期併用群では初期治療失敗までの期間の相対リスクが大きく低下しており(HR, 0.37)、日本人でもメトホルミンとDPP-4阻害薬の早期併用療法はより効果的である可能性が期待される。 今回の結果は、2型糖尿病患者に対するメトホルミンとビルダグリプチンの早期併用療法は持続的な血糖コントロール達成のために有用であることを示している。ビルダグリプチン以外の薬剤での早期併用療法が同様の結果をもたらすかどうかは 興味深い点であり、現在進行中のGRADE研究(2) (メトホルミンにSU薬、DPP-4阻害薬、GLP-1受容体 アゴニスト、インスリンのいずれかを併用し、長期の有用性を検討)を含め今後さらなるエビデンス の蓄積が望まれる。 1. Kim YG, et al. Diabetologia. 2013; 56 (4) : 696-708. 2. Nathan DM, et al. Diabetes Care. 2013; 36 (8) : 2254–2261.
2019年新型コロナウイルスのゲノム特性解析と疫学:ウイルスの起源と受容体結合の意味するところ
2019年新型コロナウイルスのゲノム特性解析と疫学:ウイルスの起源と受容体結合の意味するところ
Genomic characterisation and epidemiology of 2019 novel coronavirus: implications for virus origins and receptor binding Lancet 2020 Feb 22;395(10224):565-574. 上記論文のアブストラクト日本語訳 ※ヒポクラ×マイナビ 論文検索(Bibgraph)による機械翻訳です。 【背景】2019年12月下旬、中国・武漢で原因不明の微生物によるウイルス性肺炎を呈した患者が報告された。その後、原因病原体として新規コロナウイルスが同定され、2019 novel coronavirus(2019-nCoV)と仮称された。2020年1月26日現在、2019-nCoV感染症は2000例以上確認されており、そのほとんどが武漢在住者または訪問者であり、ヒトからヒトへの感染が確認されている。 【方法】武漢の華南水産市場を訪問した9名の入院患者から採取した気管支肺胞洗浄液と培養分離株のサンプルについて次世代シーケンサーで解析を行った。これらの個体から2019-nCoVの完全および部分ゲノム配列が得られた。ウイルスコンティグをサンガーシークエンスで連結して全長ゲノムを得、末端領域はcDNA末端の迅速増幅で決定した。これらの2019-nCoVゲノムと他のコロナウイルスのゲノムの系統解析は、ウイルスの進化の歴史を決定し、その起源と思われるものを推測するのに役立てられた。ホモロジーモデリングにより、ウイルスの受容体結合の可能性を探った。 【調査結果】9人の患者から得られた2019-nCoVの10個のゲノム配列は、99-98%以上の配列同一性を示し、極めて類似していた。注目すべきは、2019-nCoVは、2018年に中国東部の舟山で採取された2つのコウモリ由来の重症急性呼吸器症候群(SARS)様コロナウイルス、bat-SL-CoVZC45およびbat-SL-CoVZXC21と近縁(88%の同一性)でしたが、SARS-CoV(約79%)およびMERS-CoV(約50%)より遠かったということです。系統解析の結果、2019-nCoVはベタコロナウイルス属のサルベコフイルス亜属に属し、最も近縁のbat-SL-CoVZC45およびbat-SL-CoVZXC21との枝長が比較的長く、SARS-CoVとは遺伝的に異なることが判明しました。注目すべきは、相同性モデリングにより、2019-nCoVは、いくつかの重要な残基でアミノ酸が異なるにもかかわらず、SARS-CoVと同様の受容体結合ドメイン構造を持っていることが明らかになった。 【解釈】2019-nCoVはSARS-CoVから十分に分岐し、新しいヒト感染性ベータコロナウイルスと見なされる。我々の系統解析は、コウモリがこのウイルスの本来の宿主である可能性を示唆しているが、武漢の海産物市場で売られている動物が、ヒトへのウイルス出現を促進する中間宿主である可能性もある。重要なのは、構造解析により、2019-nCoVがヒトのアンジオテンシン変換酵素2受容体に結合する可能性があることが示唆されたことである。本ウイルスの今後の進化、適応、拡散について早急に調査する必要がある。 【資金提供】中国国家重点研究開発計画、中国感染症制御・予防国家重点プロジェクト、中国科学院、山東第一医科大学 第一人者の医師による解説 ヒトのアンジオテンシン変換酵素2受容体に結合する可能性を示唆 下畑 享良 岐阜大学大学院医学系研究科脳神経内科学分野教授 MMJ.June 2020;16(3) 新型 コ ロ ナ ウ イ ル ス 感染症(COVID-19)は、 2019年12月に中国武漢で報告されて以来、世界 的にパンデミックとなった。本論文は中国疾病管 理予防センター(Centers for Disease Control and Prevention;CDC)が 論文発表前に公開し、 直ちに世界で共有されたものである。COVID-19 の病原体であるSARS-CoV-2のゲノム配列を系統 解析し、その由来と構造を検討している。 まず武漢の海鮮市場を訪 れ た8人 を 含 む9人 の入院患者の気管支肺胞洗浄液と咽頭拭い液からウイルス株を培養分離し、次世代シークエンサーを用いて解析している。9人から分離したSARSCoV-2のゲノム配列は類似し、99.98%以上の同一性があった。このことからこのウイルスは、生じてからの時間経過は短いことが示唆される。 次に、取得した完全ないし部分的なゲノムシーケンスを用いてウイルスコンティグ(ゲノム断片)を接続、完全長ゲノムを取得し、さらにcDNA末端 の増幅により末端領域を決定した。このウイルス ゲノムと他のコロナウイルスの系統解析を行い、 ウイルスの進化の歴史を決定し、起源を推測した。 結果として、SARS-CoV-2は、2018年に中国東部で収集された重症急性呼吸器症候群(SARS)類似の2つのコウモリ由来のコロナウイルス(bat-SLCoVZC45およびbat-SL-CoVZXC21)と88% の同一性があったが、SARS および中東呼吸器症 候群(MERS)の 病原体 で あるSARS-CoVおよびMERS-CoVとは、同一性 が そ れ ぞ れ 約79%、 約50%と低いことが判明した。つまりSARSや MERSの病原体よりもコウモリ由来コロナウイルスに近縁であるが、同一ではなく、おそらく海鮮市場で売られていた動物がコウモリから感染し、中間宿主となり、ゲノム配列が変化したものと推測された。実際に、この時期のコウモリは冬眠中で、 コウモリからヒトへの直接の感染は考えにくかった。また、ウイルスが感染する際に使用する受容体としては、SARS-CoVと同じアンジオテンシン変換酵素2(ACE2)受容体を用いることが分かった。 結論として、系統解析において、SARS-CoV-2は βコロナウイルス属サルベコウイルス亜属に分類 さ れ、最 も 近 い 近縁 のbat-SL-CoVZC45やbatSL-CoVZXC21のように比較的長い分枝長を持ち、 SARS-CoVやMERS-CoVとは異なっていた。また、 SARS-CoV-2がヒトのACE2受容体に結合できる可能性があることが示唆された。
人から人への感染を示す2019年新型コロナウイルスに関連した肺炎の家族性クラスター:研究報告。
人から人への感染を示す2019年新型コロナウイルスに関連した肺炎の家族性クラスター:研究報告。
A familial cluster of pneumonia associated with the 2019 novel coronavirus indicating person-to-person transmission: a study of a family cluster Lancet 2020 Feb 15;395(10223):514-523. 上記論文のアブストラクト日本語訳 ※ヒポクラ×マイナビ 論文検索(Bibgraph)による機械翻訳です。 【背景】中国湖北省武漢市で、新型コロナウイルスに関連した肺炎の集団発生が報告された。罹患した患者は、潜在的な感染源である地元のウェットマーケットと地理的に関連していた。本研究では,武漢訪問後に中国広東省深-市に帰国し,原因不明の肺炎を呈した家族集団の患者5名と,武漢に旅行していない追加家族1名の疫学,臨床,検査,放射線,微生物学的所見を報告する.これらの患者の遺伝子配列の系統解析を行った。 【FINDINGS】2020年1月10日から、2019年12月29日から2020年1月4日の間に深センから武漢に渡航した患者6人の家族を登録した。武漢に渡航した家族6人のうち、5人が新型コロナウイルスに感染していることが確認された。さらに、武漢に渡航していない家族1名が、家族4名と数日間接触した後、ウイルスに感染しました。武漢の市場や動物との接触はなかったが、2名は武漢の病院を受診したことがあった。家族5名(36-66歳)が感染後3-6日で発熱,上気道症状,下痢,あるいはこれらの複合症状を呈した.発症から6-10日後に当院(香港大学深-病院)を受診した。彼らと無症状の子供1人(10歳)には、放射線学的に地上の肺の混濁が見られた。高齢者(60歳以上)には,より多くの全身症状,広範なX線上の肺の地表面変化,リンパ球減少,血小板減少,CRPと乳酸脱水素酵素値の上昇がみられた.これら6人の患者の鼻咽頭または咽頭スワブは,ポイントオブケア多重RT-PCRによって既知の呼吸器系微生物に対して陰性であったが,5人の患者(成人4人と子供)はこの新規コロナウイルスの内部RNA依存RNAポリメラーゼおよび表面スパイク蛋白をコードする遺伝子に対してRT-PCR陽性であり,サンガー配列決定によって確認された.これら5名のRT-PCRアンプリコンと2名のフルゲノムの次世代シーケンサーによる系統解析の結果、このウイルスは中国のカブトコウモリに見られるコウモリ重症急性呼吸器症候群(SARS)関連コロナウイルスに最も近い新規コロナウイルスであることが判明した。 【解釈】我々の発見は、病院や家庭環境におけるこの新規コロナウイルスの人対人感染や、他の地域の旅行者の感染報告と一致するものである。 【資金提供】The Shaw Foundation Hong Kong, Michael Seak-Kan Tong, Respiratory Viral Research Foundation Limited, Hui Ming, Hui Hoy and Chow Sin Lan Charity Fund Limited, Marina Man-Wai Lee, the Hong Kong Hainan Commercial Association South China Microbiology Research Fund, Sanming Project of Medicine (Shenzhen) and High Level-Hospital Program (Guangdong Health Commission)を含む。) 第一人者の医師による解説 今でこそ常識だが 一家族のクラスター研究がもたらした大きな一歩 岩田 健太郎 神戸大学医学部附属病院感染症内科教授 /診療科長 MMJ.June 2020;16(3) 新しい感染症(新興感染症)が勃発したときは、 その感染症の正体を突き止めることが急務となる。それは漸近的に行われる。一足でカント的「物自体」を掴み取ることは不可能だ(もちろん、ずっと時間をかけても掴み取れないかもしれないのだが)。よって、研究者らは寄ってたかって、それも“急いで”真実に近づこうとする。感染症のアウトブレイク時は「研究しながら、対策する」という「急ぎの態度」が必要だ。ゆっくりと腰を落ち着けてやる研究とは性格を異にする。 本研究も、今からルックバックすると「なーんだ」という研究だ。しかし、本研究があったからこそ「なーんだ」なのである。すべての研究が先人の肩の上に乗っているとはよく言われるが、本研究のもたらした恩恵、すなわちSARS-CoV-2(論文発 表時は2019-nCoV)がヒト -ヒト感染を起こすという、現在では誰でも知っている「常識」を初めて看破した。その過程をここで追体験したい。 本研究は2019年12月29日から翌年1月4日 に広東省深圳市から湖北省武漢まで旅行した家族 6人を調査した。そのうち5人が新型コロナウイルスに感染し、香港大学深圳病院を受診。1月1日 に1人が熱と下痢を発症し、その後他の家族も発症した。4人は症状があったが、10歳の子は無症状だった。しかし、胸部 CTではすりガラス陰影が認められた。新型コロナウイルス感染はRT-PCR法で確定診断された。さらに武漢に行かなかった別の家族1人も同じ感染症に罹患し、RT-PCR法で診断が確定した。 聞き取り調査により、当初感染源と目された海鮮市場や動物との接触は否定された。武漢で接触した親戚にも発熱や咳が認められた。また、そうした親戚の中には肺炎として12月29日に武漢の病院を受診し、別の親戚が付き添っていた。武漢に行かなかった家族の感染が判明した事実も合わせ、 ヒト─ヒト感染があったことが示唆された。その後 ウイルスの全ゲノムシークエンシングが行われ、 遺伝子情報を用いた系統樹が作成された。リニエイジ Bのベータコロナウイルスに典型的な配列が RNAポリメラーゼ、スパイク蛋白、全ゲノムから見いだされた。患者2と患者5の全ゲノムはほぼ同じで、違いは塩基2つ分だけであった。 5月中旬でも 世界中で 猛威 を ふ る っ て い る COVID-19。私の周りでも院内外のアウトブレイクが起きている。アウトブレイクは巨大な対策を要し、それは担当者にとって苦痛以外の何物でもない。しかし、そこから得る新たな学びもある。まだ まだ分からないことの多いCOVID-19。学びを重ね、 理解を深め、少しでも近づき続けたいと考えながら本論文も読み直した。
日本における院外心停止患者におけるパブリックアクセス除細動と神経学的転帰:集団ベースのコホート研究。
日本における院外心停止患者におけるパブリックアクセス除細動と神経学的転帰:集団ベースのコホート研究。
Public-access defibrillation and neurological outcomes in patients with out-of-hospital cardiac arrest in Japan: a population-based cohort study Lancet 2019 Dec 21;394(10216):2255-2262. 上記論文のアブストラクト日本語訳 ※ヒポクラ×マイナビ 論文検索(Bibgraph)による機械翻訳です。 【背景】緊急医療サービス(EMS)要員が到着する前に院外心停止(OHCA)を起こし、ショック状態の心拍動を起こしていた患者では、公衆アクセスによる除細動の80%以上が自然循環の持続的な回復をもたらさない。このような患者の神経学的転帰と生存率は評価されていない。我々は,このような患者の神経学的状態と生存成績を評価することを目的とした。 [METHODS]本研究は,2005年1月1日から2015年12月31日までの間に日本でOHCAを発症した1人の患者(1Zs_2008 ain ainakersZs_2008299 ainakersZs_2008 ainakers784)を対象とした,全国の人口ベースのプロスペクティブな登録簿から得られたコホート研究のレトロスペクティブ解析である。主要転帰はOHCA後30日目の良好な神経学的転帰(Cerebral Performance Category of 1または2)であり,副次的転帰はOHCA後30日目の生存率であった.本研究は、University Hospital Medical Information Network Clinical Trials Registry, UMIN000009918に登録されています。 【FINDINGS】我々は、傍観者から心肺蘇生法を受けた28人の傍観者立会型OHCAとショック性心拍数を持つ28人の患者を同定しました。このうち,2242人(8-0%)の患者はCPR+公衆アクセス除細動で自然循環の回復が得られず,25人(89-5%)の患者は救急隊到着前にCPRのみで自然循環の回復が得られなかった.好ましい神経学的転帰を示した患者の割合は、パブリックアクセス除細動を受けた患者の方が受けなかった患者よりも有意に高かった(845[37-7%]対5676[22-6%];傾向スコアマッチング後の調整オッズ比[OR]、1-45[95%CI 1-24-1-69]、p<0-0001)。OHCA後30日目に生存した患者の割合も、パブリックアクセス除細動を受けた患者の方が受けなかった患者よりも有意に高かった(987[44-0%] vs 7976[31-8%];傾向スコアマッチング後の調整済みオッズ比[OR]、1-31[95% CI 1-13-1-52]、p<0-0001)。 [INTERPRETATION]本知見は、パブリックアクセス除細動の利点、およびコミュニティにおける自動体外式除細動器のアクセス性と利用可能性の向上を支持するものである。 第一人者の医師による解説 AEDを用いた市民へのCPR教育活動推進の意義付けとなる成果 中島 啓裕/安田 聡(副院長・部門長) 国立循環器病センター心臓血管内科冠疾患科・心臓血管系集中治療科 MMJ.June 2020;16(3) 近年、市民による自動体外式除細動器(AED)使用が心室細動(VF)による院外心停止患者の蘇生率向上に有効である可能性が報告された(1)。日本では約60万台のAEDが設置されている。各団体のAED普及啓発活動にもかかわらず、市民によるAED除細動実施率は5%程度と低い。 市民によるAED除細動患者の約80%は救急隊現場到着時点では心停止が持続していることから、AEDを運び込むまでの心肺蘇生(CPR)中断や救急通報の遅れなどのデメリットが除細動不成功例の予後に与える影響を明らかにすれば、より適正なAED使用を推奨できると考えられる。しかしながら、ガイドラインによりAED使用が強く推奨されている現代において、AED使用群と未使用群で予後を比較する無作為化対照試験の実施は困難である。 そこで、本研究では日本の総務省消防庁によるウツタイン様式救急蘇生統計データを活用した。救急隊現場到着時点のVF持続患者を対象に、事前に市民がAEDを用いてCPRを実施した患者とAED なしのCPRを実施した患者の予後が比較された。 2005~15年に日本全国で発生した1,299,784 人の院外心停止患者から、市民による目撃があり CPRが実施されたVF心停止患者28,019人のうち、救急隊現場到着時点の心停止持続患者27,329 人(CPR+ AED併用群 2 ,242 人 、CPR単独群 25,087人)を解析した。 背景因子を傾向スコアで調整したところ、CPR+ AED併用群はCPR単独群 と比較して、発症から救急隊覚知までに中央値1分の遅れ(2分 対 1分:P<0.0001)を認めたものの、 主要評価項目である30日後の神経学的転帰良好な割合は有意に高値であった(38% 対 23%:調整 オッズ比 , 1.45; 95% CI, 1.24~1.69:P< 0.0001)。さらに、救急通報時から救急隊現着までにかかった時間(中央値8分[四分位範囲6~10 分])別の感度分析でも同様の結果が得られた。 本研究では、AEDによる除細動実施で心拍再開 (AEDの主要効果)を得られなかった患者においても、AEDが神経学的予後を改善させうることが示された。AEDの音声ガイダンスなどがCPRの質の改善に寄与していた可能性が示唆された。本結果は救急通報~救急隊現着までの時間に依存しないことから、日本全国において一般化されるものであり、市民に対するAEDを用いたCPR教育活動を推進していく意義付けとなるものと考える。 一般市民のAED使用は、たとえ救急隊の現場到着までに患者の自己心拍再開を得られなかったとしても、 その後の患者の良好な予後につながるという点が本研究からのメッセージである。 1. Kitamura T et al. N Engl J Med. 2016;375(17):1649-1659.
三尖弁逆流の軽減のための経カテーテル的edge-to-edge修復術。TRILUMINATE単群試験の6ヵ月成績
三尖弁逆流の軽減のための経カテーテル的edge-to-edge修復術。TRILUMINATE単群試験の6ヵ月成績
Transcatheter edge-to-edge repair for reduction of tricuspid regurgitation: 6-month outcomes of the TRILUMINATE single-arm study Lancet 2019 Nov 30;394(10213):2002-2011. 上記論文のアブストラクト日本語訳 ※ヒポクラ×マイナビ 論文検索(Bibgraph)による機械翻訳です。 【背景】三尖弁閉鎖不全症は、高い罹患率と死亡率を伴う有病率疾患であり、治療選択肢はほとんどない。TRILUMINATE試験の目的は、低侵襲な経カテーテル三尖弁修復システムであるトライクリップの三尖弁逆流の軽減に対する安全性と有効性を評価することです。 【方法】TRILUMINATE試験は、ヨーロッパとアメリカの21施設で行われた前向き、多施設、シングルアーム試験です。NYHA(ニューヨーク心臓協会)クラスII以上の中等度以上の三尖弁逆流を有し、適用される基準に従って十分な治療を受けている患者を登録対象とした。収縮期肺動脈圧が60mmHg以上、三尖弁形成術の既往、TriClip装着を阻害する心血管植込み型電子機器を装着している患者は除外されました。参加者はTriClip三尖弁修復システムを用いてクリップベースのedge-to-edge修復技術を使用して治療を受けた。三尖弁逆流は、標準的な米国心エコー学会の等級分類を拡張した 5 クラスの等級分類(軽度、中等度、重度、巨大、奔流)を用いて評価された。有効性の主要評価項目は、大腿静脈穿刺時に三尖弁修復術を試みたすべての患者さんを対象に、術後30日目に三尖弁逆流の重症度が1グレード以上低下したこととし、35%をパフォーマンスゴールとしました。安全性の主要評価項目は、6ヵ月後の主要有害事象の複合であり、パフォーマンスゴールは39%でした。6ヵ月後のフォローアップに至らず、それまでのフォローアップで主要な有害事象が発生しなかった患者は、安全性の主要評価項目から除外された。本試験は登録を完了し、フォローアップが進行中であり、ClinicalTrials. gov(番号NCT03227757)に登録されている。 【FINDINGS】2017年8月1日から2018年11月29日までに85例(平均年齢77~8歳[SD 7~9]、女性56例[66%])を登録し、トライクリップ植込みを成功させた。心エコー図データおよび画像診断が可能な83例中71例(86%)において、三尖弁逆流の重症度が30日時点で少なくとも1グレード低下していた。片側下限 97-5% 信頼限界は 76% であり,事前に規定したパフォーマンス目標である 35% を上回った(p<0-0001).1名の患者が6ヵ月後のフォローアップまでに重大な有害事象を発症することなく退学したため、主要安全性エンドポイントの解析から除外された。6ヵ月後、84例中3例(4%)に主要な有害事象が発生し、事前に設定したパフォーマンスゴールである39%より低かった(p<0-0001)。72例中5例(7%)に単葉着床が発生した。周術期の死亡、手術への移行、デバイスの塞栓、心筋梗塞、脳卒中は発生しなかった。6ヵ月後、84例中4例(5%)に全死亡が発生した。 【解釈】TriClipシステムは安全で、三尖弁逆流を少なくとも1グレード減少させる効果があると思われる。この減少は、術後6ヶ月での有意な臨床的改善となる可能性がある。 第一人者の医師による解説 設定基準には合格 今後の長期成績および前向き試験結果が待たれる 清末 有宏 東京大学医学部附属病院循環器内科助教 MMJ.June 2020;16(3) 三尖弁閉鎖不全症(tricuspid regurgitation; TR)の所見は心臓超音波検査において日常的に遭遇する所見であり、近年は検査機器の発達にも伴い観察が容易となり、その流速を利用しての右室圧推定が行われることは医師国家試験レベルでも広く知られている。しかし逆にその病的意義あるいは治療介入効果の評価はしばしば困難である。 日本循環器学会「弁膜疾患の非薬物療法に関するガイドライン(2012年改訂版)」でも、外科的介入がクラス Iとなっているのは「高度 TRで、僧帽弁との同時初回手術としての三尖弁輪形成術」または「高度の1次性 TRで症状を伴う場合」のみとなっており、圧倒的に頻度が高い左心不全や心房細動などに続発する2次性(機能性)TRにおいては逆流が高度でも単独での三尖弁への外科的介入はクラス Iではない。 本論文で報告されたTRILUMINATE試験は、日本でも2017年に承認された経皮的僧帽弁接合不全修復システムのMitraClip®を三尖弁用に改良したTriClip®を用いて、中等度以上の三尖弁逆流を有 す る 患者85人 を 対象 に 経皮的三尖弁接合不全修復を行い、有効性と安全性を前向き多施設共同単群試験で検討している。 その結果、有効性に関して は、事前に規定した術後30日の心臓超音波検査上の三尖弁逆流重症度の1グレード以上改善達成率が 86%、片側97.5%信頼下限は76%であったため、 目標値(信頼下限35%以上)を達成したとの評価に至った。また安全性に関しては事前に規定した術後6カ月の主要有害事象の発現率が4%で、目標値(39%)を下回ったとの評価に至った。 今回の試験は単群試験であり、有効性・安全性の 目標値設定に関しては、同じ研究グループが2017 年に報告したMitraClip®を用いた高リスク患者に対する経皮的三尖弁接合不全修復の論文が引用されているが(1)、そちらを参照しても設定根拠ははっきりしなかった。手技詳細や超音波データの変化詳細などはむしろ2017年の論文に詳しく、自覚症状や予後の信頼できるサロゲートマーカーである 6分間歩行の改善も報告されている。技術的には、 当然ではあるが僧帽弁とは異なり三尖弁は三尖あるため、どの尖間のクリッピングを選択するかというのが1つ大きなポイントであるようだ。 いずれにせよ三尖弁への低侵襲単独介入が本技術により現実的になると思うが、その予後改善効果は上記 の積極的外科介入適応が現在までなかったため、真の有効性評価についてはランダム化対照試験によるハードエンドポイント改善効果判定を待たねばなるまい。 1. Nickenig G et al. Circulation. 2017;135(19):1802-1814.
電子タバコやベイプに関連した肺損傷の臨床症状、治療、短期転帰:前向き観察コホート研究。
電子タバコやベイプに関連した肺損傷の臨床症状、治療、短期転帰:前向き観察コホート研究。
Clinical presentation, treatment, and short-term outcomes of lung injury associated with e-cigarettes or vaping: a prospective observational cohort study Lancet 2019 Dec 7;394(10214):2073-2083. 上記論文のアブストラクト日本語訳 ※ヒポクラ×マイナビ 論文検索(Bibgraph)による機械翻訳です。 【背景】電子タバコまたはベイプに関連した肺損傷(E-VALIまたはVALIとも呼ばれる)の進行中のアウトブレイクが、2019年3月に米国で開始された。この疾患の原因,診断,治療,経過は依然として不明である。 【方法】この多施設,前向き,観察,コホート研究では,2019年6月27日から10月4日の間に,米国ユタ州に拠点を置く統合医療システム,Intermountain Healthcareで受診した電子タバコまたはベイプに関連する肺損傷の全患者のデータを収集した。米国ユタ州ソルトレイクシティを拠点とするテレクリティカルケアは、症例検証、公開報告、およびシステム全体の専門知識の普及のための中央リポジトリとして使用され、電子タバコまたはベイプに関連する肺損傷の診断・治療ガイドラインの提案が含まれた。ユタ州保健局(米国ユタ州ソルトレイクシティ)が実施したカルテレビューと患者へのインタビューから、患者の提示、治療、短期フォローアップ(退院後2週間)に関するデータを抽出した。 【FINDINGS】統合医療システム内の13病院または外来診療所において、60人の患者が電子タバコまたはベイプに関連した肺損傷を呈した。60人中33人(55%)が集中治療室(ICU)に入院した。60人中53人(88%)が体質的症状、59人(98%)が呼吸器症状、54人(90%)が消化器症状を呈していた。60人中54人(90%)に抗生物質が投与され,57人(95%)にステロイドが投与された.60人中6人(10%)が2週間以内にICUまたは病院に再入院し、そのうち3人(50%)はベイプまたは電子タバコの使用で再発した。2週間以内にフォローアップを受けた患者26名のうち,臨床的・X線的な改善は全員に見られたものの,胸部X線写真(15名中10名[67%])および肺機能検査(9名中6名[67%])で異常が残存している者が多かった.2名の患者が死亡し,電子タバコまたはベイプに関連する肺損傷は,両者とも死因ではないが一因であると考えられた。認知度の向上により、抗生物質やステロイドで治療された患者の重症度が幅広く確認されるようになった。改善されたものの、短期間のフォローアップでは多くの患者に異常が残存していた。電子タバコやベイプに関連した肺損傷は、感染症や他の肺疾患と症状が重なるため、依然として臨床診断が困難である。この疾患に対する高い疑い指数を維持することは,これらの患者の原因,最適な治療,および長期転帰を理解するための作業が続く中で重要である. 第一人者の医師による解説 日本では改正健康増進法が完全施行 たばこ対策の強化を 欅田 尚樹 産業医科大学産業保健学部産業・地域看護学講座教授 MMJ.June 2020;16(3) 電子たばこはプロピレングリコール(PG)やグリセロール(GLY)を基剤としさまざまな香料を添加したリキッドを電気的に加熱してエアロゾルを発生・吸煙する。海外ではニコチンを含むものが大半で若人を中心に急速に普及しているが、国内では、薬機法でニコチン含有は規制されている。一方、たばこ葉を電気的に加熱しニコチンを吸引する加熱式たばこは、たばこ事業法の製造たばことして、世界に先駆けて販売拡大しているが、構造も法規制 も異なる。 米国 で は2019年3月 よ り 電子 た ば こ ま た は ベ イ ピ ン グ に 関連した 肺傷害(lung injury associated with e-cigarettes or vaping; E-VALI)が急増したが、原因、診断、治療、経過は 不明のままであり社会問題にもなった。 本論文は、2019年6月27日~ 10月4日に米国ユタ州で収集したE-VALI全症例を解析した多施設共同前向き観察コホート研究の結果とともに実用的な臨床ガイドラインを提示している。死亡2人を含む患者 60人の多くは全身症状、呼吸器症状のみならず消化器症状を呈し、33人は集中治療室(ICU)管理を要した。その多くはステロイド治療を受けたが、診断の不確実性から抗菌薬の投与例も多かった。重症度は多岐にわたり、症状が改善しても胸部 X線写真や肺機能検査に異常が残存していた。患者の多くは大麻成分のテトラヒドロカンナビノール(THC) を含むリキッドを使用していたが、それ以外にもさまざまなものが使用されていた。 その後の調査で、多くのE-VALI患者の気管支肺胞洗浄液では、THCあるいはその代謝物とともに、 ビタミン Eアセテートが検出され、原因物質と考えられている(1)。米国疾病対策予防センター(CDC)の報告では、THCを含む電子たばこ・リキッドのリスク認知の広がり、ビタミン Eアセテートの使用禁止などさまざまな規制が実施され患者は減少したが、2020年2月18日現在、全米で死亡68人を含む2,807人の患者が報告されている。    電子たばこのエアロゾルには、PGやGLYの熱分解により生成したホルムアルデヒドに代表される発がん物質が高濃度に発生するものがあり、中には紙巻きたばこ主流煙より高濃度を示すものもある(2)。 日本呼吸器学会からは、「加熱式たばこや電子たばこによる健康影響は不明」であり「リスクが低いという証拠はない」、さらに「使用の際には二次曝露対策が必要である」、と見解と提言が出されている。 また現在パンデミックとなり問題となっている新型コロナウイルス肺炎の危険因子として喫煙(3)がある。国内でも受動喫煙対策を義務化した改正健康増進法が完全施行される中、たばこ対策の強化が求められる。 1. Blount BC et al. N Engl J Med. 2020;382(8):697-705. 2. Uchiyama S et al. Chem Res Toxicol. 2020;33(2):576-583. 3. Guan WJ et al. N Engl J Med. 2020 Feb 28. doi: 10.1056/NEJMoa2002032.
Irbesartan in Marfan syndrome(AIMS):二重盲検プラセボ対照無作為化試験。
Irbesartan in Marfan syndrome(AIMS):二重盲検プラセボ対照無作為化試験。
Irbesartan in Marfan syndrome (AIMS): a double-blind, placebo-controlled randomised trial Lancet 2019 Dec 21;394(10216):2263-2270. 上記論文のアブストラクト日本語訳 ※ヒポクラ×マイナビ 論文検索(Bibgraph)による機械翻訳です。 【背景】マルファン症候群において、長時間作用型の選択的アンジオテンシン-1受容体阻害薬であるイルベサルタンが、解離や破裂と関連する大動脈の拡張を抑制する可能性がある。我々は、マルファン症候群の小児および成人における大動脈拡張率に対するイルベサルタンの効果を明らかにすることを目的とした。 【方法】英国の22施設で、プラセボ対照二重盲検無作為化試験を行った。臨床的にマルファン症候群と確認された6~40歳の個人を含めることができた。試験参加者は全員、イルベサルタン75mgを1日1回オープンラベルで投与され、その後、イルベサルタン150mg(忍容性により300mgまで増量)またはマッチングプラセボに無作為に割り付けられた。大動脈径は、ベースラインとその後1年ごとに心エコーで測定された。すべての画像は治療割り付けを盲検化したコアラボで解析された。主要エンドポイントは大動脈基部拡張の割合であった。本試験はISRCTNに登録されており、番号はISRCTN90011794である。 【所見】2012年3月14日から2015年5月1日の間に、192名の参加者を募集し、イルベサルタン(n=104)またはプラセボ(n=88)にランダムに割り当て、全員が最長5年間追跡調査された。募集時の年齢中央値は18歳(IQR12~28)、99人(52%)が女性、平均血圧は110/65mmHg(SD16、12)、108人(56%)がβブロッカーを服用中であった。ベースラインの平均大動脈基部径はイルベサルタン群(SD 5-8)、プラセボ群(5-5)で34-4mmであった。大動脈基部拡張率の平均はイルベサルタン群0-53mm/年(95%CI 0-39~0-67)、プラセボ群0-74mm/年(0-60~0-89)、平均値の差は-0-22mm/年(-0-41~-0-02、p=0-030)であった。大動脈Zスコアの変化率もイルベサルタンによって減少した(平均値の差-0-10/年、95%CI -0-19 to -0-01、p=0-035)。イルベサルタンは、重篤な有害事象の発生率に差は認められず、良好な忍容性を示した。 【解釈】イルベサルタンは、マルファン症候群の小児および若年成人における大動脈拡張率の低下と関連しており、大動脈合併症の発生を抑制できる。 【助成】英国心臓財団、英国マルファン協会、英国マルファン・トラスト。 第一人者の医師による解説 β遮断薬との併用も可能 広がる内科的治療の選択肢 森崎 裕子 榊原記念病院臨床遺伝科医長 MMJ.June 2020;16(3) マルファン症候群(MFS)は、全身の結合組織の脆弱化をきたす遺伝性の疾患で、頻度は0.5 ~ 1 万人に1人という稀少疾患である。主な合併症は大動脈の脆弱性による大動脈瘤で、無治療では高率に大動脈解離に至る。解離リスクは大動脈基部径と相関があることから、現行治療の中心は、基部径が拡大した 患者に 予防的大動脈基部人工血管置換術を行う外科的治療であるが、手術による生活の質 (QOL)の低下は否めない。内科的治療としてβ遮断薬が使われてきたが、喘息症状の悪化やふらつきなどの副作用から十分な量が使えず拡張抑制効 果が得られない患者も多かった。 近年、MFS大動脈病変の背景にTGFβシグナル系の過剰応答が判明し、TGFβ抑制効果のあるアンジオテンシンⅡ受容体拮抗薬(ARB)が治療薬として浮上し、モデルマウス実験でロサルタンはβ遮断薬を凌駕する治療効果を示した。β遮断薬は、心筋の収縮を抑制し血圧変化を抑えることで大動脈壁のストレスを軽減するという対症療法である。一方、ARBは発症機序を抑制する根本的治療薬として期待されたが、その後の大規模臨床試験においてβ遮断薬に対するロサルタンの優越性は証明されず、「両者の効果はほぼ同等(非劣性)」という結 論に至っている(1)。 今回報告された無作為化プラセボ対照二重盲検試験のAortic Irbesartan Marfan Study(AIMS) では、ルーチン治療(β遮断薬の使用は任意)に長時間作用型 ARBイ ル ベ サ ル タン(開始150mg、 最大300mg)を追加し、プラセボを対照として、 大動脈拡張抑制効果を比較した。英国22施設で登録した6 ~ 40歳のMFS患者192人を対象とし、 最長5年間追跡した。主要評価項目として大動脈基 部径の年間変化量、副次評価項目としてZ スコア変 化量、解離や手術などのイベント発生も解析した。 その結果、心臓超音波検査での大動脈基部径の拡大はイルベサルタン群で0.53mm/年、プラセボ群 で0.74mm/年と有意差を認め、Zスコア評価でも 有意な抑制効果が示された。抑制効果は開始1年目 から認められ、両群とも約半数の患者がβ遮断薬を服用していたにもかかわらず、イルベサルタン 群の80%の患者が300mg/日まで増量可であったと示されている。 β遮断薬には、喘息患者やふらつきなどの副作用を認める患者では使いにくい、といった難点がある。一方、ロサルタンでは血圧抑制効果が不十分という問題があった。今回、降圧効果がより強い長時 間作用型のイルベサルタンで治療効果が認められ、 β遮断薬との併用も可能なことが示されたことで、 今後、内科的治療の選択肢が拡がり、個々の患者の 病態に合わせた治療を選べる方向性がみえてきたといえる。 1. Lacro RV et al. N Engl J Med. 2014;371(22):2061-2071.
股関節骨折における加速手術と標準治療の比較(HIP ATTACK):国際無作為化比較試験。
股関節骨折における加速手術と標準治療の比較(HIP ATTACK):国際無作為化比較試験。
Accelerated surgery versus standard care in hip fracture (HIP ATTACK): an international, randomised, controlled trial Lancet 2020 Feb 29;395(10225):698-708. 上記論文のアブストラクト日本語訳 ※ヒポクラ×マイナビ 論文検索(Bibgraph)による機械翻訳です。 【背景】観察研究では、股関節骨折患者において、手術の迅速化は転帰の改善と関連することが示唆されている。HIP ATTACK試験では、手術の迅速化が死亡率や重大な合併症を減らすかどうかを評価した。 【方法】HIP ATTACK試験は、17か国69病院で行われた国際無作為化対照試験である。手術が必要な45歳以上の股関節骨折の患者を対象とした。研究担当者は、中央コンピューター無作為化システムにより、無作為にブロックサイズを変えながら、患者を加速手術(診断後6時間以内に手術の目標)または標準治療のいずれかに無作為に割り付けた(1対1)。主要アウトカムは、無作為化後90日目における死亡率と主要合併症(死亡率、非致死的心筋梗塞、脳卒中、静脈血栓塞栓症、敗血症、肺炎、生命を脅かす出血、大出血)の複合であった。患者、医療従事者、および試験スタッフは治療割り付けを認識していたが、転帰判定者は治療割り付けをマスクされた状態で行われた。患者は intention-to-treat の原則に従って分析された。本研究はClinicalTrials. gov(NCT02027896)に登録されている。 【所見】2014年3月14日から2019年5月24日までに、27701人の患者がスクリーニングされ、そのうち7780人が適格であった。このうち2970人が登録され、加速手術(n=1487)または標準治療(n=1483)を受けるよう無作為に割り付けられた。股関節骨折の診断から手術までの時間の中央値は、加速手術群で6時間(IQR4-9)、標準治療群で24時間(10-42)であった(p<0-0001)。加速手術に割り付けられた140人(9%)と標準ケアに割り付けられた154人(10%)が死亡し,ハザード比(HR)は0~91(95%CI 0~72~1~14),絶対リスク減少(ARR)は1%(-1~3,p=0~40)であった.主要合併症は、加速手術に割り付けられた321人(22%)と標準治療に割り付けられた331人(22%)に発生し、HRは0-97(0-83~1-13)、ARRは1%(-2~4、p=0-71)であった。 【解釈】股関節骨折の患者では、加速手術は標準治療に比べて死亡率や主要合併症を複合したリスクを有意には下げなかった【財源】カナダ保健研究機構(Canadian Institutes of Health Research. 第一人者の医師による解説 6時間以内の手術は有用だが 24時間以内の手術なら問題ないという解釈も可能 田島 康介 藤田医科大学病院救急科教授 MMJ.June 2020;16(3) 大腿骨近位部骨折は全世界で年間150万人以上の高齢者が受傷し、早期手術、早期離床を目指すことで合併症や日常生活動作(ADL)および生命予後を改善することが数多く報告されており(1),(2)、欧米では入院後24~48時間以内に手術を提供することが標準的治療となっている。日本では日本整形外科学会の2013年度調査によると手術までの平均待機日数は4.4日であるが、早期に手術を行う施設が昨今増加している。 このような背景から、著者らは、超早期手術により患者の疼痛や安静をより早期に解除することが 合併症や死亡率の改善につながるかどうかを検討するため、欧米・アジア(日本を含まない)など17 カ国69施設で無作為化対照試験(HIP ATTACK)を実施した。45歳以上で転倒などの低エネルギーによる外傷で受傷した患者を対象とし、登録患者 2,970人が超早期群(診断後6時間以内に手術)と 標準治療群に無作為に割り付けられた。患者背景、 既往歴、内服歴、社会歴などに差はなかった。 手術 90日後の超早期群と標準治療群の比較において、主要評価項目である死亡率(9% 対 10%[標準]) および複数の重大合併症発生率(22% 対 22%) に関して有意差はなく、副次評価項目である心筋梗塞(6% 対 5%)、心不全(2% 対 2%)、静脈血栓塞栓症(肺塞栓症、深部静脈血栓症)(1% 対 1%)、 大出血(6% 対 5%)、敗血症(5% 対 5%)のいずれも有意差はなかった。しかし、敗血症以外の感染症(11% 対 14%;P=0.032)、脳卒中(<1% 対 1%;P=0.047)、せん妄(9% 対 12%;P= 0.0089)は超早期群で有意に発生率が低かった。 既報では72、48、24時間と手術待機時間が短いほどさまざまな合併症が減少するとされ、生命予後にも医療経済的にも有利とされている1。超早期手術は術前検査が不十分であるとの指摘もあるが、今回の試験では各パラメータに差はなかった。骨折を受傷した患者は手術まで床上安静を強いられ疼痛からも解除されないことを考えると、むしろせん妄の発生率が低下するなど超早期手術は安全かつ有用であるとも言える。 日本の現状として、予定手術でうまった手術室の予定を調整して超早期手術を提供することは困難な施設が多いであろう。また、死亡率と合併症発生率に有意差がなかったことは、逆に超早期手術を行わず24時間以内に手術が行えれば問題ないという解釈も成り立つ。最後に、本試験の術後90日 死亡率(9~10%)は日本の標準的な報告よりも高いことを言及しておきたい。 1. Simunovic N et al. CMAJ. 2010;182(15):1609-1616. 2. Nyholm AM et al. J Bone Joint Surg Am. 2015;97(16):1333-1339.
Resmetirom(MGL-3196)の非アルコール性脂肪肝炎の治療:多施設、無作為、二重盲検、プラセボ対照、第2相試験。
Resmetirom(MGL-3196)の非アルコール性脂肪肝炎の治療:多施設、無作為、二重盲検、プラセボ対照、第2相試験。
Resmetirom (MGL-3196) for the treatment of non-alcoholic steatohepatitis: a multicentre, randomised, double-blind, placebo-controlled, phase 2 trial Lancet 2019 Nov 30;394(10213):2012-2024. 上記論文のアブストラクト日本語訳 ※ヒポクラ×マイナビ 論文検索(Bibgraph)による機械翻訳です。 【背景】 非アルコール性脂肪性肝炎(NASH)は、肝脂肪沈着、炎症、肝細胞障害、進行性肝線維化を特徴とする。Resmetirom(MGL-3196)は、肝臓指向性、経口活性、選択的甲状腺ホルモン受容体βアゴニストで、肝脂肪代謝を増加させ、脂肪毒性を低下させることによりNASHを改善するよう設計されています。 【方法】MGL-3196-05は、米国内の25施設で36週間の無作為化二重盲検プラセボ対照試験が実施されました。生検でNASH(線維化ステージ1~3)が確認され、MRI-proton density fat fraction(MRI-PDFF)により評価したベースライン時の肝脂肪率が10%以上の成人が適格とされました。患者は、コンピュータベースのシステムにより、resmetirom 80 mgまたはマッチングプラセボを1日1回経口投与するよう2対1に無作為に割り付けられた。12週目と36週目に肝脂肪を連続測定し、36週目に2回目の肝生検を行った。主要評価項目は、ベースラインと12週目のMRI-PDFFを測定した患者において、12週目にプラセボと比較してMRI-PDFFで評価した肝脂肪の相対変化としました。本試験は ClinicalTrials. gov(NCT02912260) に登録されている。 【所見】米国内の18施設で348名の患者がスクリーニングされ、84名がレスメチロムに、41名がプラセボにランダムに割り付 けられた。12週目(レスメチロム:-32-9%、プラセボ:-10-4%、最小二乗平均差:-22-5%、95%CI:-32-9~-12-2、p<0-0001)および36週目(レスメチロム:-37-3%、プラセボ:-8-5、34:-8%、42-0~-15-7、p<0-0001)においてプラセボ:78名と比較して肝臓脂肪の相対低下が認められ、レスメトロム:74名およびプラセボ:38名では肝臓脂肪が低下していました。有害事象は、ほとんどが軽度または中等度であり、レスメチロムで一過性の軽い下痢と吐き気の発生率が高かったことを除いて、群間でバランスがとれていた。 【解釈】レスメチロム投与により、NASH患者において12週間および36週間の投与後に肝脂肪の有意な減少が認められた。レスメチロムのさらなる研究により、組織学的効果と非侵襲的マーカーや画像診断の変化との関連を記録する可能性があり、より多くのNASH患者におけるレスメチロムの安全性と有効性を評価することができます。 【資金提供】マドリガル・ファーマスーティカルズ 第一人者の医師による解説 開発進むNASH治療薬 線維化改善作用に関しては進行中の第3相試験で検証 中原 隆志(診療准教授)/茶山 一彰(教授) 広島大学大学院医系科学研究科消化器・代謝内科学 MMJ.August 2020;16(4) 現在、世界的に非アルコール性脂肪肝炎(nonalcoholicsteatohepatitis;NASH)が急増し、社会問題化している。NASHの多くはメタボリック症候群を背景に発症するが、さまざまなホルモン分泌異常も病態に関与する(1) 。健常人と比較し、非アルコール性脂肪性肝疾患(NAFLD)では甲状腺機能低下症が有意に多く(21% 対 10%)(2) 、さらにNASHではNAFLDよりも甲状腺機能低下症が高頻度にみられる(3)。 本論文は、肝細胞に高発現する甲状腺ホルモン受容体β(THR-β)に対する特異的アゴニストであるレスメチロム(resmetirom)の有効性と安全性の評価を目的に、米国25施設で実施された無作為化プラセボ対照第2 相試験の報告である。対象はベースライン時の肝脂肪率が10%以上のNASH患者125 人で、生検でNASH( 線維化:stage 1~ 3)が確認され、MRIプロトン密度脂肪画分測定法(MRI-PDFF)により肝臓に10%以上の脂肪化が認められた患者であった。患者はレスメチロム(80mg)もしくはプラセボを1日1回経口投与する群に2対1の比で無作為に割り付けられた。12週時と36週時に肝臓の脂肪が測定され、36週時には2回目の肝生検が実施された。 その結果、12、36週時の肝脂肪率のベースラインからの低下度はレスメチロム群の方がプラセボ群よりも大きく、両群間の最小二乗平均差は12週時で- 25 .5 %(P< 0 .0001)、36 週時で-28 .8%(P<0 .0001)であった。また、アラニンアミノトランスフェラーゼ(ALT)の低下、低比重リポ蛋白(LDL)コレステロール、中性脂肪(TG)、リポ蛋白の低下や線維化マーカーや肝細胞の風船化(ballooning)と相関するサイトケラチン(CK)-18の低下を認め、36週時のNAFLD活動性スコアの改善を認めた。忍容性も良好であった。有害事象の多くは軽度~中等度で、一過性の軽度下痢および悪心の発現率がレスメチロム群で高かった以外は2群間にほとんど差はなかった。 一方、NASHの予後は、肝脂肪化ではなく、肝線維化によって規定されることが明らかとなっている。本研究では直接的な線維化の評価がされておらず、また肝細胞におけるTHR-βの発現量も評価されていない。線維化改善作用に関しては現在進行中のstage F2 ~ F3の線維症を有するNASH患者を対象とした第3相試験(MAESTRO-NASH試験)で検証されることとなる。 1. Takahashi H. Nihon Rinsho. 2019;77: 884-888. 2. Pagadala MR et al. Dig Dis Sci. 2012;57(2):528-534. 3. Carulli L et al. Intern Emerg Med. 2013;8(4):297-305.
非アルコール性脂肪肝炎の治療薬としてのオベチコール酸:多施設共同無作為化プラセボ対照第3相試験の中間解析。
非アルコール性脂肪肝炎の治療薬としてのオベチコール酸:多施設共同無作為化プラセボ対照第3相試験の中間解析。
Obeticholic acid for the treatment of non-alcoholic steatohepatitis: interim analysis from a multicentre, randomised, placebo-controlled phase 3 trial Lancet 2019 Dec 14;394(10215):2184-2196. 上記論文のアブストラクト日本語訳 ※ヒポクラ×マイナビ 論文検索(Bibgraph)による機械翻訳です。 【背景】 非アルコール性脂肪性肝炎(NASH)は、肝硬変に至ることもある一般的な慢性肝疾患の一種である。ファルネソイドX受容体アゴニストであるオベチコール酸は、NASHの組織学的特徴を改善することが示されている。ここでは、NASHに対するオベチコール酸の進行中の第3相試験の予定された中間解析の結果を報告する。 【方法】この多施設共同無作為化二重盲検プラセボ対照試験では、明確なNASH、非アルコール性脂肪性肝疾患(NAFLD)活性スコアが4以上、線維化ステージF2-F3、または少なくとも1つの合併症を伴うF1の成人患者を、対話的ウェブ応答システムを用いて1:1:1で、プラセボ、オベチコール酸10mg、オベチコール酸25mgを毎日内服するようランダムに割り当てた。肝硬変、他の慢性肝疾患、高アルコール摂取、または交絡条件が存在する患者は除外された。18ヶ月目の中間解析における主要評価項目は、NASHの悪化を伴わない線維化の改善(1ステージ以上)、または線維化の悪化を伴わないNASHの消失とし、いずれかの主要評価項目を満たした場合に試験成功したと判断されました。主要解析は、線維化ステージF2-F3の患者様で、少なくとも1回の治療を受け、事前に指定された中間解析のカットオフ日までに18ヵ月目の診察に到達した、または到達する見込みの患者様を対象に、intention to treatで実施されました。また、本試験では、NASHおよび線維化の他の組織学的および生化学的マーカー、ならびに安全性についても評価しました。本試験は、ClinicalTrials. gov、NCT02548351、EudraCT、20150-025601-6に登録され、進行中である。 【所見】2015年12月9日から2018年10月26日の間に、線維化ステージF1~F3の患者1968名が登録され、少なくとも1回の試験治療を受け、線維化ステージF2~F3の患者931名が主要解析に含まれた(プラセボ群311名、オベチコール酸10mg群312名、オベチコール酸25mg群308名)。線維化改善エンドポイントは、プラセボ群37名(12%)、オベチコール酸10mg群55名(18%)、オベチコール酸25mg群71名(23%)が達成した(p=0-0002)。NASH消失のエンドポイントは達成されなかった(プラセボ群25例[8%]、オベチコール酸10mg群35例[11%][p=0-18]、オベチコール酸25mg群36例[12%][p=0-13])。安全性集団(線維化ステージF1~F3の患者1968名)において、最も多く見られた有害事象はそう痒症(プラセボ群123例[19%]、オベチコール酸10mg群183例[28%]、オベチコール酸25mg群336例[51%])で、発現頻度は概ね軽度から中等度であり、重症度は低かったです。全体的な安全性プロファイルはこれまでの試験と同様であり、重篤な有害事象の発生率は治療群間で同様でした(プラセボ群75例[11%]、オベチコール酸10mg群72例[11%]、オベチコール酸25mg群93例[14%])。 【解釈】オベチコール酸25mgはNASH患者の線維化およびNASH疾患活性の主要成分を著しく改善させました。この予定された中間解析の結果は、臨床的に有意な組織学的改善を示しており、臨床的有用性を予測する合理的な可能性を持っています。本試験は臨床転帰を評価するために継続中である。 第一人者の医師による解説 脂肪肝炎の組織学的治癒の改善は達成せず 搔痒による忍容性の懸念も 中島 淳 横浜市立大学大学院医学研究科肝胆膵消化器病学教室主任教授 MMJ.August 2020;16(4) 非アルコール性脂肪性肝疾患(NAFLD)は飲酒習慣のない脂肪肝で、日本でも食生活の欧米化に伴い2000 万人以上の患者がいる。NAFLDの約25%は慢性進行性の肝炎である非アルコール性脂肪肝炎(NASH)になり、その後肝硬変や肝がんに進展する。また、欧米の調査ではNAFLDの死因トップは心血管イベントである。NASHに適応のある薬剤は世界的にまだ1つもなく、多くの開発治験がなされてきたがそのハードルは高い。最近では線維化抑制薬セロンセルチブの第3 相国際臨床試験が日本も含めて行われたが主要評価項目の達成に至らなかった。 本論文は、肝臓の核内受容体FXRの作動薬であるオベチコール酸のNASHに対する有用性を評価するために、20カ国332施設で実施された無作為化プラセボ対照第3相試験(REGENERATE)の中間解析結果の報告である。REGENERATE試験では、線維化ステージ1 ~ 3のNASH患者1,968人をプラセボ群、オベチコール酸10 mg群、25 mg群に無作為化し、1年半の投与後に肝生検が行われ評価された。 その結果、2つの主要評価項目のうちの1つである脂肪肝炎の悪化なき線維化の1ステージの有意な組織学的改善を25mg群でのみ達成したが(プラセボ群12% 対 25mg群23%)、もう1つの主要評価項目である脂肪肝炎の組織学的治癒(NASH resolution)は達成しなかった。重篤な有害事象は認められなかった。 主要評価項目の1つを満たしたことから米国では本剤の承認申請が行われている。確かに米国では近々FDAがオベチコール酸の早期承認を行うと報道されているが、問題もある。まず一番の問題は対プラセボでの治療効果が非常に低いことである。線維化に対して10 mgは無効で、25mgでのみ有効であったが、そのレスポンダーは23%にとどまった。しかもNASHの病理学的治癒は達成されてない。このようなパワーでは果たして今後投与を継続して4年後にハードエンドポイントであるイベント低減を達成できるだろうか。 また、薬剤独自の有害事象として痒みとLDLコレステロールの上昇が懸念されている。前者は本試験の25mgにおいて軽症~重症の搔痒を51%に認めた(プラセボ群19%)ことから忍容性が心配であろう。LDLコレステロールの上昇は25mg 群で17%(プラセボ群7%)に認めたが、これは本疾患の欧米での死因トップが心血管イベントであることを考慮すると問題かもしれない。非常に残念なことは、日本においてオベチコール酸の開発は第2相試験までで中断され、今回のグローバル試験に日本は参加できなかった点であり、当分NASHの新薬は国内で承認されることはなさそうである。
潰瘍性大腸炎における大腸がん:スカンジナビアの人口ベースコホート研究。
潰瘍性大腸炎における大腸がん:スカンジナビアの人口ベースコホート研究。
Colorectal cancer in ulcerative colitis: a Scandinavian population-based cohort study Lancet 2020 Jan 11;395(10218):123-131. 上記論文のアブストラクト日本語訳 ※ヒポクラ×マイナビ 論文検索(Bibgraph)による機械翻訳です。 【背景】潰瘍性大腸炎(UC)は大腸癌(CRC)の危険因子である。)しかし、利用可能な研究は、古い治療とサーベイランスパラダイムを反映しており、腫瘍ステージ別のCRC発生率やCRCによるステージ調整死亡率を評価するなど、サーベイランスとリードタイムのバイアスを考慮せずにCRC発生リスクを評価したものがほとんどである。我々は、UC患者におけるCRC死亡率及びCRC発症の全体的及び国別のリスクの両方を比較することを目的とした。 【方法】デンマーク(n=32 919)及びスウェーデン(n=63 528)のUC患者96 447人の人口ベースのコホート研究において、患者は、CRC発症及びCRC死亡率について1969年1月1日から2017年12月31日の間に追跡され、一般集団のマッチした参照人(n=949 207)と比較された。UC患者を国の登録から選び、(当該国の)患者登録に関連する国際疾病分類の記録が2つ以上ある場合、またはそのような記録1つと炎症性腸疾患を示唆する形態コードを持つ大腸生検報告書がある場合に解析に含めました。UC患者全員について、デンマークとスウェーデンの総人口登録から、性、年齢、出生年、居住地が一致した参照人物を選んだ。Cox回帰を用いて、腫瘍の病期を考慮したCRC発症およびCRC死亡のハザード比(HR)を算出した。 【所見】追跡期間中に、UCコホートでは1336例のCRC発症(1000人年当たり1-29例)、参照個人では9544例のCRC発症(1000人年当たり0-82例、HR1-66、95%CI1-57-1-76)が観察された。UCコホートでは、同期間に639人の患者がCRCで死亡した(1000人年当たり0-55人)のに対し、参照群では4451人(1000人年当たり0-38人、HR 1-59、95%CI 1-46-1-72)であった。UC患者のCRC病期分布は、マッチさせた参照群よりも進行していなかったが(p<0-0001)、腫瘍病期を考慮すると、UCおよびCRC患者はCRC死亡のリスクが依然として高かった(HR 1~54、95%CI 1~33~1~78)。過剰リスクは暦年間で減少した:追跡の最後の5年間(2013~17年,スウェーデンのみ),UC患者のCRC発症のHRは1~38(95%CI 1-20~1-60,または5年ごとにUC患者1058人に1例の追加),CRCによる死亡のHRは1~25(95%CI 1-03~1-51,または5年ごとにUC患者3041人に1例の追加)であった。 【解釈】UCのない人に比べて、UCのある人はCRCを発症するリスクが高く、CRCと診断されてもあまり進行しておらず、CRCによる死亡のリスクも高いが、これらの過剰リスクは時間とともに大幅に減少している。国際的なサーベイランスガイドラインにはまだ改善の余地があるようだ。 【財源】スウェーデン医学協会、カロリンスカ研究所、ストックホルム県議会、スウェーデン研究会議、スウェーデン戦略研究財団、デンマーク独立研究基金、フォルテ財団、スウェーデンがん財団 第一人者の医師による解説 長期の追跡による成果 潰瘍性大腸炎の診療ガイドライン改訂に役立つ 中山 富雄 国立がん研究センター社会と健康研究センター検診研究部部長 MMJ.August 2020;16(4) 潰瘍性大腸炎など炎症性腸疾患の大腸がんリスクについては、これまで4件のメタアナリシスが報告されているが、うち3件は2004年までの古いデータに基づいていた。今回の研究はスウェーデンとデンマークで1969~2017年に診断された95 ,000 人強の潰瘍性大腸炎患者と性・年齢・居住地をマッチさせた一般集団95万人強を最長約50年追跡して、リスクを評価した。潰瘍性大腸炎診断後の大腸がん罹患のみに限定して解析した。 その結果、潰瘍性大腸炎の患者が一般集団に比べて罹患リスクが1.66倍、死亡リスクが1.59倍高いことが示されたが、この成績は先行研究と同等であった。個別の因子として、18歳未満の潰瘍性大腸炎発症、全大腸型大腸炎、原発性硬化性胆管炎の合併、1親等の大腸がん家族歴が一般集団に比べて特にリスクが高いことが確認された。また潰瘍性大腸炎診断後の1年以内に大腸がんの診断および死亡のリスクが特に高かった。 罹患のみが高いのであれば、潰瘍性大腸炎に対する内視鏡検査で偶発的に大腸がんが見つかったというサーベイランスバイアスの可能性が高いが、死亡が増加していることは、必ずしもバイアスで説明できるものではなく、大腸炎の発病自体が発がんに影響しているのだろう。若年発症は確かにリスクが高いが、40歳以上で潰瘍性大腸炎と診断された場合は、診断後5年以降の大腸がん死亡リスクは一般集団とあまり変わらず、60歳以上での診断例は、診断直後から一般集団と差がなかった。 この長い追跡期間の間に、大腸がん罹患・死亡リスク自体は大幅に低下していた。これは前がん病変の検出やサーベイランスの変遷によるものかもしれないが、食習慣や運動などの予防の影響かもしれない。 今回の研究結果は、住民を対象とし50年近い長期の追跡期間によるもので、結果を一般化しやすい。潰瘍性大腸炎早期発症や病変範囲の広い大腸炎などが際立ってリスクが高いこと、追跡が長期化した場合は罹患も死亡もリスクが一般集団と同レベルに低下することなど、個別のリスクに応じた詳細なサーベイランス方法の設定が可能となる非常に有用なデータである。これまで高リスク者に1~2年に1回の内視鏡検査が推奨されてきたが、いつまで続けるのかは示されていなかった。今回の成績が、潰瘍性大腸炎患者の診療ガイドラインの改訂に役立つこととなるだろう。
大腸がんの転帰予測のためのディープラーニング:発見と検証の研究。
大腸がんの転帰予測のためのディープラーニング:発見と検証の研究。
Deep learning for prediction of colorectal cancer outcome: a discovery and validation study Lancet 2020 Feb 1;395(10221):350-360. 上記論文のアブストラクト日本語訳 ※ヒポクラ×マイナビ 論文検索(Bibgraph)による機械翻訳です。 【背景】早期大腸がん患者を層別化し、アジュバント療法の選択を洗練するために、予後のマーカーを改善することが必要である。本研究の目的は、深層学習を使用してスキャンした従来のヘマトキシリン・エオジン染色切片を直接分析することにより、原発性大腸がん切除後の患者の転帰のバイオマーカーを開発することである。 【方法】4コホートから明らかに良好または不良の病期を有する患者からの12 000 000以上の画像タイルを使用して、超大型異種画像の分類のために専用に構築された合計10の畳み込みニューラルネットワークを訓練するために、使用された。10 個のネットワークを統合した予後バイオマーカーを、転帰が明確でない患者を使用して決定した。このマーカーは、英国で作成されたスライドを用いて920人の患者でテストされ、次にノルウェーで作成されたスライドを用いて、カペシタビンの単剤投与を受けた1122人の患者で、あらかじめ定められたプロトコルに従って独立に検証された。すべてのコホートには、切除可能な腫瘍を有し、解析に利用できるホルマリン固定パラフィン包埋腫瘍組織ブロックを有する患者のみが含まれていた。主要アウトカムは癌特異的生存率であった。 【所見】4つのコホートから828人の患者が明確なアウトカムを有し、明確なグランドトゥルースを得るためのトレーニングコホートとして使用された。1645人の患者は明確な転帰を示さず、チューニングに使用された。バイオマーカーは、検証コホートの一次解析では、予後不良と予後良好のハザード比を3-84(95%CI 2-72-5-43; p<0-0001)、同じコホートの一変量解析で有意だった予後マーカー(pN期、pT期、リンパ管侵襲、静脈血管侵襲)を調整すると3-04(同 2-07-4-17; p<0-0001)であった。 【解釈】臨床的に有用な予後マーカーが、従来のヘマトキシリン・エオジン染色した腫瘍組織切片のデジタルスキャンと連携したディープラーニングを使用して開発された。このアッセイは独立した大規模な患者集団で広範囲に評価され,確立された分子的・形態的予後マーカーと相関し,それを上回り,腫瘍や結節の病期を越えて一貫した結果を与えることができた。このバイオマーカーは、II期とIII期の患者を十分に異なる予後グループに層別化し、非常にリスクの低いグループでの治療を避け、より強力な治療レジームから恩恵を受ける患者を特定することによって、補助治療の選択の指針として使用できる可能性がある。 第一人者の医師による解説 汎用性高い新規バイオマーカー 前向き比較試験での検討必要 小澤 毅士 帝京大学医学部附属病院外科助教/多田 智裕 武蔵浦和メディカルセンターただともひろ胃腸科肛門科理事長 MMJ.August 2020;16(4) 遺伝子検査の進歩などに伴い、さまざまな疾患において各個人に合わせた治療法の選択(プレシジョン・メディシン)が可能になってきた。進行再発大腸がんでは、RAS、BRAFなどの遺伝子変異、マイクロサテライト不安定性(MSI)、がん発生部位に応じた抗がん剤効果予測が一般的に行われている。一方、大腸がん治癒切除後の補助化学療法については、依然として深達度、リンパ節・遠隔転移を考慮したステージングを超える明確な導入判断基準はなく、新たなバイオマーカーが期待されている。 近年、深層学習(deep learning)技術の登場により人工知能(AI)の能力が飛躍的に向上した。医療現場では特にAIを用いた画像診断支援の研究が盛んで、専門医と同等の診断能を示すAIの開発が進んでおり、すでに医療現場に導入されているAI診断支援システムもある。 このような背景において、本研究では大規模な大腸がんの原発巣の病理切除標本スライドをもとに、予後予測AIを開発した。まず予後との関連付けを行ったステージI~ III大腸がんの病理切除標本スライドを用いてAIの教育を行い、別のコホートでその予後予測能を検証した。本研究でとられた手法の特徴は、腫瘍内不均一性を考慮して、標本スライド全体をそのまま予後と関連付けて学習させるのでなく、がん病変部位をタイルと呼ばれる小区域に区切り、それぞれのタイルを解析の上、最終的にがん病変をタイルのヒートマップとして学習させた点である。 結果として、ステージII~ III大腸がんにおいて、AIによる分類は有意な独立した予後因子となりうることが示唆され(ハザード比[HR],3.04;P<0 .0001)、またステージII、ステージIIIそれぞれにおいても同様の結果であった(それぞれHR, 2.71[P=0.011]、2.95[P<0.0001])。1人当たりの解析にかかった時間(中央値)は2.8分であった。 AIを用いた病理標本スライド分類は、新しいバイオマーカーとして役立つのみならず、ヒューマンエラーや観察者間相違をなくし、バイオマーカーとして高い再現性が期待できる。欧米で導入されている遺伝子発現検査などは、コスト、検査時間、検体の保管方法などに伴う再現性の問題から、なかなか広まらないのに対して、本研究で用いられるのは3μ m厚のヘマトキシリンエオジン染色スライドのみであり、汎用性が高いと考えられる。 本研究で開発されたAI分類の最終目標は、ステージII~ III大腸がんに対する術後補助化学療法の導入選択のバイオマーカーとして役立つことであり、これについては今後前向きな比較試験を行い、検討していく必要がある。
活動性の関節症性乾癬患者におけるビメキズマブ:48週間の無作為化、二重盲検、プラセボ対照、用量設定の第2b相試験の結果。
活動性の関節症性乾癬患者におけるビメキズマブ:48週間の無作為化、二重盲検、プラセボ対照、用量設定の第2b相試験の結果。
Bimekizumab in patients with active psoriatic arthritis: results from a 48-week, randomised, double-blind, placebo-controlled, dose-ranging phase 2b trial Lancet 2020 Feb 8;395(10222):427-440. 上記論文のアブストラクト日本語訳 ※ヒポクラ×マイナビ 論文検索(Bibgraph)による機械翻訳です。 【背景】 インターロイキン17A(IL17A)とインターロイキン17F(IL17F)の二重中和は、関節症性乾癬の新規治療アプローチとなる可能性があります。BE ACTIVE試験は、チェコ共和国、ドイツ、ハンガリー、ポーランド、ロシア、米国の41施設で行われた無作為化、二重盲検、プラセボ対照、用量設定の第2b相試験で、IL17AとIL17Fを選択的に中和するモノクローナル抗体、Bimekizumabが評価されました。成人発症の活動性関節症で6カ月以上の症状がある18歳以上の適格患者さんを、プラセボ、ビメキズマブ16mg、ビメキズマブ160mg、ビメキズマブ160mgと320mgの単回ロード用量、ビメキズマブ320mgにランダムに割り付け(1:1:1:1)、4週間隔で12週間皮下投与しました。12週間後、プラセボ群およびビメキズマブ16mg群に割り付けられた患者さんは、ビメキズマブ160mgまたは320mgのいずれかにランダムに(1対1で)再割り付けされ、その他の患者さんは48週間まで最初に割り当てられた用量を継続しました。参加者と研究者の双方は、最初の12週間は治療割り付けについて盲検化され、その後はビメキズマブの投与量について盲検化されました。主要評価項目は、12週目に米国リウマチ学会の奏功基準を50%以上改善した患者の割合とし、少なくとも1回の試験治療を受け、ベースライン時に有効な主要評価項目の測定値があったすべての患者を対象に評価されました。なお、本試験は、すべてのフォローアップを含めて終了しています。本試験はClinicalTrials. gov、NCT02969525に登録されています。 【FINDINGS】2016年10月27日から2018年7月16日の間に、308人の患者がスクリーニングされ、206人がランダムに割り付けられた。プラセボ群に42名、ビメキズマブ4群に各41名が割り付けられた。12週時点で、プラセボ群と比較して、ビメキズマブ16mg群(オッズ比[OR]4-2[95%CI 1-1-15-2];p=0-032) 、ビメキズマブ160mg群(8-1[2-3-28-7];p=0-0012)、ビメキズマブ160mg(ロード用量)群でACR50反応を獲得した患者は、9-7[2-7-34-3];p=0-0004 )と有意差があった。12週時点で、プラセボ群42例中24例(57%)、ビメキズマブ群164例中68例(41%)が、治療上問題となる有害事象を報告しました。これらの有害事象の多くは軽度または中等度であった。重篤な治療上緊急の有害事象は9名に発生し、そのうち8名はビメキズマブ投与を受けていました。ビメキズマブ16mgおよび160mgの投与(320mgのローディング用量の有無にかかわらず)は、プラセボと比較してACR50を有意に改善し、安全性プロファイルも許容範囲内でした。この結果は、関節症性乾癬の治療薬としてのビメキズマブの第3相試験を支持するものである。 【資金提供】UCB Pharma. 第一人者の医師による解説 生物学的製剤の種類が増えれば 他の製剤からの変更含め治療選択肢が拡大 山本 俊幸 福島県立医科大学医学部皮膚科教授 MMJ.August 2020;16(4) 乾癬性関節炎(PsA)に対する生物学的製剤は、新しい薬剤ほど重篤な副作用が減り、投与間隔も長くなり、自己注射可能なものもある。皮膚症状の改善効果は腫瘍壊死因子(TNF)阻害薬よりもインターロイキン(IL)-17やIL-23を標的とした抗体薬の方が高く、逆に重篤な関節症状への効果はTNF阻害薬の方が高いと考えられることが多かった。 欧州リウマチ学会(EULAR)ガイドラインでTNF阻害薬が第1選択になっている理由の1つは長期エビデンスの存在であるが、近年抗IL-17抗体薬でも長期エビデンスが出てきている。IL-17サブタイプ(A~ F)のうち乾癬皮疹部ではA、C、Fの発現が亢進している。IL-17 A、IL-17 FはTh17細胞以外に自然免疫担当細胞からも産生される。関節滑膜局所ではIL-17 Fの方が強く発現していると報告されている(1)。PsAの付着部炎ではIL-17が中心的な役割を果たしており、特徴的とされる骨病変(altered bone remodeling)の骨新生にも骨びらんにもIL-17が重要な役割を担っている(2)。 従来のIL-17阻害薬は主にIL-17 Aを標的としていたが、今回、IL-17 AとIL-17Fを同時に抑える抗体薬ビメキズマブのPsAに対する有効性がランダム化二重盲検プラセボ対照用量範囲設定試験で検討された。欧米6カ国41施設で成人発症の活動性PsA患者206 人がプラセボ群、実薬群(16 mg、160 mg、初回のみ320 mgローディングする160 mg、320 mg)の5 群に割り付けられ、4 週間ごとの皮下投与を12週受けた。 主要評価項目である12週時点のACR50%改善の達成率は、ビメキズマブ16mg群、160mg (ローディングあり)群のほうがプラセボ群よりも有意に高かったが、320mg群では有意差がなかった。12週時点の有害事象はプラセボ群57%、ビメキズマブ群41%に発現し、多くは軽度~中等度であった。重症な有害事象は9人に発現し、うち8人はビメキズマブ群であった。自殺念慮は1人にみられた。死亡、炎症性腸疾患、ブドウ膜炎、心血管疾患、アナフィラキシーはみられなかった。IL-17の有害事象に好中球減少症や感染症(細菌・真菌感染)があるが、IL-17 A、IL-17Fの同時抑制によってそれらのリスクが高まることはなかった。 今回高用量(320mg)群で有意な有効性がみられなかった要因として、著者らは患者集団が圧痛関節数をはじめ活動性の高い患者が多かった可能性を挙げている。生物学的製剤の種類が増えることで、他の生物学的製剤からの変更を含めた治療選択肢が広がる。今後、適正用量、長期的な持続効果、免疫原性、有害事象などを含めたさらなる検討が必要である。 1. Van Baarsen LG et al. Arthritis Res _ er. 2014;16(4):426. 2. Schett G et al. Nat Rev Rheumatol. 2017;13(12):731-741.
女性と比較した男性の心不全治療薬至適用量の特定 前向き観察コホート研究
女性と比較した男性の心不全治療薬至適用量の特定 前向き観察コホート研究
Identifying optimal doses of heart failure medications in men compared with women: a prospective, observational, cohort study Lancet. 2019 Oct 5;394(10205):1254-1263. doi: 10.1016/S0140-6736(19)31792-1. Epub 2019 Aug 22. 原文をBibgraph(ビブグラフ)で読む 上記論文の日本語要約 【背景】左室駆出率が低下した心不全(HFrEF)患者にガイドラインが推奨するアンジオテンシン変換酵素(ACE)阻害薬またはアンジオテンシン受容体拮抗薬(ARB)およびβ遮断薬の用量は、薬剤の薬物動態に性差があることが知られているにもかかわらず、男女でほぼ同じである。著者らは、HFrEF患者に用いるACE阻害薬またはARBおよびβ遮断薬の至適用量に性差があると仮定を立てた。 【方法】欧州11カ国で実施した前向き試験BIOSTAT-CHFの事後解析を実施した。この試験は、手順従ってACE阻害薬またはARBおよびβ遮断薬の投与開始と用量漸増を推奨された心不全患者を対象としたものである。著者らは、左室駆出率が40%未満の患者のみを対象とし、試験開始3カ月以内に死亡した患者を除外した。あらゆる原因による死亡または心不全による入院までの期間の複合を主要転帰とした。男性3539例、女性961例のHFrEF患者から成る独立コホートASIAN-HFで結果を検証した。 【結果】BIOSTAT-CHFのHFrEF患者(男性1308例、女性402例)で、女性のほうが男性よりも高齢(74歳vs. 70歳、P<0.0001)、低体重(72kg vs. 85kg、P<0.0001)、低身長(162cm vs. 174cm、P<0.0001)だったが、BMIに有意な差はなかった。ほぼ同じ割合の男女でガイドラインが推奨するACE阻害薬またはARB[99例(25%)vs. 304例(23%)、P=0.61]およびβ阻害薬[57例(14%) vs. 168(13%)例、P=0.54]の目標用量に達していた。死亡または心不全による入院のハザード比が最も低かったのは、男性ではACE阻害薬またはARBおよびβ遮断薬の推奨用量100%を服用していた患者であったが、女性は推奨用量のわずか50%でリスクが約30%も低く、増量してもリスクはそれ以上低下しなかった。この性差は、年齢、体表面積などの変数で補正した後もなお認められた。ASIAN-HFレジストリでは、ACE阻害薬およびβ遮断薬いずれもほぼ同じパターンが見られ、女性は推奨用量の50%でリスクが約30%低く、増量してもそれ以上の便益は見られなかった。 【解釈】この試験から、HFrEF女性患者は男性よりもACE阻害薬またはARBおよびβ遮断薬を減量する必要があることが示唆される。これは、男性と比較した女性の真の至適薬物療法が何であるかという問題を提起するものである。 第一人者の医師による解説 男女により標準治療薬の至適用量に差があることに注意が必要 瀧本 英樹 東京大学医学部附属病院循環器内科講師 MMJ. October 2020; 16 (5):133 左室駆出率(LVEF)が低下した心不全患者に対するガイドラインに準じた治療(guideline-directed medical therapy;GDMT)では現在、アンジオテンシン変換酵素(ACE)阻害薬、アンジオテンシン受容体拮抗薬(ARB)、β遮断薬を少量から導入し、性別関係なく推奨量まで漸増する。しかし、薬物の体内分布、代謝、病態には性差があり、至適用量は男女で異なる可能性が高い。  本論文はこの仮説を、欧州11カ国のBIOSTATCHF試験 の 事後解析により検証、さらにアジア11地域のASIAN-HF試験で検証した。BIOSTATCHF試験はGDMTが奏効しない要因を調べる前向き観察研究であり、2010~12年に患者登録が行われた。ACE阻害薬 /ARB、β遮断薬投与が不十分な心不全患者について、登録後3カ月間に治療薬を増量・最適化、その後6カ月間は用量を維持して評価した(追跡期間の中央値は21カ月)。対象となったLVEF 40%未満の男性1,308人と女性402人をβ遮断薬、ACE阻害薬 /ARBの推奨量到達度で4群(0%、1~49%、50~99%、100%以上)にわけて全死亡と心不全入院の複合エンドポイントを評価すると、β遮断薬、ACE阻害薬 /ARBとも女性では50~99%、男性では100%以上で最もリスクが低かった。推奨量到達度と相対リスクの関係を解析すると、β遮断薬に関して女性では推奨量の60%でリスクが最も低いU字型曲線を示し、男性では30~100%でリスクが低かった。ACE阻害薬 /ARBに関して、女性では推奨量の40%で最もリスクが低く、それ以上で低下しなかったが、男性は増量に伴ってリスクが低下し、推奨量100%で最もリスクが低くなった。  ASIAN-HF試験に登録されたLVEF40%以下の男性3,539人、女性961人を対象とした解析でもほぼ同様の結果であり、女性においてβ遮断薬は推奨量の40~50%、ACE阻害薬 /ARBは推奨量の60%で十分な相対リスク低下効果を得ることができ、それ以上でリスク低下を認めなかった。一方、男性ではβ遮断薬は推奨量100%で最も相対リスクが低く、ACE阻害薬 /ARBは推奨量50%以上でリスク低下が得られた。  すなわち、女性の心不全治療において、これら薬物は人種によらず推奨量の約半量が適切であることが示された。この機序については検討されていないが、薬物の代謝、血中濃度、副作用、病態形成の性差が考えられよう。これからは性差を考慮した医療を適切に提供することが求められている。
救急外来を受診した急性心房細動患者に用いる電気的除細動と薬物的除細動の比較(RAFF2) 部分要因無作為化試験
救急外来を受診した急性心房細動患者に用いる電気的除細動と薬物的除細動の比較(RAFF2) 部分要因無作為化試験
Electrical versus pharmacological cardioversion for emergency department patients with acute atrial fibrillation (RAFF2): a partial factorial randomised trial Lancet. 2020 Feb 1;395(10221):339-349. doi: 10.1016/S0140-6736(19)32994-0. 原文をBibgraph(ビブグラフ)で読む 上記論文の日本語要約 【背景】急性心房細動は、救急部で治療する最も頻度の高い不整脈である。著者らの主要目的は、電気駅除細動前の薬物による除細動(薬物ショック)と電気的除細動単独(ショック単独)の洞調律復帰を比較することであった。第二の目的は、電気的除細動時の2つのパッド位置の有効性を比較することであった。 【方法】カナダの大学病院11施設の救急部で、急性心房細動患者を対象とした2通りのプロトコールを用いた部分要因試験を実施した。成人急性心房細動患者を組み入れた。プロトコール1では、プロカインアミド静注(15mg/kgを30分かけて投与)による薬物的除細動後の必要に応じた電気的除細動実施(3回まで、いずれも200J以上)とプラセボ投与後の電気的除細動実施を無作為化プラセボ対照盲検下で比較した。電気的除細動を実施する際、前後と前外側のパッド位置を無作為化非盲検コホート内で比較するプロトコール2を用いた。プロトコール1では、オンライン電子データ収集システムを用いて現場の研究員が患者を無作為に(1対1の割合、試験施設で層別化)割り付けた。薬物投与30分後に洞調律に復帰しない患者をプロトコール2で無作為化し、試験施設とプロトコール1の割り付けで層別化した。患者を全研究員と救急部職員にプロトコール1の治療割り付けを伏せた。主要転帰は、無作為化後30分以降および3回のショック直後の正常洞調律復帰とした。プロトコール1はintention-to-treatで解析し、プロトコール2は電気的除細動を実施しなかった患者を除外した。この試験はClinicalTrials.govにNCT01891058番で登録されている。 【結果】2013年7月18日から2018年10月17日にかけて、396例を組み入れ、追跡脱落例はなかった。薬剤ショック群(204例)では196例(96%)、ショック単独群(192例)では176例(92%)が洞調律に復帰した(絶対差4%、95%CI 0-9、P=0.07)。退院した患者の割合は、97%(198例)と95%(183例)だった(P=0.60)。薬物ショック群の106例(52%)が、薬物注入単独で洞調律に復帰した。追跡中、重度の有害事象が発現した患者はいなかった。プロトコール2(244例)の2通りのパッド位置の洞調律復帰率は同等であった[前後群127例中119例(94%)と前外側群117例中108例(92%)]。 【解釈】救急部を受診した急性心房細動患者で、薬物ショックとショック単独戦略ともに洞調律復帰の有効性が高く、迅速で安全で、再受診を回避できた。薬物注入は、約半数に有効であり、電気的除細動を要する資源集約的な処置時の鎮静が回避できた。このほか、電気的除細動時の前後と前外側のパッド位置による有意な差は見られなかった。救急部を受診する急性心房細動の即時洞調律維持が良好な転帰をもたらすことになる。 第一人者の医師による解説 Ic群抗不整脈薬の効果はより高いと推測 日本の救急現場での治療方針は妥当 井上 博 富山県済生会富山病院顧問 MMJ. October 2020; 16 (5):127 心房細動発作で救急受診した患者では、血圧低下や心不全悪化など特別な事情がなければ、まず抗不整脈薬を静注し30分ほど様子をみて、洞調律化しない場合は電気的除細動を試みることが通例である。この2段階のアプローチは臨床現場では特に疑問視されることなく行われているが、最初から電気的除細動を試みるアプローチと厳密に比較した成績は乏しい。  本試験は、カナダの大学病院救急外来11施設で、2013年7月~18年10月に行われた無作為対照試験(RAFF2)である。心房細動が3時間以上持続し、除細動を必要とする、状態の安定した成人患者396人(平均年齢60歳)を対象とした。プロトコール1では、プロカインアミド 15mg/kg(最大1,500mg)を30分で静注し無効な場合に電気的除細動(二相性波形、200J以上で3回まで)を行うdrug-shock群(204人)と最初から電気的除細動を行うshock群(192人)に無作為化した。プロトコール2では、プロトコール1で電気的除細動を行う患者を電極配置が前-後(右鎖骨下と左肩甲骨下)と前-外側(右鎖骨下と左前腋窩)の2群に無作為化した。その結果、除細動され洞調律が30分以上持続した患者は、drug-shock群で196人(96%)、shock群で176人(92%)と両群間で有意差はなかった(P=0.07)。Drug-shock群の52%では、プロカインアミド静注のみで静注開始から中央値で23分後に洞調律化した。14日間の追跡期間終了時点で306人(77%)が洞調律であり、重篤な有害事象や脳卒中の発生はみられなかった。2種類の電極配置の間で洞調律化率に差はなかった(前-後群92% 対 前-外側群94%;P=0.68)。  発症後間もない心房細動発作を救急外来で除細動する場合、薬理学的除細動+電気的除細動あるいは電気的除細動のみを行う方法のいずれも、高い有効性と安全性を示した。薬理学的除細動のみで約半数の患者で洞調律化が可能であったことから、臨床現場ではまず薬理学的除細動を試み、無効な場合に電気的除細動を行う方針が効果的と考えられる。電極の配置は洞調律化率には影響しない。  以上の結果は、日本の救急現場で経験的に行われる治療方針が妥当であることを無作為化対照試験で改めて示したものである。注意すべき点は、わが国では心房細動の洞調律化にはプロカインアミドが使用されることはまれで、Ic群抗不整脈薬などが選択されることが多く、薬理学的除細動率は本試験の成績より高いと推測される。また、除細動とは別の治療方針として、発症後間もない心房細動発作は薬物による心拍数コントロールのみで48時間以内に69%が洞調律化するという報告(1)もあり、救急現場ではこれらの成績も考慮に入れて治療方針を選択することが望ましい。 1. Pluymaekers NAHA et al. N Engl J Med 2019;380(16):1499-1508.
1990年から2017年における慢性腎疾患の世界的、地域的、および国家的負担:世界疾病負担研究2017のシスティマティック分析
1990年から2017年における慢性腎疾患の世界的、地域的、および国家的負担:世界疾病負担研究2017のシスティマティック分析
Global, regional, and national burden of chronic kidney disease, 1990-2017: a systematic analysis for the Global Burden of Disease Study 2017 Lancet. 2020 Feb 29;395(10225):709-733. doi: 10.1016/S0140-6736(20)30045-3. Epub 2020 Feb 13. 原文をBibgraph(ビブグラフ)で読む 上記論文の日本語要約 背景: 医療制度を計画する上で、慢性腎疾患(CKD)の疫学を注意深く評価する必要がありますが、CKDの罹患率と死亡率に関するデータは、多くの国で不足しているか、存在していません。私たちは、2017年の世界疾病負担、負傷、および危険因子研究について、CKDの世界的、地域的、および全国的な負担、ならびに腎機能障害に起因する心血管疾患および痛風の負担を推定しました。全てのステージのCKDの罹患率と死亡率を調べるためにCKDという用語を使用し、心血管疾患と痛風によるCKDの追加リスクを調べるために腎機能障害という用語を使用しました。 方法: 私たちが使用した主なデータソースは、公開された文献、人口動態登録システム、末期腎疾患登録、および世帯調査でした。CKD負担の推定値は、死因アンサンブルモデル(Cause of Death Ensemble model)とベイズのメタ回帰分析ツールを使用して算出されました。ここでは、発生率、有病率、障害のあった年数、死亡率、死亡年、および障害調整生命年(DALY)を組み込みました。腎機能障害に起因する心血管疾患と痛風による負担の割合の推定には、比較リスク評価アプローチを適用しました。 結果: 世界的に、2017年に120万人(95%の不確実性区間[UI] 120万~130万)がCKDで死亡しました。CKDによる世界の全年齢死亡率は、1990年から2017年の間に41.5%(95%UI 35.2~46.5)増加しましたが、年齢標準化死亡率に有意な変化は認められませんでした(2.8%、 -1.5~6.3)。 2017年には、全ステージのCKDで6億9,750万症例(95%UI 6億4,920万~7億5,200万)が記録され、世界的な有病率は9.1%(8.5~9.8)でした。CKDの世界的な全年齢有病率は、1990年以降29.3%(95%UI 26.4~32.6)増加しましたが、年齢標準化された有病率は横ばいでした(1.2%、-1.1から3.5)。CKDのDALYは、2017年に3,580万(95%UI 3,370万~3,800万)となり、糖尿病性腎症がほぼ3分の1を占めました。CKDの負担のほとんどは、社会人口統計指数(SDI)の五分位の下位3つに集中していました。いくつかの地域、特にオセアニア、サハラ以南のアフリカ、ラテンアメリカでは、CKDの負担は経済発展レベルから予想されるよりもはるかに高かったのに対し、サハラ以南の西、東、中央アフリカ、東アジア、南アジア、中央および東ヨーロッパ、オーストラリア、西ヨーロッパでは、負担は予想を下回っていました。心血管疾患関連では140万例(95%UI 120万~160万)の死亡と2,530万(2,220万~2,890万)の心血管疾患DALYが、腎機能障害に起因していると考えられました。 解釈: 腎疾患は、世界的な病的状態と死亡の直接的な原因として、そして心血管疾患の重要な危険因子として、健康に大きな影響を及ぼします。殆どのCKDは予防と治療が可能であり、特にSDIが低/中程度の地域では、グローバルヘルスポリシーの意思決定において、より大きな注意を払う必要があります。 研究資金: Bill & Melinda Gates Foundation. 第一人者の医師による解説 一般の認知度は10%未満 世界的な認知度向上と早期発見体制の確立が必要 山縣 邦弘 筑波大学医学医療系腎臓内科学教授 MMJ. October 2020; 16 (5):140 慢性腎臓病(CKD)は末期慢性腎不全(ESKD)への進展だけでなく心血管死の重要な危険因子であることから、全世界的に疾病対策の重要性が唱えられてきた。本論文では、全世界で7億人以上が罹患し、年間120万人以上が死亡するCKDについて、障害調整生存年(DALYs)という指標から論じている。DALYsによる疾患の評価については、世界保健機関(WHO)のGlobal Burden of Disease (GBD)プロジェクトが指標として使用し、GBDでは現在133疾患について検討が進められている。CKDはGBD対象疾患における死因の12番目を占め、結核や後天性免疫不全症候群(AIDS)による死亡よりも多く、自動車事故死と同等とされている。近年世界的にESKD患者数が増加し、腎代替療法(透析や腎移植療法)を必要とする患者数が今後も増加すると予測されている。このことは先進国を中心に、国民所得が高い国々の集計結果をもとに検討されてきた。しかし近年では、発展途上国でも高血圧や糖尿病などによるCKD発症・進展リスクにさらされるようになり、ESKD患者数は全世界的に急速に増加している。毎年200万人以上の患者が新たに腎代替療法を開始しているものの、ほぼ同数の患者が腎代替療法を受けることができずに死亡してい るという事実も知られている(1)。  本論文では腎代替療法導入やCKD罹患に伴う負担と、腎代替療法非導入を含めたCKDのために死亡することによる負担を合わせた数値指標であるDALYsを地球規模、地域別、国別に算出することによって、今後のCKD対策の方向性を検討した。過去27年間で、CKD患者の罹病率、死亡率とも有意に上昇しており、この理由として人口の高齢化や、糖尿病、高血圧といったCKD発症の危険因子保有者の増加が挙げられた。一方、人口の高齢化の影響を考慮した年齢標準変動率では、全世界でCKDによる人口あたりのDALYsは過去27年間で8.6%低下していたが、CKDにおけるこの低下は他の非感染性疾患(NCD)に比べ少なく、CKD対策が不十分であることを示していた。特に発展途上国を中心に経済的に困窮した地域では先進国に比べ15倍以上 DALYsが高い。このような中で一般人におけるCKDの認知度は先進国、発展途上国とも同様に10%未満で認知度が極めて低い。また早期のCKDを発見し、進行を抑制することは医療経済的にも有用性が証明されているものの、各国のCKDスクリーニング体制が不十分であり、CKDの早期発見、腎機能が高度に悪化する前での治療体制の確立が世界的に求められている。また透析や腎移植といった腎代替療法をどの国でも同等に実施できる施設面での整備を急ぐことも重要な課題として挙げられた。 1. Liyanage T, et al. Lancet. 2015;385(9981):1975-1982.
70歳以上の非ST上昇型急性冠症候群患者に用いるクロピドグレルのチカグレロルまたはプラスグレルとの比較 無作為化非盲検非劣性試験
70歳以上の非ST上昇型急性冠症候群患者に用いるクロピドグレルのチカグレロルまたはプラスグレルとの比較 無作為化非盲検非劣性試験
Clopidogrel versus ticagrelor or prasugrel in patients aged 70 years or older with non-ST-elevation acute coronary syndrome (POPular AGE): the randomised, open-label, non-inferiority trial Lancet. 2020 Apr 25;395(10233):1374-1381. doi: 10.1016/S0140-6736(20)30325-1. 原文をBibgraph(ビブグラフ)で読む 上記論文の日本語要約 【背景】現行ガイドラインでは、急性冠症候群後の患者のチカグレロルまたはプラスグレルを用いた強力な抗血小板療法が推奨されている。しかし、高齢者の最適な抗血小板阻害に関するデータが不足している。著者らは、非ST上昇型急性冠症候群(NSTE-ACS)高齢患者に用いるクロピドグレルのチカグレロルまたはプラスグレルと比較した安全性および有効性を明らかにすることを試みた。 【方法】オランダの12施設(病院10施設および大学病院2施設)で非盲検無作為化試験POPular AGEを実施した。70歳以上のNSTE-ACS患者を組み入れ、ブロックサイズを6としたインターネットを用いた無作為化法で、クロピドグレル300mgまたは600mg、チカグレロル180mgまたはプラスグレル60mgの負荷投与の後、標準治療と併用した12カ月間の維持投与(クロピドグレル1日1回75mg、チカグレロル1日2回90mg、プラスグレル1日1回10mgのいずれか)に1対1の割合で無作為化に割り付けた。患者と治療担当医師に治療の割り付けを知らせておいたが、結果評価者には治療の割り付けを伏せた。主要出血転帰を血小板凝集阻害と患者転帰[PLATelet inhibition and patient Outcome(PLATO):大出血または小出血(優越性の仮説)]とした。全死因死亡、心筋梗塞、脳卒中、PLATO大出血および小出血(非劣性の仮説、マージン2%)を主要複合評価項目(ネットクリニカルベネフィット)とした。追跡期間は12カ月間であった。intention-to-treat集団を解析対象とした。この試験はNetherlands Trial Register(NL3804)、ClinicalTrials.gov(NCT02317198)およびEudraCT (2013-001403-37)に登録されている。 【結果】2013年7月10日から2018年10月17日の間に、1002例をクロピドグレル(500例)、チカグレロルまたはプラスグレル(502例)に無作為に割り付けた。チカグレロルまたはプラスグレル群の475例(95%)にチカグレロル投与したため、このグループをチカグレロル群とした。チカグレロルに割り付けたチカグレロル群502例中238例(47%)、クロピドグレルに割り付けた500例中112例(22%)が早期中止に至った。クロピドグレル群[500例中88例(18%)]の大出血がチカグレロル群[502例中118例(24%)]よりも低かった(ハザード比0.71、95%CI 0·54~0·94、優越性のP=0.01)。複合評価項目はクロピドグレル群のチカグレロルに対する非劣性が示された[139例(28%) vs. 161例(32%)、絶対リスク差-4%、95%CI -10.0~1.4、非劣性のP=0.03]。最も重要な中止の理由は、出血(38例)、呼吸困難(40例)および経口抗凝固薬を用いた治療の必要性(35例)であった。 【解釈】NSTE-ACSを呈した70歳以上の患者で、クロピドグレルは全死因死亡、心筋梗塞、脳卒中および出血の複合評価項目が増加することなく出血イベントを抑制するため、チカグレロルに取って代わる有効な選択肢である。クロピドグレルは、特に出血高リスクの高齢患者のP2Y12阻害薬の代替薬になると思われる。 第一人者の医師による解説 個人差の大きい高齢者 複合リスクの見極めが肝要 中村 正人 東邦大学医療センター大橋病院循環器内科教授 MMJ. October 2020; 16 (5):129 PLATO試験、TRITON-TIMI38試験の結果から急性冠症候群に対しては強力な抗血小板薬(チカグレルまたはプラスグレル)が欧米のガイドラインではクラス Iで推奨されている。出血リスクを上回る虚血イベント抑制効果がこれらの試験で示されたからである。しかし、出血リスクの高い患者にこの治療戦略が有効であるかどうかは明らかではない。そこで近年、出血リスクの高い患者を対象に出血リスクを軽減させるさまざまな戦略の妥当性が検証されている。70歳以上の高齢者に対する今回のPOPular AGE試験も同様である。非 ST上昇型心筋梗塞という血栓イベントリスクの高い病態に出血を考慮したDAPT(クロピドグレル)と血栓イベントを優先するDAPT(プラスグレルまたはチカグレル)の優劣が比較された。主要エンドポイントである出血の発生率は抗血小板作用の弱いクロピドグレルによるDAPTの方が低かった。この結果は想定範囲内である。ポイントは血栓イベントを加えた複合エンドポイントで非劣性が示された点にある。ランダム化前に98%の患者がローディングされており、その7割がチカグレルであった点、イベントの内訳としてステント血栓症はクロピドグレル群のみで認められた点は留意すべきであるが、総合的にみてクロピドグレルによるDAPTの妥当性が実証されたと結論されている。  しかし、他にも抗血小板療法をde-escalationさせる策としては、遺伝子多型や血小板凝集能をチェックする方法、短期 DAPT とP2Y12阻害薬単剤の組み合わせ、低用量のP2Y12阻害薬によるDAPTなどがある。このため、本研究で高齢者に対する戦略の結論が得られたとは言い難い。現在最もエビデンスが豊富な戦略は短期 DAPTとP2Y12阻害薬単剤の組み合わせである。実際、出血高リスク例に対しこの戦略が日本のガイドラインでは推奨されている(1)。また、日本ではプラスグレルの用量は海外の3分の1で、出血リスクが考慮されている。この点からも本研究の結果を日本の実臨床へ外挿する場合にはさらなる検証が必要である。近年、Academic Research Consortium(ARC)により出血高リスク(HBR)の定義が提唱され、これによると75歳以上 の 高齢者はminor criteriaに該当する。単独ではなく複合でHBRに分類される(2)。高齢者を一律 HBRと定めることはできない。高齢者は個人差が大きく、他の出血リスク因子の有無を見極めることが肝要である。わが国をはじめとした東アジア諸国は欧米諸国より出血リスクが高いとされる(3)。このため高齢化社会を迎えている日本における独自の検討が必要である。 1. Nakamura M, et al. Circ J. 2020;84(5):831-865. 2. Urban P, et al. Eur Heart J. 2019;40(31):2632-2653. 3. Levine GN, et al. Nat. Rev. Cardiol. 2014; 11: 597-606.
急性期脳梗塞治療に用いるnerinetideの有効性および安全性(ESCAPE-NA1試験) 多施設共同二重盲検無作為化対照試験
急性期脳梗塞治療に用いるnerinetideの有効性および安全性(ESCAPE-NA1試験) 多施設共同二重盲検無作為化対照試験
Efficacy and safety of nerinetide for the treatment of acute ischaemic stroke (ESCAPE-NA1): a multicentre, double-blind, randomised controlled trial Lancet. 2020 Mar 14;395(10227):878-887. doi: 10.1016/S0140-6736(20)30258-0. Epub 2020 Feb 20. 原文をBibgraph(ビブグラフ)で読む 上記論文の日本語要約 【背景】nerinetideは、シナプス後肥厚タンパク質95を阻害するエイコサペプチドで、前臨床の脳梗塞モデルで有効性が確認されている神経保護薬である。この試験では、急性期脳梗塞患者の急速な血管内血栓除去に伴って起きるヒトの虚血・再灌流に用いるnerinetideの有効性および安全性を評価した。 【方法】8カ国の急性期病院48施設で実施された多施設共同二重盲検プラセボ対照無作為化試験では、大血管閉塞による急性期脳梗塞発症12時間以内の患者を組み入れた。無作為化時点で後遺症を伴う18歳以上の脳梗塞があり、発症前は地域で自立した生活を送っており、Alberta Stroke Program Early CTスコア(ASPECTS)5点以上で、側副血行路の充満度が中等度ないし良好な(多相CT血管造影で判定) 患者を適格とした。インターネットを用いたリアルタイムの動的、層別、最小化法を用いて、被験者をnerinetide 2.6mg/kg、推定または(分かれば)実際の体重を基に最大用量270mgを静脈内単回投与するグループと生理食塩水のプラセボを投与するグループに1体1の割合で無作為に割り付けた。アルテプラーゼ静脈内投与と申告された血管内デバイスの選択で被験者を層別化した。全試験担当者と被験者には順序と割り付けを伏せておいた。全例に血管内血栓除去を実施し、適応があれば標準治療としてアルテプラーゼを投与した。主要転帰は無作為化90日後の良好な機能的転帰とし、修正ランキン尺度(mRS)スコア0~2点と定義した。神経学的障害、日常生活行動の機能的自立、きわめて良好な機能的転帰(mRS 0~1点)および死亡を副次評価項目とした。解析は、intention-to-treat集団で実施し、年齢、性別、試験開始時のNational Institutes of Health Stroke Scaleスコア、ASPECTS、閉塞部位、施設、アルテプラーゼ投与の有無および申告された最初のデバイスで調整した。安全性評価集団は、被験薬を投与した全例を対象とした。この試験は、ClinicalTrials.govにNCT02930018番で登録されている。 【結果】2017年3月1日から2019年8月12日にかけて、1105例をnerinetide(549例)とプラセボ(556例)に無作為に割り付けた。nerinetide群549例中337例(61.4%)およびプラセボ群556例中329例(59.2%)が90日時にmRSスコア0~2点を達成した(調整後リスク比1.04、95%CI 0.96~1.14、P=0.35)。副次転帰は両群でほぼ同じだった。治療効果の修飾の根拠が認められ、アルテプラーゼを投与した患者で治療効果が阻害された。重度有害事象が両群でほぼ同じ割合で発現した。 【解釈】nerinetideによって、血管内血栓除去後に良好な臨床的転帰を得た患者の割合がプラセボと比較して、改善することがなかった。 第一人者の医師による解説 アルテプラーゼ非使用例で機能予後改善と脳梗塞容積減少 今後の研究に期待 上坂義和 虎の門病院脳卒中センターセンター長 MMJ. October 2020; 16 (5):136 再潅流療法が現在の脳梗塞急性期治療の大きな柱である。しかし、虚血や再潅流障害からの脳保護を目指した脳保護薬の開発も種々試みられてきた。ネリネチド(nerinetide)は興奮毒性をもたらすシグナル伝達を抑制することで神経保護作用を有することが動物実験で示されている。  今回報告されたESCAPE-NA1試験は、前方循環大血管閉塞に血栓回収療法を実施した患者におけるネリネチドの有効性を検討するために、欧米6カ国、オーストラリア、韓国で実施された多施設共同二重盲検ランダム化試験である。対象は、発症前に障害はなく、National Institutes of Health Stroke Scale(NIHSS)スコア6点以上、18歳以上、発症後12時間以内でCT血管造影法にて頭蓋内頸動脈か中大脳動脈水平部に閉塞が確認された脳梗塞急性期患者である。Alberta Stroke Program Early CT Score(ASPECTS)スコア5点以上で軟膜動脈経由の側副血行が中大脳動脈領域の50%以上にみられる比較的側副血行良好な1,105人が組み入れられた。全例で血管内治療による血栓回収療法(EVT)が行われた。アルテプラーゼ静注療法は臨床的判断に基づき実施された。患者はネリネチド群とプラセボ(生食)群にアルテプラーゼ投与有無も均等化されるようランダムに割り付けられた。主要評価項目は発症90日後の修正 Rankin Scale(mRS)0~2の 割合で、2次評価項目としてmRS 0~1の割合、死亡、3次評価項目として脳梗塞容積が解析された。その結果、mRS 0~2はネリネチド群で61.4%、プラセボ群で59.2%、mRS 0~1はそれぞれ40.4%、40.6%、死亡率は12.2%、14.4%、平均脳梗塞容積は71.1mL、73.1mLといずれも有意差を認めなかった。しかしながら、アルテプラーゼ非投与患者446人の解析では、mRS 0~2はネリネチド群で59.3%、プラセボ群で49.8%、死亡率はそれぞれ12.8%、20.3%、平均脳梗塞容積は67.8mL、87.2mLといずれも有意差を認めた。  ネリネチドはアルテプラーゼの作用に影響を与えないが、アルテプラーゼ投与で生じるプラスミンによって開裂されるアミノ酸配列を有する。本研究でもネリネチドの血中ピーク濃度はアルテプラーゼ投与患者で非投与患者の40%程度に低下しており結果に影響を与えた可能性がある。発症から割り付けまでの時間はアルテプラーゼの適応を反映して非投与群で平均270~275分、投与群で152~161分と非投与群では114~118分遅くなっている。発症からやや時間が経過した患者などのアルテプラーゼ非適応患者におけるネリネチドの脳保護作用を期待させるものであり、今後の研究が待たれる。
活動性強直性脊椎炎に用いるupadacitinibの有効性および安全性(SELECT-AXIS 1) 多施設共同第II/III相二重盲検プラセボ対照無作為化試験
活動性強直性脊椎炎に用いるupadacitinibの有効性および安全性(SELECT-AXIS 1) 多施設共同第II/III相二重盲検プラセボ対照無作為化試験
Efficacy and safety of upadacitinib in patients with active ankylosing spondylitis (SELECT-AXIS 1): a multicentre, randomised, double-blind, placebo-controlled, phase 2/3 trial Lancet. 2019 Dec 7;394(10214):2108-2117. doi: 10.1016/S0140-6736(19)32534-6. Epub 2019 Nov 12. 原文をBibgraph(ビブグラフ)で読む 上記論文の日本語要約 【背景】JAK経路は強直性脊椎炎の治療の標的となる可能性がある。この試験では、強直性脊椎炎に用いる選択的JAK1阻害薬upadacitinibの有効性および安全性を評価した。 【方法】この多施設共同第II/III相二重盲検プラセボ対照並行群間無作為化試験には20カ国62施設から成人患者を組み入れた。改訂ニューヨーク基準を満たし、生物学的疾患修飾性抗リウマチ薬による治療歴がなく、非ステロイド系抗炎症薬(NSAID)が2種類以上で効果不十分、NSAID不耐または禁忌の活動性強直性脊椎炎患者を適格とした。被験者を自動応答技術を用いて、第1期の14週間に経口upadacitinib 15mg 1日1回服用するグループまたは経口プラセボを服用するグループに無作為に割り付けた。ここでは第1期のデータのみを報告する。主要評価項目は、14週時のAssessment of SpondyloArthritis international Society (ASAS) 40反応率で測定した複合転帰とした。無作為化し被験薬を1回以上投与した全例の完全解析を実施した。この試験は、ClinicalTrials.govのNCT03178487番に登録されている。 【結果】2017年11月30日から2018年10月15日にかけて、187例をupadacitinib 15mg(93例)とプラセボ(94例)に無作為に割り付け、178例(95%、upadacitinib群の89例とプラセボ群の89例)が被験薬の第1期を完遂した(2019年1月21日までに完了)。upadacitinib群のほうがプラセボ群よりも14週時のASAS40反応率が有意に高かった[92例中48例(52%) vs. 94例中24例(26%)、P= 0.0003、治療群間差26%(95%CI 13~40)]。upadacitinib群93例中58例(62%)とプラセボ群94例中52例(55%)に有害事象が報告された。upadacitinib群に最も多く発現した有害事象は、クレアチン・ホスホキナーゼ上昇(updacitinib群9% vs. プラセボ群2%)だった。重篤な感染、帯状疱疹、悪性腫瘍、静脈血栓塞栓イベントや死亡は報告されなかった。各群1例に重度の有害事象が発現した。 【解釈】upadacitinib 15mgは、NSAIDの効果不十分または禁忌の活動性強直性脊椎炎に有効で忍容性も良好であった。このデータは、強直性脊椎炎治療に用いるupadacitinibのさらに詳細な調査を支持するものである。 第一人者の医師による解説 ウパダシチニブは有効かつ耐容可能なAS治療薬になりうることを示唆 高相 晶士 北里大学医学部整形外科学主任教授 MMJ. October 2020; 16 (5):143 強直性脊椎炎(AS)の治療としては非ステロイド系抗炎症薬(NSAID)が推奨され、従来の抗リウマチ薬や糖質コルチコイドは軸性症状の改善に乏しいと言われている。近年、関節リウマチ、乾癬性関節炎、潰瘍性大腸炎の適応でJAK阻害薬が承認されており、JAK経路はASの治療標的としても着目されている。ASに対するJAK阻害薬(トファシチニブおよびフィルゴチニブ)の有効性については、2つの第2相試験で示されたところである。  本論文で報告されたSELECT-AXIS 1試験は、生物学的疾患修飾性抗リウマチ薬(bDMARD)の使用歴がなく、NSAIDが効果不十分または不耐容である18歳以上の活動性 AS患者18人を対象に、ウパダシチニブ 15mg/日を14週間経口投与した際の有効性と安全性評価を目的とした多施設共同無作為化プラセボ対照二重盲検試験である。本試験には20カ国(北米、西欧、東欧、アジア、オセアニア)の62施設が参加。対象患者はASのNew York基準を満たし、仙腸関節 X線写真で診断され、従来の抗リウマチ薬、経口ステロイド薬、NSAID使用は組み入れ可とされた。主要評価項目は投与後14週目におけるAssessment of Spondylo Arthritis international Society(ASAS)40反応を達成した患者の割合とされた。割り付け結果はウパダシチニブ群93人、プラセボ群94人、全体の背景は平均年齢45.4歳、男性71%、HLA-B27陽性率76%であった。  有効性の評価において、投与後14週目のASAS40達成率は、ウパダシチニブ群で有意に改善した(ウパダシチニブ群52% 対 プラセボ群26%;P=0.0003)。疾患活動性を示すBath Ankylosing Spondylitis Disease Activity Index(BASDAI)50 改善率、ASAS 部 分 寛 解率、Ankylosing Spondylitis Disease Activity Score(ASDAS)変化量、機能評価としてのBath Ankylosing Spondylitis Functional Index(BASFI)変化量、MRI 変化(SPARCC MRI spine score)についても、ウパダシチ二ブ群で有意な改善が認められた。  有害事象はウパダシチニブ群62%、プラセボ群55%に発生し、ウパダシチニブ群における主な有害事象はクレアチニンキナーゼの上昇であった。重篤な感染症の発生はみられず、感染症発生率および有害事象による試験中止率は両群ともに同程度であった。ウパダシチニブの安全性情報については関節リウマチに対する過去の試験で報告されたものと一致した。  今回、ウパダシチニブ 15mg経口投与により活動性 AS患者の疾患活動性、機能、MRI所見で改善を認め、忍容性も良好であったことから、本剤は有効かつ耐容可能なAS治療薬となりうることが示唆された。
ドラベ症候群の発作治療に用いる塩酸フェンフルラミン 無作為化二重盲検プラセボ対照試験
ドラベ症候群の発作治療に用いる塩酸フェンフルラミン 無作為化二重盲検プラセボ対照試験
Fenfluramine hydrochloride for the treatment of seizures in Dravet syndrome: a randomised, double-blind, placebo-controlled trial Lancet. 2019 Dec 21;394(10216):2243-2254. doi: 10.1016/S0140-6736(19)32500-0. Epub 2019 Dec 17. 原文をBibgraph(ビブグラフ)で読む 上記論文の日本語要約 【背景】ドラベ症候群は、まれな治療抵抗性の発達性てんかん性脳症であり、さまざまな種類の発作が頻繁に起こるのが特徴である。フェンフルラミンは、光過敏性発作とドラベ症候群の観察研究で抗発作作用が報告されている。この試験の目的は、ドラベ症候群患者に用いるフェンフルラミンの有効性および安全性を評価することであった。 【方法】この無作為化二重盲検プラセボ対照試験は、若年成人および小児のドラベ症候群患者を対象とした。試験開始時の1カ月のけいれん発作頻度(MCSF;明らかな運動徴候がある半側間代発作、強直発作、間代発作、強直・脱力発作、全般性強直・間代発作、焦点発作をけいれん発作と定義)を確かめる6週間の観察期間の後、ウェブ自動応答システムを用いて、患者を抗てんかん薬に上乗せしてプラセボ、フェンフルラミン1日0.2mg/kg、同薬剤1日0.7mg/kgのいずれかを14週間投与するグループに1対1対1の割合で無作為に割り付けた。主要評価項目は、プラセボ群と比較した1日0.7mg/kg投与群の試験開始時と比較した治療期間中の1カ月の平均痙攣発作頻度とした。プラセボ群と比較した1日0.2mg/kg投与群の1カ月の平均けいれん発作頻度を副次評価項目とした。修正intention-to-treat集団で解析を実施した。被験薬を1回以上投与した全例を安全性解析の対象とした。この試験は、ClinicalTrials.govに2通りのプロトコールNCT02682927とNCT02826863に登録されている。 【結果】2016年1月15日から2017年8月14日の間に173例を評価し、そのうち119例(平均年齢9.0歳、54%が男児)をフェンフルラミン1日0.2mg/kg群(39例)、同薬剤1日0.7mg/kg群(40例)、プラセボ群(40例)に無作為に割り付けた。治療期間中、発作頻度の低下率中央値はフェンフルラミン0.7mg/kg群で74.9%(28日当たりの中央値20.7回から4.7回に低下)、同薬0.2mg/kg群で42.3%(同17.5回から12.6回に低下)、プラセボ群で19.2%(同27.3回から22.0回に低下)だった。試験は主要有効性評価を達成し、プラセボと比較すると、平均MCSFがフェンフルラミン1日0.7mg/kgで62.3%(95%CI 47.7-72.8、P<0.0001)、同薬0.2mg/kgで32.4%(同6.2-52.3、P=0.0209)低下した。最も頻度の高かった有害事象(被験者の10%以上に発現し、フェンフルラミン群で頻度が高かった)は食欲減退、下痢、疲労、無気力、傾眠および体重減少であった。試験期間中の心エコー検査で、全例に生理学的正常範囲内の弁機能を確認し、肺高血圧症の徴候は見られなかった。 【解釈】ドラベ症候群で、フェンフルラミンは、プラセボよりもけいれん発作頻度を有意に抑制し、忍容性も良好で、心臓弁膜症や肺高血圧症は認められなかった。フェンフルラミンはドラベ症候群の新たな治療選択肢になると思われる。 第一人者の医師による解説 ドラベ症候群治療薬として期待も 効果・有害事象とも長期評価が必要 今井 克美 国立病院機構静岡てんかん・神経医療センター副院長・小児科 MMJ. October 2020; 16 (5):137 ドラベ症候群は1歳未満で発症し、発熱や入浴で誘発されやすい全身・半身けいれんを反復し、5分以上持続するけいれん重積状態が起こりやすく、1歳以降に発達の伸びが鈍化する難治性てんかんで、約80%の患者はNaチャネル遺伝子 SCN1Aの変異を有し、約4万人に1人が罹患するとされる。バルプロ酸、臭化物、トピラマート、スチリペントール、クロバザムなどの抗てんかん薬がよく使われるが効果不十分な場合が多く、より有効な治療法の開発が喫緊の課題である(1)。  本論文は、ドラベ症候群のけいれん発作に対するフェンフルラミンの有効性と安全性を北米、欧州西部、オーストラリアで検討した無作為化二重盲検プラセボ対照試験の報告である。臨床的に診断された2~18歳のドラベ症候群患者を対象に、服用中の抗てんかん薬は継続し、119人(平均年齢9.0歳、男性54%)がフェンフルラミン低用量0.2mg/kg、高用量0.7mg/kg、プラセボの3群に割り付けられた。前観察期間6週間に対する、割り付け後14週間におけるけいれん頻度の低下率は、高用量群74.9%、低用量群42.3%、プラセボ群19.2%で、高・低用量群ともにプラセボ群に比べ有意にけいれんが減少した。有害事象は食欲低下、下痢、易疲労性、倦怠、眠気、体重減少が10%以上にみられ、有害事象による中止は9人(高用量群6人、プラセボ群3人)であった。心エコー検査では心臓弁膜症や肺高血圧などの重篤な合併症は認められなかった。  フェンフルラミンはセロトニン作動薬で、食欲抑制目的の健康食品で使用されていた時に用量や誘導体含有などの問題に関連する心臓弁膜症、肺高血圧、肝障害が報告され、それ以降使用されなくなったが、ドラベ症候群を含む難治性てんかんにおいて著効例がベルギーから複数報告されたことから、本試験が実施された。用量を減らし品質管理を徹底して実施された本試験において、けいれんの有意な減少が示され、最長8カ月の長期安全性試験でも重篤な有害事象はなく、日本でもドラベ症候群への適応承認が期待される。今回の試験ではドラベ症候群にのみ保険適応のあるスチリペントールは併用禁忌であったが、スチリペントールとの併用でも同等の有効性と安全性が報告されている(2)。今後の課題として、けいれん抑制効果が年余にわたって継続するか、けいれん重積の頻度も低下するのか、年単位の長期服用でも心臓弁膜症、肺高血圧、著明な体重減少、肝障害を生じないか、などの検討が必要である。 1. Takayama R, et al. Epilepsia. 2014;55(4):528-538. 2. Nabbout R, et al. JAMA Neurol. 2019;77(3):300-308.
重症急性腎障害に用いる腎代替療法の遅延と早期開始 系統的レビューと無作為化試験の患者個人データのメタ解析
重症急性腎障害に用いる腎代替療法の遅延と早期開始 系統的レビューと無作為化試験の患者個人データのメタ解析
Delayed versus early initiation of renal replacement therapy for severe acute kidney injury: a systematic review and individual patient data meta-analysis of randomised clinical trials Lancet . 2020 May 9;395(10235):1506-1515. doi: 10.1016/S0140-6736(20)30531-6. Epub 2020 Apr 23. 原文をBibgraph(ビブグラフ)で読む 上記論文の日本語要約 【背景】生命を脅かす合併症がない場合の重症急性腎障害に腎代替療法(RRT)を実施するタイミングは活発に議論されている。著者らは、早期RRT実施と比較した遅延RRTが重症急性腎障害の重症患者の28日時生存率に影響を及ぼすかを評価した。 【方法】この系統的レビューと患者個人データのメタ解析では、MEDLINE(PubMed経由)、Embase、Cochrane Central Register of Controlled Trialsで、2008年4月1日から2019年12月20日に出版され、重症急性腎障害に用いるRRT遅延と早期開始戦略を比較した無作為化試験を検索した。急性腎障害(Kidney Disease: Improving Global Outcomes[KDIGO]の急性腎障害分類2または3、KDIGOが使われていない場合、腎Sequential Organ Failure Assessment[SOFA]スコア3点以上と定義)がある18歳以上の重症患者を対象とした試験を適格とした。各試験の研究生責任者に連絡を取り、患者データの提供を依頼した。対象とした試験から、急性腎障害がない患者、無作為化しなかった患者を患者個人データのメタ解析から除外した。主要転帰は、無作為化28日後の総死亡率とした。この試験は、PROSPERO(CRD42019125025)として登録されている。 【結果】特定した試験1031件のうち、1件は適格基準を満たしたが組み入れ時期が古いため除外し、10件(対象2143例)を解析対象とした。9件(2083例)の患者個人データが入手でき、1879例に急性腎障害があり、946例(50%)を遅延RRT群、933例(50%)を早期RRT群に無作為に割り付けた。遅延RRT群に割り付けデータが入手できた929例中390例(42%)はRRTを受けなかった。28日時までに死亡した患者の割合は、遅延RRT群(837例中366例[44%])と早期RRT群(827例中355例[43%])で有意差がなく(リスク比1.01、95%CI 0.91-1.13、P=0.80)、全体のリスク差は0.01(95%CI -0.04-0.06)であった。試験間に異質性はなく(I2=0%、τ2=0)、ほとんどの試験でバイアスリスクが低かった。 【解釈】重症急性腎障害を呈した重症患者で、緊急RRTの適応がない場合のRRT導入のタイミングは、生存率に影響を及ぼすことがない。患者を緊密にモニタリングしながらRRT導入を遅らせることで、RRT実施率が低下し、ひいては医療資源を節約することになる。 第一人者の医師による解説 AKIでの腎代替療法は「伝家の宝刀」 むやみに使わず迷ったら使う 寺脇 博之 帝京大学ちば総合医療センター第三内科(腎臓内科)教授・腎センター長 MMJ. December 2020;16(6):172 『広辞苑』によると、「伝家の宝刀」とは「代々家宝として伝わっている名刀。転じて、いよいよという時以外にはみだりに使用しない、とっておきの物・手段など」…つまり、のっぴきならない窮地に立たされて初めて抜く刀、とされている。今回紹介する研究(メタアナリシス)は、重症の急性腎障害(acute kidney injury;AKI)を治療する上で、腎代替療法(renal replacement therapy;RRT)がまさにその「伝家の宝刀」であることを示唆する結論、すなわち「AKIに対してRRTを一律に早期導入しても予後への好影響は確認されなかった」を導き出している。  著者らは2008年4月1日~19年12月20日に発表された1,031件の研究から、早期導入戦略(early strategy:何らかの基準に従いAKIと診断された時点でRRTを導入)と晩期導入戦略(delayed strategy:高カリウム血症、肺水腫などのため生命の危険が迫った時点でRRTを導入)を比較した無作為化対照試験(RCT)を抽出。最終的に8件の研究から抽出された早期導入群827人、晩期導入群837人(年齢、性比、入院理由、Sequential Organ Failure Assessment[SOFA]スコア、併存疾患、敗血症合併率、割り付け時の利尿薬使用率に関する差なし)の2群を対象に、以下の項目について比較した:主要評価項目(割り付け後28日以内の全死亡)、副次評価項目(死亡までの期間[28日後まで])、60日全死亡、90日全死亡、院内死亡、入院期間、RRTを要しなかった日数[28日後まで])。  その結果、主要評価項目の28日全死亡について、晩期導入群におけるリスク比は早期導入群に対して1.01(95%信頼区間 , 0.91~1.13;P=0.80)と有意差はなく、さらに副次評価項目についても両群間に差は認められなかった。なお晩期導入群では42%の患者がRRTを1回も受けていなかった:このことは、早期導入群の42%においてRRTは不要であった可能性を示唆する。  今回の研究における「早期導入戦略」にとっての救いは、早期導入群では予後が優れていなかったものの、劣ってもいなかったことである。すなわち、早期導入群の42%にとってRRTは不要だったかもしれないが、不利益ももたらさなかったわけである。結局、今回の研究から得られた教訓は、「むやみには使わない(早期導入で予後は改善されないから)」「でも使うか使わないか迷ったら使う(不利益はもたらさないから)」という、AKIにおける伝家の宝刀たるRRTの“正しい振るい方”だと言うことができる。
中等症ないし重症アトピー性皮膚炎の青少年・成人患者に用いるabrocitinibの有効性および安全性 第III相多施設共同二重盲検プラセボ対照無作為化試験
中等症ないし重症アトピー性皮膚炎の青少年・成人患者に用いるabrocitinibの有効性および安全性 第III相多施設共同二重盲検プラセボ対照無作為化試験
Efficacy and safety of abrocitinib in adults and adolescents with moderate-to-severe atopic dermatitis (JADE MONO-1): a multicentre, double-blind, randomised, placebo-controlled, phase 3 trial Lancet . 2020 Jul 25;396(10246):255-266. doi: 10.1016/S0140-6736(20)30732-7. 原文をBibgraph(ビブグラフ)で読む 上記論文の日本語要約 【背景】経口選択的JAK1阻害薬abrocitinibは、第IIb相試験で、中等症ないし重症アトピー性皮膚炎成人患者に有効で忍容性も良好であった。著者らは、中等症ないし重症アトピー性皮膚炎の青少年および成人患者に用いるabrocitinib単独療法の有効性および安全性を評価した。 【方法】この第III相多施設共同二重盲検プラセボ対照無作為化試験(JADE MONO-1)では、オーストライラ、カナダ、欧州および米国69施設で、12歳以上で体重40kg以上の中等症ないし重症アトピー性皮膚炎(IGAスコア3点以上、EASIスコア16点以上、罹患面積の体表面積に対する割合10%以上かつ最大掻痒の順序尺度[NRS]スコア4点以上)患者を組み入れた。患者を経口abrocitinib 100mg、同200mg、プラセボを12週間にわたって経口投与するグループに(2対2対1の割合で)無作為に割り付けた。双方向自動応答システムを用いて無作為化し、ベースラインの疾患および年齢で層別化した。多重主要評価項目は、IGAスコアが改善した(0点[皮疹消失]または1点[皮疹ほぼ消失]を達成した患者の割合で、試験開始時から2段階以上改善)患者の割合および試験開始時からEASIスコアが75%以上改善した患者の割合(EASI-75達成率)とし、いずれも12週時に評価した。無作為化し治験薬を1回以上投与した全例の完全解析集団を対象に、有効性を評価した。無作為化した全例を対象に安全性を評価した。この試験は、ClinicalTrials.govにNCT03349060番で登録されている。 【結果】2017年12月7日から2019年3月26日にかけて、387例を組み入れ、156例をabrocitinib 100mg群、154例を同200mg群、77例をプラセボ群に割り付けた。組み入れた全例に治験薬を1回以上投与したため、12週間の有効性の評価対象となった。I12週時の主要評価項目のデータが入手できた患者で見ると、GAスコア改善達成率は、aborcitinib 100mg群(156例中37例[24%] vs. 76例中6例[8%]、P=0.0037)、同200mg群(153例中67例[44%]vs. 76例中6例[8%]、P<0.0001)の方がプラセボ群より有意に高かった。12週時の主要評価項目のデータが入手できた患者で、プラセボ群と比較すると、EASI-75を達成した患者の割合は、abrocitinib 100mg群(156例中62例[40%]vs. 76例中9例[12%]、P<0.0001)および同200mg群(153例中96例[63%] vs.76例中9例[12%]、P<0.0001)の方が高かった。abrocitinib 100mg群156例中108例(69%)と同200mg群154例中120例(78%)、プラセボ群77例中44例(57%)に有害事象が報告された。abrocitinib 100mg群156例中5例(3%)、同200mg群154例中5例(3%)、プラセボ群77例中3例(4%)に重度有害事象が報告された。治療関連の死亡は報告されなかった。 【解釈】中等症ないし重症アトピー性皮膚炎の青少年・成人患者に、経口abrocitinibの1日1回単独投与が有効で忍容性も良好であった。 第一人者の医師による解説 アブロシチニブは1日1回の単剤投与で 有用な新規経口全身療法薬となりうる 伊藤 友章(准教授)/大久保 ゆかり(教授) 東京医科大学皮膚科学分野 MMJ. December 2020;16(6):159 アトピー性皮膚炎の基本的な治療は、ステロイド外用薬となるが、重症患者では改善が乏しい。ステロイド内服療法は有効だが、副作用が多い。シクロスポリン内服療法は継続治療ができず治療効果に乏しい。近年、IL-4/13受容体阻害薬デュピルマブが、重症アトピー性皮膚炎治療に用いられ、その有効性は高い。しかし、皮下注射製剤であること、ときに結膜炎の合併が生じ、眼科医のケアを必要とする。アトピー性皮膚炎ではIL-4、IL-13、IL-22、IL-31、thymic stromal lymphopoietin(TSLP)などの炎症サイトカインによるJAK1経路の活性化が病態に関与しており、JAK1が治療標的として注目されている。  本論文は、経口JAK1阻害薬アブロシチニブのアトピー性皮膚炎に対する有効性と安全性を検討した多施設共同、無作為化、プラセボ対照、第3相試験(JADE MONO-1)の報告である。対象は12歳以上、EASIスコア(湿疹面積・重症度指数)16点以上、IGA(皮膚症状重症度の全般評価)3点以上、BSA(アトピー性皮膚炎に罹患した体表面積)10%以上、PP-NRSスコア(最高痒み数値評価尺度)4点以上の患者とした。患者387人が登録され、アブロシチニブ100mg、200mgまたはプラセボを1日1回投与する群に2対2対1の比で割り付けられ、12週間の治療を受けた:100mg群156人(18歳未満22%、EASI31.3*、BSA50.8*)、200mg群154人(18歳未満21%、EASI30.6*、BSA49.9*)、プラセボ群77人(18歳未満22%、EASI28.7*、BSA47.4*;*平均値)。評価項目は、12週の時点におけるIGAスコアが1点以下かつベースラインから2点以上低下した患者の割合(IGA≦1達成割合)、およびEASIスコアがベースラインから75%以上改善した患者の割合(EASI-75達成割合)とされた。  結果は、12週時のIGA≦1達成率は100mg群24%、200mg群44%、プラセボ群8%であり、EASI-75達成割合は100mg群40%、200mg群63%、プラセボ群12%であり、いずれもプラセボ群に比べ有意に高値であった(P<0.0001)。PPNRSは12週時で、100mg群および200mg群とも、ベースラインと比較して、有意に痒みを抑えた。治療に関連のある有害事象として、単純ヘルペスウイルス感染症の発現は100mg群1人、200mg群3人で、帯状疱疹はそれぞれ1人と2人、口腔ヘルペスはそれぞれ3人と1人に発現した。治療関連死の報告はなかった。  結論として、アブロシチニブは、外用薬でコントロールできない、12歳以上の中等症~重症アトピー性皮膚炎患者の治療において、1日1回の単剤投与で有用な新規の経口全身療法薬となりうる。
乳児のアトピー性皮膚炎予防のための皮膚保湿剤と早期補完食(PreventADALL) 多施設共同多因子クラスター無作為化試験
乳児のアトピー性皮膚炎予防のための皮膚保湿剤と早期補完食(PreventADALL) 多施設共同多因子クラスター無作為化試験
Skin emollient and early complementary feeding to prevent infant atopic dermatitis (PreventADALL): a factorial, multicentre, cluster-randomised trial Lancet . 2020 Mar 21;395(10228):951-961. doi: 10.1016/S0140-6736(19)32983-6. Epub 2020 Feb 19. 原文をBibgraph(ビブグラフ)で読む 上記論文の日本語要約 【背景】乳児期早期の皮膚保湿剤によってアトピー性皮膚炎が予防でき、早期補完食導入によって高リスク乳児の食物アレルギーが減少すると思われる。この試験は、一般の乳児で、生後2週間の定期的な皮膚保湿剤使用や生後12-16週齢の間の早期補完食導入によって生後12カ月時までのアトピー性皮膚炎発症を抑制できるかを明らかにすることを目的とした。 【方法】この住民対象の2×2要因無作為化臨床試験は、ノルウェー・オスロ市のオスロ大学病院およびエーストフォール病院トラスト、スウェーデン・ストックホルム市のカロリンスカ大学病院で実施された。妊娠18週時のルーチンの超音波検査実施時に出生前の乳児を登録し、2015年から2017年の間に出生した新生児を以下のクラスターごとに無作為に割り付けた――(1)スキンケアに関して特別な助言はしないが、乳児の栄養に関して国の指針に従うよう助言した対照群(非介入群)、(2)皮膚保湿剤使用(入浴剤やクリーム;皮膚介入群)、(3)ピーナツ、牛乳、小麦、卵の補完食早期導入(食物介入群)、(4)皮膚および食物介入(複合介入群)。コンピュータ生成クラスター無作為化法を用いて、参加者を92の地理的居住地域と3カ月ごと8期間を基に(1対1対1対1の割合で)割り付けた。1週間当たり4日以上、保護者に介入方法を指導した。主要転帰は生後12カ月までのアトピー性皮膚炎とし、介入の割り付けをふせておいた試験担当医師による3、6、12カ月時の診察を基に判定した。12カ月間の追跡期間を完遂後、アトピー性皮膚炎を評価し、UK Working PartyとHanifin and Rajka(12カ月時のみ)の診断基準を満たしているかを診断した。主要有効性解析は、無作為化した全例を対象としたintention-to-treat解析で実施した。2020年に全例の3歳時の診察が終了するとき、食物アレルギーの結果を報告することとした。これは、ORAACLE(the Oslo Research Group of Asthma and Allergy in Childhood; the Lung and Environment)が実施した試験である。この試験は、clinicaltrials.govにNCT02449850番で登録されている。 【結果】2014年12月9日から2016年10月31日の間に、女性2697例を登録し、2015年4月14日から2017年4月17日の間に出生した新生児2397例を組み入れた。非介入群の乳児596例中48例(8%)、皮膚介入群575例中64例(11%)、食物介入群642例中58例(9%)、複合介入群583例中31例(5%)にアトピー性皮膚炎が見られた。皮膚保湿剤、補完食早期導入ともにアトピー性皮膚炎の発症を抑制できず、皮膚介入のリスク差3.1%(95%CI -0.3-6.5)、食物介入で1.0%(-2.1-4.1)となり、対照を支持するものであった。介入による安全性の懸念はなかった。皮膚介入群、食物介入群および複合介入群で報告された皮膚症状や徴候(掻痒、浮腫、皮膚乾燥、蕁麻疹)は、非介入群と比べて頻度は高くなかった。 【解釈】早期皮膚保湿剤や早期補完食導入では、生後12カ月までのアトピー性皮膚炎発症を抑制することができなかった。試験は、生後12カ月までのアトピー性皮膚炎を予防するために、乳児にこの介入法を用いることを支持するものではない。 第一人者の医師による解説 スキンケア方法や離乳食の開始法、その頻度の影響を検討する必要あり 大矢 幸弘 国立成育医療研究センターアレルギーセンター センター長 MMJ. December 2020;16(6):160 アレルギー家系の乳児に新生児期から保湿剤を塗布するスキンケアを行うことでアトピー性皮膚炎の発症予防効果を示した100人規模の2つのランダム化比較試験(RCT)が2014年に発表された(1),(2)。その後、離乳食を3カ月という早期から開始した場合と生後6カ月から開始する場合を比較したEAT試験が2016年に発表され、卵とピーナツに関してはそれぞれのアレルギーの予防効果が示されている(3)。また、コホート研究の中には、離乳食の開始が早い方が食物アレルギーだけでなくアトピー性皮膚炎の発症も少ないという報告もある。  本研究は、生後2週間からバスオイルによる保湿スキンケアと生後12~16週で離乳食を早期開始するという2つの介入の単独または併用を対照群と比較する4群比較 RCTである。主要評価項目は生後12カ月時点でのアトピー性皮膚炎(UK Working PartyまたはHanifinとRajkaの診断基準)と3歳時での食物アレルギーであるが、今回の論文では前者のみ報告されている。対象は、今回解説を併載したBEEP試験のような高リスク家系ではなく一般人口の乳児である。スキンケア介入は、水8Lあたり0.5dLのバスオイルを入れて5~10分入浴し顔全体にクリームを塗布し、石鹸は使用しない。早期離乳食介入は、生後12~16週にピーナツバター、1週遅れて牛乳、翌週小麦のおかゆ、4週目にスクランブルエッグを開始する。スキンケア、離乳食とも週4日以上の実行が指示された。  主要評価項目アトピー性皮膚炎の発症率は、非介入群8%(48/596)、スキンケア群11%(64/575)、早期離乳食群9%(58/642)、併用介入群5%(31/583)であり、介入によるアトピー性皮膚炎の発症予防は実証できなかった。ちなみに、バスオイルを週平均4.5日以上実行した割合はスキンケア群32%、併用介入群33%、週平均5.5日以上はそれぞれ13%と14%であった。早期離乳食のアドヒアランス(4種類のうち3種類以上を生後18週までに開始し、週3~5日以上、5週間以上実施)率は食事介入単独群35%、併用介入群27%であった。  このようにスキンケアを週7日実施した参加者がほとんどいないRCTでスキンケアによるアトピー性皮膚炎の予防効果を実証することは困難と思われるが、研究が行われた北欧では、毎日入浴する習慣がなく実行可能性を考慮して週4日以上というプロトコールとなった。バスオイルでの入浴と入浴後に保湿剤を塗布する効果が同じかどうかは不明であるが、BEEP試験と同じく、中途半端なスキンケアではアトピー性皮膚炎は予防できないという結果を示している。 1. Horimukai K, et al. J Allergy Clin Immunol. 2014;134(4):824-830.e6. 2. Simpson EL, et al. J Allergy Clin Immunol. 2014;134(4):818-823. 3. Perkin MR, et al. N Engl J Med. 2016 May 5;374(18):1733-1743.
アトピー性皮膚炎予防のための1日1回の保湿剤塗布 BEEP無作為化比較試験
アトピー性皮膚炎予防のための1日1回の保湿剤塗布 BEEP無作為化比較試験
Daily emollient during infancy for prevention of eczema: the BEEP randomised controlled trial Lancet . 2020 Mar 21;395(10228):962-972. doi: 10.1016/S0140-6736(19)32984-8. Epub 2020 Feb 19. 原文をBibgraph(ビブグラフ)で読む 上記論文の日本語要約 【背景】アトピー性皮膚炎発症の前に皮膚バリア機能障害が起きる。著者らは、生後1年間の日常的な保湿剤使用によって高リスク乳児のアトピー性皮膚炎が予防できるかを検証した。 【方法】英国の病院12施設とプライマリ・ケア4施設で、多施設共同実用的並行群間無作為化比較試験を実施した。アトピー性皮膚炎発症高リスク(医師が診断したアトピー性皮膚炎、アレルギー性鼻炎または喘息があると両親が報告した一等親近縁者が1人以上いるなど)の正期産新生児(在胎37週以上)を登録するため、出生前または出生後の診察を利用して家族に打診した。アトピー性疾患の家族歴がある正期産新生児を1年間の1日1回保湿剤塗布(DiprobaseクリームとDoubleBaseゲルのいずれか)+標準的なスキンケアの助言(保湿群)と標準的なスキンケアの助言単独(対照群)に(1体1の割合で)無作為に割り付けた。無作為化のスケジュールは、コンピュータが生成したコード(組み入れ施設とアトピー性疾患がある一等親近縁者数で層別化)を用いて作成し、インターネットによる無作為化システムを用いて参加者を割り付けた。主要評価項目は2歳時のアトピー性皮膚炎(UK Working Party診断基準で判定)とし、評価項目のデータが得られた参加者を割り付け遵守に関係なく無作為化したグループとして解析し、層別化変数で調整した。この試験は、ISRCTNにISRCTN21528841番で登録されている。長期追跡のためのデータ収集が進行中だが、試験の登録は終了している。 【結果】2014年11月19日から2016年11月18日にかけて、新生児1394例のうち693例を保湿群、701例を対照群に割り付けた。保湿群で質問票の回答を完遂した参加者の遵守率は、3カ月時88%(532例中466例)、6カ月時82%(519例中427例)、12カ月時74%(506例中375例)だった。2歳時、評価項目のデータが得られた保湿群598例中139例(23%)、対照群612例中150例(25%)にアトピー性皮膚炎があった(調整相対リスク0.95[95%CI 0.78-1.16]、P=0.61、調整リスク差-1.2%[-5.9-3.6])。評価項目の結果は、その他のアトピー性皮膚炎の定義から裏付けられた。1年目の乳児1例当たりの平均皮膚感染数が対照群0.15(SD 0.46)に対して保湿群0.23(SD 0.68)で、調整発生率比1.55(95%CI 1.15-2.09)であった。 【解釈】高リスク乳児で1日1回保湿剤塗布による生後1年間のアトピー性皮膚炎予防に何ら根拠がないことが明らかになり、根拠から皮膚感染のリスクが上昇することが示唆された。試験から、アトピー性皮膚炎、喘息、アレルギー性鼻炎の家族が新生児のアトピー性皮膚炎予防のために1日1回の保湿剤を塗布してはならないことが明らかになった。 第一人者の医師による解説 アドヒアランスが低い大規模研究の問題点を露呈 中途半端なスキンケアでは予防につながらない 大矢 幸弘 国立成育医療研究センターアレルギーセンター センター長 MMJ. December 2020;16(6):161 新生児期から全身に保湿剤を塗布することで、アトピー性皮膚炎の発症率が低下することを示したランダム化対照試験(RCT)が2014年に2件報告され(1),(2)、そのうちの1つ(2)は、今回報告されたBEEP試験のパイロット研究の位置づけであった。  BEEP試験はアレルギー家系の乳児1,394人を対象に行われた多施設共同研究である。主要評価項目は1歳から2歳になるまでの1年間のアトピー性皮膚炎(UK Working Partyの 疫学的診断基準)の発症率で、結果は、保湿群23%(139/598)、対照群25%(150/612)と両群間に有意差はなかった。保湿剤塗布期間は生後1年までであったが、1歳時のアトピー性皮膚炎有病率は両群とも20%で差はなかった。スキンケアという面倒な介入の効果はアドヒアランスに依存すると思われるが、筆者らは良好だったと記載している(生後3カ月時は88%、6カ月時は82%、生後12カ月時は74%)。  では、なぜ、パイロット研究(2)と乖離した結果が出たのであろうか。本研究におけるアドヒアランス良好の定義は、週3~4日以上、頭頸部、四肢、体幹の2カ所以上に保湿剤を塗布していればよいことになっている(パイロット研究では全身)。アドヒアランスの詳細を論文補遺から調べて、パイロット研究と比較したところ、以下のように大きな違いがあった。今回、介入群693人のうち生後12カ月目のアンケートに507人(73%)が回答し、そのうち週7日間保湿剤を塗布していたのは248人(49%)であった。一方、パイロット研究では生後6カ月の主要評価項目の評価時点でのアドヒアランスは54人中44人(81.5%)が週7日保湿剤を塗布していた。介入群では保湿剤塗布が週0日のノンアドヒアランスは0%だったのに対してBEEP試験では12カ月時点で58人(11%)もいた。パイロット研究と同じ生後6カ月時点のアドヒアランスを比べてもBEEP試験では週0日塗布が6%、週7日塗布が56%と著しく低い。しかも、対照群で週3日以上保湿剤を塗布していた人は生後6カ月と12カ月で28%もいたのである。Pragmatic trialというと聞こえは良いが、プロトコールの遵守率が低下する大規模研究の問題点が露呈したとも言える。アトピー性皮膚炎の治療効果はアドヒアランスに依存するが予防効果も同様であったことを本研究は示した。中途半端なスキンケアでは予防はできない、ということである。 1. Horimukai K, et al. J Allergy Clin Immunol. 2014;134(4):824-830.e6. 2. Simpson EL, et al. J Allergy Clin Immunol. 2014;134(4):818-823.
駆出率が低下した心不全に用いる包括的疾患修飾薬物療法の生涯にわたる便益の推測 無作為化比較試験3件の比較分析
駆出率が低下した心不全に用いる包括的疾患修飾薬物療法の生涯にわたる便益の推測 無作為化比較試験3件の比較分析
Estimating lifetime benefits of comprehensive disease-modifying pharmacological therapies in patients with heart failure with reduced ejection fraction: a comparative analysis of three randomised controlled trials Lancet . 2020 Jul 11;396(10244):121-128. doi: 10.1016/S0140-6736(20)30748-0. Epub 2020 May 21. 原文をBibgraph(ビブグラフ)で読む 上記論文の日本語要約 【背景】駆出率が低下した心不全(HFrEF)に3つの薬剤クラス(ミネラルコルチコイド受容体拮抗薬[MRA]またはアンジオテンシン受容体ネプリライシン阻害薬[ARNI]、ナトリウム・グルコース共益輸送担体2[SGLT2]阻害薬)を用いると、アンジオテンシン変換酵素(ACE)阻害薬またはアンジオテンシン受容体拮抗薬(ARB)とβ遮断薬を用いた従来療法よりも死亡率が低下する。各クラスはこれまで別々の基礎療法で検討されていたが、併用療法として予想される便益は知られていない。ここに、これまで報告された無作為化比較試験3件のデータを用いて、慢性HFrEFに用いる包括的治療と従来治療によって獲得される無イベント生存および全生存を推定した。 【方法】このcross-trial解析では、3件のきわめて重要な試験、EMPHASIS-HF(2737例)、PARADIGM-HF(8399例)およびDAPA-HF(4744例)を間接的に比較することによって、慢性HFrEFに用いる総括的疾患修飾薬物療法(ARNI、β遮断薬、MRAおよびSGLT2阻害薬)の従来の薬物療法と比較した治療効果を推定した。主要評価項目は、心血管死または心不全による初回の入院の複合とした。このほか、3つの評価項目を個別に評価し、総死亡率を評価した。この関連のある治療効果は長い期間をかけて一貫して見られると仮定して、EMPHASIS-HF試験の対照群(ACE阻害薬またはARBとβ遮断薬)で、包括的疾患修飾療法によって長期的に得られる無イベント生存および全生存の増加分を見積もった。 【結果】従来療法と比較して包括的疾患修飾療法が主要評価項目(心血管死または心不全による入院)にもたらす帰属集計効果のハザード比(HR)は0.38(95%CI 0.30-0.47)だった。このほか、心血管死単独(HR 0.50、95%CI 0.37-0.67)、心不全による入院単独(同0.32、0.24-0.43)、全死因死亡(0.53、0.40-0.70)のハザード比も有利であった。従来療法に比べると、包括的疾患修飾薬物療法によって心血管死または心不全による初回入院が2.7年(80歳時)から8.3年(55歳時)、生存が1.4年(80歳時)から6.3年(55歳時)長くなると推定された。 【解釈】HFrEFで、早期包括的疾患修飾薬物療法で得られると推定される治療効果はきわめて大きく、新たな標準的治療としてARNI、β遮断薬、MRAおよびSGLT2阻害薬の併用を支持するものである。 第一人者の医師による解説 ARNI、SGLT2阻害薬、MRAの導入 生命予後改善に有用 鈴木 秀明(助教)/安田 聡(教授) 東北大学大学院医学系研究科循環器内科学分野 MMJ. December 2020;16(6):169 日本における死亡総数は、心疾患による死亡が悪性新生物に次ぎ2番目に多い。心疾患の内訳では心不全死が最多を占め、心不全入院患者数は依然増加し続けている。駆出率が低下した心不全(HFrEF)では、アンジオテンシン変換酵素(ACE)阻害薬/アンジオテンシンII受容体拮抗薬(ARB)とβ遮断薬のエビデンスは確立しており、この2剤に加えミネラルコルチコイド受容体拮抗薬(MRA)、アンジオテンシン受容体・ネプリライシン阻害薬(ARNI)、Na+/グルコース共役輸送担体(SGLT)2阻害薬の導入は生命予後をさらに改善することが報告されている。しかし心不全治療の現場において、HFrEF患者に対するARNI、SGLT2阻害薬、MRAの導入は必ずしも進んでいない。  本研究では、HFrEFに対するARNI、SGLT2阻害薬、MRAの有用性を示した3件の臨床試験(それぞれPARADIGM-HF[n=8,399]、DAPA-HF[n=4,744]、EMPHASIS-HF[n=2,737])を比較することで、ACE阻害薬/ARB+β遮断薬の2剤が導入されたHFrEF患者に対し、ARNI・SGLT2阻害薬・MRAの3剤導入が与える効果について検証が行われた。結果として、これら3剤の導入は、1次エンドポイントである心血管死・心不全入院を62%減少させ(ハザード比[HR], 0.38)、心血管死(HR, 0.50)、心不全入院(HR, 0.32)、全死亡(HR, 0.53)も減少させた。こうした予後改善効果は、55歳、80歳時点の導入において心血管死・心不全入院をそれぞれ8.3年、2.7年間遅らせ、全死亡を6.3年、1.4年間遅らせることに相当すると推定された。  本研究の限界として、①新しいランダム化比較試験を行ったわけではなく、過去の臨床試験を比較検討した内容、②薬物の中止やアドヒアランスを考慮していない、③有害事象や費用面の検討を行っていない、④イバブラジン(HCNチャネル遮断薬)やベルイシグアト(可溶性グアニル酸シクラーゼ刺激薬)といったHFrEFにエビデンスのある他の薬剤や、デバイスなどの非薬物治療の効果を検討していない、などが挙げられる。しかし、こうした点を考慮しても、ACE阻害薬/ ARB+β遮断薬の2剤による従来治療に加え、ARNI・SGLT2阻害薬・MRAの3剤を導入することはHFrEF患者の生命予後を改善する上で有用であることが本研究で改めて明らかになったと言えよう。
21の高所得国、中所得国および低所得国の15万5722例の修正可能な危険因子、心血管疾患および死亡率(PURE研究) 前向きコホート研究
21の高所得国、中所得国および低所得国の15万5722例の修正可能な危険因子、心血管疾患および死亡率(PURE研究) 前向きコホート研究
Modifiable risk factors, cardiovascular disease, and mortality in 155 722 individuals from 21 high-income, middle-income, and low-income countries (PURE): a prospective cohort study Lancet . 2020 Mar 7;395(10226):795-808. doi: 10.1016/S0140-6736(19)32008-2. Epub 2019 Sep 3. 原文をBibgraph(ビブグラフ)で読む 上記論文の日本語要約 【背景】世界で共通の修正可能な危険因子が心血管疾患と死亡率に及ぼす作用の推定の大部分が、異なる方法を用いた別々の研究データに基づいている。Prospective Urban Rural Epidemiology(PURE)研究は、経済水準で層別化した21カ国(5大陸)で、修正可能な危険因子が心血管疾患および死亡率に及ぼす作用を前向きに測定するのにほぼ同じ方法を用いてこの欠点を克服している。 【方法】この国際共同前向きコホート研究では、高所得国(HIC)、中所得国(MIC)および低所得国(LIC)21カ国から登録した、心血管疾患の既往歴がない参加者15万5722例を対象に、14の修正可能な危険因子と死亡率・心血管疾患の関連を調査した。この論文の主要評価項目は、心血管疾患事象(心血管死、心筋梗塞、脳卒中および心不全と定義)および死亡率とした。有病率、ハザード比(HR)および行動因子クラスター(喫煙週間、血圧、アルコール、食事法、運動および塩分摂取など)、代謝因子(脂質、血圧、糖尿病、肥満など)、社会経済的および心理社会的因子(教育、うつ症状など)、握力、家庭内および環境汚染と関連を示す心血管疾患と死亡率の人口寄与割合(PAF)を明らかにした。多変量Coxフレイルティモデルを用いて危険因子と転帰の関連を証明し、コホート全体ではPAFを用いて、このほか国の所得水準別の分類別でこの関連を明らかにした。 【結果】2005年1月6日から2016年12月4日野間に、15万5722例を組み入れ、追跡子て危険因子を測定した。1万7249例(11.1%)がHIC、10万2680例(65.9%)がMIC、3万5793例(23.0%)がLICから組み入れた参加者であった。試験全体の対象者でみた心血管疾患と死亡の約70%が修正可能な危険因子によるものであった。代謝因子が心血管疾患の最も大きな危険因子(PAFの41.2%)で、高血圧が最大であった(PAFの22.3%)。クラスターとして、行動危険因子のほとんどが死亡の寄与因子(PAFの26.3%)であったが、最も大きな危険因子は低学歴であった(PAFの12.5%)。大気汚染に心血管疾患のPAFの13.9%との関連が見られたが、この解析では異なる統計手法を用いた。MICとLICで、家庭内空気汚染、質の悪い食生活およぎ低握力がHICよりも心血管疾患および死亡率に大きな作用を及ぼしていた。 【解釈】ほとんどの心血管疾患と死亡が数少ない共通の修正可能な危険因子によるものであった。世界の広範囲にわたって影響を及ぼす因子(高血圧、学歴など)がある一方で、国の経済水準によって異なる因子(家庭内空気汚染、質の悪い食生活)もあった。健康政策には、世界的に心血管疾患や死亡率を回避するのに最も大きな効果がある危険因子に焦点を当て、さらに、特定の国に大きな重要性がある危険因子に力を入れるべきである。 第一人者の医師による解説 低所得国では教育年数や大気汚染の影響大 日本への適用には注意 山岸 良匡筑波大学医学医療系社会健康医学教授・茨城県西部メディカルセンター/磯 博康 大阪大学大学院医学系研究科公衆衛生学教授 MMJ. December 2020;16(6):180 本論文のPURE研究は、21カ国の一般集団において、14の介入可能な循環器危険因子について、循環器病や全死亡との関連および寄与リスクを示すことを目的としている。従来、このようなテーマについては各国で別々の手法を用いたコホート研究が行われてきたが、本研究は、21カ国で統一した手法で行ったことを1つの売りとして、特にそれらの関連や寄与危険度が国の富裕度(所得)で異なるか、という点に焦点を当てている。  実際、14の危険因子は、国の富裕度によって分布が大きく異なることが示された。論文では危険因子の関連の強さの比較について(行動関連危険因子では喫煙が最も関連が強いなどといった)議論がなされているが、これについては危険因子の定義によって結果が異なる可能性に留意する必要がある。それよりも、それらの関連の強さが国の富裕度によって異なることを示した点が本研究の重要な知見と言える。例えば全死亡について、高所得国では喫煙との関連が強いのに対し、低所得国ではアルコールや教育年数との関連が強い、などである。寄与リスクの観点からは、循環器病の71%が介入可能な危険因子によるものであり、その割合は低所得国の方が高い、すなわち介入の余地が大きいことが示されている。そのほか、循環器病、心筋梗塞、脳卒中ごとの各危険因子の寄与の違いや、それらの国の富裕度による違い(低所得国では教育年数、食事、家庭レベルの大気汚染の影響が大きい)など、興味深い知見が得られている。  公衆衛生学的には興味深い論文であるが、日本人の読者にとって注意が必要なのは今回の参加国である。例えば、本研究の高所得国(カナダ、サウジアラビア、スウェーデン、アラブ首長国連邦)には米国や西欧諸国は含まれていない。東アジアでは中国が中所得国として含まれているが、日本や韓国、台湾、シンガポールなどは含まれていない(このため、食塩摂取量は中国が含まれる中所得国で最も高い)。したがって、本研究の特記すべき成果は、グローバルに共通する強力な危険因子(高血圧、喫煙、食事など)と、国の富裕度により寄与が異なる危険因子(教育年数や大気汚染など)の存在を示したことといえる。日本では、PUREの各国とは、危険因子の分布だけでなく疾病構造(欧米と比較して脳卒中が多く心筋梗塞が少ない)も異なる(1)ことを念頭に置く必要があり、わが国のことは自国のデータで論じる必要がある。日本にも同様に危険因子の寄与リスクを算出した研究(2)があり、喫煙と高血圧の寄与が大きいことが示されているが、より最近のデータを用いてPUREの結果と対比することが有用であろう。 1. Brunner E, et al. (Ed) Health in Japan: Social Epidemiology of Japan since the 1964 Tokyo Olympics. Oxford University Press 2020. 2. Ikeda N, et al. Lancet. 2011;378(9796):1094-1105.
2010〜19年の世界の発がん頻度と危険因子の解析 男性50.6%、女性36.3%が危険因子起因のがんで死亡
2010〜19年の世界の発がん頻度と危険因子の解析 男性50.6%、女性36.3%が危険因子起因のがんで死亡
The global burden of cancer attributable to risk factors, 2010-19: a systematic analysis for the Global Burden of Disease Study 2019 Lancet. 2022 Aug 20;400(10352):563-591. doi: 10.1016/S0140-6736(22)01438-6. 上記論文のアブストラクト日本語訳 ※ヒポクラ×マイナビ 論文検索(Bibgraph)による機械翻訳です。 [背景]潜在的に修正可能な危険因子に起因するがんの負担の大きさを理解することは、効果的な予防および緩和戦略の開発に不可欠です。 2019 年の疾病、傷害、危険因子の世界的負担に関する研究(GBD)の結果を分析し、がん対策計画の取り組みを世界的に知らせました。 [方法] GBD 2019 の比較リスク評価フレームワークを使用して、行動、環境、および職業、および代謝の危険因子。世界がん研究基金の基準に基づいて、合計 82 のリスクと結果のペアが含まれていました。 2019 年の推定がん死亡数と障害調整生存年数 (DALY)、および 2010 年から 2019 年までのこれらの指標の変化が示されています。 % 不確実性区間 4.01-4.94) の死亡と 1 億 500 万 (95.0-116) の DALY を合わせて、がんによる全死亡の 44.4% (41.3-48.4) と 42.0すべての DALY の % (39·1-45·6)。男性では2.88百万(2.60-3.18)のリスクに起因する癌による死亡があり(すべての男性の癌による死亡の50.6%[47.8-54.1])、1.58百万(1. 36-1.84) 女性のリスクに起因する癌による死亡 (全女性の癌による死亡の 36.3% [32.5-41.3])。 2019 年の両性を合わせたリスクに起因するがんによる死亡と DALY について、世界で最も詳細なレベルでの主要な危険因子は喫煙であり、アルコール使用と高 BMI がそれに続きました。リスクに起因するがんの負荷は、世界の地域と社会人口統計学的指標 (SDI) によって異なり、2019 年のリスクに起因するがんの DALY が低い SDI の場所では、喫煙、危険なセックス、アルコールの使用が 3 つの主要な危険因子でした。 SDI の場所は、上位 3 つの世界的な危険因子ランキングを反映しています。 2010 年から 2019 年にかけて、世界のリスクに起因するがんによる死亡は 20.4% (12.6-28.4) 増加し、DALY は 16.8% (8.8-25.0) 増加し、代謝の増加率が最も高かったリスク (34.7% [27.9-42.8] および 33.3% [25.8-42.0]).危険因子は、2010 年から 2019 年の間に最大の増加を示しました。これらの修正可能な危険因子への曝露を減らすことで、世界中のがん死亡率と DALY 率が低下し、政策は地域のがん危険因子負担に合わせて適切に調整する必要があります。[資金提供]ビル & メリンダ ゲイツ財団。 第一人者の医師による解説 がんの半数近くは対策可能な危険因子による予防が可能なことを明示 西原 広史 慶應義塾大学医学部腫瘍センターゲノム医療ユニット教授 MMJ.February 2023;19(1):22 予防医学的に対応可能な危険因子に起因するがんの発症頻度を把握しておくことは、がんの発症回避や早期発見・治療につながり、総合的ながん対策として重要である。世界の疾病・傷害・危険因子負荷研究(GBD)2019からは、204国・地域におけるがんの発症数・死亡数、障害調整生存年(DALY)(1)、危険因子(2)の2019年推計値、2010~19年の推移が報告されており、本論文はさらに、がんの発症にどのような危険因子がどの程度関わっているのかを明らかにした大変興味深い報告である。著者らは、がん発症との因果関係が証明されている危険因子(日常的な行動、生活環境・職業、代謝関連[食事、BMIなど])とアウトカム(がん死亡など)の合計82のペア(がん23種類、危険因子34種類)を対象に、危険因子、がん種、地域別に2019年のがん死亡数、がんDALYを推計し、2010年の数値との比較を行った。 その結果、全体では、何かしらの危険因子に起因するがん死亡者の割合は、男性で50.6%、女性で36.3%であった。そのうちがん死亡とDALYへの影響が最も大きい危険因子は男女ともに喫煙であり、続いてアルコールとBMI高値であった。喫煙、大気汚染、職業因子は肺がん、アルコールは男性では喉頭・咽頭がんと消化管がん、女性では乳がんと消化管がん、薬物摂取歴は肝がん、食生活は消化管がん、BMI高値は男性では肝がんと消化管がん、女性で子宮がん、乳がんと強い関連が認められた。この結果は先行疫学研究の報告を裏付けるとともに、がんの発症機序を考えれば当然のことと言える。社会人口指数(SDI:収入、教育、出生率の混合指標)の低い地域では、高い地域に比べ、がん死亡者数が多く、感染対策なしの性交渉が高い危険因子となっているのはヒトパピローマウイルス(HPV)感染による子宮頸がんやヒト免疫不全ウイルス(HIV)による発がんが考えられる。2010~19年にかけて、危険因子に起因するがん死亡者数は20.4%、DALYは16.8%増加した。特に代謝関連リスクの上昇が顕著なのは世界的な生活レベル上昇とそれに伴う肥満や糖尿病者の増加に起因すると考えられる。 本研究は、がんの半数近くは対策が可能な危険因子によるものであり、がんは予防が可能であることを明示している。日本は年間約100万人の新規発症がん患者を抱える国3であり、がん予防対策は喫緊の課題である。近年は喫煙者が減っている一方で、肥満や糖尿病などの代謝関連疾患患者が増えており、こうした部分に対する集中的な対策が重要だと考えられる。 1. Global Burden of Disease 2019 Cancer Collaboration. JAMA Oncol.2022;8(3):420-444. 2. GBD 2019 Risk Factors Collaborators. Lancet. 2020;396(10258):1223-1249. 3. 最新がん統計(がん情報サービス)https://ganjoho.jp/reg_stat/statistics/stat/summary.html